Download as pdf or txt
Download as pdf or txt
You are on page 1of 324

MatheMatics-ii

B.Tech. I-Year II-Sem (Common to All Branches)


JNTU - Anantapur
Contents
Introduction to the Subject
Syllabus as per R15 Curriculum
List of Important Formulae L.1 - L.4
Latest exam question paper with soLutions
May-18 (R15) QP.1 - QP.16
previous exam question papers with soLutions
May/June-17 (R15) QP.1 - QP.16
May/June-16 (R15) QP.1 - QP.20
miD-wise objective type & essay type questions with Key
MID-I (Units-1& 2) M.1 - M.8
MID-II (Units- 3, 4 & 5) M.9 - M.17
Model Question Papers with Solutions (As per the New External Exam Pattern)
Model Paper-I to III MP.1 - MP.6

unit-wise short & essay questions with soLutions

unit no. unit name question nos. page nos.


topic no. topic name

UNIt - I LAPLACE tRANSFoRM Q1 - Q60 1.1 - 1.56

Part-A ShoRt QUEStIoNS wIth SoLUtIoNS Q1 - Q12 1.1 - 1.4


Part-B ESSAy QUEStIoNS wIth SoLUtIoNS Q13 - Q60 1.5 - 1.56
1.1 Laplace Transform of Standard Functions Q13 - Q18 1.5
1.2 Inverse Transform Q19 - Q24 1.9
1.3 First Shifting Theorem Q25 - Q28 1.20
1.4 Transforms of Derivatives and Integrals Q29 - Q32 1.23
1.5 Unit Step Function Q33 - Q34 1.26
1.6 Second Shifting Theorem Q35 - Q39 1.27

1.7 Dirac’s Delta Function Q40 1.29

1.8 Convolution Theorem Q41 - Q44 1.30

1.9 Laplace Transform of Periodic Function Q45 - Q47 1.34

1.10 Differentiation and Integration of Transform Q48 - Q53 1.37

1.11 Application of Laplace Transforms to Ordinary Differential


Equations of First and Second Order Q54 - Q60 1.41
UNIt - II FoURIER SERIES Q1 - Q39 2.1 - 2.54
Part-A ShoRt QUEStIoNS wIth SoLUtIoNS Q1 - Q10 2.1 - 2.4
Part-B ESSAy QUEStIoNS wIth SoLUtIoNS Q11 - Q39 2.5 - 2.54
2.1 Determination of Fourier Coefficients Q11 - Q16 2.20
2.2 Fourier Series, Even and Odd Functions Q17 - Q22 2.26
2.3 Fourier Series in an Arbitrary Interval Q23 - Q27 2.36
2.4 Even and Odd Periodic Continuation Half-range
Fourier sine and cosine Expansions Q28 - Q36 2.50
2.5 Parseval’s Formula-Complex Form of Fourier Series Q37 - Q39 2.54
UNIt - III FoURIER tRANSFoRM Q1 - Q40 3.1 - 3.32
Part-A ShoRt QUEStIoNS wIth SoLUtIoNS Q1 - Q17 3.1 - 3.5
Part-B ESSAy QUEStIoNS wIth SoLUtIoNS Q18 - Q40 3.6 - 3.32
3.1 Fourier Integral Theorem (Only Statement) – Fourier
sine and cosine integrals Q18 - Q20 3.6

3.2 Fourier Transform – Fourier Sine and Cosine


Transforms – properties Q21 - Q31 3.8

3.3 Inverse Transforms Q32 - Q34 3.20

3.4 Finite Fourier Transforms Q35 - Q40 3.23

UNIt - IV PARtIAL DIFFERENtIAL EQUAtIoNS Q1 - Q52 4.1 - 4.52

Part-A ShoRt QUEStIoNS wIth SoLUtIoNS Q1 - Q10 4.1 - 4.4

Part-B ESSAy QUEStIoNS wIth SoLUtIoNS Q11 - Q52 4.5 - 4.52

4.1 Formation of Partial Differential Equations by Elimination of


Arbitrary Constants and Arbitrary Functions Q11 - Q26 4.5

4.2 Method of Separation of Variables Q27 - Q33 4.13

4.3 Solutions of One-Dimensional Wave Equation,


Heat Equation Q34 - Q47 4.18

4.4 Two Dimensional Laplace Equation Under Initial


and Boundary Conditions Q48 - Q52 4.46
UNIt - V Z-tRANSFoRM Q1 - Q48 5.1 - 5.40

Part-A ShoRt QUEStIoNS wIth SoLUtIoNS Q1 - Q10 5.1 - 5.7

Part-B ESSAy QUEStIoNS wIth SoLUtIoNS Q11 - Q48 5.8 - 5.40

5.1 Introduction Q11 - Q14 5.8

5.2 Inverse Z-transform Q15 - Q22 5.11

5.3 Properties – Damping Rule – Shifting Rule Q23 - Q29 5.16

5.4 Initial and Final Value Theorems Q30 - Q33 5.21

5.5 Convolution Theorem Q34 - Q38 5.26

5.6 Solution of Difference Equation by Z-transforms Q39 - Q29 5.40


Syllabus
UNIT-I

Laplace transform of standard functions – Inverse transform – First shifting theorem, Transforms of derivatives

and integrals – Unit step function – Second shifting theorem – Dirac’s delta function – Convolution theorem –

Laplace transform of periodic function.

Differentiation and integration of transform – Application of Laplace transforms to ordinary differential equations

of first and second order.

UNIT-II

Fourier Series: Determination of Fourier coefficients – Fourier series – Even and odd functions – Fourier series

in an arbitrary interval – Even and odd periodic continuation – Half-range Fourier sine and cosine expansions –

Parseval’s formula – Complex form of Fourier series.

UNIT-III

Fourier integral theorem (only statement) – Fourier sine and cosine integrals. Fourier transform – Fourier sine

and cosine transforms – Properties – Inverse transforms – Finite Fourier transforms.

UNIT-IV

Formation of partial differential equations by elimination of arbitrary constants and arbitrary functions – Method

of separation of variables – Solutions of one dimensional wave equation, Heat equation and two - dimensional

Laplace’s equation under initial and boundary conditions.


UNIT-V

Z-transform – Inverse Z-transform – Properties – Damping rule – Shifting rule – Initial and final value theorems.
Convolution theorem – Solution of difference equations by Z-transforms.
INTRODUCTION TO THE SUBJECT
Mathematics is a language in which every symbol and every combination has a specific meaning which can be known
by applying logical rules. This is the most challenging and important subject at all the levels of student education.
The mathematical skills are not only required in various work environments ranging from services to manufacturing
but they are also a must if you want to continue your education.

At the basic level, mathematical education begins with learning skills such as identifying different shapes, measuring
space, counting things, using operations such as addition, subtraction, multiplication and division. It also introduces
time which measures duration of a complete day from sunrise to sunset and sunset to sunrise.

Based on this ground knowledge of mathematics, more advanced concepts and operations such as sets and real
numbers, polynomials, coordinate geometry, progressions, mensuration, trigonometry and statistics are developed.
However, you come across all these concepts in various levels in schooling (till 10th class). Indeed, these mathematical
courses are the required disciplines for entering college and university programs in science, biology, engineering,
economics and business.

The tools and techniques explained in the subject Mathematics-II are used in various engineering disciplines for
easy analysis and simplification of concepts.

The table below gives the complete idea about the number of questions that can be asked from each unit in external
examination along with their weightage. This will be helpful for the students to plan and score good marks in their
exams.

No. of Weightage
Unit No. Unit Name Description
Questions of marks

Short Essay

1 Laplace Transform 2 1 14 This unit presents a brief study on Laplace


transform of standard functions, inverse
transform, transforms of derivatives and
integrals laplace transform of periodic
function. It also includes application of
Laplace transforms to ordinary differential
equations of first and second order.
2 Fourier Series 2 1 14 This unit deals with the determination of
Fourier coefficients, even and odd functions,
Fourier series in an arbitrary interval, even
and odd periodic continuation, half-range
Fourier sine and cosine expansions,
Parseval’s formula and complex form of
Fourier series.
3 Fourier Transform 2 1 14 This unit emphasizes the concepts of
Fourier integral theorem and Fourier sine and
cosine integrals. It also includes Fourier
transform, Fourier sine and cosine transform
properties, inverse transforms and finite
Fourier transforms.
4 Partial Differential 2 1 14 This unit introduces Formation of partial
Equations differential equations by elimination of
arbitrary constants and arbitrary functions.
It also includes solutions of one dimensional
wave equation, heat equation and two
dimensional Laplace’s equation under
initial and boundary conditions.

5 Z-Transform 2 1 14 This unit motivates the study of Z-transform


and inverse Z-transform. It also includes
initial and final value theorems and
Convolution theorem.
MID-I: (Units - 1 & 2) M.1

MID - I
(UNIT S-1 & 2)

OBJECTIVE TYPE &


ESSAY QUESTIONS WITH KEY

SPECTRUM ALL-IN-ONE JOURNAL FOR ENGINEERING STUDENTS SIA GROUP


M.2 MATHEMATICS-II [JNTU-ANANTAPUR]

MID-I (Objective Type & Essay Questions with Key)


Objective Type

I. Fill in the Blanks

1. The Laplace transform of a unit step function is _____ .

6(s + 200)
2. If I(s) = , initial value of i(t) is, _____.
s(s+ 100)

3. The final value theorem is Lt f (t ) = _____ .


t →∞

d2
4. If δ(t) is a unit impulse function, Laplace transform of δ(t) is _____.
d2

5. The Fourier series expansion of an even function in (– c, c) has only _____ terms.

6. The Fourier expansion of an odd function has only _____ terms.

7. In the Fourier series expansion of f(x) = |sinx| in (– π, π), the value of bn = _____.

8. If f(x) = x4 in (– 1, 1) then the Fourier coefficent, bn = _____.

9. If f(x) is a periodic function with period 2T, then bn = _____.

10. The condition for expansion of a function in series are known as _____ conditions.

II. Multiple Choice

1
1. The inverse Laplace transform of in the region δ < a is _________ . [ ]
s−a

(a) –e–atu(t) (b) eatu(t)

(c) eatu(–t) (d) –eatu(–t)

2(s + 1)
2. If X(s) = , then x (0+) and x( ∞ ) are given by _________ and _________ respectively. [ ]
s 2 2s + 5

(a) 0, 0 (b) 2, 0

(c) 0, 2 (d) 2, 2

Look for the SIA GROU P LOGO on the TITLE COVER before you buy
MID-I: (Units - 1 & 2) M.3

1
3. The inverse Laplace transform of (1–e–as) is _________. [ ]
s

(a) u(t) – u(t – a) (b) u(t) + u(t – a)

(c) u(t – a) – u(t) (d) u(t) – u(t + a)

4. The final value theorem in Laplace transform determines the _________value of the system output. [ ]

(a) Marginal state (b) Transient state

(c) Steady state (d) None of the above

5. Laplace transform of e–2t sin h 2t is _________. [ ]

2 2
(a) (s + 4) (b) s(s+ 4)

2 2
(c) (s − 4) (d) s(s− 4)

⎧⎪eβt , for t < 0


6. If f(t) = ⎨ αt , its Laplace transform will exist if and only if _________ . [ ]
⎪⎩e for t > 0

(a) α > Re(s) < β (b) α < Re(s) < β

(c) α > Re(s) > β (d) α < Re(s) > β

7. The convolution of x(t) and y(t) is given by x(t)*y(t) = _________. [ ]

∞ t
(a)
∫ x (t ) y(t – τ) dτ (b) ∫ x(t ) y(t – τ) dτ
0
0

t t
(c)

0
x(t − τ) y(t) dτ (d)
∫ x(t ) y(t – τ) dτ
0

8. The advantage of Laplace transform is _________ . [ ]

(a) The total solution is more systematic

(b) The solution is in frequency domain only

(c) Initial conditions are incorporated in very first step

(d) All the above

SPECTRUM ALL-IN-ONE JOURNAL FOR ENGINEERING STUDENTS SIA GROUP


M.4 MATHEMATICS-II [JNTU-ANANTAPUR]
9. A function f(x) defined for 0 < x < 1 can be extended to an odd periodic function in _________. [ ]

(a) (1, 1) such that f(x) = f(–x) (b) (–1, 1) such that f(x) = f(–x)

(c) (1, 1) such that f(x) = 0 (d) None

10. In the Fourier series expansion of a function, the Fourier coefficient a0 represents _________
value of the function. [ ]

(a) Average (b) Maximum

(c) Minimum (d) Mean

11. The formulae for finding half range cosine series for the function f(x) in (0, 1) are _________. [ ]

l l
2 2 nπ x
(a) a0 =
l ∫0
f ( x)dx, an =
l ∫ f ( x) cos
0
l
dx

l l
1 1 nπx
(b) a0 =
2l ∫
0
f ( x) dx, an =
2l ∫ f ( x) cos
0
l
.dx

l l
nπ x
(c) ∫
a0 = 2l f ( x) dx, a n = 2l f ( x) cos
0

0
l
dx

l l
1 1 nπx
(d) a0 =
l ∫
0
f ( x) dx , a n =
l ∫ f ( x) cos
0
l
dx

12. The half range sine series for 1 in (0, T) is _________. [ ]

1⎧ sin 3 x ⎫
(a) ⎨sinx + + sin 5 x + ...⎬
π⎩ 3 ⎭

⎧ sin 3 x ⎫
(b) 4 π ⎨sinx + + sin 5 x + ...⎬
⎩ 3 ⎭

4⎧ sin 3 x ⎫
(c) ⎨sinx + + sin 5 x + ...⎬
π⎩ 3 ⎭

1 ⎧ sin 3 x ⎫
(d) ⎨sinx + + sin 5 x + ...⎬
4π ⎩ 3 ⎭

Look for the SIA GROU P LOGO on the TITLE COVER before you buy
MID-I: (Units - 1 & 2) M.5
13. If f(x) = x cos x in (–π, π), then b1 = _________. [ ]

(a) –2 (b) –1

(c) 0 (d) 1

⎧− x, x<0
14. If f(x) = ⎨ then f(x) is _________ function. [ ]
⎩ x, 0 < x < π

(a) Odd (b) Even and odd

(c) Even (d) None

15. The half range sine series for f(x) = ex in (0, π) is _________. [ ]

∑n
9 n
(a) 2
[1 + ( −1) n +1 e x ] sinx
2 n =1 +1

∑n
2 n
(b) 2
[1 + ( −1) n +1 e x ] sinx
9 n =1 +1


n2 +1

2
(c) [1 + ( −1) n +1 e x ]
9 n =1
n


n2 +1

9
(d) [1 + ( −1) n +1 e x ]
2 n =1
n

III. Match the Following

(i)

⎛ a 2 + p2 ⎞
1. Fourier expansion of odd function has [ ] (a) 2 log ⎜⎜ 2 2 ⎟⎟. 0 < b < a
⎝b +p ⎠

–d
2. Fourier expansion of even function has [ ] (b) Fc (S)
ds

⎡ e – bt – e – at ⎤
3. Fc ⎢ ⎥= [ ] (c) – s2[F(U)]
⎢⎣ t ⎥⎦

4. Fs[x f(x)] [ ] (d) sine terms

5. Fourier expansion of derivative U(x, t) [ ] (e) cosine terms

SPECTRUM ALL-IN-ONE JOURNAL FOR ENGINEERING STUDENTS SIA GROUP


M.6 MATHEMATICS-II [JNTU-ANANTAPUR]
(ii)

s
1. L[sin (at)] [ ] (a)
s – a2
2

a
2. L[eat sinh (bt)] [ ] (b)
s + a2
2

ssin(b) + acos(b)
3. L[t cos(at)] [ ] (c)
s2 + a 2

s2 – a 2
4. L[cos h (at)] [ ] (d)
(s 2 + a 2 ) 2

b
5. L[sin (at + b)] [ ] (e)
(s – a) 2 – b 2

KEY

I. Fill in the Blanks

1
1.
s

2. 6A

3. Lt s F(s)
s→ 0

4. s2

5. cosine

6. sine

7. 0

8. 0

α+2T
1 nππ
9.
T ∫
α
f(x) sin
T
. dx

10. Dirichlet’s

Look for the SIA GROU P LOGO on the TITLE COVER before you buy
MID-I: (Units - 1 & 2) M.7
II. Multiple Choice

1. (d)

2. (b)

3. (a)

4. (c)

5. (b)

6. (b)

7. (b)

8. (d)

9. (b)

10. (d)

11. (a)

12. (c)

13. (d)

14. (a)

15. (b)

III. Match the Following

(i)

1. (d)

2. (e)

3. (a)

4. (b)

5. (c)

SPECTRUM ALL-IN-ONE JOURNAL FOR ENGINEERING STUDENTS SIA GROUP


M.8 MATHEMATICS-II [JNTU-ANANTAPUR]
(b)

1. (b)

2. (e)

3. (d)

4. (a)

5. (c)

Look for the SIA GROU P LOGO on the TITLE COVER before you buy
MID-I: (Units - 1 & 2) M.9

ESSAY QUESTIONS WITH KEY

Q1. Prove that,

a
(a) L{sin at} = ,s>a
s + a2
2

s
(b) L{cos at} = , s > a. (Refer Unit-I / Q15)
s + a2
2

Q2. Find L–1[(2s + 3)/(s3 – 6s2 + 11s – 6)]. (Refer Unit-I / Q20

Q3. Prove that,

b
(i) L[eat sinh bt] =
(s – a)2 – b2

s–a
(ii) L[eat cosh bt] = . (Refer Unit-I / Q26)
(s – a)2 – b2

Q4. Show that L{fn(t)} = sn f(s) – sn – 1f(0) – sn–2f' (0)..... fn– 1(0), where L{f (t)} = f (s). (Refer Unit-I / Q29)

Q5. Find the Laplace transform of periodic function f(t) with period T,

4Et T 4Et T
where f(t) = – E, 0 ≤ t ≤ = 3E – , ≤ t ≤ T. (Refer Unit-I / Q47)
T 2 T 2

Q6. Using Laplace transform, solve (D2 + 4D + 5)y = 5, given that y(0) = 0, y"(0 ) = 0. (Refer Unit-I / Q54)


1
Q7. Obtain the Fourier series for the function, f(x) = x + x in [–π, π], deduce the value of
2
∑n
n=1
2
. (Refer Unit-II / Q12)

⎧ -1
⎪⎪ 2 ( π + x) for - π ≤ x ≤ 0
Q8. Find the Fourier series in [–π, π] for the function f(x) = ⎨ . (Refer Unit-II / Q16)
⎪ 1 ( π - x) for 0 ≤ x ≤ π
⎪⎩ 2

Q9. Obtain the Fourier series for the function f(x) = |cos x| in (–π, π). (Refer Unit-II / Q19)

Q10. Find the half-range sine series of f(x) = (x – 1)2 in the interval (0, 1). (Refer Unit-II / Q28)

SPECTRUM ALL-IN-ONE JOURNAL FOR ENGINEERING STUDENTS SIA GROUP


M.10 MATHEMATICS-II [JNTU-ANANTAPUR]

⎧1 1
⎪⎪ 4 - x , 0<x<
2
Q11. Find the half-range sine series for f(x) = ⎨ . (Refer Unit-II / Q29)
⎪x - 3 , 1
< x <1
⎪⎩ 4 2

l
⎡1 ∞ ⎤
Q12. Prove that ∫ ⎢⎣ 2

[f(x)]2 dx = l ⎢ a 02 +
n=1
(anna + bbn )⎥ provided the Fourier series for
⎥⎦
–l

f(x) converges uniformly in (– l, l). (Refer Unit-II / Q37)

Look for the SIA GROU P LOGO on the TITLE COVER before you buy
MID-II: (Units - 3, 4 & 5) M.11

MID - II
(UNIT S-3, 4 & 5)

OBJECTIVE TYPE &


ESSAY QUESTIONS WITH KEY

SPECTRUM ALL-IN-ONE JOURNAL FOR ENGINEERING STUDENTS SIA GROUP


M.12 MATHEMATICS-II [JNTU-ANANTAPUR]

MID-II (Objective Type & Essay Questions with Key)


Objective Type

I. Fill in the Blanks

1. The Fourier sine integral representation of a function f(x) is _____ .

2. Fourier transform is a _____ operation.

1
3. The Fourier sine transform of is _____.
x

4. Fourier cosine transform of f(t) is _____.

5. If F{f(x)} = F(s), then F{f(x – a)} = _____ .

6. A partial differential equation is said to be of _____ order if number of arbitrary constants equals to the number of
independent variables.

7. Elimination of two arbitrary functions gives a partial differential equation of order _____.

8. A partial differential equation is said to be _____ if the dependent variable Z and its derivatives are of degree 1.

9. The general form of a second-order P.D.E in the function ‘u’ of the two independent variables x, y is _____.

10. The general solution of two-dimensional heat equation, u(x, y, t) = _____.

∂ 2u
11. Two-dimensional wave equation is of the form, 2 = _____.
∂t

12. In a two-dimensional wave equation, C2 is a ____ constant.

13. In a vibrating rectangular membrane, change in time changes the _____ periodically.

14. The function Unm(x, y, t) in a rectangular membrane is known as _____ harmonic of the rectangular drum.

15. Z-transform is defined as _____.

16. The Z-transform of δ(n) is, _____.

17. Initial value theorem states that _____ .

18. If x(n) ⇔ x(z) the z-transform of x(n – n0) is, _____.

19. The ROC cannot contain any _____.

20. The region of convergence of the z-transform of a unit step function is, _____.

Look for the SIA GROU P LOGO on the TITLE COVER before you buy
MID-II: (Units - 3, 4 & 5) M.13

II. Multiple Choice

1. If Fourier transform of f(x) is F(s) then the inversion formula is _________. [ ]

∞ ∞
1
∫ F(s) e
−isx
f(x) =
∫ F(s) e
− isx dx
(a) f(x) = 2 π . dx (b) 2π
−∞ −∞
∞ ∞
1
∫ F(s) e ∫ F(s) e
− isx − isx
(c) f(x) = π dx (d) f(x) = dx
π
−∞ −∞

2. The Fourier sine transform of f(x) in the interval (0, 1) is. [ ]

l l
nπ x nπ x
(a) ∫
− nπ l f(x) cos
l
dx (b) ∫
nπ l f(x) cos
l
dx
0 0
l l
− nπ nπ x nπ nπ x
(c)
l ∫
f(x) cos
l
. dx (d)
l ∫
f(x) cos
l
dx
0 0

3. If Fourier transform of f(x) = f(s) then Fourier cosine transform of e–x is _________. [ ]

⎛s⎞ ⎛s⎞
(a) − 2 F⎜ ⎟ (b) 2 F⎜ ⎟
⎝2⎠ ⎝2⎠

1 ⎛s⎞ 1 ⎛s⎞
(c) − F⎜ ⎟ (d) F⎜ ⎟
2 ⎝2⎠ 2 ⎝2⎠

4. The Fourier cosine transform of e–x is _________. [ ]

(a) s2 + 1 (b) s2 – 1

1 1
(c) (d)
s +1
2
s −1
2

5. The period of a constant function is _____. [ ]

(a) Defined (b) Not defined

(c) Both (d) None

6. If F(λ) is the Fourier transform of f(x), then the Fourier transform of F(ax) is _________ . [ ]

⎡λ ⎤ 1 ⎡λ⎤
aF ⎢ ⎥ F
a ⎢⎣ a ⎥⎦
(a) (b)
⎣a ⎦

⎡λ⎤
(c) F⎢ ⎥ (d) F[λ]
⎣a ⎦

SPECTRUM ALL-IN-ONE JOURNAL FOR ENGINEERING STUDENTS SIA GROUP


M.14 MATHEMATICS-II [JNTU-ANANTAPUR]

7. The inverse finite Fourier sine transform of Fs(n) is F(x) = _________ . [ ]

∞ ∞

∑ F (n) sin
2 nπ x

2 nπ x
(a) Fs (n) cos (b) s
π n =1
n π n =1
n
∞ ∞
∑ F (n) sin
π nπ x
∑ F (n) cos
π nπ x
(c) s (d) s
2 n =1
n 2 n
n =1

8. Which among the following denotes a partial differential equation? [ ]

⎛ ∂z ∂z ∂ 2 z ⎞ ⎛ ∂z ∂z ∂ 2 z ⎞
(a) F⎜⎜ x , y, z, , , 2 ...⎟⎟ = 1 (b) F⎜⎜ , , 2 ...⎟⎟ = 0
⎝ ∂x ∂y ∂x ⎠ ⎝ ∂x ∂y ∂x ⎠
⎛ ∂z ∂z ∂ 2 z ⎞
(c) F(x, y, z) = 1 (d) F⎜⎜ x, y, z, , , 2 ...⎟⎟ = 0
⎝ ∂x ∂y ∂x ⎠

9. A partial differential equation is said to be of higher order if, [ ]

(a) Number of arbitrary constants = Number of independent variables

(b) Number of arbitrary constants > Number of independent variables

(c) Number of arbitrary constants < Number of independent variables

(d) Number of arbitrary constants = 0

10. Obtain partial differential equation by eliminating arbitrary constants from z = ax2 + by2. [ ]

(a) z = x zx + y zy (b) z=x+y

(c) 2z = x zx + y zy (d) 2z = x + y

11. What is the order of the P.D.E for z = a(x + y) + b(x – y) + abt + c? [ ]

(a) 1st order (b) 2nd order

(c) 3rd order (d) 4th order

12. A P.D.E is said to be _____, if each term contains the dependent variable or its derivative. [ ]

(a) Linear (b) Quasi-linear

(c) Non-linear (d) Homogeneous

Look for the SIA GROU P LOGO on the TITLE COVER before you buy
MID-II: (Units - 3, 4 & 5) M.15

13. A P.D.E is said to be parabolic if, [ ]

(a) B2 – 4AC < 0 (b) B2 – 4AC = 0

(c) B2 – 4AC > 0 (d) B2 – AC = 0

∂ 2u ∂ 2u
14. + = 0 is known as __________, [ ]
∂x 2 ∂y 2

(a) Two-dimensional Laplace equation (b) One-dimensional Laplace equation

(c) Laplace equation (d) Flowing heat equation

15. In a two-dimensional wave equation, C2 = _____. [ ]

T
(a) Tρ (b)
ρ

(c) T + ρ (d) T–ρ

16. The region of convergence is required for evaluating _________ from X(z). [ ]

(a) Inverse Z transform (b) Z transform

(c) Both (d) None

17. If X1(n) ⇔ x1(z) and x2(n) ⇔ x2(z) then z-transform of x1(n) * x2(n) is _________. [ ]

x1 (z)
(a) x1(z) – x2(z) (b)
x 2 (z)

(c) x1(z) + x2(z) (d) x1(z) x2(z)

z
18. The Z-tranform of a system is X(z) = , if the ROC is | z | < 0.2 then x(n) is _________. [ ]
z − 0.2

(a) (0.2)–n u(–n –1) (b) –(0.2)–n u(–n –1)

(c) (0.2)n u(–n –1) (d) –(0.2)n u(–n –1)

SPECTRUM ALL-IN-ONE JOURNAL FOR ENGINEERING STUDENTS SIA GROUP


M.16 MATHEMATICS-II [JNTU-ANANTAPUR]

19. The Z-transform of the function F(nt) = ant is _________. [ ]


z− a t
(a) z – at (b)
z
z
(c) (d) None
z− a t

20. The ROC of nan.u(n) is _________ for | z | > | a |. [ ]

z az
(a) (b)
(z − a) 2 z− a

az az
(c) (d)
z+ a (z − a) 2

III. Match the Following

(i)


xcosx– sinx x π –a
1. ∫
0
x 3
cos
2
dx [ ] (a)
2
e


xcosx– sinx π – ax
2. ∫
0
x 3
dx [ ] (b)
2
e

∞ 2
⎛ sinx ⎞ –3π
3. ∫0

⎝ 2x ⎠
⎟ dx [ ] (c)
16

s –π
4. ∫a 2
+s 2
sin s x ds [ ] (d)
4
0

xsinax π
5. ∫ 1+ x
0
2
dx [ ] (e)
8

(ii)

1. The solution of Uxy = 0 is [ ] (a) Z = F1(y) + x F2(y) + x2F3(y)

∂ 3z
2. The solution of = 0 is [ ] (b) xp + yq = z
∂x 3

∂ 2z ∂ 2z
3. The solution of
∂x 2
=
∂y 2
[ ] (c) u= ∫ f ( y)dy + φ(x)
x3y
4. The partial differential equation [ ] (d) u= + xf ( y) + φ( y)
6

from z = zx + by + c is,

5. Solution of Uxx = xy [ ] (e) z = F1(y + x) + F2(y – x)

Look for the SIA GROU P LOGO on the TITLE COVER before you buy
MID-II: (Units - 3, 4 & 5) M.17
(iii)

1. (an) [ ] (a) e1/z

⎛ 1 ⎞ z
2. ⎜ ⎟ [ ] (b)
⎝ n +1 ⎠ (z– a)

⎛1⎞ ⎛ z– 1 ⎞
3. ⎜ ⎟ [ ] (c) – z log⎜ ⎟
⎝ n! ⎠ ⎝ z ⎠

z sinθ
4. (cos x θ) [ ] (d) 2
z – 2 zcosθ + 1

z(z– cosθ)
5. (sin x θ) [ ] (e)
z – 2 zcosθ + 1
2

KEY

I. Fill in the Blanks

∞ ∞
1. ∫ ∫
2 π sinλi f(x) sinλ t dt .dλ
0 0

2. Linear

s2
3.
2

4. ∫
Fc(s) = f(t) cosst dt
0

5. eisa F(s)

6. First

7. 2

8. Linear

∂ 2u ∂ 2u ∂ 2u ⎛ ∂u ∂u ⎞
9. A(x, y). 2 + B(x, y). + C(x, y). 2 + f ⎜⎜ x, y , u, , ⎟⎟ = 0
∂x ∂x ∂y ∂y ⎝ ∂x ∂y ⎠

∞ ∞
⎛ mπ x ⎞ ⎛ nπ y ⎞
10. ∑∑ A
m =1 n =1
mn sin⎜
⎝ a
⎟. sin ⎜
⎠ ⎝ b ⎠
⎟ .exp(–λ2 C2t)
mn

SPECTRUM ALL-IN-ONE JOURNAL FOR ENGINEERING STUDENTS SIA GROUP


M.18 MATHEMATICS-II [JNTU-ANANTAPUR]

⎛ ∂ 2u ∂ 2u ⎞
C2 ⎜⎜ + ⎟
∂y 2 ⎟⎠
11.
⎝ ∂x
2

12. Positive

13. Amplitude

14. (n, m)th

α
15. X(z) = ∑ X(n) z
n =0
–n

16. 1

17. Lim X(z) = x(0)


z– ∞

18. Z–n0. x(z)

19. Poles

20. |z|>1

II. Multiple Choice

1. (b)

2. (c)

3. (d)

4. (c)

5. (b)

6. (b)

7. (b)

8. (d)

9. (b)

10. (c)

11. (a)

12. (d)

13. (b)

14. (a)

15. (b)

Look for the SIA GROU P LOGO on the TITLE COVER before you buy
MID-II: (Units - 3, 4 & 5) M.19
16. (a)

17. (d)

18. (d)

19. (c)

20. (d)

III. Match the Following

(i)
1. (c)

2. (b)

3. (e)

4. (d)

5. (a)

(ii)

1. (c)

2. (a)

3. (e)

4. (b)

5. (d)

(iii)

1. (b)

2. (c)

3. (a)

4. (d)

5. (e)

SPECTRUM ALL-IN-ONE JOURNAL FOR ENGINEERING STUDENTS SIA GROUP


M.20 MATHEMATICS-II [JNTU-ANANTAPUR]

ESSAY QUESTIONS WITH KEY

Q1. ∫
Solve the integral equation f(x) cosαx dx= e–αx. (Refer Unit-III / Q20)
0


cos xt

π –|x|
Q2. Find the Fourier transforms of f(x) = e–|x| and deduce that dt = e .
0
1+ t 2 2

4s
Hence show that F(xe–|x|) = i . (Refer Unit-III / Q23)
(1+ s 2 )2

2
–x
Q3. Find the Fourier cosine transform of e . (Refer Unit-III / Q25)

1
Q4. Find the fourier sine transform of . (Refer Unit-III / Q26)
x(x + a 2 )
2

π x2
Q5. Find the finite cosine transform of, f(x) = – x+ , 0 < x < π. (Refer Unit-III / Q35)
3 2π

Q6. Find the finite Fourier sine and cosine transform of f(x) = x (π – x) in 0 < x < π. (Refer Unit-III / Q39)

Q7. Obtain the partial differential equation by eliminating the arbitrary constants

(a) z = (x – a)2 + (y – b)2 + 1

2
(b) z = ae – b t
cos bx. (Refer Unit-IV / Q12)

Q8. Form the PDE by eliminating ‘f’ from f(x + y + z, x2 + y2 + z2) = 0. (Refer Unit-IV / Q24)

Q9. Solve by separation of variables 3ux + 2uy = 0 with u(x, 0) = 4e– x. (Refer Unit-IV / Q29)

Q10. A tightly stretched string with fixed end points x = 0 and x = 1 is initially at rest in
its equilibrium position. If itisset on vibrating by giving each of its points a velocity
λx(1–x), find the displacement of the string at any distance x from end at any time ‘t’. (Refer Unit-IV / Q35)

Q11. Find the deflection u(x, y, t) of the square membrane with a = b = 1 and c = 1,
if the initial velocity is zero and the initial deflection is f(x, y) = A sin πx sin 2πy. (Refer Unit-IV / Q37)

Q12. A bar of length L is laterally insulated with its ends A and B kept at 0° and 100°
respectively until steady state condition is reached. The temperature at A is raised to 30°
and that at B is reduced to 80° simultaneously. Find the temperature in the rod at a later time.

(Refer Unit-IV / Q46)

Look for the SIA GROU P LOGO on the TITLE COVER before you buy
MID-II: (Units - 3, 4 & 5) M.21

3z 2 + z
Q13. Find the inverse Z-transform of . (Refer Unit-V / Q16)
(5z – 1)(5z – 2)

⎡ 1 ⎤
Q14. Find Z–1 ⎢ 3⎥
when | z | > 5. Determine the region of convergence. (Refer Unit-V / Q17)
⎣ (z – 5) ⎦

Q15. Find the linearity property of Z-transforms,

⎛ nπ π ⎞
(i) cos ⎜ + ⎟
⎝ 2 4⎠

⎛ nπ ⎞
(ii) cosh ⎜ + θ⎟ . (Refer Unit-V / Q24)
⎝ 2 ⎠

2 z 2 + 4 z+12
Q16. If Z(un) = find u2. (Refer Unit-V / Q30)
(z-1)4

⎪⎧ z2 ⎪⎫
Q17. Evaluate Z–1 ⎨ ⎬ using convolution theorem. (Refer Unit-V / Q34)
⎪⎩ (z – 1)(z – 3) ⎪⎭

Q18. Solve the difference equation, using Z-transforms un +2 – un = 2n where u0 = 0, u1 = 1. (Refer Unit-V / Q43)

SPECTRUM ALL-IN-ONE JOURNAL FOR ENGINEERING STUDENTS SIA GROUP


May/June-2017 (R15) Question Paper with Solutions QP.1
Code No. : 15A54201/R15
I B.Tech II Semester Regular Examinations R15
May / June - 2017 Solutions
MATHEMATICS-ii
(Common to All)
Time: 3 Hours Max. Marks: 70

PART-A
(Compulsory Questions)
1. Answer the following: (10 × 02 = 20) Marks
(a) Find L[t2.et.cos4t]. (Unit-I)
(b) Define unit step function Laplace transform. (Unit-I)
(c) If f(x) = x4 in (–1, 1) then find the Fourier coefficient of bn. (Unit-II)
(d) What is Fourier even function (– π, π)? (Unit-II)
(e) Write the Fourier sine transform of f(t). (Unit-III)
(f) Find the value of Z(an cosnt). (Unit-V)
(g) Find the general solution of uxx = xy. (Unit-IV)
(h) Find the Z-transform of the sequence {x(n)} where x(n) is n.2n. (Unit-V)
JK 1 NO
(i) Find z–1 KK OO . (Unit-V)
L z – 3P
(j) What do you mean by steady state and transient state? (Unit-IV)

PART-B

(Answer all five units, 5 × 10 = 50 Marks)

Unit-I
JK 1 NO
2. (a) Apply convolution theorem for L–1 KKK 3 2 OO . (Unit-I, Topic No. 1.8)
s ^s + 1h O
JK t N L P
KK –1 sint OOO
(b) Evaluate L Ke
KK t
#dtO . (Unit-I, Topic No. 1.10)
OO
0
L P OR

d2 x JK π ON
3. Solve K O
2 + 9x = cos 2t, if x(0) = 1, x K 2 O = – 1. (Unit-I, Topic No. 1.11)
dt L P
Unit-II
1 1 1 1 π
4. Find the Fourier series expansion of f(x) = 2x – x2 in (0, 3) and hence deduce that 2 – 2 + 2 – 2 + ...... 3 = 12 .
1 2 3 4
(Unit-II, Topic No. 2.3)

OR
Z] l
]] kx, 0# x #
]] 2 1 1 1
5. Obtain half range cosine series for f(x) = [] . Deduce the sum of the series 2 – 2 + 2 + ..... 3 .
]] l 1 3 5
]] k ]l – xg , # x # l
2
\
(Unit-II, Topic No. 2.4)

SPECTRUM ALL-IN-ONE JOURNAL FOR ENGINEERING sTUDENTs SIA Group


QP.2 MATHEMATICS-II [JNTU-anantapur]
Unit-III

6. (a) Find the Fourier sine transform of e–|x|. (Unit-III, Topic No. 3.2)

(b) Write the conditions of Parseval's identity for Fourier transforms. (Unit-III, Topic No. 3.2)

OR
2
7. Verify convolution theorem for f(x) = g(x) = e –x . (Unit-III, Topic No. 3.2)

Unit-IV
JK xy NO
8. (a) Form the partial differential equation z = f KK OO by eliminating the arbitrary function. (Unit-IV, Topic No. 4.1)
L 2 P
2u 2u
(b) Use the method of separation of variables, solve =4 where u(x, 0) = 8e–3y. (Unit-IV, Topic No. 4.2)
2x 2y
OR
2
2 u 2 u 2 JK n π x NO
9. Solve the Laplace equation 2 +
K
2 = 0 subject to the conditions u(0, y) = u(i, y) = u(x, 0) = 0 and u(x, a) = sin K l O
O
2x 2y L P
(Unit-IV, Topic No. 4.4)
Unit-V

10. (a) Find the z-transformation of sinnθ. (Unit-V, Topic No. 5.1)
2z 2 + 5z + 14
(b) If U(z) = , evaluate U2, U3 using initial value theorem. (Unit-V, Topic No. 5.4)
]z – 1g4
OR

11. Solve the differential equation un + 2 – 2un + 1 + un = 3n + 5 using z-transforms. (Unit-V, Topic No. 5.6)

L o o k f o r t h e SIA Group logo o n t h e title cover before you buy


May/June-2017 (R15) Question Paper with Solutions QP.3

Solutions to May/June-2017, (R15), QP


PART-A
(10 × 2 = 20 Marks)
Q1. (a) Find L[t2.et.cos4t].
Answer : May/June-17, R(15), Q1(a)
Given function is,
t2 et cos 4t
d2
L[t2 et cos 4t] = ]– 1g2 . 7L 6et cos 4t@A
ds 2
d 2 SR s – 1 WVW
= 1. 2 SSS W
ds ]s – 1g2 + 4 2 W
T XV
d 2 SRS s –1 WW
= 2 S W
ds s + 1 – 2s + 16 W
S 2

R T X
d SS d JKK s – 1 NOVWW
= S OW
ds SS ds K s 2 – 2s + 17 OWW
RTS L PX VW
S d d 2
2
d SS ^ s – 2s + 17h ]s – 1g – ]s – 1g ^ s – 2s + 17h WWW
= ds ds
ds SSS WW
^ s 2 – 2s + 17h2 W
RTS 2 VW X
d S ^ s – 2s + 17h ]1g – ]s – 1g]2s – 2g W
= SS W
WW
ds S ^ s 2 – 2s + 17h2
RTS 2 V X
d S s – 2s + 17 – ^2s 2 – 2s – 2s + 2h WW
= SS W
WW
ds S ^ s 2 – 2s + 17h2
RT VX
d SSS s 2 – 2s + 17 – 2s 2 + 2s + 2s – 2 WWW
=
ds SS ^ s 2 – 2s + 17h2
W
W
RST 2 VW X
d SS – s + 2s + 15 WW
=
ds SS ^ s 2 – 2s + 17h2 WW
T X
2 d d
^ s – 2s + 17h . ^ – s 2 + 2s 15h – ^ – s 2 + 2s + 15h . ^ s 2 – 2s + 17h2
2

= ds ds
2
_^ s 2 – 2s + 17h2i
^ s 2 – 2s + 17h2 . ]– 2s + 2g – ^ – s 2 + 2s + 15h 2 ^ s 2 – 2s + 17h ]2s – 2g
=
^ s 2 – 2s + 17h4
2
^ s 2 – 2s + 17h 2 ]– s + 1g – 2 ^ s 2 – 2s + 17h ^ – s 2 + 2s + 15h ]2s – 2g
=
^ s 2 – 2s + 17h4
2 ^ s 2 – 2s + 17h ^^ s 2 – 2s + 17h ]– s + 1g – ^ – s 2 + 2s + 15h ]2s – 2gh
=
^ s 2 – 2s + 17h4
2 ^^ s 2 – 2s + 17h ]– s + 1g – ^ – s 2 + 2s + 15h ]2s – 2gh
=
^ s 2 – 2s + 17h3
2 ^^ – s3 + 3s 2 – 19s + 17h – ^ – 2s3 + 6s 2 + 26s – 30hh
=
^ s 2 – 2s + 17h3
2 ^ – s3 + 3s 2 – 19s + 17 + 2s3 – 6s 2 – 26s + 30h
=
^ s 2 – 2s + 17h3
2 ^ s3 – 3s 2 – 45s + 47h
=
^ s 2 – 2s + 17h3
2 ^ s3 – 3s 2 – 45s + 47h
\ L[t2 et cos 4t] = .
^ s 2 – 2s + 17h3

SPECTRUM ALL-IN-ONE JOURNAL FOR ENGINEERING sTUDENTs SIA Group


QP.4 MATHEMATICS-II [JNTU-anantapur]
(b) Define unit step function Laplace transform. 3 J –1hn nN
/ KKK ^e a z + ^e –it a z –1h OO
it
= O
Answer : May/June-17, R(15), Q1(b) n=0L 2 P
RS 3 3 VW
For answer refer Unit-I, Q33. =
1
2
/
SS
SS ^ it
e az –1hn
+ / ^e a z WW
–it –1hnW
W
n=0 n=0
(c) If f(x) = x in (–1, 1)then find the Fourier
4
T X
coefficient of bn. 1 RSS 1 1 VW
W
= S + W
2 S1 – eit a z –1 1 – e –it a z –1 W
Answer : May/June-17, R(15), Q1(c) T X
R WVW
1 SS z z
Given function is, = S + W
2 SS ^ z – aeit h ^ z – ae – it h WW
T X
f (x) = x4 R V
1 SS z ^ z – ae –it h + z ^ z – aeit h WW
= SS W
As f (x) is even function, it doesn't contain sine terms. 2 S ^ z – eit h ^ z – ae –it h WW
T X
R V
1 1 SS z 2 – zae –it + z 2 – zaeit WW
= S W
i.e., # f ] xg .sin nπx dx = 0 2 SS z 2 – aze –it – azeit + 1 WW
T X
R V
–1 1 SS 2z 2 – az ^e –it + eit h WW
= S W
\ Coefficient of bn = 0. 2 SS z 2 – az ^eit + e –it h + 1WW
T X
(d) What is Fourier even function (– π, π)? R V
1 SS 2z 2 – a2z cos t WW
= S W
Answer : May/June-17, R(15), Q1(d) 2 SS z 2 – 2az cos t + 1WW
T X
If a function f (x) = f (– x), then f (x) is an even function. 1 JKK z 2 – az cos t NOO
= .2
2 KK z 2 – 2az cos t + 1 OO
The Fourier series of even function in (– π, π) is L P
expressed as, z 2 – az cos t
=
3
z 2 – 2az cos t + 1

a
f (x) = 0 + /
a cos nx
2 n=1 n
z 2 – az cos t
\ Z[ancosnt] = 2 .
π z – 2az cos t + 1
1
Where a0 =
π
# f ] xgdx (g) Find the general solution of uxx = xy.
–π
π Answer : May/June-17, R(15), Q1(g)
1
an =
π
# f ] xg cos nx dx . Given equation is,
–π
uxx = xy
(e) Write the Fourier sine transform of f(t).
22 u
Answer : May/June-17, R(15), Q1(e) Þ = xy
2x 2
For answer refer Unit-III, Q7, Topic: Fourier sine
Integrating with respect to x (keeping y constant) on
Transform.
both sides,
(f) Find the value of Z(an cosnt). 2

Answer : May/June-17, R(15), Q1(f)


# 22xu dx = # xy dx
2

2u
Given that, Þ
2x
= y # xdx
x(n) = an cosnt 2u x2
Þ = y. + f ^ yh
The Z-transform of the signal x(n) is given as, 2x 2
3 Integrating with respect to x (keeping y constant) on
Z[x(n)] = X(Z) = / x]ng.z –n
both sides.
n = –3
3
Z[ancosnt] = / a cos nt.z n –n
# 22ux dx = 2y # x dx + # f ^ yhdx
2
n=0
3 R V
S int + e – int WW n –n y x3
= / SSS e 2
WW a z Þ u= . + f ^ yh
2 3
# 1dx
n=0T X
L o o k f o r t h e SIA Group logo o n t h e title cover before you buy
May/June-2017 (R15) Question Paper with Solutions QP.5
x3 y
Þ u= + f ^ yh] xg + φ ^ yh
6
x3 y
Þ u= + x f ^ yh + φ ^ yh
6
x3 y
\ The general solution is, u = + x f ^ yh + φ ^ yh
6
where f (y) and f(y) are arbitrary functions.

(h) Find the Z-transform of the sequence {x(n)} where x(n) is n.2n.
Answer : May/June-17, R(15), Q1(h)
For answer refer Unit-V, Q2(i).
JK 1 NO
(i) Find Z–1 KK OO .
Lz – 3P
Answer : May/June-17, R(15), Q1(i)
Given that,
SR 1 WVW
Z –1 SSS W
z – 3W
T X
RS 1 VW
As, Z –1 SSS WW = an – 1u(n – 1)
z – aW
T X
Here, a = 3
RS 1 VW
Z –1 SSS WW = 3n – 1u(n – 1)
z – 3W
T X
= 3n.3– 1u(n – 1)
3n
= u ]n – 1g
3
RS 1 VW n
\ Z –1 SSS WW = 3 u ]n – 1g .
z – 3W 3
T X
(j) What do you mean by steady state and transient state?
Answer : May/June-17, R(15), Q1(j)
Steady State
If the temperature is independent of time it is known as Steady State. In this state, heat flow is constant.
Transient State
If the temperature is dependent of time it is known as Transient State. In this state, heat flow varies with the time.

PART-B
(5 × 10 = 50 Marks)
JK 1 NO
Q2. (a) Apply convolution theorem for L–1 KKK 3 2 OO .
s ^s + 1h O
L P
Answer : May/June-17, R(15), Q2(a)
Given function is,
]Z] 1 b_b
L–1 ][ 3 2 `
] s ^ s + 1h bb
\ a
1 1 t2
Let, f (s) = 3 ; f (t) = L–1 ( 3 2 =
s s 2!
t2
Þ f (t) =
2

SPECTRUM ALL-IN-ONE JOURNAL FOR ENGINEERING sTUDENTs SIA Group


QP.6 MATHEMATICS-II [JNTU-anantapur]
1 1
Let, g (s) = 2 ; g (t) = L–1 ( 2 2 = sint
s +1 s +1
Þ g(t) = sint
By convolution theorem,
]Z] 1 b_b t

L–1 ][ 3 2
] s ^ s + 1h bb
`= # f ]ug g ]t – ug du
\ a 0
t
u2
= # 2
. sin ]t – ug du
0
t
1
=
2
# u . sin ]t – ugdu
2

1 RSS 2 d 2 VWt
=
2
SSu #
sin ]t – ug du – #
du
#
^u h . sin ]t – ug du duWW
W0
T X
1 SRS 2 JK – cos ]t – ug NO JK – cos ]t – ug NO WVWt
= Su K
2 S KL –1
OO – 2u KK # –1
OO duW
W
T P L P X0
1 RSS 2 VWt
=
2 S
S #
u cos ]t – ug – 2 u cos ]t – ug duWWW
0
T X
R
1 SS 2 R
S d VWVWt
=
2 #
SSu cos ]t – ug – 2 SSSu cos ]t – ug du – # du
#
]ug . cos ]t – ug du duWWWWWW
T T XX0
1 RSS 2 JK sin ]t – ug NO J sin ]t – ug NO VWWt
=
2
SSu cos ]t – ug – 2 (u KK
–1 P
#
OO – 1. KKK
– 1 OP W0
O du 2W
T L L X
1 RSS 2 RS VWVWt
=
2 SS
#
u cos ]t – ug – 2 SSS– u sin ]t – ug + sin ]t – ug duWWWWWW
0
T T XX
R R V V
KJ – cos ]t – ug ONOWWWW
t
1 SS 2 S
= Su cos ]t – ug – 2 SSS– u sin ]t – ug + KK OWWWW
2S L –1 PXX0
T T
1 2
7u cos ]t – ug – 2 6– u sin ]t – ug + cos ]t – ug@A0
t
=
2

6u cos ]t – ug + 2u sin ]t – ug – 2 cos ]t – ug@t0


1 2
=
2

= 76t 2 cos ]t – t g + 2t sin ]t – t g – 2 cos ]t – t g@ – 60 2 cos ]t – 0g + 2 ]0g sin ]t – 0g – 2 cos ]t – 0g@A


1
2

76t cos ]0g + 2t sin ]0g – 2 cos ]0g@ – 50 + 0 – 2 cos t?A


1 2
=
2

6t ]1g + 2t ]0g – 2 ]1g – 0 – 0 + 2 cos t@


1 2
=
2

6t + 0 – 2 + 2 cos t@
1 2
=
2

6t – 2 + 2 cos t@
1 2
=
2
t2
= – 1 + cos t
2
Z] 1 _b 2
] b t
\ L–1 [] 3 2 `b = – 1 + cos t .
] s ^ s + 1h b 2
\ a

L o o k f o r t h e SIA Group logo o n t h e title cover before you buy


May/June-2017 (R15) Question Paper with Solutions QP.7
JK t N s
KK –1 sint OOO L{x"} + 9L{x} =
(b) Evaluate L KKe
K 0 t
dtO .
OO
# s + 22
2
s
L P s2.L{x} – s.x(0) – x'(0) + 9L{x} =
Answer : May/June-17, R(15), Q2(b) s2 + 4
Substituting x(0) = 1, x'(0) = k(say) in above equation,
Given function is,
s
t
s2.L{x} – s(1) – k + 9L{x} =
e –1
# sin t
t dt
2
s +4
s
0 L{x} (s2 + 9) – s – k = 2
t t
s +4
L >e –1 t dtH = e .L
s
# sin t –1
# sint t dt L{x} (s2 + 9) =
s2 + 4
+s+k
0 0
SRS t WV s + s ^ s 2 + 4h + k ^ s 2 + 4h
SSe –1 sin t dtWWW JK 1 NO
LS
SS
# t W
WW
= e –1 KK fr ] s gOO
s
... (1) L{x} (s2 + 9) =
^ s 2 + 4h
0 L P
T X
RS SRS t WVW VW s + s3 + 4s + ks 2 + 4k
SS WW L{x} =
SSa L SSS
SS SS
# W 1
f ]ug duWW = f ] s gWW
WW s WW
^ s 2 + 4h^ s 2 + 9h
0
T T X X s3 + ks 2 + 5s + 4k
Consider sin t, L{x} = ... (1)
^ s 2 + 4h^ s 2 + 9h
1
L{sin t} = 2
Applying partial fractions to equation (1),
s +1
Then, s3 + ks 2 + 5s + 4k As + B Cs + D
3 2 2 = 2 + 2 ... (2)
1 ^ s + 4h^ s + 9h s +4 s +9
L (
sin t
t
2 = # 2
s +1
s s3 + ks 2 + 5s + 4k ] As + Bg ^ s 2 + 9h + ]Cs + Dg^ s 2 + 4h
2 2 =
^ s 2 + 4h^ s 2 + 9h
f ] s g = 6tan –1 ] s g@s
3
^ s + 4h^ s + 9h
Þ
Þ f ] s g = tan–1(∞) – tan–1(s) s3 + ks 2 + 5s + 4k = As3 + 9As + Bs 2 + 9B + Cs3 + 4Cs + Ds 2 + 4D
π s3 + ks 2 + 5s + 4k = ] A + C g s3 + ]B + Dg s 2 + s ]9A + 4C g + 9B + 4D
Þ f ] s g = – tan –1s
2
Þ f ] s g = cot–1s ... (2) Comparing corresponding coefficients on both sides.

Substituting equation (2) in equation (1) A + C = 1 ... (3)


t B + D = k ... (4)
L >e t dtH = e KK s cot sOO
J
–1 K 1
N

–1
# sin t –1 O
9A + 4C = 5 ... (5)
0 L P
9B + 4D = 4k ... (6)
1
= cot –1s
se Solving equations (3) and (5),
t 1 4
L >e –1 t dtH = se cot s .
1
\ # sin t –1 A=
5
,C=
5
0 Solving equations (4) and (6),
OR B = 0, D = k
2
d x KKJ π NOO Substituting the values of A, B, C and D in equation (2),
Q3. Solve 2 + 9x = cos 2t, if x(0) = 1, x K 2 O = – 1.
dt L P
1 4
Answer : May/June-17, R(15), Q3 s+0 s+k
s3 + 5s 2 + 5s + 4k 5 5
2 = +
Given differential equation is, ^ s + 4h^ s 2 + 9h s2 + 4 s2 + 9
d2 x
2 + 9x = cos2t s3 + 5s 2 + 5s + 4k s 4s k
dt = + + ... (7)
^ s 2 + 4h^ s 2 + 9h 5 ^ s 2 + 4h 5 ^ s 2 + 9h s 2 + 9
Þ x" + 9x = cos2t
Substituting equation (7) in equation (1),
Applying laplace transform on both sides, s 4s k
Þ L{x} = 2 + 2 + 2
L{x" + 9x} = L{cos2t} 5 ^ s + 4h 5 ^ s + 9h s + 9
SPECTRUM ALL-IN-ONE JOURNAL FOR ENGINEERING sTUDENTs SIA Group
QP.8 MATHEMATICS-II [JNTU-anantapur]
Applying inverse laplace transform on both sides,
s 4s 5
L–1(L{x}) = L–1 ) 2 3 + L–1 ) 2 3 + L–1 ( 2 2
5 ^ s + 4h 5 ^ s + 9h s +9
1 –1 s 4 s 1
Þ x(t) = L ( 2 2+ ( 2 2 + kL–1 ( 2 2
5 s +4 5 s +9 s +9
1 4 k
Þ x(t) =cos 2t + cos 3t. + sin 3t ... (8)
5 5 9
π
Substituting t = in equation (8),
2
JK π NO 1 KJ 2 π NO 4 JK 3π NO k JK 3 π NO
x KK OO = cos KK OO + cos KK OO + sin KK OO
L2P 5 L 2 P 5 L 2 P 9 L 2 P
1 4 k
Þ –1= ]– 1g + ]0g + ]– 1g
5 5 9
–1 k
Þ –1= –
5 9
k –1
Þ = +1
9 5
k 4
Þ =
9 5
4 36
Þ k=×9 =
5 5
36
Substituting k = in equation (8),
5
1 4 36
x(t) = cos 2t + cos 3t + sin 3t
5 5 5×9
1 4 4
= cos 2t + cos 3t + sin 3t
5 5 5

= 5cos 2t + 4 cos 3t + 4 sin 3t?


1
5

x(t) = 5cos 2t + 4 cos 3t + 4 sin 3t? .


1
\
5
1 1 1 1 π
Q4. Find the Fourier series expansion of f(x) = 2x – x2 in (0, 3) and hence deduce that 2 – 2 + 2 – 2 + ...... 3 = 12
1 2 3 4
Answer : May/June-17, R(15), Q4
Note
1 1 1 1 π2 1 1 1 1 π
In equation 2 – 2 + 2 – 2 + ..... 3 = 12 is misprinted as 2 – 2 + 2 – 2 + ...... 3 = 12 .
1 2 3 4 1 2 3 4
Fourier Series
For answer refer Unit-II, Q27 (up to equation (6))
\ The Fourier series of f (x) = 2x – x2 is given as,
3 J N 3 J N
2x – x2 =
–9
π2
/ n1 cos KKK 2 n3πx OOO + π3 / 1n . sin KKK 2n3πx OOO
2
n=1 L P n=1 L P
Deduction
3
Substituting x =
in above equation,
2
KJ 3 ON KJ 3 ON –9 3 1 KKJ 2n π 3 ON 3 J N
2 3
2 KK OO – KK OO = 2
2 2 π n=1 n
/
2 cos K 3 ×
O+
2 OP π
/ 1n . sin KKK 23n π × 23 OOO
L P L P L n=1 L P

L o o k f o r t h e SIA Group logo o n t h e title cover before you buy


May/June-2017 (R15) Question Paper with Solutions QP.9
3 3
9 –9
3 –
4
= 2
π
/ n1 cos ]n πg + π3 / 1n sin ]nπg
2
n=1 n=1

3 3
12 – 9 9

4
= 2
π
/ –n 1 ]– 1g + π3 / 1n ]0g
2
n

n=1 n=1

3 π2 3 n
×
4 9
= / – ]n– 1g 2 +0
n=1

π2 – ]– 1g1 JKK ]– 1g2 ONO JKK – ]– 1g3 ONO KJK – ]– 1g4 ONO
= + KK – O+K O+K O + ....
12 12 L 2 2 OP KL 3 2 OP KL 4 2 OP
π2 1 1 1 1
= 2 – 2 + 2 – 2 + .....
12 1 2 3 4
1 1 1 1 π2
2 – 2 + 2 – 2 + ...... 3 = .
1 2 3 4 12
OR

Z] l
]]kx, 0# x #
]] 2
Q5. Obtain half range cosine series for f(x) = [] . Deduce the sum of the series
l
]]k ]l – xg, # x # l
]]
1 1 1 2
\
2 – 2 + 2 + ..... 3 .
1 3 5
Answer : May/June-17, R(15), Q5
Given function is,
Z] l
]] kx, 0# x #
]] 2
f(x) = ][
]] l
]] k ]l – xg, # x # l
2
\
Half-range cosine series is given as,
3
npx
f(x) = 20 +
a
/
an cos b l l ... (1)
n=1
l
2
Where, a0 = l # f (x) dx
0
RS l VW
SS 2 l WW
2 SS WW
=
lS
#
SS f ] xg dx + #
f ] xg dxWW
WW
SS0 l WW
S 2
TR XV
SS l WW
S 2 l W
2 SS kxdx + k (l – x) dxWW
# #
= lS S WW
SS0 l WW
S 2 W
RST l XV
WW
SS 2 l WW
S
=
2S
lS
#
SSk xdx + k ]l – xg dxWWW # WW
SS 0 l WW
S 2
TR l VW X
SS WW
S 2 l
2k SS W
=
l S
# #
SS xdx + ]l – xg dxWWW
WW
SS0 l WW
S 2
T X
SPECTRUM ALL-IN-ONE JOURNAL FOR ENGINEERING sTUDENTs SIA Group
QP.10 MATHEMATICS-II [JNTU-anantapur]
l
= l >; x E – < (l – x) F l H
2 2 2 l
2k
2 0 2 2
R V
2k 1 SS 2 2l l W W
= . SS^ x h0 – ^]l – xg2h l WW
l 2S 2
W
T X
R V
k SS l 2 J l N2W
= SS – 0 – ]l – l g2 + KKKl – OOO WWW
l S4 2P W
T L X
R
k SS l 2 V
l 2 WW
= SS – 0 – 0 + WW
l 4 4
T X
k 2l 2 kl
= l. 4 = 2
kl
\ a0 = 2
l
npx
2
an = l # f (x) cos b l l dx
0
RS l VW
SS 2 W
JK nπ x NO WWW
l
2 SS JK nπx NO
=
l SS
# S f ] xg cos KK #
OO dx + f ] xg cos KK
l OP WW
O dxW
SS0 L l P l L WW
S 2 W
T X
RS l VW
SS 2 l WW
2 SS kx cos npx dx + k (l – x) cos npx dxWW
# #
= l SS l l WW
SS0 l WW
S 2 W
T X
RS l VW
SS 2 l WW
2 SS JK n πx NO JK nπ x NO WW
=
l SS
#Sk x cos KK
l OP
#
O dx + k ]l – xg cos KK
l OP WW
O dxW
SS 0 L l L WW
S 2 W
T X
RS l VW
SS 2 l WW
2k SS J n πx ON J nπ x ON WW
=
l SS
# S x cos KK K
#
OO dx + ]l – xg cos KK K OO dxWW
SS0 L l P L l P WW
S
l WW
2
T X
RS V
2k SSRSS JK n π xNO JK nπ x NO VW 2l SR J π N J N VWl WWW
d d π
= # S
l SSSSx cos K l O dx –
K O # dx
# ] xg KK
l
#
OO dx dxWW + SS]l – xg cos KKK
W0 S
n
#l
x OO
O dx – dx
#]l – xg cos KKK
m
l
x OO W
O dx dxWW l WWW
ST L P L P X T L P L P X 2 WX
T
RS WV
2k SSRSS JK JK n π x NONO l JK nπx NO l VW 2l RS JK JKn π x NO l NO J JK n π xNO l ON VWl WW
= S K K OO #
l SSSSx Ksin K l OO . nπ – 1. sin K l O n π dxWW0 + SS
K O W S]l – xgKKsin KK # O . O – ]– 1gKKsin KK
l OP n π OP
K W
O . O dxW W
l OP nπ OP W l WWW
ST L L PP L P X T L L L L X 2X
T
RS l V
W
2k SSRSS xl J N JK nπx NO VWW 2 RSS l ]l – xg JK nπx NO l JK n π xNO WWVl WW
= SSS sin KKK nπx OOO – l # sin KK OO dxWW + SS sin KK # OO + nπ sin KK l OO dxWW l WWW
l SS n π
ST L l P nπ L l P X0 T nπ L l P L P X 2W
T X
RS l VW
2k SSRSS xl JK n π x NO l JK JK nπ x NO l NOVWW 2 RSS l ]l – xg JK nπx NO l J
K JK nπx NO l NOWWVl WW
= SS K O K K O OW S K O K K O OW W
l SSS n π sin K l O – nπ K – cos K l O . nπ OW0 + S nπ sin K l O + nπ K – cos K l O . nπ OW l WW
ST L P L L P PX T L P L L P PX 2 W
T X
RS l V
WW
2k SSSS xl SR JK n πx NO V R
JKnπ x NOWW 2 SS l ]l – xg JK nπx NO V l
JK nπx NOWW WW
l2 l2
= S K O K OW S K O K OW
l SSSS n π sin K l O + n 2 π 2 cos K l OWW0 + SS n π sin K l O – n 2π 2 cos K l OWW l WW
ST L P L PX T L P L PX 2 W
T X
L o o k f o r t h e SIA Group logo o n t h e title cover before you buy
May/June-2017 (R15) Question Paper with Solutions QP.11
RSSR 2 V
2k SS l
SSSS
JK nπl ON l2
O + 2 2 cos KKK
J nπ l ON 0.l KJ nπ ]0g OON l2 KJ n π]0g OONWWW
= sin KK O O–
O sin KK O – 2 2 cos KK O W
l SS 2n π π
TT L 2l P n π L 2l P n π L P n π L l PWX
RS JK l NO VWVW
SS K O WW
+ SS l ]l – l g KKJ nπl NOO l 2
KKJ n πl OON KL
l l –
2 OP KKJ n π l OON l 2 JK n πl NOWWWW
W
SS nπ sin K O – 2 2 cos K l O – n π sin K 2l O + 2 2 cos KK 2l OOWWWWW
L l P n π L P L P n π L PXX
T
R 2
S JK n π NOWVW
2k S l ]n πg l 2
]n πg l 2
l ]0 g l 2
l 2
KJ nπ ON l2
= SS sin + 2 2 cos – 0 – 2 2 cos ]0g + sin ]n πg – 2 2 cos ]n πg – sin KK OO + 2 2 cos KK OOWW
l S 2n π nπ 2 nπ L 2 P n π
TR 2
2 n π 2 n π n π L 2 PWX
V
2k SS 2l J n πN l 2
–l 2 W
= SS 2 2 cos KKK OOO – 2 2 ]1g + 0 2 2 cos n πWWW
l Sn π W
TR 2 L 2 P n π nπ
X
S JK nπ ON 2 V
W
2k S 2l l W
= S cos K O – 2 2 ]1 – cos n πgW
K O W
l SS n 2π 2 2 n π W
T 2 R L P V X
2k l SS JK n π NO n
WW
= . S2 cos KK OO – 1 – ]– 1g WW
l n 2π 2 S L 2 P
T X
2kl SS R JK n π ON VW
= 2 2 SS2 cos KK OO – 1 – ]– 1gnWWW

T L 2 P X
2kl RSS KJK n π ONO
VW
\ an = 2 2 SS2 cos K O – 1 – ]– 1gnWWW

T L 2 P X
Substituting the values of a0 and an in equation (1),
kl
3
2kl KJ J N N J N
f (x) = 2 + KK2 cos KKK n πOOO – 1 – ]– 1gnOOO cos KKKnπ x OOO
/
2 n=1 n π
2 2
L 2 P
L P L l P
kl 2kl 3 1 JK J N N J N
= + 2 KK2 cos KKK nπ OOO – 1 – ]– 1gnOOO cos KKK n π xOOO
/
4 π n=1 n L2
L 2 P P L l P
kl 2kl 3
KKJ N JK nπ x NO
\ f (x) =
4
+ 2
π
/ n1 2
nπ nO
K2 cos 2 – 1 – ]– 1g OO cos KK l OO ... (2)
n=1 L P L P
Deduction

Substituting x = 0 in equation (2)


3 J J N N J N
f (0) =
kl 2kl
4
+ 2
π
/ n1 KKK2 cos KKK n2πOOO – 1 – ]– 1g OOO cos KKK n πl]0g OOO
2
n

n=1 L L P P L P
kl 2kl 3
1 JK JK n π NO NO
0 =
4
+ 2
π
/ K K O ] gnO ] g
2 K2 cos K 2 O – 1 – – 1 O cos 0
n=1 n L L P P
J N
/ n1 KKK2 cos JKKK n2πNOOO – 1 – ]– 1g OOO
3
– kl 2kl n
Þ = 2
4 π L P
n=1
2
L P
– kl π 2 J 3 J N N SRS JK nπ NO WV
KK2 cos KKK n π OOO – 1– ]– 1gnOOO
1K
Þ
4
×
2kl
= 2 / SSa 2 cos KK OO – 1 – ]– 1gn = 0 if n is oddWWW
n = 2, 4, 6 n L L 2 P P T L 2 P X
2 R J N V R J N V R J N V
–π 1S K 2πO W 1S K 4π O W 1S K 6π O W
Þ = 2 SSS2 cos KK OO – 1 – ]– 1g2WWW + 2 SSS2 cos KK OO – 1 – ]– 1g4WWW + 2 SSS2 cos KK OO – 1 – ]– 1g6WWW
8 2 L P2 4 L P 2 6 L P2
T X T X T X
R V R V
1 SS KKJ 8 π ONO W 1 S J
K 10 π N
O W
+ 2 SS2 cos K O – 1 – ]– 1g8WWW + 2 SSS2 cos KK O – 1 – ]– 1g10WW + .....
8 L 2P 10 L 2 OP W
T X T X
– π2
= 2 62 cos ]πg – 1 – 1@ + 2 62 cos ]2 πg – 1 – 1@ + 2 62 cos ]3π g – 1 – 1@ + 2 62 cos ] 4 πg – 1 – 1@
1 1 1 1
Þ
8 2 4 6 8

62 cos ]5πg – 1 – 1@ + ...


1
+
10 2
SPECTRUM ALL-IN-ONE JOURNAL FOR ENGINEERING sTUDENTs SIA Group
QP.12 MATHEMATICS-II [JNTU-anantapur]
– π2 1 1 1 1 1
Þ = 2 62 ]– 1g – 1 – 1@ + 2 62 ]1g – 1 – 1@ + 2 62 ]– 1g – 1 – 1@ + 2 62 ]1g – 1 – 1@ + 2 62 ]– 1g – 1 – 1@ + .....
8 2 4 6 8 10
– π2
= 2 5– 4? + 2 50? + 2 5– 4? + 2 50? + 2 5– 4? + .....
1 1 1 1 1
Þ
8 2 4 6 8 10
– π2 1 1
Þ = –1+ 0 – + 0 – + .....
8 9 25
– π2 RS 1 1 1 VW
Þ = – SS 2 + 2 + 2 + .....WWW
8 S1 3 5
T X
2
π 1 1 1
Þ = 2 + 2 + 2 + .....
8 1 3 5
1 1 1 π2
\ + + + ..... = .
12 32 52 8
Q6. (a) Find the Fourier sine transform of e–|x|.
Answer : May/June-17, R(15), Q6(a)
Given function is,
f (x) = e–|x|
The fourier sine transform of f (x) is given as,
3
Fs[f(x)] = # f (x) . sin sx dx
0
3

= # e –x sin sx dx [ a |x| = x, x ≥ 0]
0
RS – x VW3 RS VW
S e e –ax
= SS 2
Ss +1
]– sin sx – s cos sxgWWW
W0
SS
SSa # e –ax sin bx dx =
a2 + b2
] – a sin bx – b cos bx g WW
WW
T X T X
SRS e– 0 WVW
= SS0 – 2 ^ – sin s ]0g – s cos s ]0ghWW
S s +1 W
T X
RS – 1 VW
= SS 2 ^ – 0 – s ]1ghWW
Ss + 1 W
T X
–1
= 2 ]– sg
s +1
s
= 2
s +1
s
\ Fourier sine transform of e–|x| is .
s2 + 1
(b) Write the conditions of Parseval's identity for Fourier transforms.
Answer : May/June-17, R(15), Q6(b)
The conditions of Parseval's Identity for Fourier Transforms are given as,
3 3
(i)
1
2p # F (s) . G (s) ds = # f (x) .g (x) dx
–3 –3
3 3
(ii)
1
2p # | F (s) | 2 ds = # | f (x) | 2 dx
–3 –3

Where F(s) and G(s) are Fourier transform of f(x) and g(x) respectively,

L o o k f o r t h e SIA Group logo o n t h e title cover before you buy


May/June-2017 (R15) Question Paper with Solutions QP.13
Parseval's identity for Fourier sine and cosine transforms
3 3
(i)
2
p # Fc (s) .Gc (s) ds = # f (x) .g (x) dx
0 0
3 3
(ii)
2
p # Fs (s) .Gs (s) ds = # f (x) g (x) dx
0 0
3 3
(iii)
2
p # | Fc (s) | 2 ds = # | f (x) | 2 dx
0 0
3 3
(iv)
2
p # | Fs (s) | 2 ds = # | f (x) | 2 dx
0 0

OR
–x 2
Q7. Verify convolution theorem for f(x) = g(x) = e .
Answer : May/June-17, R(15), Q7
Given functions are,
2
f (x) = g(x) = e –x
From the definition of convolution theorem,
F[f (x)*g(x)] = F(s).G(s)
Consider,
3 3

F[f(x)*g(x)] = # # f ]ug .g ] x – ug du eisx dx


–3 –3
3 3

= # # 2 2
e –u .e –]x – ug du eisx dx
–3 –3
3 3

= # # 2 2
e –u .e –] x – ug .eisu e –isu eisx dx du
–3 –3
3 3

= # # 2 2
e –u .e –]x – ug du.eisu .eis]x – ug dx
–3 –3
3 3

= # 2
e –u .eisu du # 2
e –]x – ug .eis]x – ug dx
–3 –3
Let, x – u = t
dx = dt
U.L : x = ∞, t = ∞
L.L : x = – ∞, t = – ∞
3 3

= # e –u 2 isu
.e du. # 2
e –t .eist dt
–3 –3
3 3

= # 2
e –t eist dt # 2
e –t eist dt (Replacing u by t)
–3 –3
RS 3 VW2
SS W
= SS
SS
–t2 ist W
e e dtWW
WW
#
–3
T X
SPECTRUM ALL-IN-ONE JOURNAL FOR ENGINEERING sTUDENTs SIA Group
QP.14 MATHEMATICS-II [JNTU-anantapur]
RS 3 VW2 3 KJJK N
is NO2 JK is NO2OO
SS W –KKKK x – O –K O O
= SS
SS
# e –_t 2 – ist i W
dtWW
WW
= # e
K
LL 2 OP KL 2 OP O
P dx
–3 –3
T X
RS 3 KJK 2 J is ON2 KJ is ON2ONO V 2 is NO2 JK is NO2
–KKt – ist + KK OO – KK OO OO W
K 3 JK
SS WW –KK x – O + KK OO
= #
SS
SS
e L 2
L P L P P dtW 2
WW = # e L 2 OP L2P dx
–3 W –3
T X
RS 3 JK is NO2 JK si NO2 VW2 3 JK is NO2 JK is NO2
SS W KK OO
# –KKt – OO + KK OO –KK x – O
= SS
SS
e L 2 P L 2 P dtWWW
WW
= # e L 2 OP
.e L 2 P dx

–3 –3
T X
SRS 3 WV2 KJK is ONO2 3 JK is NO 2
– KK x – 2 OO
KJ is ON J si N2 W
#
2 K O
SS –KKt – OO KKK OOO WW = eL 2 P e L P dx
= # SS
SS
e L 2 P .eL 2 P dtWW
WW is –3
T
–3
X Let, x – = q
2
RS J N2 3 J N2 V W 2
SS KK OO K is O K is O WW dx = dq
= #
SSeL 2 P
SS
–KKt – OO
e L 2 P dtWW
WW
T
–3
X U.L : x = ∞, q = ∞
RS 2 3 KJ is ON 2 VW2 L.L : x = – ∞, q = – ∞
SS – s WW
= #SSe 4 .
SS
e L
–KKt – OO
2 P dtW
WW
W s2 3

#
–3 – 2
si T X = e 4 e – q dq
Let t – =y
2 –3
dt = dy s2 3

U.L : t = ∞, y = ∞ = e

4 .2
2
e – q dq #
0
L.L : t = – ∞, y = – ∞
RS s2 3 VW2 s2
2^ πh
SS – W = e

= SSe 4
SS
e dyWWW
– y 2

WW
# 4 .
2
–3
T X s2

RS s2 3 VW2 = πe 4
SS – W
= SSe .2 e dyWW
SS
4 – y2 W
#
WW s2
0 –
T X \ F(s) = πe 4
SRS – s2 KJ π NOWVW2
= SS2e 4 . KK OOWW Similarly,
S 2 W
T L PX s2
R 2V –
s2 ^ 2 G(s) = G SSe –x WW = π .e 4
–2 πh T X
= 4.e 4 .
4 Consider,
s2
– s2 s2
= πe 2
F(s).G(s) = π .e

4 . π .e

4
s2
– 2s 2
\ F[f (x) * g(x)] = πe 2 ... (1) –
= πe 4

Consider,
3 – s2
R 2V
F(s) = F SSe –x WW =
T X
# e –x2
.e dxisx = πe 2

–3 – s2
3 \ F(s).G(s) = πe 2 ... (2)
= # e –_ x
2 – isx i
dx
From equations (1) and (2)
–3

KJ N – s2
3 KJ is ON2 KJ is ON2O
–KK x 2 – isx + KK OO – KK OO OO F[f (x)*g(x)] = F(s).G(s) = πe 2
= # e L
K 2L P 2 O L P P dx

–3 Hence, convolution theorem is verified.

L o o k f o r t h e SIA Group logo o n t h e title cover before you buy


May/June-2017 (R15) Question Paper with Solutions QP.15
JK xy NO
Q8. (a)Form the partial differential equation z = f KK OO by eliminating the arbitrary function.
L 2 P
Answer : May/June-17, R(15), Q8(a)

Given equation is,


xy
z = f c m ... (1)
2
Where f is an arbitrary function.

Differentiating equation (1) partially with respect to 'x' on both sides,


2z xy y
2x = f ' c m . 2
2
xy y
i.e., p = f ' c m . 2 ... (2)
2
Again differntiating equation (1) partially with rspect to 'y' on both sides,

2z xy x
2y = f ' c m . 2
2
xy x
i.e., q = f ' c m. ... (3)
2 2
Dividing equation (2) and equation (3),
JK xy NO x
K O
p f 'K 2 O. 2
= JL NP
q K xy O y
f ' KK OO .
L2P 2
p y
Þ q = x
Þ px = qy

\ px = qy is the required partial differential equation.


2u 2u
(b) Use the method of separation of variables, solve =4 where u(x, 0) = 8e–3y.
2x 2y
Answer : May/June-17, R(15), Q8(b)

For answer refer Unit-IV, Q31.


OR
22 u 22 u
Q9. Solve the Laplace equation + = 0 subject to the conditions u(0, y) = u(i, y) = u(x, 0) = 0 and
2x 2 2y 2
JK nπx NO
u(x, a) = sin KK OO .
L l P
Answer : May/June-17, R(15), Q9

Note: In question u(l, y) is misprinted as u(i, y).

For answer refer Unit-IV, Q48.

Q10. (a) Find the z-transformation of sinnθ.

Answer : May/June-17, R(15), Q10(a)


Given that,
U(n) = sinnθ
3
Z{U(n)} = /U z n
–n

n=0

SPECTRUM ALL-IN-ONE JOURNAL FOR ENGINEERING sTUDENTs SIA Group


QP.16 MATHEMATICS-II [JNTU-anantapur]
3
Z{sin nθ} = / sin nθz –n

n=0

iθn
– e –iθn –n
3
= / (e 2i
2z
n=0

3 iθn 3 –iθn
= / e2i z –n
– / e2i z –n
n=0 n=0

3 3 n
1
=
2i
/ ^e iθ –1hn
z –
1
2i
/ ^e – iθ –1h
z
n=0 n=0

1 1 1 RSS 1 VW
WW
= ( 2 = S
2i 1 – eθi z –1 2i S1 – e –θi z –1 W
T X
1 1 1
= ( – 2
2i 1 – eθi z –1 1 – e –θi z –1
Z _
1 ]] 1 – e –θi z –1 – 1 + eθi z –1 bb
= [] `
2i ] ^1 – eθi z –1h ^1 – e –θi z –1h bb
\ a
Z _b
1 ]] ^eθi – e –θih z –1 b
= [ `
2i ]] 1 – ^eθi + e –θih z –1 + z –2 bb
R\ V a
1 SS z 2 ^eiθ – e –iθh z –1 WW
= S W
2i SS z 2 – z ^eiθ + e –iθh + 1WW
T X
^eiθ – e –iθh
z
= 2i
JK eiθ + e –iθ NO
z – 2z KK
2 OO + 1
L 2 P
z sin θ
= 2
z – 2z cos θ + 1
z sin θ
\ Z[sin nθ] = 2 .
z – 2z cos θ + 1
2z 2 + 5z + 14
(b) If U(z) = , evaluate U2, U3 using initial value theorem.
]z – 1g4
Answer : May/June-17, R(15), Q10(b)
For answer refer Unit-V, Q31.
OR
Q11. Solve the differential equation un + 2 – 2un + 1 + un = 3n + 5 using z-transforms.
Answer : May/June-17, R(15), Q11
For answer refer Unit-V, Q45.

L o o k f o r t h e SIA Group logo o n t h e title cover before you buy


May/June-16 (R15) Question Paper with Solutions QP.1
Code No.: 15A54201/R15
R15
B.Tech I Year II Semester Regular Examinations Solutions
May/June - 2016
MATHEMATICS - II
( Common to All )
Time: 3 Hours Max. Marks: 70
PART-A
( Compulsory Question)
*****
1. Answer the following: (10 × 02 = 20 Marks)
(a) Find L[t2.et.cos4t] (Unit- I)
sin 2t
(b) Find the Laplace Transform of t . (Unit- I)
(c) What are Dirichlet’s conditions? (Unit- II)
(d) Express f(x) = x as a Fourier series from – p to p . (Unit- II)
(e) Write the formula of the Fourier cosine integral of f(x). (Unit- III)
(f) Write the formula for the inverse Fourier transform of F(s) in ( – 3 , 3 ) (Unit- III)
(g) Find the value of Z (an cos nt). (Unit- IV)
(h) Find the Z-transform of the sequence {x(n)} where x(n) is n.2n (Unit- IV)
(i) Derive a partial differential equation by eliminating the arbitrary function f from the relation:
f(x2 + y2, x2 – z2) = 0 (Unit- V)
(j) Form the PDE from the relation z = f(x +it) + g(x – it). (Unit- V)

PART-B
( Answer all five units 5 × 10 = 50 Marks)

UNIT-I
s
2. Find the inverse Laplace Transform of 2 2 2 by using Convolution theorem. (Unit-I, Topic No. 1.8)
(s + a )
OR
3. Solve (D2 – D – 2)y = 20 sin2t where y(0) = 1, y'(0) = 2.(Unit-I, Topic No. 1.11)

UNIT-II
p2 1 1 1 1
4. Find a Fourier Series to represent x – x2 from x = – p to x = p and deduce that 12 = 2 – 2 + 2 – 2 + ...
1 2 3 4
(Unit-II, Topic No. 2.3)
OR
p p
5 If f(x) = 3 , 0 # x # 3
p 2p
= 0, 3 # x # 3
p 2p
= – 3, 3 #x#p

2 <
cos x – 5 cos 5x + 7 cos 7x + ... F (Unit-II, Topic No. 2.4)
1 1
Then f(x) =
3

SIA GROUP
QP.2 MATHEMATICS-II [JNTU-ANANTAPUR]

UNIT-III
3

6. Show that # sin pl sin lx


1 – l2
dl = p sin x, for 0 # x # p
2
0

= 0 for x > p (Unit-III, Topic No. 3.1)


OR 3

7. Find Fourier transform of f(x) = 1 – x for x # 1 = 0 for x $ 1 and hence find


2
# x cos x – sin x
x3
x
cos 2 dx
(Unit-III, Topic No. 3.2) 0

UNIT-IV
8. Find the partial differential equation of all spheres whose centre lie on Z - axis and given by equation

x2 + y2 + (z – a)2 = b2, and b being constants (Unit-IV, Topic No. 4.1)


OR
px
9. A string is stretched and fastened to two points l apart. Motion is started by displacing the string in the form y = a sin l
from which it is released at a time t = 0. Show that the displacement of any point at a distance x from one end at time t is

given by y (x,t) = a sin d l n cos d l n . (Unit-IV, Topic No. 4.3)


px pct

UNIT-V
10. Solve the difference equation, using Z-transorm un+2 – un = 2n, Where u0 = 0 and u1 = 1 (Unit-V, Topic No. 5.6)
OR
2
2z + 3z + 4
11. If f(z) = , ; z ; >3, then find the values of f(1), f(2), f(3). (Unit-V, Topic No. 5.4)
(z – 3) 3

L o o k f o r t h e SIA GROUP LOGO o n t h e TITLE COVER before you buy


May/June-16 (R15) Question Paper with Solutions QP.3

SOLUTIONS TO MAY/JUNE-2016, R15, QP


PART-A
Q1. (a) Find L[t2.et.cos4t]
Answer : May/June-16, (R15), Q1(a)
The given function is,
t2etcos4t
Consider, et cos4t
< a L [e at cos bt] = F
s –1 s–a
L[et cos4t] =
( s – 1) 2 + ( 4) 2 (s – a) 2 + b 2
s –1
= ... (1)
(s – 1) 2 + 16
Consider, [t2et cos4t]

d2 < 8 t
L e cos 4t BF < a L [t n f (t)] = (–1) n d n [ f (s)] F
n
L [t2et cos4t] = (–1)2 ... (2)
ds 2 ds
Substituting equation (1) in equation (2)

< F
d2 s –1
L [t2et cos4t] = (–1)2
ds 2 (s – 1) 2 + 16
RS V
SS ` (s – 1) 2 + 16 j d (s – 1) – (s – 1) d 8 (s – 1) 2 + 16 B WWW
= ds SSS WW
d ds ds
8 (s – 1) + 16 B
SS 2 2 WW
W
TR V X
SS WW
d S ` (s – 1) + 16 j (1) – (s – 1) 2 (s – 1) (1) WW
2
= ds SS WW
SS 8 (s – 1) 2 + 16 B
2
S WW
T X
= ds > H
2 2
d (s – 1) + 16 – 2 (s – 1)
8 (s – 1) 2 + 16 B
2

= ds > 2H
2
d 16 – (s – 1)
8 (s – 1) 2 + 16 B

<` (s – 1) 2 + 16 j (–2) (s–1) – 816 – (s – 1) 2 B 2 8 (s – 1) 2 + 16 B 2 (s – 1) F


2

= 2
d ` (s – 1) 2 + 16 j n
2

– 2 (s – 1) ` (s – 1) 2 + 16 j< (s – 1) 2 + 16 + 2 ` 16 – (s – 1) 2 jF
=
` (s – 1) 2 + 16 j
4

– 2 (s – 1) 8 (s – 1) 2 + 16 + 32 – 2 (s – 1) 2 B
= 3
` (s – 1) 2 + 16 j

– 2 (s – 1) 8 48 – (s – 1) 2 B
= 3
` (s – 1) 2 + 16 j

2 (s – 1) 8 (s – 1) 2 – 48 B
= 3
` (s – 1) 2 + 16 j

2 (s – 1) 8 (s – 1) 2 – 48 B
\ L {t2et cos4t} =
8 (s – 1) 2 + 16 B
3

SIA GROUP
QP.4 MATHEMATICS-II [JNTU-ANANTAPUR]
sin 2t
(b) Find the Laplace Transform of t .
Answer : May/June-16, (R15), Q1(b)
The given function is,
sin 2t
t
Let, f(t) = sin2t
3 SRS 3 WVW
L< t F =
SS W
sin 2t
# f (s) ds SS a L d t n =
f ( t)
# f (s) ds WW
WW
s SS s W
3 T X
= # L [sin 2t] ds
s
3

# < a L [sin at] = F


2 a
= ds
s2 + 4 s2 + a2
s
3

=2 # 1
s2 + 4
ds
s

= 2 < 2 tan –1 2 F
3
1 s
s

= < tan 2 F
3
–1 s
s
3 s
= tan –1 2 – tan –1 2

= tan –1 d tan 2 n – tan –1 2


π s < a tan π = 3 and tan –1 (tan θ) = θ F
2
π s
= 2 – tan –1 2

\ L < t F = 2 – tan –1 2
sin 2t π s

(c) What are Dirichlet’s conditions?


Answer : May/June-16, (R15), Q1(c)
For answer refer Unit-II, Q3.
(d) Express f(x) = x as a Fourier series from – p to p
Answer : May/June-16, (R15), Q1(d)
For answer refer Unit-II, Q11.
(e) Write the formula of the Fourier cosine integral of f(x).
Answer : May/June-16, (R15), Q1(e)
For answer refer Unit-III, Q18 excluding case (i).
(f) Write the formula for the inverse Fourier transform of F(s) in ( – 3 , 3 )
Answer : May/June-16, (R15), Q1(f)
For answer refer Unit-III, Q13.
(g) Find the value of Z (an cos nt).
Answer : May/June-16, (R15), Q1(g)
n
Given function is a cosnt
Consider, [cosnt]
z (z – cos nt)
Z[cosnt] = 2 ...(1)
(z – 2z cos nt + 1)

>a Z [cos θ] = H
z (z – cos θ)
(z 2 – 2z cos θ + 1)

L o o k f o r t h e SIA GROUP LOGO o n t h e TITLE COVER before you buy


May/June-16 (R15) Question Paper with Solutions QP.5
From time scaling property of Z-Transform,
z
an x(n) « X ` a j
z
Z[an cosnt] = X ` a j
z` z
– cos nt j
\ Z[an cosnt] = z a2 a z
` j – 2 cos nt + 1
a a
z
2 ( z – a cos nt)
= 2a
(z – 2za cos nt + a 2)
a2
z (z – a cos nt)
= 2
z – 2az cos nt + a 2
z (z – a cos nt)
\ Z[an cosnt] = 2
(z – 2az cos nt + a 2)
(h) Find the Z-transform of the sequence {x(n)} where x(n) is n.2n May/June-16, (R15), Q1(h)
Answer :
For answer refer Unit-V, Q2 (i).
(i) Derive a partial differential equation by eliminating the arbitrary function f from the relation:
f(x2 + y2, x2 – z2) = 0. May/June-16, (R15), Q1(i)
Answer :
Given function is,
f(x2 + y2, x2 – z2) = 0 ...(1)
Let, x2 + y2 = u and x2 – z2 = v ... (2)
\ From equations (1) and (2)
f(u,v) = 0 ... (3)
Differentiating equation (3) with respect to ‘x’ by chain rule,
2f 2u 2f 2u 2z 2f 2v 2f 2v 2z
2u 2x + 2u 2z 2x + 2v 2x + 2v 2z 2x = 0
2f 2f 2f 2f
Þ 2u (2x) + 2u (0) (p) + 2v (2x) – (2z) 2v p
2z
Where, p = 2x
2f 2f
Þ 2u (2x) + 2v (2x – 2pz) = 0

Þ 2 < x 2u + (x – pz) 2v F = 0
2f 2f

2f 2f
Þ x 2u + (x – pz) 2v = 0 ... (4)
Differentiating equation (3) with respect to ‘y’ by using chain rule,
2f 2u 2f 2u 2z 2f 2v 2f 2v 2z
2u 2y + 2u 2z 2y + 2v 2y + 2v 2z 2y = 0
2f 2f 2f 2f
Þ 2u (2y) + 2u (0) (q) + 2v (0) + 2v (–2z) q = 0
2z
Where, q = 2y

2 < 2u y + (–qz) 2v F = 0
2f 2f
Þ
2f 2f
Þ 2u y – qz 2v = 0 ...(5)
2f 2f
Eliminating 2u and 2v from equations (4) and (5),

SIA GROUP
QP.6 MATHEMATICS-II [JNTU-ANANTAPUR]
x x – pz
=0
y – qz
Þ – xqz – yx + pyz = 0
Þ (py – xq)z = xy
This is the required partial differential equation.
(j) Form the PDE from the relation z = f(x +it) + g(x – it).
Answer : May/June-16, (R15), Q1(j)
The given function is,
z = f(x + it) + g(x – it)
Let, x + it = u
and x – it = v
z = f(u) + g(v) ...(1)
Differentiating equation (1), partially with respect to ‘x’,
2z l 2u l 2v
2x = f (u) 2x + g (v) 2x
= f l (u) (1) + g l (v) (1) < a 2u = 1 and 2v = 1 F
2x 2x
2z
2x = f (u) + g (v)
l l ...(2)
Differentiating equation (2), partially with respect to ‘x’,
22 z 2u 2v
= f m (u) 2x + g m (v) 2x
2x 2
= f m (u) + g m (v) ...(3)
Differentiating equation (1) partially with respect to ‘t’,
2z 2u l 2v
2t = f (u) 2t + g (v) 2t
l

= f l (u) (i) + g l (v) (– i) < a 2u = i, 2v = – i F


2t 2t
2z
2t = if (u) – ig (v)
l l ... (4)
Differentiating equation (4) partially with respect to ‘t’,
22 z 2u 2v
= if m (u) 2t – ig m (v) 2t
2t 2
= i 2 f m (u) + i 2 g m (v)
22 z
= – f m (u) – g m (v) ... (5)
2t 2
Adding equations (3) and (5),
22 z 22 z
2 + = f m (u) + g m (v) – f m (u) – g m (v) = 0
2x 2t 2
22 z 22 z
\ 2 + =0
2x 2t 2
This is the required partial differential equation.
PART-B
UNIT-I
s
Q2. Find the inverse Laplace Transform of 2 by using Convolution theorem.
(s + a 2) 2
Answer : May/June-16, (R15), Q2
The given function is,
s
(s 2 + a 2) 2
s
Let, f (s) = 2
s + a2

L o o k f o r t h e SIA GROUP LOGO o n t h e TITLE COVER before you buy


May/June-16 (R15) Question Paper with Solutions QP.7

Then, f(t) = L–1 < F >a L–1 < F = cos at H


s s
(s 2 + a 2) ( s 2 + a 2)

= cosat
1
Let, g (s) = 2
s + a2
Then, g(t) = L–1 < F
1
(s 2 + a 2)

>a L–1 < F = a sin at H


1 1 1
= a sin at
(s 2 + a 2)
From convolution theorem,
RS VW
t
SS t
WW
L < 2
–1 s
(s + a 2) 2
F= # f (u) g (t – u) du SS a L–1 [ f (s) g (s) = f (t) ) g (t) =
SS
# f (u) g (t – u) du WW
WW
0 S 0 W
T X
1
= (cosat) ) a (sinat)
t
1
= a # cos au sin a (t – u) du
0
t

# [sin (au + a (t – u) – 2sin (au – a (t – u)] du < a cos A sin B = F


1 sin (A + B) – sin (A – B)
= a 2
0
t
1
= 2a # [sin (au + at – au) – sin (au – at + au)] du
0
t
1
= 2a # [sin at – sin (2au – at)] du
0
SRS t t WVW
1 SS W
# #
= 2a S sin at du – sin (2au – at) du WW
SS WW
S0 0 W
T X
1 * F 4
= 2a sin at 8 (u) B0 + <
t
t cos (2au – at)
2a 0

= 2a * 8 t sin at – 0 B + < F4
1 cos (2at – at) cos (0 – at)
2a – 2a

= 2a < t sin at + 2a – 2a F
1 cos at cos at

1
= 2a [t sin at + 0]
1
= 2a t sin at

\ L–1 < F = 2a t sin at


s 1
(s 2 + a 2) 2
OR
Q3. 2
Solve (D – D – 2)y = 20 sin2t where y (0) = 1, y'(0) = 2. May/June-16, (R15), Q3
Answer :
Given differential equation is,
(D 2 – D – 2) y = 20 sin 2t
y (0) = 1, yl (0) = 2
<a D = d F
d 2 y dy
Þ – – 2y = 20 sin 2t
dx 2 dx dx
Þ y m – yl – 2y = 20 sin 2t ... (1)

SIA GROUP
QP.8 MATHEMATICS-II [JNTU-ANANTAPUR]
Applying Laplace Transform on both sides of equation (1),
L 8 y m – yl – 2y B = L 8 20 sin 2t B

Þ L 8 y m B – L 8 ylB – 2L 8 y B = 20 L 8sin 2t B

Þ 8 s 2 L (y) – sy (0) – yl (0) B – 8 s L [y] – y (0) B – 2 L 8 y B = 20 2


... (2)
s 2 + (2) 2

< a L 8 y n B = s n L (y) – s n – 1 y (0) – s n – 2 yl (0), L 8 sin at B = F


a
s2 + a2

s 2 L 8 y B – s ( 1 ) – ( 2) – < s L 8 y B – 1 F – 2 L 8 y B =
40
Þ
s2 + 4
40
Þ s 2 L (y) – s – 2 – s L [y] + 1 – 2 L [y] =
s2 + 4
40
Þ (s2 – s – 2) L [y] – s – 2 + 1 =
s2 + 4
40
Þ (s2 – s – 2) L [y] – (s + 1) = 2
s +4
40
Þ (s2 – s – 2) L [y] = 2 + (s + 1)
s +4
40 ( s + 1)
Þ L [y] = 2 +
(s + 4) (s 2 – s – 2) (s 2 – s – 2)
40 (s + 1)
Þ L [y] = +
(s + 1) (s – 2) (s 2 + 4) (s + 1) (s – 2)
40 1
Þ L [y] = + ... (3)
(s – 2) (s + 1) (s 2 + 4) (s – 2)
40
Consider,
(s – 2) (s + 1) (s 2 + 4)
Applying partial fractions,
40 A B Cs + D
= (s – 2) + (s + 1) + 2 ... (4)
(s – 2) (s + 1) (s 2 + 4) ( s + 4)
Þ 40 = A (s + 1) (s 2 + 4) + B (s – 2) (s 2 + 4) + [Cs + D) (s – 2) (s + 1)] ... (5)
3 2 3 2
Þ 40 = A (s + s + 4s + 1) + B (s – 2s + 4s – 8)
+ Cs3 – Cs3 – Cs 2 – 2Cs + Ds 2 – Ds – 2D ... (6)
Substituting s = 2 in equation (5),
40 = A (2 + 1) (2 2 + 4) + B (0) + (Cs + D) (0)
Þ 40 = A (3) (8) + 0
40
Þ A = 24
5
` A= 3
Substituting s = – 1 in equation (5),
40 = A (0) + B (– 1 – 2) (( – 1) 2 + 4) + 0 + 0
Þ 40 = – 3B (5)
40
Þ B = – 15
–8
` B= 3
Substituting s = 0 in equation (5),

40 = A (1) (4) + B ( – 2) (4) + C (0) + D ( – 2) (1)


Þ 40 = 4A – 8B – 2D

40 = 4 d 3 n – 8 d 3 n – 2D
5 –8
Þ

L o o k f o r t h e SIA GROUP LOGO o n t h e TITLE COVER before you buy


May/June-16 (R15) Question Paper with Solutions QP.9
20 + 64
Þ 40 = 3 – 2D
84
Þ – 2D = 40 – 3
Þ – 2D = 40 – 28
Þ – 2D = 12
Þ D =–6
` D=–6
Comparing the co-efficients of s3 on both sides of equation (6),
0= A+B+C
5 8+
Þ 3 – 3 C=0
3
Þ C= 3 =1
` C=1
Substituting the corresponding values in equation (4),
5 d –8 n
40 3 3 1 ( s) – 6
2 = (s – 2) + (s + 1) + 2
(s – 2) (s + 4) s +4
5 8 (s – 6)
= 3 (s – 2) – 3 (s + 1) + 2
(s + 4)
Substituting the corresponding values in equation (3),
5 8 (s – 6)
L [y] = 3 (s – 2) – 3 (s + 1) + 2 + 1
s +4 s– 2
8 8 s 6
= 3 ( s – 2) – 3 ( s + 1 ) + 2 –
s + 4 s2 + 4
L [y] = 3 < (s – 2) – (s + 1) F + 2
8 1 1 s 6
– 2
s +4 s +4
Applying inverse Laplace transform on both sides,

L – 1 > 3 < (s – 2) – (s + 1 ) + < 2 FH


8 1 1 s ( 3 × 2)
L -1 [L (y)] = – 2
s +4 s +4
8 8 2t –t
B + cos 2t – 3 sin 2t
y= 3 e –e

>a L – 1 < s 1– a H = eat L -1 <


s
F = cosat L -1 <
a
F = sin at B
s2 + a2 s2 + a2

` y = 3 8 e 2t – e –t B + cos 2t – 3 sin 2t
8

UNIT-II
p2 1 1 1 1
Q4. Find a Fourier Series to represent x – x2 from x = – p to x = p and deduce that 12 = 2 – 2 + 2 – 2 + ...
1 2 3 4
Answer : May/June-16, (R15), Q4
The given function is,
f (x) = x – x2 in (–π, π)
From the definition of trigonometric Fourier series,
∞ ∞

∑ ∑b
a0
f(x) = + an cos nx + n sin nx ... (1)
2 n =1 n =1

π π π
1 1 < x 2 x3 F
Where, a0 =
π ∫ f ( x)dx = 1
π # (x – x ) dx = 2
π 2 – 3 –π
−π –π

SIA GROUP
QP.10 MATHEMATICS-II [JNTU-ANANTAPUR]

= p > 2 – 3 – d 2 + 3 nH = p < 2 – 3 – 2 – 3 F
1 p 2 p3 (–p) 2 (–p) 3 1 p 2 p3 p 2 p3

= p < 3 F= 3
–1 2p3 – 2p 2

– 2p 2
\ a0 = 3
p p
(x – x 2) cos nx.dx = π >< (x – x 2) n F – # (1 – 2x) sinnnx dx H
p
1 1 sin nx
an = π # –p
–p –p
SRS J NOWW
V
SS p KK p
O WW
= π SS< (x – x 2) n F – KKK > (1 – 2x) d dx H OOOWW
–p
1
SS
sin nx
–p K
– cos nx
n 2 n – (– 2)
(– cos nx)
n 2 # OOWW
SS K –p
PWW
–p
L
T X
>a # uvdx = u # vdx – # d dxd u # vdx ndx H
SRS J NWW
V
SS p K K p O O W
= π SS< (x – x 2) n F – KKK > (1 – 2x) d H OOWW
p
n d n
1 sin nx – cos nx sin nx
– 2.
SS –p K
K n2 –p n3 –p OOOWWW
SS L PWW
T X
= π >< (x – x 2) n F + < (1 – 2x) F + < 2. 3 F H
p p p
1 sin nx cos nx sin nx
–p n 2 –p n –p

= π >< (π – π) 2 n + (1 – 2π) 2. 3 F – < (– π – (– π) 2) FH


1 sin nπ cos nπ + sin nπ sin n (–π) cos n (–π) sin n (–π)
+ (1 – 2 (–π)) + 2.
n2 n n n2 n3

= π < 0 + (1 – 2π) 2 + 0 – 0 – (1 + 2π) – 0F


n
8 a sin nπ = 0 B
1 (–1) cos nπ
n n2

= π < (1 – 2π) 2 – (1 + 2π) 2 F 8 a cos nπ = (–1) n B


n n
1 (–1) (–1)
n n

= π< 2 – F
1 (–1)
n
2π (–1) n (–1) n 2π (–1) n
– –
n n2 n2 n2

= π < 4π 2 F
–1 (–1) n
n
n
– 4 (–1)
=
n2
– 4 (–1) n
` an =
n2
p
1
bn = π # (x – x2) . sin nx.dx
–p
RS VW
SS p W
= π SS> (x – x 2) d
– cos nx n H
p
– cos nx WW
1
SS n – ( #
1 – 2 x ) . n dx WW
S –p –p WW
TR X VW
SS p WW
= π SS> (x – x 2) d
– cos nx n H +
p

> H
S
< F
p
1
SS n ( 1 – 2 x )
sin nx
n 2 – (#–2 ) d
sin nx
n 2 dx n WW
WW

S –p p
–p W
T p X
1> n H + < ( 1 – 2 x) 2 F + > 2 d n H
p p
= π ( x – x 2) d
– cos nx sin nx – cos nx
n
–p
n –p n3 –p
p

= π > (x – x 2 ) d H
1 – cos nx n + sin nx 2 cos nx
(1 – 2x) 2 –
n n n3
–p

L o o k f o r t h e SIA GROUP LOGO o n t h e TITLE COVER before you buy


May/June-16 (R15) Question Paper with Solutions QP.11
RS VW
SS sin n (–π) 2 cos n (–π) WWW
= π SS (π – π 2) d
1
SS
– cos nπ n +
n ( 1 – 2π )
sin nπ
– 2
cos nπ
– > (– π – (– π ) 2 d – cos n (–π) n
) n + ( 1 + 2 π) – HWW
n2 n3 n2 n3 WW
S
T X
= π (π – π ) (– 1) n + (1 – 2π) .0 – 2. 3 + (π + π) (–1) n – (1 + 2π) .0 + 2. 3 F
1< 2 ( – 1) n (– 1) n 2 (– 1) n (– 1) n
n n

= π < – (π – π 2) n + (–π 2 – π) n F
n n
1 ( – 1) ( – 1)

= π < –π n + π 2 n – π 2 n – π n F = π < –2π n F =


1 (– 1) n (– 1) n (– 1 ) n (– 1 ) n 1 (– 1) n – 2 ( – 1) n
n
n
– 2 (– 1)
` bn = n
Substituting the corresponding values in equation (1),
3 3
– 2p 2 (–1) n (–1) n
f(x) = 3 (2) + /
( – 4) 2 cos nx +
n
/
( – 2) n sin nx
n=1 n=1
3 n 3 n
=
– p2
3 –4 / (–n1)
2 cos nx – 2 / ( –n1) sin nx
n=1 n=1

< – sin x + sin 2x – sin 3x + .... F


3 – 4 < 1 2 + 2 2 – 3 2 + ... F – 2
– p2 – cos x cos 2x cos 3x
= 1 2 3

f(x) = x – x2 = 3 + 4 < 2 – + ... F


2
–p cos x cos 2x cos 3x

1 22 32

+ 2 < 1 – 22 + 33 – ... F
sin x sin x sin x
... (2)

Substituting x = 0 in equation (2),

0 = – 3 + 4 < 2 – 2 + 2 – 2 + ... F + 2 < 1 – 2 + 3 ... F > H


p2 1 1 1 1 0 0 0 a cos 0o = 1
1 2 3 4 sin 0o = 0
– p2
3 + 4 d 1 2 – 2 2 + 3 2 – 4 2 + ... n
1 1 1 1
0=

p2
4 d 2 – 2 + 2 – 2 + ... n = 3
1 1 1 1
Þ
1 2 3 4
1 1 1 1 p2
Þ 2 – 2 + 2 – 2 + ... = 3 × 4
1 2 3 4
1 1 1 1 p2
Þ 2 – 2 + 2 – 2 + ... = 12
1 2 3 4
p2 1 1 + 1 1
` 12 = 1 2 – 2 2 3 2 – 4 2 + ...
Hence proved.
OR
π π
Q5 If f(x) = 0#x# 3
3,
= 0, π 2π
3 #x# 3
=– π
3,

3 #x#π
2 <
cos x – 5 cos 5x + 7 cos 7x + ... F
1 1
Then f(x) =
3
Answer : May/June-16, (R15), Q5
Given that,
Z] π π _b
]] , 0 # x # 3 bb
]] 3 bb
]] π 2π bb`
f (x) = ][ 0, 3 # x # 3 bb
]] bb
]] π 2π bb
]] – , # x # π b
3 3
\ a
SIA GROUP
QP.12 MATHEMATICS-II [JNTU-ANANTAPUR]
Here, the interval is 0 to 2 p . half-range cosine series is to be determined
Half - range cosine series is expressed as,
3
f(x) = a0 + / a cos nx
n ... (1)
n=1

Where,
p
2
a0 = p # f (x) dx
0

And
p
2
an = p # f (x) cos nx dx
0
RS p/3 VW
S 2p/3 p
WW
2 SS
a0 = p S #
SS
f (x) dx + # f (x) dx + # f (x) dx WW
WW
S0 p/3 2p/3 W
T X V
SRS p/3 2p/3 p WW
S W
= pS d
2S
SS
# p
3 n dx + #0 dx + # < – p
3 F dx WW
WW
S0 p/3 2p/3 W
RS p/3 T V X
SS p WW
W
2 p
= p SS 3
SS
# dx + 0 3
–p
#
dx WW
WW
S 0 2p/3 W
T X
2 p <8 Bp/3 8 Bp F
= p × 3 x 0 – x 2p/3

= 3 > 3 – 0 – < p – 3 FH
2 p 2p

= 3 <3 – 3F
2 p p

=0
` a0 = 0

Similarly,
SRS p/3 2p/3 p WVW
2 SS W
an = p SS # f (x) cos nx dx + # f (x) cos nx dx + # f (x) cos nx dx WW
WW
SS 0 p/3 2p/3 W
RST p/3 VW X
S 2p/3 p
WW
2 SS
= pS S # p
3 cos nx dx + # 0 cos nx dx + # d p n cos nx dx WW
3 WW
SS 0 p/3 2p/3 W
TR V X
SS p/3 p WW
2 SS p W
= p SS 3 # cos nx dx + 0 + d – 3 n
p
# cos nx dx WW
WW
SS 0 2p/3 W
T X
2 p >< sin nx F < F H
p/3 p
sin nx
= p 3 n 0 – n 2p/3

= 3 > n <sin n 3 – sin (0) F H – n <sin np – sin 2n 3 F


2 1 p 1 p

= 3n <sin n 3 – 0 + sin 2n 3 F > H


2 p p a sin np = 0
sin (0) = 0

= 3n >sin n 3 + sin d
2 p 3np – np n H
3

L o o k f o r t h e SIA GROUP LOGO o n t h e TITLE COVER before you buy


May/June-16 (R15) Question Paper with Solutions QP.13

= 3n >sin n 3 + sin d np – 3 n H
2 p np

= 3n >sin n 3 + ( – 1) n sin d – 3 n H 8 a sin (np + q) = ( – 1) n sin q B


2 p np

= 3n <sin n 3 – ( – 1) n sin 3 ) F 8 a sin ( – q ) = – sin q B


2 p p

= 3n sin n 3 81 – ( – 1) n B
2 p

` an = 3n sin n 3 81 – ( – 1 ) n B
2 p

For n = 1
Þ a1 = 3 (1) sin (1) 3 81 – ( – 1) 1 B
2 p

= 3 sin 3 81 + 1 B
2 p
4 p
= 3 sin 3
For n = 2
Þ a2 = 3 (2) sin 2 3 81 – 1 B
2 p

=0
` an = 0 for even values of n
4 p
= 3n sin n 3 for odd values of n.
Substituting a0 and an values in equation (1),
3
f(x) = 0 + /4 p
3n sin n 3 cos nx
n=1

/ < 1n sin n p3 cos nx F


3
4
= 3
n=1

= 3 <sin 3 cos x + 3 cos 3x (0) + 5 sin 5 3 cos 5x + 7 sin 7 3 cos 7x + ... F


4 p 1 1 p 1 p

= 3 < 2 cos x – 2 5 cos 5x + 2 7 cos 7x + ... F


4 3 31 31 >a sin 5 p3 = sin (2p – p3 ) = sin p3 = d 23 n H

= 3 × 2 < cos x – 5 cos 5x + 7 cos 7x + ... F


4 3 1 1

2 <
cos x – 5 + 7 + ... F
cos 5x cos 7x
=
3
2 <
cos x – 5 + 7 + ... F
cos 5x cos 7x
` f(x) =
3
Hence proved.

UNIT-III
3
Q6. Show that # sin pl sinlx
1 – l2
p
dl = 2 sin x, for 0 # x # p
0
=0 for x > p
Answer : May/June-16, (R15), Q6
Given integral is,
3
π
# sin pl sin lx
1 – l2
dl = 2 sin x for 0 £ x £ p
0
=0 for x > p
Consider the function

SIA GROUP
QP.14 MATHEMATICS-II [JNTU-ANANTAPUR]
p
f(x) = 2 sin x for 0 < x < p
=0 for x > p ... (1)
Here, the interval is 0 to p
Fourier sine integral is expressed as,
3 3
2
f(x) = p # sin lx # f (t) sin lt dt dl ... (2)
0 0

Substituting the equation (1) in equation (2),


3 p
a f (x) = f (t)
# sin lx # > H
2 p
f(x) = p 2 sin t sin lt dt dl
0 0 =p2 sin t
3 p
2 p
= p×2 # sin lx # sin t sin lt dt dl
0 0
3 3
1< ` < ` sin A sin B = 1 cos (A – B) – cos (A + B) F
= # sin lx # 2 cos lt – t j – cos (lt + t) F dt dl 2
0 0
3 3
18
= # sin lx # 2 cos t (l – 1) – cos t (l + 1) B dt dl
0 0
3

# sin lx< (l 1– 1) sin (l – 1) t – (l 1+ 1) sin (l + 1) t F


p
1
= 2 dl
0
0
3
1
= 2 # sin lx< (l 1– 1) 8sin (l – 1) p – sin (l – 1) (0) BF – (l 1+ 1) 8sin (l + 1) p – sin (l + 1) 0 Bdl
0

# sin lx * < (l1– 1 8sin (pl – p) – 0 F – sin((lpl+ 1+)p) –0 B 4 dl


3
1
= 2
0

# sin lx * < – sin (lp––1pl) F – < sin((lpl+ 1+)p) F 4 dl


3
1
= 2
0

# sin lx * < sin(1(p– –l)pl F – < –(lsin+ pl F 4 dl


3

> H
1 a sin (p – q) = sin q
= 2 1)
0 sin (p + q) = – sin q
3
1
= 2 # sin lx< sin pl sin pl F
1 – l + 1 + l dl
0
3
1
= 2 # sin lx sin pl< 1 –1 l + 1 +1 l Fdl
0
3
1
= 2 # sin lx sin pl< (11 +– ll)+(11 +– ll) Fdl
0
3
1
= 2 # sin lx sin pl< 1 –2l 2 F dl
0
3
1
= 22 # sin lx sin pl
1 – l2
dl
0
3

Þ # sin lx sin pl
1 – l2
dl = f(x)
0
3

` # sin lx sin pl
1 – l2
p
dl = 2 sin x
0

Hence proved.

L o o k f o r t h e SIA GROUP LOGO o n t h e TITLE COVER before you buy


May/June-16 (R15) Question Paper with Solutions QP.15
OR 3

Q7. Find Fourier transform of f(x) = 1 – x for x # 1 = 0 for x 2 1 and hence find
2
# x cos x – sin x
x3
x
cos 2 dx .
Answer : 0
May/June-16, (R15), Q7
Given function is,
f(x) = * 1 – x for – 1 # x # 1 4
2

0 for x 1 – 1 and x > 1


By definition, the Fourier transform of f (x) is,
∞ 1
1 1
2π ∫ 2π ∫
F(s) = f ( x) . eisx dx = 2
(1 − x ) . e isx
dx
−∞ −1
1
1  2 e
isx   isx   isx 
 − (−2 x) e  + (−2) e 
= (1 − x )    i2s2   i 3s3 
2π   is      −1
1
1  2 e
isx   isx
 − (2 x) e
 2 isx 
= (1 − x )    s2
+
 is 3 e 
2π   is     −1

1   e is  2 is    e −is  2 −is 


= 0 − (2) 2 + e  − 0 + 2 + e 
2π  s  is 3    s2  is 3 
     
1  2 is −is 2 is −is 
= − (e + e ) + is 3 (e − e )
2π  s 2 
1  2 2  RS VW
− (2 cos s ) + is 3 (2i sin s )
iq –iq
SS a cos θ = e + e WW
= 2π  s 2  SS 2 WW
SS e iq
– e –iq WW
SS sin θ = WW
2 2 1 2i
F(s) = − . ( s cos s − sin s ) T X ... (1)
π s3
Applying the inverse fourier transform to equation (1),
3
1

# F (s) .e –isx ds = f (x)
–3

1 2 2 1
− ∫ (s cos s − sin s )e −isx ds = f (x)
2π −∞
π s3

2 1
3 1 − x 2 , | x |< 1
Þ π #s3
( s cos s – sin s ) e –isx
ds =  ... (2)
–3  0, otherwise
1
Substituting x = 2 in equation (2),

2 1
π ∫s
−∞
3
( s cos s − sin s ) e −isx ds = 1–


1  s s π
∫s
−∞
3
( s cos s − sin s ) cos − i sin  ds =
 2 2
×
2

1 s ∞
∫s ( s cos s − sin s ) cos  ds − i ( s cos s − sin s ) sin s  ds
−∞
3
2
−∞
s3 ∫ 2
= ... (3)

Comparing the real and imaginary parts both sides of equation (3),

1 s
∫s
−∞
3
( s cos s − sin s ) cos  ds =
2
... (4)

SIA GROUP
QP.16 MATHEMATICS-II [JNTU-ANANTAPUR]
3
sin d 2 n ds = 0
(s cos s – sin s)
And # s3
s
... (5)
–3

The integrand of equation (5), indeed, is an odd function of s and hence the integral vanishes. i.e. 0
The integrand on the L.H.S of equation (4) is an even function of s. Hence, equation (4) becomes,
 ∞ ∞ 

( s cos s − sin s )
2 ∫ s3
s
cos  ds
2
=

8

 ∫ ∫
f ( x)dx = 2 f ( x)dx
 ... (6)
0  −∞ 0 
Replacing s with x and simplifying, we get from equation (6),
3

cos d 2 n dx = 8
x cos x – sin x 3π
2 # x3
x
0
3

cos d 2 n dx =
x cos x – sin x 3π
\ # x3
x
0 16
UNIT-IV
Q8. Find the partial differential equation of all spheres whose centre lie on Z - axis and given by equation
x2 + y2 + (z – a)2 = b2, a and b being constants.
Answer : May/June-16, (R15), Q8
Given equation of all spheres whose center lie on Z-axis is,
(x – 0)2 + (y – 0)2 + (z – a)2 = b2
x2 + y2 + (z – a)2 = b2 ...(1)
Where a, b are constants
Differentiating equation (1) partially with respect to ‘x’,
2z
2x + 0 + 2(z – a) 2x = 0

Þ x + (z – a) (p) = 0 < a 2z = p F
2x
Þ (z – a)p = – x
–x
Þ (z – a) = p
–x
\ (z – a) = p ... (2)
Differentiating equation (1) partially with respect to ‘y’,
2z
0 + 2y + (z – a) 2x = 0

Þ 2y + 2(z – a)q = 0 < a q = 2z F


2y
Þ (z – a)q = – y
–y
Þ (z – a) = q ... (3)
Substituting equation (3) in equation (2),
–x –y
p = q
x y
Þ p = q
Þ xq = py
Þ xq – py = 0
\ xq – py = 0
This is the required partial differential equation.
OR

L o o k f o r t h e SIA GROUP LOGO o n t h e TITLE COVER before you buy


May/June-16 (R15) Question Paper with Solutions QP.17
Q9. A string is stretched and fastened to two points l apart. Motion is started by displacing the string in
px
the form y = a sin l from which it is released at a time t=0. Show that the displacement of any point
at a distance x from one end at time t is given by y ( x ,t) = a sin d l n cos d l n .
px pct

Answer : May/June-16, (R15), Q9


y

y (x, t)
o x
l
Wave equation is,
22 y 22 y
= 2 ... (1)
2t 2 2x 2
Boundary conditions are,
(i) y(0, t) = 0
(ii) y(l, t) = 0
2y
(iii) 2t (x, 0) = 0
2y πx
(iv) 2t (x, 0) = a sin l
y(x, t) = (c1 cos px + c2 sin px) (c3cos pct + c4sin pct) which satisfies the boundary conditions. ... (2)
Applying condition (i) in equation (2),
y(0, t) = c1(c3 cos pct + c4 sin pct)
Where, c3 cos pct + c4 sin pct ¹ 0
` c1 = 0
Substituting in equation (2),
y(x, t) = (c2 sin px)(c3 cos pct + c4 sin pct)
Applying condition (ii) in equation (3),
y(l, t) = c2 sin pl(c3 cos pct + c4 sin pct)
0 = c2 sin pl(c3 cos pct + c4 sin pct)
When c2 ¹ 0 and c3 cos pct + c4 sin pct ¹ 0
\ sin pl = n π
Þ pl = n π

p= 6
Substituting p value in equation (3),
nπx nπct nπct
y(x, t) = c2 sin l ` c3 cos l + c4 sin l j ... (4)

Differentiating equation (4) with respect to ‘t’,


2y nπx d nπc nπct nπc nπct n
2t (x, t) = c2 sin l – c3 l sin l + c4 l cos l
Applying condition (iii),
2y nπx d nπc n
2t (x, 0) = c2 sin l c4 l

SIA GROUP
QP.18 MATHEMATICS-II [JNTU-ANANTAPUR]
nπx
0 = c2 sin l (c4 nπc)

` c4 = 0 < a c2 ! 0 and sin nπx ! 0 F


6
Substituting above value in equation (4),
nπx nπct
y(x, t) = c2 sin l c3 cos l
nπx nπct
y(x, t) = bn sin l cos l [Put c2, c3 = bn]
\ General solution is,
3
y(x, t) = / bn sin nπl x cos nπlct
n=1

Applying boundary conditions (iv) in equation (5),


px
y(x, 0) = a sin l
3
= / px
bn sin n l
n=1
px px px
Þ a sin l = b1 sin l + b2 sin 2 l + ...
px
Equating the co-efficient of sin l ,
b1 = a and b2, b3...... bn = 0
For n = 1,
p (1) x p (1) ct
y (x, t) = a sin l cos l
px pct
= a sin l cos l

` y (x, t) = a sin d l n cos d l n


px p ct

Hence proved.

UNIT-IV
Q10. Solve the difference equation, using Z-transorm un+2 – un = 2n, Where u0 = 0 and u1 = 1
Answer : May/June-16, (R15), Q10
For answer refer Unit-V, Page No. 5.31, Q43.
OR
2z2 + 3z + 4
Q11. If f(z) = , z > 3 , then find the values of f(1), f(2), f(3).
( z – 3) 3
Answer : May/June-16, (R15), Q11
Given that,
2z 2 + 3z + 4
f(z) = ... (1)
( z – 3) 3
f(1) = ?
f(2) = ?
f(3) = ?
z2 < 2 + z + 2 F
3 4
z
f(z) =
z3 <1 – z F
3
3

RS V
3 4 WWW
S < 2 + z + z 2 F WW
SS
1S
Þ f(z) = z SS WW ... (2)
SS 3
W
SS d 1 – z n WWW
3
S W
T X
L o o k f o r t h e SIA GROUP LOGO o n t h e TITLE COVER before you buy
May/June-16 (R15) Question Paper with Solutions QP.19
From initial value theorem,
f(0) = Lim f (z) ... (3)
z"3
Substituting equation (2) in equation (3),
RS V
SS 2 + 3 + 4 WWW
1S z z WW
2
f(0) = Lim z SS 3 W
z"3 S
SS <1 – F WWW
3
S z W
T X
1
= 3
=0
f(0) = 0
f(1) is expressed as,
f(1) = Lim z 8 f (z) – f (0) B ... (4)
z"0
Substituting the corresponding values in equation (4),
RS R V VW
SS SS 3 4 WW WW
SS 1 SS 2 + z + 2 WW WW
f(1) = Lim z SS z SS z W W
3 W – 0W
z"3 S S
SS SS d 1 – 3 n WWW WW
WW
SS S z W W
T X
RS T VW X
SS 2 + 3 + 4 WW
S z z2 W
= Lim SS 3 W
W
z"3 S
SS d 1 – 3 n WWW
S z W
3 T 4 X
2+ 3 + 3
= 3
d1 – 3 n
3
2+0+0
= 1– 0
=2
\ f(1) = 2
f(2) is expressed as
f(2) = Lim z 2 [f (z) – f (0) – f (1) z –1] ... (5)
z"3
Substituting the corresponding values in equation (5),
RS R V VW
SS SS 3 4 WW WW
SS 1 SS 2 + z + 2 WW W
z –1 W
f(2) = Lim z 2 SS z SS W
3 W – 0 – 2z W
W
SS SS WW WW
SS SS d 1 – z n WW
z"3 3
WW
S T X W
TR V X
SS W
SS < 2 + + 2 F 3 4 W
WW
S1 z z 2 WW
= Lim z 2 SS z – z WW
SS 3
d1 – 3 n WW
z"3
SS z WW
S
TR X V
SS 3W
SS 2 + 3 + 4 – 2 d 1 – 3 n WWW
z2 S z z2 z WW
= Lim z SS WW
S 3
SS d1 – 3 n WW
z"3
SS z WW
T X
SRS WV
SS 2 + + 2 – 2 <1 – + 2 – 13 F WWW
3 4 3 3
8 a (a – b) 3 = a3 + b3 – 3ab + 3ab 2 B
S z z z z z WW
= Lim z SS WW
z"3 S
3
SS d1 – n 3 WW
SS z WW
T X
SIA GROUP
QP.20 MATHEMATICS-II [JNTU-ANANTAPUR]
RS V
SS 2 + 3 + 4 – 2 + 6 + 6 – 2 WWW
S z z2 z z 2 z3 W
= Lim z SS 3
WW
z"3 S WW
SS d1 – n3
WW
S z
TRS VW X
SS 9 – 2 + 2 WW
S z z 2 z3 WW
= Lim z SS 3 W
z"3 S
SS d 1 – 3 n WWW
S z W
RS T VW X
SS 9 – + 2 2 W
S z z 2 WW
= Lim SS 3 W
W
z"3 S
SS d 1 – 3 n WWW
S z W
9 – 0T + 0 X
= 1– 0
=9
\ f(2) = 9
f(3) is expressed as
f(3) = Lim z3 [f (z) – f (0) – f (1) z –1 – f (2) z –2] ... (6)
z"3
Substituting the corresponding values in equation (6),
RS R V VW
SS SS 3 4 WW WW
S 1 SS 2 + +
z z 2 WW WW
3S 2 9
f(3) = Lim z SS z SS W
3 W–0– z – 2W
W
SS SS 3 n WW z WW
d
z"3
SS SS 1 – z WW WW
S T X W
RST VW X
J
SSK 2z 2 + 3z + 4 O N WW
= Lim z SSK K O – 2z – 9 WW
3 O
z " 3 SK O WW
SSKK z d 1 – 3 n OO WW
SSKL z O
P W
RTS X VW
< 3 j3 F – 9z ` 1 – 3 j3 WW
SS 2
SS 2 z + 3z + 4 – 2 z z ` 1 – z z WW
= Lim z SS WW
z`1 – 3 z j
3
z"3 S WW
SS W
RS T X VW
SS 2z 2 + 3z + 4 – 2z 2 1 – 3 + 3 – 1 3 + 3 – 1 n WW
SS d z z2 z3 n – 9 z d 1 – z z2 z3 WW
= Lim SS WW
`1 – 3 z j
3
z"3 S WW
SS W
RTS VW X
SS WW
SS d 2z + 3z + 4 – 2z + 6z – 6 + z n – 9z + 27 – z + z 2 WW
2 2 2 27 9
= Lim SS WW
`1 – 3 z j
3
z"3 S WW
SS W
TRS VW X
SS < 9z – 9z – 2 + 27– z – z + z 2 F WW
SS 2 27 9 WW
= Lim SS 3
WW
z"3 S `1 – 3 z j WW
SS W
RTS 25 9 VW X
SS 0 + 25 – + 2 WW
S z z WW
= Lim SS 3 WW
z"3 S S ` 1 – 3 j W
z
RTS 25 9 VW X
SS 25 – W
S z + z 2 WW
= Lim SS 3 W
W
z"3 S S ` 1 – 3 z j WW
T X
25 – 0 + 0
= 1– 0
= 25
\ f(3) = 25
\ f(1) = 2, f(2) = 9 and f(3) = 25

L o o k f o r t h e SIA GROUP LOGO o n t h e TITLE COVER before you buy


May-2018 (R15) Question Paper with Solutions QP.1
Code No.: 15A54201
Jawaharlal Nehru Technological University Anantapur
B.Tech. I Year II Semester Regular & Supplementary Examinations
R15
Solutions
May - 2018
MATheMATics-ii
( Common to all )
Time: 3 Hours Max. Marks: 70

Part-a
(Compulsory Question)
1. Answer the following: (10 × 2 = 20 Marks)
(a) Find the Laplace transform of cos2t. (Unit-I)
s
(b) Find the inverse Laplace transform of . (Unit-I)
s 2 + s + 13
(c) Find the half-range sine series for the function f(t) = t – t2 0 < t < 1. (Unit-II)
(d) Obtain the Fourier series for f(x) = x in the interval (0,2). (Unit-II)
(e) State convolution theorem of the inverse transforms. (Unit-III)
2
(f) Find the Fourier transformation of e–x . (Unit-III)
(g) Solve py3 + qx2 = 0 by method of separation of variables. (Unit-IV)
2
x2 + y
(h) Form the partial differential equation by eliminating the arbitrary constants 2z = 2 2 . (Unit-IV)
a b
(i) Find the Z-transformation of (nan). (Unit-V)
1
(j) Find the inverse Z-transformation of z – 2 . (Unit-V)

Part-B

(Answer all five units, 5 × 10 = 50 Marks)

Unit-I
Z]
]]Esin wt, 0 < t < p/w
2. (a) Find the Laplace transform of the function f(t) = [] p having period 2p/w. (Unit-I, Topic No. 1.9)
]]0, w < t < 2p/w
\
1 at bt
(b) Find the Laplace transform of t (e – e ) (Unit-I, Topic No. 1.4)
OR
3. Solve by Laplace transform method y'' – 3y' + 2y = 4, where y(0) = 2, y'(0) = 3. (Unit-I, Topic No.1.11)

Unit-II
4. (a) Find the Fourier series to represent (x – x2) from x = –p to x = p. (Unit-II, Topic No. 2.2)
(b) Find the half range cosine series for the function f(x) = x2 in the range 0 # x # p. (Unit-II, Topic No. 2.4)
OR
5. (a) Find the complex form of the Fourier series of f(x) = e–x in – 1 ≤ x ≤ 1 (Unit-II, Topic No. 2.5)
Z]
]] 1 – x if 0 < x < 1
]4 2
(b) Expand f(x) = ][ as the Fourier series of sine terms. (Unit-II, Topic No. 2.4)
]] x – 3 if 1 < x < 1
] 4 2
\
SPECTRUM ALL-IN-ONE JOURNAL FOR ENGINEERING STUDENTS SIA Group
QP.2 MatheMatics-ii [JNtU-aNaNtapUr]

Unit-III
3

Find the Fourier transform of f(x) = ( #


1, |x |< 1 sinx
6.
0, |x |> 1
Hence deduce that x dx (Unit-III, Topic No. 3.2)
0
OR
2 x2
7. Find the Fourier transformation of e –a , a > 0. (Unit-III, Topic No. 3.2)

Unit-IV
8. (a) Form the partial differential equation by eliminating the arbitrary functions f(x2 + y2, z–xy) = 0.
(Unit-IV, Topic No. 4.1)
(b) The ends A and B of a rod 20 cm long have the temperature at 30°C and 80°C until steady state prevails. The
temperature of the ends is changed to 40°C and 60°C respectably. Find the temperature distribution in the rod at
time t. (Unit-IV, Topic No. 4.3)
OR
9. (a) Find the partial differential equation by eliminating the arbitrary constants from:
(x – a)2 + (y – b)2 + z2 = c2. (Unit-IV, Topic No. 4.1)
(b) A tightly stretched string of length 1 with fixed ends is initially in equilibrium position. It is set vibrating by giving
px
each point a velocity v0sin3 ` 1 j . (Unit-IV, Topic No. 4.3)

Unit-V
z2
10. (a) Evaluate Z-transformation of (z – 1) (z – 3) using convolution theorem. (Unit-V, Topic No. 5.5)

1 1
(b) Using Z-transforms solve yn + 4 yn – 1 = un + 3 un–1 where un is a unit step sequence. (Unit-V, Topic No. 5.6)
OR
z2
11. (a) Evaluate Z-transformation of using convolution theorem. (Unit-V, Topic No. 5.5)
(z – 1) 3
(b) Using Z-transforms solve Un+2 – 2Un+1 + Un = 3n + 5. (Unit-V, Topic No. 5.6)

L o o k f o r t h e SIA Group logo o n t h e TITlE CovER before you buy


May-2018 (R15) Question Paper with Solutions QP.3

solUTioNs To MAy-2018, R15, QP


Part-a
Q1. (a) Find the laplace transform of cos2t.
Answer : May-18, (R15), Q1(a)
Given function is,
cos2t
1 + cos 2t 0
L{cos2t} = L & 2
1
= 2 [L{1 + cos2t}]

1
= 2 [L{1} + L{cos2t}]

1 1 s 0
= 2 &s + 2
s +4
1 RS s 2 + 4 + s 2 WVW
= 2 SSS
s (s 2 + 4) WW
T X
1 RSS 2s 2 + 4 VWW
= 2 SS
s (s 2 + 4) WW
T X
2
2 (s 2) +
=
2s ( s 2 + 4 )
s2 + 2
=
s ( s 2 + 4)
s2 + 2
\ L{cos2t} =
s ( s 2 + 4)
s
(b) Find the inverse laplace transform of .
s 2 + s + 13
Answer : May-18, (R15), Q1(b)
Given function is,
s
s 2 + s + 13
Z] _
]] s + 1 – 1 bbb
L $ 2
–1 s
. = L [] J
–1 2 2 `
]] KK s + 1 OON + 51 bbb
2
s + s + 13
\ L 2P 4 a
Z] 1 1 _b
]] s+ 2 bb
= L–1 [] J 1 N2 51 – J 1 N2 51 `b 2
]] KK s + OO + KK s + OO + b
\L 2P 4 L 2P 4 ba
]Z] 1 b_b ]Z] 1 b_b
] s+ 2 b ] 2 b
= L [] J 1 N2 51 `b – L [] J 1 N2 51 `b
–1 –1
]] KK s + OO + b ]] KK s + OO + b
\L 2P 4 ba \L 2P 4 ba
Z] _b
]] + 12 bb
= L–1 ][]
s b 1 L–1 ]Z]] 1 b_b
2` – 2b
J
]] K 1 O KKN 2 J 51 N
O b b 2 [ J N J
]] KK s + 1 OO + KKK
2
51 N
OO `b
] K s + 2 O + K 2 OO bb ] 2 2 O bb
\L P L P a \L P L P a
RS VW
2 0 = e
SSa L –1 & s+a
J N
–at
cos b t WW
–1 51 1 K 1 – 1 51 O SS + 2
(s a ) b + WW
= e 2 t cos 2 t – 2 KK e 2 t sin 2 t OO SS –1 W
SS L ' (s + a) 2 + b 2 1 = b e sin btWWW
KK 51 OO 1 1 –at

L 2 P T X
SPECTRUM ALL-IN-ONE JOURNAL FOR ENGINEERING STUDENTS SIA Group
QP.4 MatheMatics-ii [JNtU-aNaNtapUr]
–t 51 1 –1 51
= e 2 cos 2 t – e 2 t sin 2 t
51
R V
–t S 51 W
= e 2 SSScos 2 t – sin 2 t WWW
51 1
S 51 W
T X
RS VW
. = e 2 SSScos 2 t – 1 sin 2 t WWW .
s – 51 51
L–1 $ 2
t
\
s + s + 13 S 51 W
T X
(c) Find the half-range sine series for the function f(t) = t – t2, 0 < t < 1.
Answer : May-18, (R15), Q1(c)
For answer refer Unit-II, Q31.
(d) obtain the Fourier series for f(x) = x in the interval (0, 2).
Answer : May-18, (R15), Q1(d)
Given function is,
f(x) = x, (0, 2)

The Fourier series of f(x) in the interval (0, 2L) is given as,

/ /
3 3
a n px n px
f(x) = 20 + an cos ` L j + b n sin ` L j
n=1 n =1

Here, L = 1
2L
1
a0 = L # f (x) dx
0

2
1
= 1 # x dx
0

J x2 N 2
= KK 2 OO
L P0
1
= 2 (22 – 02)

=2
2L
1
an = L # f (x) cos n px d x
0
2
1
= 1 # x cos npx dx
0
2 2

=x # cos npx dx – # d
dx (x) # cos npx dx dx
0 0

RS J sin npx NVW 2 2

= SSx KK np OOWW – # (1)KJK sinnnppx ONO dx


T L PX 0 0 L P
2
1 1
= np [2 sin np(2) – 0 sin np(0)] – np # sin npx dx
0

1 1 – cos npx 2
= np [2(0) – 0] – np ` np j 0

= np (0) + 2 2 6cos npx@ 02


1 1
np
1
=0+ [cos 2np – cos np(0)]
n2 p2

1
= (1 – 1) = 0
n2 p2

L o o k f o r t h e SIA Group logo o n t h e TITlE CovER before you buy


May-2018 (R15) Question Paper with Solutions QP.5
2L
1
bn = L # f (x) sin npx dx
0
2
1
= 1 # x sin npx dx
0

2 2

=x # sin npx dx – # d
d x (x ) # sin npx dx dx
0 0

2
R – cos npx WV 2
= SSx ` np jW 0 – # (1)` – cosnpnpx j dx
T X 0

2
–1 1
= np [2cos 2np – 0 cos np(0)] + np # cos npx dx
0

1 1 JK sin npx NO 2
= – np [2(1) – 0] + np K O
L np P 0
1 1
= – np (2) + 2 2 (sin np(2) – sin np(0))
np
2 1
= – np + 2 2 (0 – 0)
np
2
= – np

Substituting the values of a0, an and bn in f(x),


JK –2 NO
/ /
3 3
2 npx npx
⇒ f(x) = 2 + (0) cos ` 1 j + K np O sin ` 1 j
n=1 n=1 L P
/ sin npx .
3
2
⇒ f(x) = 1 – np
n=1

(e) State convolution theorem of the inverse transforms.

Answer : May-18, (R15), Q1(e)


Let the Fourier transform of f(t) and g(t) be F(w) and G(w) respectively.
i.e., F[f(t)] = F(W)
F[g(t)] = G(W)
Then, by convolution theorem,
f * g = F–1{F(W). G(W)}
2
(f) Find the Fourier transformation of e–x .
Answer : May-18, (R15), Q1(f)
Given function is,
f(x) = e–x2
From the definition of Fourier transform,
3
F{f(x)}= # f (x) .eisx dx
–3
3
= # 2
e –x .e isx dx
–3
3
= # e –x
2
+isx
dx
–3

SPECTRUM ALL-IN-ONE JOURNAL FOR ENGINEERING STUDENTS SIA Group


QP.6 MatheMatics-ii [JNtU-aNaNtapUr]
3
= # e – (x
2
–isx)
dx
–3
3
# e –;(x) – (2)(x)b 2 l + b 2 l –b 2 l E
2 is is 2 is 2
= dx
–3
3
–;(x) 2 – (2)(x)b i s l + b i s l E + b 2 l
is 2
#
2

= e 2 2 dx
–3
3 2
s l2
= # e –b x – 2 l + i
is 2b
2 dx
–3
3 2 s 2
= # e – b x – 2 l .e i
is 2b 2 l
dx
–3
s 2 3
–b 2 l 2

= e # e –b x – 2 l dx
is

–3
is
Let, x – 2 = t
⇒ dx – 0 = dt
⇒ dx = dt
is
U.L : If x = + ¥, ( ¥) – 2 = t Þ t = ¥

is
L.L : If x = – ¥, (– ¥) – 2 = t Þ t = – ¥
Substituting the corresponding values in above integral,
3
s 2
F{f(x)} = e –b 2 l
# 2
e –t .1dt
–3

2
3
= e –b 2 l
s
# 2
e –t dt
–3

>a e –x dx = p H
3

=e
s 2
–b 2 l
^ ph # 2

–3
s2
= p .e – 4

s2
F{e–x } = p .e – 4
2
\
(g) Solve py3 + qx2 = 0 by method of separation of variables.

Answer : May-18, (R15), Q1(g)


Given equation is,
py3 + qx2 = 0 ... (1)
Let the solution of equation (1) be,
z = XY ... (2)
Partially differentiating equation (2) with respect to x and y.
2z
2x = X'Y ⇒ p = X'Y

2z
2y = XY' ⇒ q = XY'

L o o k f o r t h e SIA Group logo o n t h e TITlE CovER before you buy


May-2018 (R15) Question Paper with Solutions QP.7
Substituting p and q values in equation (1)

(X'Y)y3 + (XY')x2 = 0

⇒ X'Yy3 = – XY'x2

X' 1 – Y' 1
⇒ X . x2 = Y . y 3

Each side is equated to constant k.


X' 1 – Y' 1
& X . 2 = k Y . y3 = k
x
X'
& X = k x2 Y ' = – k y 3
Y
Integrating on both sides,
KJ y ON
4
J x3 N log Y = – k KK 4 OO + log C2
log X = k KK 3 OO + log C 1
L P L P
kx3 – ky 4
& log X – log C 1 = 3 log Y – log C2 = 4
kx3 – ky 4
& log (X/C1) = 3 log (Y/C2) = 4
X kx3 – ky 4
&C =e3 Y
C =e
4
1 2
kx3 – ky 4
& X = C1 e 3
& Y = C2 e 4

Substituting the values of X and Y in equation (2)


kx3 –ky 4
z = C1 e 3 C2 e 4

JK 3 N
KK x y 4 OO
– 4 OO
= A e kKL 3 P [a C1 C2 = A]
\ The solution is,
KJK x3 y 4 OON
K – 4 OO
z = A e kKL 3 P

2
x2 y
(h) Form the partial differential equation by eliminating the arbitrary constans 2z = + .
a2 b2
Answer : May-18, (R15), Q1(h)

For answer refer Unit-IV, Q11(b).

(i) Find the Z-transformation of (nan).

Answer : May-18, (R15), Q1(i)

For answer refer Unit-V, Q6.


1
(j) Find the inverse Z-transformation of z – 2 .

Answer : May-18, (R15), Q1(j)

Given function is,


1
z–2
RS 1 VW RS –1 RS 1 VW V
z–1 SS z – 2 WW = 2n – 1 u(n – 1) Sa z S z – a W = a n –1 u (n – 1)WW
T X T T X X
2n
= 2 u(n – 1)
RS 1 VW 2 n
\ z–1 SS z – 2 WW = 2 u(n – 1)
T X
SPECTRUM ALL-IN-ONE JOURNAL FOR ENGINEERING STUDENTS SIA Group
QP.8 MatheMatics-ii [JNtU-aNaNtapUr]
Part-B
Unit-I
Z]
]]Esin wt, 0 < t < p/w
Q2. (a) Find the laplace transform of the function f(t) = ][ p having period 2p/w.
]]0, w < t < 2 p /w
Answer : \ May-18, (R15), Q2(a)

Given function is,


Z]
]] E sin wt, 0 < t < p
] w
f(t) = ][ 2p
]]0, p
] w <t< w
\
2p
Period, T = w

The Laplace transform for periodic functions is given as,

L{f(t)} =
1
1 – e –sT
# f (t) e –st dt
0

2p/w

=
1
1 – e –s(2p/w)
# f (t) e –st dt
0

RS p/w 2p/w VW
SS WW
=
1
–2ps
1– e w
SS # f ( t ) e –st
dt + # f ( t ) e –st
dt WW
0 p/w
T X
RS p/w 2p/w VW
SS W
=
1
–2ps
1– e w
#
SS E sin wt e dt +
–st
# 0.e dtWW
–st

W
0 p/w
T X
RS p/w VW
SS W
=
1
–2ps
1– e w
# SS e sin wt dt + 0WWW
–st

0
T X
p/w

=
E
–2ps
1– e w
# e –st sin wt dt
0

RS e –st VW p/w R VW
=
E
–2ps
1– e w
SS 2
s +w 2 (– s sin wt – w cos wt)WW0 SSSa #e –ax
sin bx dx =
e –ax
2
a +b 2 (– a sin bx – b cos bx)WW
T X T X

E RSRS e –s(p/w) VW RS e –s(0) VWVW


= –2ps SSSS 2 2 (– s sin w (p/w) – w cos w (p/w))WW – SS 2 2 (– s sin w (0) – w cos w (0))WWWW
1– e w s +w s +w
TT X T XX

E RSRS e –sp/w VW RS 1 VWVW


= –2ps SSSS 2 (– s sin p – w cos p)WW – SS 2 (– s sin (0) – w cos (0))WWWW
1– e w s + w2 s + w2
TT X T XX

E RSRS e –sp/w VW RS 1 VWVW


= –2ps SSSS 2 2 (– s (0) – w (–1))WW – SS 2 2 (– s (0) – w (1))WWWW
1– e w s +w s +w
TT X T XX

E RS e –sp/w 1 VW
= –2ps SS 2 2 (w ) – 2 2 (–w)WW
1– e w s +w s +w
T X

=
E
–2ps ` 2
w
j 6e –sp/w + 1@
1– e w s + w2

L o o k f o r t h e SIA Group logo o n t h e TITlE CovER before you buy


May-2018 (R15) Question Paper with Solutions QP.9
–sp/w
Ew (1 + e )
= [a(1 – e–2a) = (1 + e–a) (1 – e–a)]
s + w (1 – e –sp/w) (1 + e –sp/w)
2 2

Ew
=
(s 2 + w 2) (1 – e –sp/w)

Ew
\ L{f(t)} =
(s 2 + w 2) (1 – e –sp/w)
1
(b) Find the laplace transform of t (eat – ebt)

Answer : May-18, (R15), Q2(b)


Given function is,
1 at bt
t (e – e )
Let f(t) = eat – ebt
3 RS 3 VW
SS W
1
L $ t (e at – e bt) . = # fr(s) ds
f ( t)
SSa L & t 0 = # fr(s) dsWW
W
T X
s s
3

= # L {e at – e bt} ds
s
3

= # [L {e } – L {e }] ds
at bt

s
3

= # JKK s 1– a – s 1– b NOO ds
s L P
3 3

= # 1
s – a ds – # 1
s – b ds
s s

= log (s – a) – log (s – b) 3s
3
s

s–a 3
= log ` s – b j s
s–a
= 0 – log ` s – b j
s–a
= – log ` s – b j
Js – bN
= log KK s – a OO
L P
1 JK s – b NO
\ L $ t (e – e ) . = log K s – a O
at bt

L P
oR
Q3. Solve by laplace transform method y'' – 3y' + 2y = 4, where y(0) = 2, y'(0) = 3.
Answer : May-18, (R15), Q3
Given differential equation is,
y'' – 3y' + 2y = 4.
Applying Laplace transform on both sides,
L{y'' – 3y' + 2y] = L[4]
4
⇒ L{y''} – 3L{y'} + 2L{y} = s
4
⇒ [s2 L{y} – sy(0) – y'(0)] – 3[s L{y} – y(0)] + 2L{y} = s

SPECTRUM ALL-IN-ONE JOURNAL FOR ENGINEERING STUDENTS SIA Group


QP.10 MatheMatics-ii [JNtU-aNaNtapUr]
4
⇒ [s2 L{y} – s(2) – 3] – 3[s L{y} – 2] + 2L{y} = s

4
⇒ s2 L{y} – 2s – 3 – 3s L{y} + 6 + 2L{y} = s

4
⇒ s2 L{y} – 3s L{y} + 2L{y} = s + 2s + 3 – 6

4
⇒ L{y} (s2 – 3s + 2) = s + 2s – 3

4 + 2s 2 – 3s
⇒ L{y} (s2 – 3s + 2) = s
2s 2 – 3s + 4
⇒ L{y} =
s (s 2 – 3s + 2)

2s 2 – 3s + 4
⇒ L{y} = s (s – 1) (s – 2) ... (1)

Applying partial fractions,

2s 2 – 3s + 4 A B C
s (s – 1) (s – 2) = s + s – 1 + s – 2 ... (2)

2s 2 – 3s + 4 A (s – 1) (s – 2) + B s (s – 2) + C s (s – 1)
⇒ s (s – 1) (s – 2) = s ( s – 1 ) (s – 2 )
⇒ 2s2 – 3s + 4 = A(s – 1) (s – 2) + Bs(s – 2) + Cs(s – 1) ... (3)
Substituting s = 0 in equation (3),
2(0)2 – 3(0) + 4 = A(0 – 1) (0 – 2) + B(0) (0 – 2) + C(0) (0 – 1)
⇒ 4 = 2A + 0 + 0
⇒ A = 2.
Substituting s = 1 in equation (3),
2(1)2 – 3(1) + 4 = A(1 – 1) (1 – 2) + B(1) (1 – 2) + C(1) (1 – 1)
⇒ 2 – 3 + 4= 0 – B + 0
⇒ B=–3
Substituting s = 2 in equation (3),
2(2)2 – 3(2) + 4 = A(2 – 1) (2 – 2) + B(2) (2 – 2) + C(2) (2 – 1)
⇒ 8 – 6 + 4 = 0 – 0 + 2C
⇒ C=3
Substituting the values of A, B and C in equation (2),
2s 2 – 3s + 4 2 3 3
s (s – 1) (s – 2) = s – s – 1 + s – 2 ... (4)

Substituting equation (4) in equation (1),


2 3 3
L{y} = s – s – 1 + s – 2

Applying inverse Laplace transform on both sides,


2 3 3
L–1[L{y}] = L–1 & s – s – 1 + s – 2 0
2 3 3
⇒ y = L–1 $ s . – L–1 & s – 1 0 + L–1 & s – 2 0
1 1 1
= 2L–1 $ s . – 3 L–1 & s – 1 0 + 3L–1 & s – 2 0
= 2(1) – 3(et) + 3(e2t)
= 2 – 3et + 3e2t
\ y = 2 – 3et + 3e2t

L o o k f o r t h e SIA Group logo o n t h e TITlE CovER before you buy


May-2018 (R15) Question Paper with Solutions QP.11
Unit-II
Q4. (a) Find the Fourier series to represent (x – x2) from x = –p to x = p.
Answer : May-18, (R15), Q4(a)
For answer refer May/June-16, Q4, (Excluding Deduction).
(b) Find the half range cosine series for the function f(x) = x2 in the range 0 ≤ x ≤ p.
Answer : May-18, (R15), Q4(b)
Given function is,
f (x) = x2 in [0, p]
The half-range cosine series of a function is given as,
a0 3 nπ x
f(x) = + Σ an cos ... (1)
2 n=1 l
Here,
l=p

∑a
a0
f(x) = x = 2 +
2 n cos n x ... (2)
n =1

Where,
π
2
a0 =
π ∫ f ( x)dx
0

π π
2 2 2  x3 
a0 =
π
0

x dx =  
π  3 
0

2  π3 − 0  2π 3
=   =
π  3  3π

π π
2 2 2
an =
π0 ∫
f ( x) cos nx dx ===
π0
x cos nx dx ∫
π
2 2 d 2  
=
π ∫  dx ∫
 x cos nxdx −  x cos nxdx dx 
 0 ∫
π
2  2 sin nx sin nx 
=
π x
n
− 2x
n ∫ dx  =
0
π
2  2 sin nx cos nx sin nx 
=  x + 2x 2 − 2 3 
π n n n 0

2   2 sin nπ cos nπ sin nπ   2sin n(0)  


=  π + 2π − 2 3  − 0 + 0 − 
π   n n 2
n   n3 

2  cos n π  4
2π = cos n π
π  n 2  n 2
=

4
∴ an = ( −1) n
n2
SPECTRUM ALL-IN-ONE JOURNAL FOR ENGINEERING STUDENTS SIA Group
QP.12 MatheMatics-ii [JNtU-aNaNtapUr]
Substituting a0 and an in equation (2),

π2 ∞
(− 1) cos n x n
2
x =
3
+ 4 ∑
n =1 n
2

oR

Q5. (a) Find the complex form of the Fourier series of f(x) = e–x in – 1 ≤ x ≤ 1

Answer : May-18, (R15), Q5(a)

For answer refer Unit-II, Q39.


]Z] 1 1
]] 4 – x if 0 < x < 2
(b) Expand f(x) = ] [ as the Fourier series of sine terms.
]] x – 3 if 1 < x < 1
] 4 2
Answer : \ May-18, (R15), Q5(b)

For answer refer Unit-II, Q29.

Unit-III
3

Find the Fourier transform of f(x) = (


1, | x | < 1 sinx
Q6.
0, | x | > 1
Hence deduce that # x dx
0
Answer : May-18, (R15), Q6

Given function is,

f (x) = (
1, | x | < 1
0, | x | > 1
The Fourier transform of the function f(x) is,
3

F[f(x)] = #e isx
f (x) dx
–3

–1 1 3

⇒ F(s) = # isx
e f (x) dx + #e isx
f (x) dx + #e isx
f (x) dx
–3 –1 1

⇒ F(s) = 0 + #e isx
(1) dx + 0
–1

⇒ F(s) = #e isx
dx
–1

RS eisx VW1
⇒ F(s) = SS is WW
T X
–1

eis – e –is
⇒ F(s) = is

2
= is sinh(is) SRSa sinh x = e x – e –x WVW
T 2 X

2
= is (i sin s) 6a sinh (ix) = i sin x@

2 sin s
= s

2 sin s
\ F(s) = s

L o o k f o r t h e SIA Group logo o n t h e TITlE CovER before you buy


May-2018 (R15) Question Paper with Solutions QP.13
Deduction
By inversion formula,
3
1
f(x) = 2p # F (s) .e –isx ds
–3

3
1
= 2p # 2 sin s
s (cos sx – i sin sx)ds
–3

3 3
1
= 2p # 2 sin s i
s cos sx ds – 2p # 2 sin s
s sin sx ds
–3 –3

3
1
= 2p # sin s
s cos sx ds – 0 [a sin sx is an odd function]
–3

3
1
= p # sin s
s cos sx ds
–3

⇒ p f(x) = # sin s
s cos sx ds
–3

⇒ p( #
1, | x | < | sin s
= s cos sx ds
0, | x | > | –3

( #
p, | x | < | sin s
⇒ = s cos sx ds
0, | x | > | –3

At x = 0, p = # sin s
s cos s(0) ds
–3
3

⇒ p=2 # sin s
s (1) ds
0
3


p
2 = # sin s
s ds
0
3


p
2 = # sin x
x dx
0
3

\ # sin x p
x dx = 2
0

oR
2 2
Q7. Find the Fourier transformation of e–a x , a > 0.
Answer : May-18, (R15), Q7
Given function is,
f(x) = e–a2x2
From the definition of Fourier transform,
3
F{f(x)}= # f (x) .eisx dx
–3
3
= # e –a
2 2
x
.eisx dx
–3
3
= # e –a
2 2
x + isx
dx
–3

SPECTRUM ALL-IN-ONE JOURNAL FOR ENGINEERING STUDENTS SIA Group


QP.14 MatheMatics-ii [JNtU-aNaNtapUr]
3 (b) The ends A and B of a rod 20 cm long have
= # e – (a
2 2
x –isx)
dx the temperature at 30°C and 80°C until
–3 steady state prevails. The temperature
3 of the ends is changed to 40°C and
# e –;(ax) –(2)(ax)b 2a l + b 2a l –b 2a l E
2 is is 2 is 2
= dx 60°C respectably. Find the temperature
–3 distribution in the rod at time t.
3
# e –;(ax) –(2)(ax)b 2a l + b 2a l + b 2a l E
2 is is 2 is 2 Answer : May-18, (R15), Q8(b)
= dx
–3 For answer refer Unit-IV, Q40.
3 2
s l2
= # e –b ax – 2a l + i
is 2b
2a dx oR
–3
Q9. (a) Find the partial differential equation by
3 eliminating the arbitrary constants from:
2 s 2
= # e –b ax – 2a l .ei
is 2b 2a l
dx (x – a)2 + (y – b)2 + z2 = c2.
–3
Answer : May-18, (R15), Q9(a)
s 2 3
–b 2a l 2

= e # e –b ax – 2a l dx
is
Given equation is,
–3
(x – a)2 + (y – b)2 + z2 = c2 ... (1)
is
Let, ax – 2a = t
Differentiating equation (1) partially with respect to x,
adx – 0 = dt 2z
2(x – a) + 0 + 2z 2x = 0
adx = dt JK 2z N
⇒ 2(x – a) + 2zp = 0 Ka p = 2x OO
1 L P
dx = a dt
⇒ 2(x – a + zp) = 0
is
U.L : If x = + ¥, a( ¥) – 2a = t Þ t = ¥ ⇒ (x – a) + zp = 0
is
L.L : If x = – ¥, a( ¥) – 2a = t Þ t = – ¥ ⇒ x – a = – zp ... (2)
Differentiating equation (1) partially with respect to y,
Substituting the corresponding values in above integral,
3 2z
s 2 0 + 2(y – b) + 2z 2y = 0
F{f(x)} = e –b 2a l
# 1
–t 2
e . a dt
JK 2z N
–3 ⇒ 2(y – b) + 2zq = 0 Ka q = 2y OO
L P
2
⇒ 2(y – b + zq) = 0
e –b 2a l
s 3
= a # 2
e –t dt
⇒ (y – b) + zq = 0
–3
⇒ y – b = – zq ... (3)
>a dx = p H
2 3
e –b 2a l ^ p h
s

= # e –x 2
Substituting equations (2) and (3) in equation (1).
a –3
(– zp)2 + (– zq)2 + z2 = c2
p s2
= a .e – 4a2 ⇒ z2 p2 + z2 q2 + z2 = c2
⇒ z2 (p2 + q2 + 1) = c2
a2x2 p –s 2
\ F{e– } = a 4a 2 \ The partial differential equation is,
e
z2 (p2 + q2 + 1) = c2 .
Unit-IV
(b) A tightly stretched string of length 1
Q8. (a) Form the partial differential equation by with fixed ends is initially in equilibrium
eliminating the arbitrary functions f(x2 + position. It is set vibrating by giving each
px
y2, z – xy) = 0. point a velocity v0sin3 ` 1 j .

Answer : May-18, (R15), Q8(a) Answer : May-18, (R15), Q9(b)

For answer refer Unit-IV, Q23. For answer refer Unit-IV, Q35.

L o o k f o r t h e SIA Group logo o n t h e TITlE CovER before you buy


May-2018 (R15) Question Paper with Solutions QP.15
Unit-V
z2
Q10. (a) Evaluate Z-transformation of using convolution theorem.
(z – 1) (z – 3)
Answer : May-18, (R15), Q10(a)

For answer refer Unit-V, Q34.


1 1
(b) Using Z-transforms solve yn + 4 yn – 1 = un + 3 un–1 where un is a unit step sequence.

Answer : May-18, (R15), Q10(b)


Given equation is,
1 1
yn + 4 yn – 1 = un + 3 un – 1

Applying Z-transform on both sides


1 1
Z & yn + 4 yn –1 0 = Z &u n + 3 u n –1 0

1 1
⇒ Z{yn} + 4 Z{yn –1} = Z{un} + 3 Z{un –1}

1 Y 1 J 1 N
Y + 4 ` z j = z – 1 + 3 KK z – 1 OO
z

L P
J 1 N 1
Y KK1 + 4z OO = z – 1 + 3 (z – 1)
z

L P
J 4z + 1 N 3z + 1
⇒ Y KK 4z OO = 3 (z – 1)
L P
(3z + 1) 4z
⇒ Y = 3 (z – 1) (4z + 1)

(3z + 1) 4z
⇒ Y= J 1N
3 (z – 1) 4 KK z + 4 OO
L P
z (3z + 1)
⇒ Y= J 1N ... (1)
3 (z – 1) KK z + 4 OO
L P
Applying partial fractions,
3z + 1 A B
JK 1 N = z –1 +
O 1 ... (2)
(z – 1) K z + 4 O z+ 4
L P
J 1N
⇒ (3z + 1) = A KK z + 4 OO + B(z – 1) ... (3)
L P
–1
Substituting z = 4 in equation (3),

J –1 N J –1 1 N J –1 N
3 KK 4 OO + 1 = A KK 4 + 4 OO + B KK 4 – 1OO
L P L P L P
–3 5B
⇒ 4 +1=0– 4
1 – 5B
⇒ 4 = 4
–1
⇒ B= 5

Substituting z = 1 in equation (3)

SPECTRUM ALL-IN-ONE JOURNAL FOR ENGINEERING STUDENTS SIA Group


QP.16 MatheMatics-ii [JNtU-aNaNtapUr]
J 1N JK z 2 z NO
3(1) + 1 = A KK1 + 4 OO + B(1 – 1) z 3
Z–1 ` z – 1 j = Z–1 KK
L P ( z – 1 ) 2 .
z – 1 OO
L P
5
⇒ 3 + 1 = A `4j + 0 = [1n.(n + 1)]1n

/1
n
5A
⇒ 4= 4 = m
(m + 1) 1 n–m
m=0

/ (m + 1)
n
16
A= 5 = 1n
m=0

Substituting the values of A and B in equation (2) = 1n (1 + 2 + 3 + - - - (n + 1))

3z + 1 16 1 (n + 1) (n + 2)
=
J 1 N = 5 (z – 1) – JK + 1 ON 2
(z – 1) KK z + 4 OO 5K z 4 O
L P L P z 3 (n + 1) (n + 2)
Substituting the above value in equation (1), \ Z–1 ` z – 1 j = 2
RS 16 VW (b) Using Z-transforms solve Un+2 – 2Un+1 + Un
z SS 1 W
Y= 3 –
SS 5 (z – 1) 5 KK z + 1 OO WWW
J N = 3n + 5.
L 4P
T X Answer : May-18, (R15), Q11(b)
R V
⇒ Y = 15 SSS (z – 1) –
1 16z 1 WW
SS 1W For answer refer May/June-17, Q11.
z + 4 WW
T X
Applying inverse Z-transform on both sides,

R NOWVW
1 S JK 16z NOO –1 JKK z
Z–1(y) = 15 SSSZ –1 KK O– Z K OOW
SS L z – 1 P KK 1 OOWW
S z+ WW
L 4 PX
T
1 RS J –1 NnVW
\ yn = 15 SS16 (1) – KK 4 OO WW .
T L PX
oR
z3
Q11. (a) Evaluate Z-transformation of
(z – 1) 3
using convolution theorem.

Answer : May-18, (R15), Q11(a)

Given function is,


z3
(z – 1) 3
z
Z–1 ` z – 1 j = 1n

z 2 z z
Then, Z–1 ` z – 1 j = Z–1 ` z – 1 . z – 1 j

= 1n.1n

/1
n

= m
.1 n–m
m=0

[a From convolution theorem]

/1
n

= 1n . m
.1 –m
m=0

= 1n (1 + 1 + 1 + . . . . .+1)

= 1n (n + 1)

L o o k f o r t h e SIA Group logo o n t h e TITlE CovER before you buy


Model Question Papers with Solutions MP.1

MODEL
R15
Jawaharlal Nehru Technological University Anantapur
B.Tech. I Year II Semester Examination
PAPER 1
MATHEMATICS-II
(Common to All Branches)

Time: 3 Hours Max. Marks: 70


PART - A
(20 MARKS)
Solutions
1. (a) Define the Laplace transform and state its domain and Kernel. (Unit-I / Q1)
(b) Find the Laplace transform of eat. (Unit-I / Q3)
(c) Give the Euler’s formulae for Fourier series. (Unit-II / Q2)
(d) Give the Fourier series expansion for odd periodic functions. (Unit-II / Q7)
(e) State Fourier integral theorem. (Unit-III / Q1)
(f) Find Fourier sine transform series of xe–ax. (Unit-III / Q9)
(g) Define order and degree with reference to partial differential equation. (Unit-IV / Q1)
(h) Form partial differential equation by eliminating the arbitrary
1 2 2y
constants z = axey + a e + b. (Unit-IV / Q4)
2

(i) State and prove the linearity property of Z-transform. (Unit-V / Q4)
(j) State and prove convolution theorem. (Unit-V / Q10)

PART - B
(50 MARKS)

UNIT - I
2. Prove that,

a
(a) L{sin at} = ,s>a
s + a2
2

s
(b) L{cos at} = , s > a. (Unit-I / Q15)
s2 + a 2
OR
3. Using Laplace transform, solve (D2 + 4D + 5)y = 5, given that y(0) = 0, y"(0 ) = 0. (Unit-I / Q54)
UNIT - II

 -1
 2 ( π + x) for - π ≤ x ≤ 0
Q4. Find the Fourier series in [–π, π] for the function f(x) =  . (Unit-II / Q16)
 1 ( π - x) for 0 ≤ x ≤ π
 2

OR
5. Find the half-range sine series of f(x) = (x – 1)2 in the interval (0, 1). (Unit-II / Q28)

SPECTRUM ALL-IN-ONE JOURNAL FOR ENGINEERING STUDENTS SIA GROUP


MP.2 MATHEMATICS-II [JNTU-ANANTAPUR]

UNIT - III


cos xt π –|x|
6. Find the Fourier transforms of f(x) = e –|x|
and deduce that ∫ 1+ t
0
2
dt =
2
e .

4s
Hence show that F(xe–|x|) = i . (Unit-III / Q23)
(1+ s 2 )2

OR

π x2
7. Find the finite cosine transform of, f(x) = – x+ , 0 < x < π. (Unit-III / Q35)
3 2π

UNIT - IV
8. Form the PDE by eliminating ‘f’ from f(x + y + z, x2 + y2 + z2) = 0. (Unit-IV / Q24)
OR
9. Find the deflection u(x, y, t) of the square membrane with a = b = 1 and c = 1,
if the initial velocity is zero and the initial deflection is f(x, y) = A sin πx sin 2πy. (Unit-IV / Q37)
UNIT - V

 1 
10. Find Z–1  3
when | z | > 5. Determine the region of convergence. (Unit-V / Q17)
 (z – 5) 

OR
11. Find the linearity property of Z-transforms,

 nπ π 
(i) cos  + 
 2 4

 nπ 
(ii) cosh  + θ . (Unit-V / Q24)
 2 

Look for the SIA GROU P LOGO on the TITLE COVER before you buy
Model Question Papers with Solutions MP.3

MODEL
R15 Jawaharlal Nehru Technological University Anantapur
B.Tech. I Year II Semester Examination
PAPER 2
MATHEMATICS-II
(Common to All Branches)

Time: 3 Hours Max. Marks: 70

PART - A Solutions
(20 MARKS)

1. (a) Define second shifting theorem. (Unit-I / Q8)

 3(s2 – 2)2 
(b) Find the inverse Laplace transformation of  5 . (Unit-I / Q12)
 2s 

(c) Give the Dirichlet conditions for Fourier expansion. (Unit-II / Q3)
(d) Give the Fourier series expansion for even periodic functions. (Unit-II / Q6)
(e) State the properties of Fourier transform. (Unit-III / Q5)

1
(f) Find Fourier sine transform of . (Unit-III / Q10)
x

∂ u ∂2 u
(g) Write the three possible solutions of = . (Unit-IV / Q9)
∂ t ∂ x2
(h) Form partial differential equation by eliminating the arbitrary constants
z = ax + by + a2 + b2. (Unit-IV / Q2)

az
(i) Show that, Z(nan) = . (Unit-V / Q6)
(z - a)2

 z2 + 2 
(j) Find Z–1  2. (Unit-V / Q9)
 (z – 1) 

PART - B
(50 MARKS)

UNIT - I
2. Find L–1[(2s + 3)/(s3 – 6s2 + 11s – 6)]. (Unit-I / Q20)
OR

3. Prove that,

b
(i) L[eat sinh bt] =
(s – a)2 – b 2
s–a
(ii) L[eat cosh bt] = . (Unit-I / Q26)
(s – a)2 – b 2

SPECTRUM ALL-IN-ONE JOURNAL FOR ENGINEERING STUDENTS SIA GROUP


MP.4 MATHEMATICS-II [JNTU-ANANTAPUR]

UNIT - II

4. Obtain the Fourier series for the function f(x) = |cos x| in (–π, π). (Unit-II / Q19)

OR

l
1 2 ∞ 
5. Prove that ∫
[f(x)]2
dx = l  a0 +
 2 n=1

(anan + bbn ) provided the Fourier series for

–l

f(x) converges uniformly in (– l, l). (Unit-II / Q37)

UNIT - III
2
–x
6. Find the Fourier cosine transform of e . (Unit-III / Q25)

OR

7. Find the finite Fourier sine and cosine transform of f(x) = x (π – x) in 0 < x < π. (Unit-III / Q39)

UNIT - IV
8. Solve by separation of variables 3ux + 2uy = 0 with u(x, 0) = 4e– x. (Unit-IV / Q29)

OR

9. A tightly stretched string with fixed end points x = 0 and x = 1 is initially at rest in
its equilibrium position. If itisset on vibrating by giving each of its points a velocity
λx(1–x), find the displacement of the string at any distance x from end at any time ‘t’. (Unit-IV / Q35)

UNIT - V

2 z 2 + 4 z+12
10. If Z(un) = find u2. (Unit-V / Q30)
(z-1) 4

OR

 z2 
11. Evaluate Z–1   using convolution theorem. (Unit-V / Q34)
 (z – 1)(z – 3) 

Look for the SIA GROU P LOGO on the TITLE COVER before you buy
Model Question Papers with Solutions MP.5

MODEL
R15 Jawaharlal Nehru Technological University Anantapur
B.Tech. I Year II Semester Examination
PAPER 3
MATHEMATICS-II
(Common to All Branches)

Time: 3 Hours Max. Marks: 70


PART - A Solutions
(20 MARKS)
1. (a) Write about the existence of Laplace transform. (Unit-I / Q2)

(b) Using Laplace transform, evaluate ∫ te


0
–t
sin t dt. (Unit-I / Q6)

(c) Define even and odd function. (Unit-II / Q5)


(d) Define half-range Fourier cosine series of f(x). (Unit-II / Q10

1– cos πλ π
(e) Using Fourier integral show that, ∫ λ
sin(xλ)dλ =
2
, 0 < x < π. (Unit-III / Q3)
0
(f) Define inverse finite Fourier sine and cosine transforms. (Unit-III / Q16)
(g) Form the partial differential equation by eliminating the arbitrary
function from z = f(x2– y2). (Unit-IV / Q5)
(h) Write the boundary conditions for the following problem. A rectangular plate
is bounded by the line x = 0,y = 0 and y = b. Its surfaces are insulated.
The temperature along x = 0 and y = 0 are kept at 0ºC and the others are kept at 100ºC. (Unit-IV / Q10)
(i) Write the properties of multiplication by n and division by n of Z-transforms. (Unit-V / Q7)
(j) Find the Z-transform of,
(i) 2n + 3
(ii) cos 3n. (Unit-V / Q3)
PART - B
(50 MARKS)
UNIT - I
n n n–1
2. Show that L{f (t)} = s f(s) – s f(0) – s f' (0)..... fn– 1(0), where L{f (t)} = f (s).
n–2
(Unit-I / Q29)
OR
3. Find the Laplace transform of periodic function f(t) with period T,
4Et T 4Et T
where f(t) = – E, 0 ≤ t ≤ = 3E – , ≤ t ≤ T. (Unit-I / Q47)
T 2 T 2
UNIT - II

∑n
1
4. Obtain the Fourier series for the function, f(x) = x + x2 in [–π, π], deduce the value of 2
. (Unit-II / Q12)
n=1

OR

1 1
 4 - x , 0 < x < 2
5. Find the half-range sine series for f(x) =  . (Unit-II / Q29)
x - 3 , 1 < x < 1
 4 2

SPECTRUM ALL-IN-ONE JOURNAL FOR ENGINEERING STUDENTS SIA GROUP


MP.6 MATHEMATICS-II [JNTU-ANANTAPUR]

UNIT - III

6. ∫
Solve the integral equation f(x) cosαx dx= e–αx.
0
(Unit-III / Q20)

OR

1
7. Find the fourier sine transform of . (Unit-III / Q26)
x(x 2 + a 2 )

UNIT - IV
8. Obtain the partial differential equation by eliminating the arbitrary constants
(a) z = (x – a)2 + (y – b)2 + 1
2
(b) z = ae – b t
cos bx. (Unit-IV / Q12)
OR
9. A bar of length L is laterally insulated with its ends A and B kept at 0° and 100°
respectively until steady state condition is reached. The temperature at A is raised to 30°
and that at B is reduced to 80° simultaneously. Find the temperature in the rod at a later time. (Unit-IV / Q46)
UNIT - V

3z 2 + z
10. Find the inverse Z-transform of . (Unit-V / Q16)
(5z – 1)(5z – 2)

OR
11. Solve the difference equation, using Z-transforms un +2 – un = 2n where u0 = 0, u1 = 1. (Unit-V / Q43)

Look for the SIA GROU P LOGO on the TITLE COVER before you buy
List of Important Formulae L.1

LIST OF IMPORTANT FORMULAE

UNIT - I
1. Laplace Transform: Laplace transform is a mathematical tool that transforms a time-domain function to a frequency domain
function and vice-versa.
2. Shifting Theorem: Let f(t) be a function defined for all the positive values of ‘t’. Then, the first shifting theorem states that,

For L[f(t)]= f (s )

L[eat f(t)]= f ( s − a), where ( s − a) > 0

3. Unit Step Function: The unit step function or Heaviside unit function is defined as,

⎧ 0, t < a
u(t – a) or H(t – a) = ⎨
⎩1, t > a

Where, a > 0
4. Second Shifting Theorem: The second shifting theorem states that,

For L[f(t)] = f (s) and

⎧ f (t − a) ; t > a
g(t) = ⎨
⎩0 ; t<a

Laplace transform of g(t) is given by,

L[g(t)] = e–as f (s )

5. Dirac’s Delta Function: The Dirac’s delta function or unit impulse function can be defined as the limiting form of,

⎧ 0, t < a
⎪1

fε(t – a) = ⎨ , a ≤ t ≤ a + ε
⎪ε
⎪⎩ 0 , t > a

6. Laplace Transform of a Periodic Function: The Laplace transform of a periodic function is given by,

∫e
− st
f (t ) dt
0
L{f(t)} =
1 − e − st

Where, T – Period of f(t).


7. Linearity Property of Laplace Transform

If L[f(t)] = f (s) and

L[g(t)] = g (s ) , then

L [c1 f(t) + c2 g(t)] = c1 L[f(t)] + c2 L[g(t)]

= c1 f (s) + c2 g (s)

SPECTRUM ALL-IN-ONE JOURNAL FOR ENGINEERING STUDENTS SIA GROUP


L.2 MATHEMATICS-II [JNTU-ANANTAPUR]

UNIT - II
1. Periodic Function: A function f (x) defined for all real numbers is said to be periodic, if there is a real number T < 0 such that,
f (x + T) = f (x) for all x
2. Euler’s Formulae for Fourier Series: The Fourier series expansion of function f(x) in the interval k ≤ x ≤ k + 2π is given by,

a0 ∞
f(x) = +
2 n =1 ∑
(an cos nx + bn sin nx)

3. Dirichlet’s Conditions
1. f (x) is uniformly bounded on the fundamental interval (–π, π) of period 2π, that is, | f (x)| < M for all –π < x < π where M
is a constant.
2. f (x) has no more than a finite number of finite discontinuities in (–π, π).
3. f (x) has a finite number of strict maxima or minima in (–π, π).
4. Even Function: A function f(x) is said to be an even function if f(–x) = f(x)
5. Odd Function: A function f(x) is said to be an odd function if f(–x) = –f(x).
6. Half-Range Fourier Sine Series: The half-range Fourier sine series of f(x) defined in the interval (0, π) is represented
as,

f(x) = ∑ b sin nx
n =1
n

7. Half-Range Fourier Cosine Series: The half-range Fourier cosine series of f(x) defined in the interval (0, π) is
represented as,

∑a
a0
f(x) = + n cos nx
2 n=1

UNIT - III
1. Fourier integral theorem: The Fourier integral theorem of f(x) is defined as,
∞ ∞
1
f(x) =
π ∫ ∫ f (t ) cos λ(t − x)dt.dλ
0 −∞

2. Fourier Sine Integral: The Fourier sine integral of f(x) is defined as,
∞ ∞
2
f(x) =
π ∫
0

sin λx f (t ) sin λt dt dλ
0

3. Fourier Cosine Integral: The Fourier cosine integral of f(x) is defined as,
∞ ∞
2
f(x) =
π ∫
0

cos λx f (t ) cos λ t dt dλ
0

4. Fourier Transform of Function f(x): The Fourier transform of a function f(x) is defined as,

∫ f ( x) e
isx
F(s) = F.T{f(x)} = dx
−∞

5. Linear Property: If f(x) and g(x) are two functions whose Fourier transform are given as F(s) and G(s) respectively, then,
F[a f(x) + b g(x)] = a. F(s) + b G(s)
Where, a and b are arbitrary constants.

Look for the SIA GROU P LOGO on the TITLE COVER before you buy
List of Important Formulae L.3
6. Scaling Property: If f(x) is a function whose complex Fourier transform is given by F(s),

Then,

1 ⎛s⎞
F{f(ax)} = F⎜ ⎟
a ⎝a⎠

Where, a ≠ 0.

7. Shifting Property: If a function f(x) has a complex Fourier transform F(s),

Then,

F{f(x–a)} = eisa.F(s)

8. Modulation Property: If a function f (x) has a complex Fourier transform F(s), then,

1
F{f(x) cos ax} = [F(s + a) + F(s – a)]
2

9. Fourier Sine Transform: The Fourier sine transform of the function f(x) is given as,


Fs(S) = FST{f(x)} = ∫ f ( x) sin sx dx
0

10. Fourier Cosine Transform: The Fourier cosine transform of the function f(x) is defined as,

Fc(S) = FCT {f(x)} = ∫ f ( x) cos sx dx


0

11. Inverse Fourier Transform: The inverse Fourier transform of f(S) is defined as,


1
∫ F ( S )e
−isx
f(x) = I.F.T {F(S)} = ds

−∞

UNIT - IV
1. Order: The order of a partial differential equation is equal to the order of highest partial derivative for a particular equation.

2. Degree: The degree of a partial differential equation is equal to the power of highest partial derivative.

3. Wave Equation: The expression for one dimensional wave equation is given as,

∂2 y 2 ∂ y
2
= c
∂t 2 ∂x 2

4. Heat Equation: The general solution of the heat equation is given as,

∂u 2 ∂ u
2
=C
∂t ∂x 2

SPECTRUM ALL-IN-ONE JOURNAL FOR ENGINEERING STUDENTS SIA GROUP


L.4 MATHEMATICS-II [JNTU-ANANTAPUR]

UNIT - V
1. Linearity Property Transform: Let x, y, z are said to be any constants and un, vn, wn be any distinct functions then.

Z(xun + yvn – zwn) = xZ(un) + yZ(vn) – Z(wn)

2. Damping Rule (or) Change of Scale of Property: The two change of scale properties or damping rule is given as,

1. If Z(un) = u (z ) then Z(a–n un) = u (az)

⎛z⎞
2. If Z(un) = u (z ) , then z(an un) = u ⎜ ⎟
⎝a⎠

3. Initial Value Theorem of Z-transform: If z(f (n)) = f (z), then Lt f (z) = f (0)
z →∞

4. Final Value Theorem of Z-Transform: If zf (n) = f (z), then nLt


→∞
f (n) = Lt (z – 1)f (z)
z →1

5. Convolution Theorem Statement: If Z–1[f(z)] = f (α) and Z–1[g(z)] = g(α)

α
Then, Z–1[f(z).g(z)] = f (α)* g(α) = Σ f(β) g(α – β)
β=0

Look for the SIA GROU P LOGO on the TITLE COVER before you buy
UNIT-1 (Laplace Transform) 1.1

UNIT LAPLACE TRANSFORM

1
PART-A
SHORT QUESTIONS WITH SOLUTIONS
Q1. Define the Laplace transform and state its domain and Kernel.
Ans: Model Paper-I, Q1(a)
Laplace Transform: Laplace transform is a mathematical tool that transforms a time-domain function to a frequency domain
function and vice-versa.
Definition: Laplace transform of a function f(t) is a linear integral transform which is defined as,

f (s) = L{f(t)}

∫ f (t ).e
− st
= .dt
0

Where,
f(t) – Time-domain function
e–st – Kernel of the transform [k(s, t)]
L – Laplace transform operator
s – Complex variable = σ + jω

f (s) – Complex variable function.


Laplace transform helps in better understanding of both time-domain and frequency domain functions and their properties.
Q2. Write about the existence of Laplace transform.
Ans: Model Paper-III, Q1(a)
Existence of Laplace Transform: Laplace transform of any function f(t) exists, if it satisfies the following two conditions,
Condition 1: f(t) is Piece-wise Continuous: A function is said to be piece-wise continuous in an interval [a, b], if it is continuous
and has finite limits at the extreme points, in any sub-interval range of [a, b].
Condition 2: f(t) is of Exponential Order ‘b’: A function, f(t) is said to be of exponential order ‘b’, if there exists ‘M’ and ‘b’ such
that,
|f(t)| < Mebt
Q3. Find the Laplace transform of eat . Model Paper-I, Q1(b)
Ans: Given function is eat
Let, f(t) = eat
From the definition of Laplace transform,

∫e
− st
L{f(t)} = . f (t ) dt
0

SPECTRUM ALL-IN-ONE JOURNAL FOR ENGINEERING STUDENTS SIA GROUP


1.2 MATHEMATICS-II [JNTU-ANANTAPUR]

On substituting, n = n – 2, in equation (1), we get,

n−2
∫e
− st
∫e
at −( s − a ) t
∴ L{eat} = .e .dt = .dt L{tn–2} = L{t n − 3 } ... (3)
0 0
s
∴ On generalizing equation (1), we get,

=
[ ]

e −( s − a ) t 0
=
0 −1
L{tn} =
n n −1 n − 2 n − 3 2 1
. . . .... − L{t n − n }
− (s − a) − (s − a) s s s s s s
n ( n − 1)( n − 2 )( n − 3).... 2 . 1
= L{1}
1 s × s × s × s....
=
s−a
n! 1 n! ⎛ k 1⎞
= . = n +1 ⎜Q L{k } = ⇒ L{1} = ⎟
1
n
s s s ⎝ s s⎠
∴ L{e at } =
s−a n!
∴ L{t n } = n +1
n! s
Q4. Prove that L{tn} = , n ≥ 0.
s n+ 1 Hence proved.
Ans: Given that, Q5. State first shifting theorem.
Ans: Let f(t) be a function defined for all the positive values of
n! ‘t’. Then, the first shifting theorem states that,
L{tn} =
s n +1 For L[f(t)] = f (s )
Consider L.H.S,
L[eat f(t)] = f ( s − a), where ( s − a) > 0 .
L{tn}

∫ te
From the definition of Laplace transform, −t
Q6. Using Laplace transform, evaluate sin t dt.
∞ 0
Ans:

− st Model Paper-III, Q1(b)
L{f (t)} = e f (t ) dt Given function is,
0

∫ te
−t
∞ sin t dt ... (1)


− st n
∴ L{tn} = e .t dt 0
Consider, sin t
0
1 ⎛ a ⎞
L[sin t] = ⎜Q L[sin at ] = 2 ⎟
∞ s 2 +1 ⎝ s + a2 ⎠
⎡ ∞ ⎤ ∞⎡
d n − st ⎤

∫ ∫ ∫ −d
− st
= ⎢t n
e .dt ⎥ − ⎢ t e .dt ⎥ dt
⎢ 0 ⎥ ⎢ dt ⎥ Then, L[t sint] = L[sin t]
⎣ ⎦0 0 ⎣ 0 ⎦ ds
⎛ n ⎞
(Q Integration by Parts) ⎜Q L[t n f (t )] = (−1) n d f (s ) ⎟
⎜ ds n ⎟
⎝ ⎠
[ ]

⎡ e − st ∞ ⎤ ∞
⎛ e −st ⎞ −d ⎡ 1 ⎤
= ⎢t .
0 ⎥
− ⎜ n.t n −1. ∫ ⎟dt
n
⎜ ⎟ =


−s ⎥
⎦0 0 ⎝
−s ⎠ ds ⎢⎣ s 2 + 1 ⎥⎦
⎡ (s 2 + 1)(0) − (1)(2s ) ⎤
∞ = −⎢ ⎥
n −st n −1 ⎣⎢ ( s 2 + 1) 2 ⎦⎥
= (0 ) +
s ∫
e .t dt
⎛ ⎛ u ⎞ ' vu ′ − u v ′ ⎞
0 ⎜Q ⎜ ⎟ = ⎟
⎜ ⎝v⎠ v2 ⎟
⎝ ⎠

n − st n −1

⇒ n
L{t } = e .t dt 2s
s =
0 ( s + 1) 2
2


n n −1
L{t } = L{t }
n
s
... (1) ∫
L[f(t)] = e −st f (t ) dt
0
On substituting, n = n – 1, in equation (1), we get, ∞
2s
∫e
− st
n −1 ⇒ L[tsint] = (t sin t ) dt =
n–1
L{t } = L{t n − 2 } ... (2) ( s 2 + 1) 2
s 0

Look for the SIA GROU P LOGO on the TITLE COVER before you buy
UNIT-1 (Laplace Transform) 1.3
On substituting s = 1, we get,

2 (1) 2 1
∫e
−t
t sin t dt = = =
0
(1 + 1) 2 4 2


1

∴ e−t t sint dt =
0
2

Q7. Define unit step function.


Ans: The unit step function or Heaviside unit function is defined as,

⎧ 0, t < a
u(t – a) or H(t – a) = ⎨
⎩1, t > a
Where, a > 0
The graphical representation of unit step function is as illustrated in figure below.

u(t – a)

0 a t
Figure: Unit Step Function
Q8. Define second shifting theorem.
Ans: Model Paper-II, Q1(a)

Statement: Let f(t) be a function defined for all the positive values of ‘t’.
Then, the second shifting theorem states that,

For L[f(t)] = f (s) and

⎧ f (t − a) ; t > a
g(t) = ⎨
⎩0 ; t<a

Laplace transform of g(t) is given by,

L[g(t)] = e–as f (s )
Another Form

⎧1 t > 0
For L{F(t)} = f (s) and H(t) = ⎨
⎩0 t < 0

Then, L{F(t – a) H(t – a)} = e–as f (s)


Q9. Define Dirac’s delta function.
Ans: The Dirac’s delta function or unit impulse function can be defined as the limiting form of,

⎧ 0, t < a
⎪1

fε(t – a) = ⎨ , a ≤ t ≤ a + ε
⎪ε
⎪⎩ 0 , t > a

SPECTRUM ALL-IN-ONE JOURNAL FOR ENGINEERING STUDENTS SIA GROUP


1.4 MATHEMATICS-II [JNTU-ANANTAPUR]
This function is graphically represented in figure below.
fε(t – a)

1
ε

t
a a +ε
Figure: Unit Impulse Function
In general, Dirac delta function is given by,

δ(t – a) = εLt { f ε (t − a)}


→0
Q10. Define the Laplace transform of a periodic function.
Ans: A function f(t) is said to be periodic, if it satisfies f(t + T) = f(t).
The Laplace transform of a periodic function is given by,
T

∫e
− st
f (t ) dt
0
L{f(t)} =
1 − e − st
Where, T – Period of f(t).
Q11. Define inverse Laplace transform.
Ans: If f (s ) is considered as the Laplace transform of f(t), then f(t) is called as inverse Laplace transform of f (s ) .

i.e., f(t) = L−1{ f (s )}


Where, L–1 – Inverse Laplace transform operator.

⎡ 3(s 2 − 2) 2 ⎤
Q12. Find the inverse Laplace transformation of ⎢ 5 ⎥. Model Paper-II, Q1(b)
⎣⎢ 2s ⎦⎥
Ans: The given function is,
–1
⎡ 3( s 2 − 2) 2 ⎤
L ⎢ 5 ⎥
⎣ 2s ⎦

⎡ 3( s 2 − 2) 2 ⎤ 3 −1 ⎡ s 4 + 4 − 4 s 2 ⎤
–1
⇒ L ⎢ 5 ⎥ = L ⎢ ⎥
⎣ 2s ⎦ 2 ⎣⎢ s5 ⎦⎥

3 ⎡ −1 ⎡ s 4 ⎤ −1 ⎡ 4 ⎤ −1 ⎡ 4s 2 ⎤ ⎤ 3 ⎡ −1 ⎡ 1 ⎤ −1 ⎡ 1 ⎤ −1 ⎡ 1 ⎤ ⎤
= ⎢L ⎢ 5 ⎥ + L ⎢ 5 ⎥ − L ⎢ 5 ⎥⎥ = ⎢ L ⎢ ⎥ + 4 L ⎢ 5 ⎥ − 4 L ⎢ 3 ⎥ ⎥
2 ⎢⎣ ⎣⎢ s ⎦⎥ ⎣s ⎦ ⎣⎢ s ⎦⎥ ⎥⎦ 2 ⎣ ⎣ ⎦ s ⎣s ⎦ ⎣ s ⎦⎦

⎛ ⎡1 ⎤ ⎞
⎜Q L−1 ⎢ ⎥ = 1 and ⎟
⎜ ⎣s⎦ ⎟
3 ⎡ 4×t 4 t2 ⎤ ⎜ n ⎟
= ⎢1 + − 4. ⎥
⎜ L−1 ⎛⎜ 1 ⎞⎟ = t ⎟
2 ⎣⎢ 4! 2! ⎦⎥ ⎜ ⎟
⎝ ⎝ s n +1 ⎠ n! ⎠

3⎡ t4 t2 ⎤ 3 ⎡ t4 ⎤ 3 t4
= ⎢1 + 4. − 4. ⎥ = ⎢1 + − 2t 2 ⎥ = + − 3t 2
2 ⎢⎣ 24 2 ⎥⎦ 2 ⎢⎣ 6 ⎦⎥ 2 4
⎡ 3( s 2
− 2 ) 2 ⎤ 3 t 4
∴ L−1 ⎢ 5 ⎥ = + − 3t
2

⎣⎢ 2 s ⎥⎦ 2 4

Look for the SIA GROU P LOGO on the TITLE COVER before you buy
UNIT-1 (Laplace Transform) 1.5

PART-B
ESSAY QUESTIONS WITH SOLUTIONS
1.1 LAPLACE TRANSFORM OF STANDARD FUNCTIONS
Q13. Define Laplace transform, domain of the function and Kernel for the Laplace transform. Also write
about the existence of Laplace transform.
Ans:
For answer refer Unit-I, Q1 and Q2.
Q14. List out the various properties of Laplace transform.
Ans:
Properties of Laplace Transform
(i) Linearity Property

If L[f(t)]= f (s) and

L[g(t)] = g (s ) , then
L [c1 f(t) + c2 g(t)] = c1 L[f(t)] + c2 L[g(t)]

= c1 f (s) + c2 g (s)

Where, c1 and c2 are constants

k
(ii) L [k] = (s > 0) and k is a constant
s

1
(iii) L [t] =
s2

n!
(iv) L [tn] = , where n is a positive integer
s n +1

1
(v) L [eat] = , where (s – a) > 0
s−a

a
(vi) L [sin at] = (if s > 0)
s2 + a2

s
(vii) L [cos at] = (if s > 0)
s + a2
2

a
(viii) L [ sin h at] = (if s > |a|)
s2 − a 2

s
(ix) L [cos h at] = (if s > |a|)
s − a2
2

n!
(x) L [eat.tn] =
( s − a) n +1

b
(xi) L [eat sin bt] =
( s − a) 2 + b 2

SPECTRUM ALL-IN-ONE JOURNAL FOR ENGINEERING STUDENTS SIA GROUP


1.6 MATHEMATICS-II [JNTU-ANANTAPUR]

s−a 1 ⎛ s + ja − s + ja ⎞
(xii) L [eat cos bt] = = ⎜ ⎟
( s − a) 2 + b 2 2 j ⎜⎝ (s − ja )(s + ja ) ⎟⎠

b
(xiii) L [eat sin h bt] = 1 ⎡ 2 ja ⎤
( s − a) 2 − b 2 = ⎢ ⎥
2 j ⎢⎣ s 2 − ( ja ) 2 ⎥⎦
s−a (Q (a – b) (a + b) = a2 – b2)
(xiv) L [eat cos h bt] =
( s − a) 2 − b 2
1 ⎛ 2 ja ⎞ a
= ⎜ ⎟ = 2
1 ⎛s⎞ 2 j ⎝ s2 + a2 ⎠ s + a2
(xv) L[f(at)] = f ⎜ ⎟
a ⎝a⎠
a
Q15. Prove that, ∴ L{sin at} =
s2 + a2
a
(a) L{sin at} = ,s>a Hence proved.
s2 + a 2
s
s (b) L{cosat} = ,s > a
(b) L{cos at} = , s > a. s + a2
2
s2 + a 2
Ans: Model Paper-I, Q2 Consider L.H.S,
L{cos at}
a
(a) L{sinat} = ,s > a By the definition of Laplace transform,
s2 + a2
Consider L.H.S, ∞


− st
L{sin at} L{f(t)} = e . f (t ) dt
By the definition of Laplace transform, 0

∞ ∞

∫ ∫
− st − st
L{f (t)}= e . f (t )dt ∴ L{cos at}= e . cos at dt
0 0

∞ ∞
− st ⎛ e jat + e − jat ⎞

− st
∴ L{sin at}= e .sin at dt
0
=
0
e ∫ .⎜

⎝ 2
⎟ dt



− st ⎛ e
jat
− e − jat ⎞
1 ⎡ − st jat ⎤
∞ ∞
= e .⎜⎜∫⎝ 2j
⎟.dt

⎠ =
2⎢
⎢ e .e∫ .dt + ∫
e − st .e − jat .dt ⎥
0 ⎥
⎣0 0 ⎦

1 ⎡ −st jat ⎤
∞ ∞

∫ ∫ 1 ⎡ −( s− ja ) t ⎤
∞ ∞
= ⎢ e .e dt − e −st .e − jat dt ⎥
2j ⎢
⎣0 0

⎦ =
2⎢ ∫
⎢ e

.dt + e −( s + ja )t .dt ⎥

⎣0 0 ⎦
1 ⎡ −( s − ja ) t ⎤
∞ ∞

= ⎢ e
2j ⎢
⎣0
∫ 0

.dt − e −( s + ja )t .dt ⎥

⎦ = ⎢
− −
[
1 ⎡ e ( s ja ) t 0
+
] [

e −( s + ja )t 0 ]
∞ ⎤

2 ⎢ − (s − ja) − (s + ja ) ⎥
⎣ ⎦
[ ∞
1 ⎡⎢ e − ( s − ja ) t 0] [

]∞
e − ( s + ja ) 0 ⎤

=
2 j ⎢ − ( s − ja ) − ( s + ja ) ⎥ 1 ⎡ 0 −1 0 −1 ⎤
+
2 ⎢⎣ − ( s − ja) − ( s + ja ) ⎥⎦
⎣ ⎦ =

1 ⎡ 0 −1 0 −1 ⎤ 1⎡ 1 1 ⎤
= ⎢ − ⎥ +
2 j ⎣ − ( s − ja ) − ( s + ja ) ⎦ = ⎢ ⎥
2 ⎣ s − ja s + ja ⎦

1 ⎡ 1 1 ⎤ 1 ⎡ s + ja + s − ja ⎤
= −
2 j ⎣ s − ja s + ja ⎥⎦
⎢ =
2 ⎢⎣ ( s − ja )(s + ja ) ⎥⎦

Look for the SIA GROU P LOGO on the TITLE COVER before you buy
UNIT-1 (Laplace Transform) 1.7

1⎡ ⎤ 3 3
2s = L{cos 7t} + L{cos t}
= ⎢ 2 2⎥ 2 2
2 ⎣⎢ s − ( ja ) ⎦⎥
3⎛ s ⎞ 3⎛ s ⎞
[Q (a – b) (a + b) = a2 – b2] = ⎜ ⎟+ ⎜ 2 ⎟
2 ⎝ s2 + 72⎠ 2 ⎝ s +1⎠
1 ⎡ 2s ⎤ ⎡ s ⎤
⎢Q L{cos at} = 2 ⎥
2 ⎢⎣ s 2 + a 2 ⎥⎦
= s + a2 ⎦

3⎡ s s ⎤
s = + 2
=
s2 + a 2

2 ⎣ s + 49 s + 1 ⎥⎦
2

3⎡ s s ⎤
s ∴ L{3 cos 3t cos 4t} = ⎢ + 2 ⎥
∴ L{cos at} = 2 ⎣ s + 49 s + 1 ⎦
2
s2 + a2
Q17. Find the Laplace Transform of the following
Hence proved. functions,
(i) f(t) = kt, k is a real constant >1
Q16. Find L[3 cos 3t cos 4t].
(ii) f(t) = tδ δ '(t).
Ans: Given Laplace function is, L[3 cos 3t cos 4t] Ans:
Consider (cos 3t ⋅ cos 4t) (i) f(t) = kt, k is a Real Constant >1
Given that,
On multiplying and dividing by 2, we get,
f(t) = kt
1 Where,
cos 3t.cos 4t = ( 2 cos 3t . cos 4t )
2 K is real constant > 1
From the definition of Laplace transform,
1
= [cos(3t + 4t) + cos(3t – 4t)] ∞
2
∫k e − st dt
t
f (s ) = L[f(t)]= ... (1)
[Q 2 cos A cos B = cos( A + B ) + cos( A − B )] 0

1 ∞ ∞
⎡ t e − st ⎤ e − st t

= [cos 7t + cost (–t)]
2 ⇒ f (s ) = ⎢ k ⎥ − k . log k dt
⎣ − s ⎦0 0 − s
1
= [cos 7t + cos t ] ⎡ d x ⎤
⎢Q dx (a ) = a log a⎥
2 x

[Q cos (– t) = cost] ⎣ ⎦

1 1 ∞
1 log k − st t
=
2
cos 7t +
2
cos t
⇒ f (s ) = +
s s 0
e k dt ∫
1 1
∴ cos 3t cos 4t = cos 7t + cos t ... (1) 1 log k
2 2 ⇒ f (s ) = + . f (s)
s s
Taking Laplace transform of 3 cos 3t cos 4t, we get,
[Q From equation (1)]
L{3 cos 3t cos 4t} = L{3(cos 3t . cos 4t )} ⎡ log k ⎤ 1
⇒ f ( s ) ⎢1 − ⎥ = s
⎣ s ⎦
⎧ ⎛1 1 ⎞⎫
= L ⎨3⎜ cos 7t + cos t ⎟ ⎬
⎩ ⎝2 2 ⎠⎭ ⎡ s − log k ⎤ 1
[Q From equation (1)] ⇒ f ( s) ⎢ ⎥=
⎣ s ⎦ s
⎧3 3 ⎫ 1
= L ⎨ cos 7t + cos t ⎬ ⇒ f (s ) =
⎩ 2 2 ⎭ ( s − log k )
1
⎧3 ⎫ ⎧3 ⎫ ∴ L[ k t ] =
= L ⎨ cos 7t ⎬ + L⎨ cos t ⎬ ( s − log k )
⎩2 ⎭ ⎩2 ⎭

SPECTRUM ALL-IN-ONE JOURNAL FOR ENGINEERING STUDENTS SIA GROUP


1.8 MATHEMATICS-II [JNTU-ANANTAPUR]
(ii) δ'(t)
f(t) = tδ On applying Laplace transform on both sides,
Given that,
L[f(t)] = L[sin(2ωt) cos(2ωt0) – cos(2ωt) sin(2ωt0)]
f(t) = t δ'(t)
Considering R.H.S and integrating, = L[sin(2ωt) cos(2ωt0)] – L[cos(2ωt) sin(2ωt0)]

∫ t.δ' (t)dt = t.∫ δ' (t ).dt − ∫ (t ' ∫ δ' (t ).dt ) dt = cos(2ωt0) × L[sin 2ωt)] – sin(2ωt0) × L[cos(2ωt)]

2ω s

= t .δ(t ) − 1.δ(t ).dt ... (2) = cos( 2ωt0 ) ×
s + (2ω)
2 2
− sin 2ωt0 × 2
s + (2ω) 2
Here, δ(t) = 1 for t = 0. But, at the same instant t.δ(t) = 0.
⎡ a ⎤
On substituting above value in equation (2), we get, ⎢Q L (sin at ) = s 2 + a 2 ⎥
⎢ ⎥
⇒ ∫ t.δ' (t).dt = t.(0) – ∫ δ(t).dt ⎢ L (cos at ) =
⎢⎣
s
2 ⎥

s +a ⎦
2

⇒ ∫ t.δ' (t).dt = 0 – ∫ δ(t).dt


2ω cos(2ωt 0 ) s. sin(2ωt 0 )

∫ t.δ' (t).dt = – ∫ δ(t).dt
=
⇒ s 2 + 4ω2 s 2 + 4ω2

∴ t.δ' (t ) = − δ(t ) ... (3)


2ω cos( 2ωt 0 ) − s. sin( 2ωt 0 )
∴ L[ f (t )] =
By definition of Laplace transform, s 2 + 4ω 2
F(s) = L [f(t)]
= L [t.δ'(t)] (ii) ωt + θ)
t cos(ω
= L [– δ(t)] Let, f(t) = t cos(ωt + θ)
[Q From equation (3)]
cos(A + B) = cosA cosB – sinA sinB

∫ − δ(t ).e
− st
⇒ F(s) = dt ∴ f(t) = t cos(ωt + θ) = t[cosωt cosθ – sinωt sinθ]
0
= t cosωt cosθ – t sinωt sinθ
And also, for an unit impulse function or signal,
On applying Laplace transform on both sides, we get,
⎧1, t = 0
δ(t) = ⎨
⎩0, else where ⇒ L[f(t)] = L[t cosωt cosθ – t sinωt sinθ]

− st = L[t cosωt cosθ] – L[t sinωt sinθ]


∴ f (s) = – (1) e |t =0 = – e–s(0) = – 1
= cosθ L[t cosωt] – sinθ L[t sinωt] ... (1)
∴ L[t.δ' (t )] = L[−δ(t )] = −1

Q18. Find the Laplace transform for the following ⎡ s 2 − ω2 ⎤ ⎡ 2ωs ⎤


functions, = cosθ × ⎢ 2 2 ⎥ – sinθ × ⎢ 2 2 2⎥
⎣ (s + ω ) ⎦ ⎣ (s + ω ) ⎦
2

(i) ω (t – t0)
sin 2ω
⎡ s2 − a2 2 as ⎤
(ii) ω t + θ ).
t cos(ω ⎢Q L(t cos at ) = 2 and L(t sin at ) = 2 ⎥
⎣⎢ (s + a )2 2
(s + a 2 ) 2 ⎦⎥
Ans:
(i) ω(t – t0)
sin 2ω
( s 2 − ω2 ) cos θ 2ωs sin θ
Let, f(t) = sin 2ω(t – t0) = sin(2ωt – 2ωt0) = − 2
( s 2 + ω2 ) 2 ( s + ω2 ) 2
Since,
sin(A – B) = sinA cosB – cosA sinB
( s 2 − ω2 ) cos θ − 2ωs. sin θ
f(t) = sin(2ωt – 2ωt0) ∴ L[ f (t )] =
(s 2 + ω2 ) 2
= sin(2ωt) cos(2ωt0) – cos(2ωt) sin(2ωt0)

Look for the SIA GROU P LOGO on the TITLE COVER before you buy
UNIT-1 (Laplace Transform) 1.9

1.2 INVERSE TRANSFORM


⎡ s+3 ⎤
Q19. Find L–1 ⎢ 2 ⎥.
⎣ s − 10s + 29 ⎦
Ans: The given function is,

⎡ s+3 ⎤
L–1 ⎢ 2 ⎥
⎣ s − 10 s + 29 ⎦
The numerator and denominator can be written as,
s + 3 = (s – 5) + 8 and
⇒ s2 – 10s + 29 = (s – 5)2 + 22

⎡ s+3 ⎤ ⎡ ( s − 5) + 8 ⎤
⇒ L–1 ⎢ 2 ⎥ =L ⎢
–1

⎣ s − 10 s + 29 ⎦ ⎢⎣ ( s − 5) 2 + 2 2 ⎥⎦

⎡ ( s − 5) ⎤ ⎡ 8 ⎤ ⎡ −1 ⎡ s−a ⎤ ⎤
+ L–1 ⎢ ⎢Q L ⎢ 2⎥
= e at cos bt ⎥
= L–1 ⎢ 2⎥ 2 ⎥ ⎢⎣ ⎢⎣ ( s − a ) + b ⎥⎦
2
⎥⎦
⎣⎢ ( s − 5) + 2 ⎦⎥
2
⎢⎣ ( s − 5) + 2 ⎥⎦
2

⎡ 4× 2 ⎤
= e5t.cos 2t + L–1 ⎢ 2⎥
⎢⎣ (s − 5) + 2 ⎥⎦
2

⎡ 2 ⎤ ⎡ −1 ⎡ b ⎤ at

= e5t.cos 2t + 4L–1 ⎢ 2⎥
⎢Q L ⎢ ⎥ = e sin bt ⎥
⎣ ( s − 5) + 2 ⎦ ⎢⎣ ( s − a ) + b ⎦⎥
2 2 2
⎢⎣ ⎥⎦

= e5t cos 2t + 4e5t sin 2t

= e5t [cos 2t + 4 sin 2t ]

⎡ s+3 ⎤
∴ L−1 ⎢ 2 ⎥ = e (cos 2t + 4 sin 2t )
5t
⎣ s − 10s + 29 ⎦

Q20. Find L–1[(2s + 3)/(s3 – 6s2 + 11s – 6)]. Model Paper-II, Q2

Ans: The given function is,


L–1[ (2s + 3)/(s3 – 6s2 + 11s – 6)]

2s + 3
Consider,
s − 6s 2 + 11s − 6
3

⎡ 2s + 3 ⎤ ⎡ 2s + 3 ⎤
⇒ ⎢ 3 2 ⎥ = ⎢ ⎥
⎣ s − 6 s + 11s − 6 ⎦ ⎣ ( s − 1)( s − 2)( s − 3) ⎦
On applying partial fractions, we get,

2s + 3 A B C
∴ = + + ... (1)
( s − 1)( s − 2)( s − 3) s −1 s−2 s −3

2s + 3 A( s − 2)( s − 3) + B ( s − 1)( s − 3) + C ( s − 1)( s − 2)


⇒ =
( s − 1)( s − 2)( s − 3) ( s − 1)( s − 2)( s − 3)
⇒ 2s + 3 = A(s – 2) (s – 3) + B(s – 1)(s – 3) + C(s – 1)(s – 2) ... (2)

SPECTRUM ALL-IN-ONE JOURNAL FOR ENGINEERING STUDENTS SIA GROUP


1.10 MATHEMATICS-II [JNTU-ANANTAPUR]
On substituting s = 1 in equation (2), we get,
2(1) + 3 = A(1 – 2)(1 – 3) + B(1 – 1)(1 – 3) + C(1 – 1)(1 – 2)
⇒ 2 + 3 = A(–1)(–2) + B(0) (–2) + C(0) (–1)
⇒ 5 = A(2) + 0 + 0
⇒ 5 = 2A

5
∴A= ... (3)
2
On substituting s = 2 in equation (2), we get,
2(2) + 3 = A(2 – 2)(2 – 3) + B(2 – 1)(2 – 3) + C(2 – 1)(2 – 2)
⇒ 4 + 3 = A(0)(–1) + B(1)(–1) + C(1)(0)
⇒ 7 = 0 + B(–1) + 0
⇒ 7 = –B

∴ B = −7 ... (4)

On substituting s = 3 in equation (2), we get,


2(3) + 3 = A(3 – 2)(3 – 3) + B(3 – 1)(3 – 3) + C(3 – 1)(3 – 2)
⇒ 6 + 3 = A(1)(0) + B(2)(0) + C(2)(1)
⇒ 9 = 2C

9
∴C = ... (5)
2
On substituting equations (3), (4) and (5) in equation (1), we get,

2s + 3 5 2 ( −7 ) 9 2
= + +
( s − 1)( s − 2)( s − 3) ( s − 1) ( s − 2 ) ( s − 3)

5 7 9
= – +
2( s − 1) s−2 2( s − 3)

On applying inverse Laplace transform on both sides, we get,

⎧ 2s + 3 ⎫ L−1 ⎧ 5 − 7 + 9 ⎫
L−1 ⎨ 3 ⎬= ⎨ ⎬
2
⎩ s − 6 s + 11s − 6 ⎭ ⎩ 2( s − 1) s − 2 2 ( s − 3) ⎭

⎧ 2s + 3 ⎫ L−1 ⎧ 5 ⎫ −1 ⎧ 7 ⎫ −1 ⎧ 9 ⎫
L−1 ⎨ 3 ⎬= ⎨ ⎬ – L ⎨ ⎬ + L ⎨ ⎬
2
⎩ s − 6 s + 11s − 6 ⎭ ⎩ 2( s − 1) ⎭ ⎩s − 2⎭ ⎩ 2( s − 3) ⎭

5 −1 ⎧ 1 ⎫ −1 ⎧ 1 ⎫ 9 −1 ⎧ 1 ⎫
= L ⎨ ⎬ –7 L ⎨ ⎬ + L ⎨ ⎬
2 ⎩ s − 1⎭ ⎩ s − 2 ⎭ 2 ⎩ s − 3⎭

5 t 9 ⎛ ⎛ 1 ⎞ ⎞
= .e – 7e2t + e3t ⎜⎜Q L−1 ⎜ ⎟ = e at ⎟⎟
2 2 ⎝ ⎝s−a⎠ ⎠

⎧ 2s + 3 ⎫ 5 t 9e 3t
∴ L−1 ⎨ 3 ⎬ = e − 7 e 2t
+
⎩ s − 6 s 2 + 11s − 6 ⎭ 2 2

Look for the SIA GROU P LOGO on the TITLE COVER before you buy
UNIT-1 (Laplace Transform) 1.11
–1 3 3
Q21. Find L [1/(s + 1)s ].
Ans: The given function is,

⎧⎪ 1 ⎫⎪
L−1 ⎨ 3 3⎬
⎪⎩ ( s + 1) s ⎪⎭

Adding and subtracting s3 in numerator, we get,

⎧⎪ ⎫⎪ ⎪1 + s − s ⎫⎪
−1 ⎧
3 3
1
L−1 ⎨ 3 ⎬ = L ⎨ ⎬
⎪⎩ ( s + 1) s 3 ⎪⎭ ⎪⎩ ( s 3 + 1) s 3 ⎪⎭

−1 ⎧
⎪ (1 + s ) ⎫⎪
3
s3
= L ⎨ 3 − ⎬
⎪⎩ ( s + 1) s 3 ( s 3 + 1) s 3 ⎪⎭

−1 ⎧ 1 1 ⎫
= L ⎨ 3− 3 ⎬
⎩ s s + 1⎭

−1 ⎧ 1 ⎫ −1 ⎧ 1 ⎫
= L ⎨ ⎬− L ⎨ 3 ⎬
⎩ s3 ⎭ ⎩ s + 1⎭

t2 ⎧ 1 ⎫ ⎡ −1 ⎧ 1 ⎫ tn ⎤
= − L−1 ⎨ 3 ⎬ ⎢Q L ⎨ n +1 ⎬ = ⎥
2! ⎩ s + 1⎭ ⎣⎢ ⎩ s ⎭ n! ⎦⎥

t2 ⎧ 1 ⎫
= − L−1 ⎨ 3 ⎬
2 ⎩ s + 1⎭

t2 ⎧⎪ 1 ⎫⎪
= − L−1 ⎨ ⎬ [Q ( a 3 + b 3 ) = ( a + b )( a 2 − ab + b 2 )] ... (1)
2 ⎪⎩ ( s + 1)( s − s + 1) ⎪⎭
2

1
Resolving into partial fraction, we get,
( s + 1)( s 2 − s + 1)

1 A Bs + C
= + ... (2)
( s + 1)( s 2 − s + 1) s +1 s2 − s +1

⇒ 1 = A(s2 – s + 1) + (Bs + C) (s + 1) ... (a)


On substituting s = –1, we get,
1 = A((–1)2 – (–1) + 1) + (B(–1) + C) (–1 + 1)
⇒ 1 = A(1 + 1 + 1) + (– B + C)0
⇒ 1 = 3A + 0

1
∴ A=
3

1
On substituting A = in equation (a), we get,
3

( s 2 − s + 1)
1= + ( Bs + C )( s + 1)
3

s2 s 1
⇒ 1= − + + Bs 2 + Bs + Cs + C ... (b)
3 3 3

SPECTRUM ALL-IN-ONE JOURNAL FOR ENGINEERING STUDENTS SIA GROUP


1.12 MATHEMATICS-II [JNTU-ANANTAPUR]
On comparing the constant terms on L.H.S and R.H.S in equation (b), we get,

1
1 = +C
3

1
⇒ C =1–
3

3 −1 2
⇒ C = =
3 3

2
∴C =
3

2
On substituting C = in equation (b), we get,
3

s2 s 1 2 2
1 = − + + Bs 2 + Bs + s + ... (c)
3 3 3 3 3
On comparing the coefficient of ‘s’ on L.H.S and R.H.S in equation (c), we get,
−1 2
0= +B+
3 3
1 2
⇒ B= −
3 3
1− 2
⇒ B=
3

−1
∴B =
3
On substituting the values of A, B and C in equation (2), we get,

1 −1 2
s+
1 3 + 3 3
∴ =
( s + 1)( s 2 − s + 1) s + 1 ( s 2 − s + 1)

1 ⎛ 1 ⎞ 1 ⎛⎜ − s + 2 ⎞
⎜ ⎟+ ⎟
3 ⎜⎝ ( s + 1) ⎟⎠ 3 ⎜⎝ ( s 2 − s + 1) ⎟⎠
=

1⎛ 1 −s+2 ⎞
= ⎜ + ⎟
3 ⎝ s +1 s2 − s +1⎠
On substituting the above partial fractions in equation (1), we get,

⎧⎪ 1 ⎫⎪ t 2 ⎧1 ⎛ 1 − s + 2 ⎞⎫
L–1 ⎨ 3 − L−1 ⎨ ⎜ + 2 ⎟⎬
3⎬
=
⎪⎩ ( s + 1) s ⎪⎭ 2 ⎩ 3 ⎝ s + 1 s − s + 1 ⎠⎭

t 2 1 −1 ⎧ 1 ⎫ 1 −1 ⎧ − s + 2 ⎫
= − L ⎨ ⎬− L ⎨ ⎬
2 3 ⎩ s + 1 ⎭ 3 ⎩ s 2 − s + 1⎭

t 2 1 −t 1 −1 ⎧ s − 2 ⎫ ⎡ −1 ⎧ 1 ⎫ − at ⎤
= − e + L ⎨ 2 ⎬ ⎢Q L ⎨ ⎬=e ⎥ ... (3)
2 3 3 ⎩ s − s + 1⎭ ⎣ ⎩s+ a⎭ ⎦

Look for the SIA GROU P LOGO on the TITLE COVER before you buy
UNIT-1 (Laplace Transform) 1.13
s−2
Consider, 2
s − s +1
2
⎛1⎞
Adding and subtracting ⎜ ⎟ in denominator.
⎝2⎠
s−2 s−2 s−2
⇒ 2
= 2 2
= 2 2
s − s +1 ⎛1⎞ ⎛1⎞ ⎛1⎞ ⎛1⎞
s − s +1 − ⎜ ⎟ + ⎜ ⎟
2 s2 − s + ⎜ ⎟ +1 − ⎜ ⎟
⎝2⎠ ⎝2⎠ ⎝ ⎠
2 ⎝2⎠
s−2 s−2 s 2
= 2 = 2 = 2
− 2
⎛ 1⎞ 3 ⎛ 1 ⎞ ⎛ 3 ⎞⎟
2
⎛ 1 ⎞ ⎛⎜ 3 ⎞⎟
2
⎛ 1 ⎞ ⎛ 3 ⎞⎟
2
⎜s − ⎟ + ⎜ s − ⎟ + ⎜⎜ ⎜s − ⎟ + ⎜ ⎜ s − ⎟ + ⎜⎜
⎝ 2⎠ 4 ⎝ 2 ⎠ ⎝ 2 ⎟⎠ ⎝ 2 ⎠ ⎝ 2 ⎟⎠ ⎝ 2 ⎠ ⎝ 2 ⎟⎠

1 1
s− +
2 2 2
= 2
− 2
1 ⎞ ⎛⎜ 3 ⎞⎟ 1 ⎞ ⎛⎜ 3 ⎟⎞
2 2
⎛ ⎛
⎜ s − ⎟ + ⎜ s − ⎟ +
⎝ 2 ⎠ ⎝⎜ 2 ⎠⎟ ⎝ 2 ⎠ ⎝⎜ 2 ⎟⎠

1 1
s−
2 2 2
= 2
− 2
− 2
1 ⎞ ⎜⎛ 3 ⎞⎟ 1 ⎞ ⎜⎛ 3 ⎞⎟ 1 ⎞ ⎜⎛ 3 ⎞⎟
2 2 2
⎛ ⎛ ⎛
⎜s − ⎟ + ⎜ ⎜s − ⎟ + ⎜ ⎜s − ⎟ + ⎜
⎝ 2 ⎠ ⎝ 2 ⎠⎟ ⎝ 2 ⎠ ⎝ 2 ⎠⎟ ⎝ 2 ⎠ ⎝ 2 ⎠⎟

1 1 1 3
s− −2 s−
2 + 2 2 − 2
= 2 2 = 2 2
1 ⎞ ⎛⎜ 3 ⎞⎟ 1 ⎞ ⎜⎛ 3 ⎞⎟ 1 ⎞ ⎜⎛ 3 ⎞⎟ 1 ⎞ ⎛⎜ 3 ⎞⎟
2 2 2 2
⎛ ⎛ ⎛ ⎛
⎜s − ⎟ + ⎜ ⎜s− ⎟ +⎜ ⎜s − ⎟ + ⎜ ⎜s − ⎟ + ⎜
⎝ 2 ⎠ ⎝ 2 ⎠⎟ ⎝ 2 ⎠ ⎝ 2 ⎠⎟ ⎝ 2 ⎠ ⎝ 2 ⎠⎟ ⎝ 2 ⎠ ⎝ 2 ⎠⎟

1 3 3 1 3
s− × s−
2 3 2 3 2
= 2 − − .
2 2 = 2 2
⎛ 1 ⎞ ⎛⎜ 3 ⎟⎞
2
⎛ 1 ⎞ ⎛⎜ 3 ⎟⎞
2

2
1⎞ ⎜ 3⎟ ⎛ ⎞ 3 ⎛ 1 ⎞ ⎛⎜ 3 ⎞⎟
2

⎜s − ⎟ + ⎜ ⎜s − ⎟ + ⎜ ⎜ s − ⎟ + ⎜ s − ⎟ +
⎝ 2 ⎠ ⎝ 2 ⎟⎠ ⎝ 2 ⎠ ⎝ 2 ⎟⎠ ⎝ 2 ⎠ ⎜⎝ 2 ⎟⎠ ⎝ 2 ⎠ ⎜⎝ 2 ⎟⎠

1 3
s−
s−2 2 2
⇒ 2 = 2
− 3. 2
s − s +1 ⎛ 1⎞
2
⎛ 3⎟⎞ ⎛ 1 ⎞ ⎛ 3 ⎞⎟
2
⎜ s − ⎟ + ⎜⎜ ⎜ s− ⎟ +⎜
⎝ 2 ⎠ ⎝ 2 ⎟⎠ ⎝ 2 ⎠ ⎜⎝ 2 ⎟⎠
On applying inverse Laplace transform on both sides, we get,
⎧ ⎫
⎪ 1 3 ⎪
⎪ s− ⎪⎪
−1 ⎧ s − 2 ⎫ −1 ⎪ 2 2
L ⎨ 2 ⎬= L ⎨ 2
− 3. 2 ⎬
⎩ s − s + 1⎭ ⎪⎛ 1 ⎞ ⎛⎜ 3 ⎞⎟ 1 ⎞ ⎛⎜ 3 ⎞⎟ ⎪
2 2

⎪⎜ s − ⎟ + ⎜ ⎜s − ⎟ +⎜ ⎪
⎪⎩ ⎝ 2 ⎠ ⎝ 2 ⎟⎠ ⎝ 2 ⎠ ⎝ 2 ⎟⎠ ⎪⎭

⎧ ⎫ ⎧ ⎫
⎪ 1 ⎪ ⎪ 3 ⎪
⎪ s− ⎪⎪ ⎪ ⎪⎪
−1 ⎪ 2 −1 ⎪ 2
= L ⎨ 2 ⎬ − 3L ⎨ 2 ⎬
⎪⎛ 1 ⎞ ⎛⎜ 3 ⎞⎟ ⎪ ⎪⎛ 1 ⎞ ⎛⎜ 3 ⎞⎟ ⎪
2

⎪⎜ s − ⎟ + ⎜ ⎪ ⎪⎜ s − ⎟ + ⎜ ⎪
⎪⎩ ⎝ 2 ⎠ ⎝ 2 ⎟⎠ ⎪⎭ ⎪⎩ ⎝ 2 ⎠ ⎝ 2 ⎟⎠ ⎪⎭

SPECTRUM ALL-IN-ONE JOURNAL FOR ENGINEERING STUDENTS SIA GROUP


1.14 MATHEMATICS-II [JNTU-ANANTAPUR]

⎛ ⎧ s−a ⎫⎪ ⎞
⎜Q L−1 ⎪⎨ ⎬ = e cos bt ⎟
at
⎛ 3⎞ ⎛ 3⎞ ⎜ ⎪⎩ (s − a) + b ⎪⎭
2 2 ⎟
= e cos⎜⎜ ⎟⎟ t − 3e sin⎜⎜ ⎟t ⎜ ⎟
t/2 t/2
⎟ ⎜ ⎟
−1 ⎧ ⎫⎪
2
⎝ ⎠ 2
⎝ ⎠ ⎪ b
⎜⎜ L ⎨ 2⎬
= e sin bt ⎟
at
⎪⎩ ( s − a) + b ⎪⎭
2 ⎟
⎝ ⎠

⎧ s−2 ⎫ ⎡ ⎛ 3⎞ ⎤
⇒ L−1 ⎨ 2 ⎬ = e ⎢cos⎜⎜
t/2 ⎟ t − 3 sin 3 t ⎥ ... (4)
⎩ s − s + 1⎭ 2 ⎟⎠
⎣⎢ ⎝
2 ⎥

On substituting equation (4) in equation (3), we get,

t 2 1 −t 1 t / 2 ⎡ ⎛ 3 ⎞ ⎛ 3⎞ ⎤
⇒ − e + e ⎢cos ⎜⎜ ⎟⎟ t − 3 sin ⎜⎜ ⎟⎟ t ⎥
2 3 3 ⎣⎢ ⎝ 2 ⎠ ⎝ 2 ⎠ ⎥⎦

⎧⎪ 1 ⎫⎪ t 2 1 −t 1 t / 2 ⎡ ⎛ 3 ⎞ ⎛ ⎞ ⎤
∴ L−1 ⎨ 3 = − e + e ⎢cos⎜ ⎟ t − 3 sin⎜ 3 ⎟ t ⎥
3⎬
⎪⎩ (s + 1)s ⎪⎭ 2 3 3 ⎢⎣ ⎜⎝ 2 ⎟⎠ ⎜ 2 ⎟ ⎥
⎝ ⎠ ⎦

5s − 2
Q22. Find inverse Laplace transforms of .
s 2 (s + 2)(s − 1)

⎧⎪ 5s − 2 ⎫⎪
Ans: The given function is, ⎨ 2 ⎬
⎪⎩ s ( s + 2)(s − 1) ⎪⎭

On taking partial fractions, we get,


5s − 2 A B C D
= + 2 + +
s ( s + 2)(s − 1)
2
s s s+2 s −1
⇒ 5 s – 2 = A s(s + 2) (s – 1) + B (s+2) (s – 1) + C (s – 1) s2 + D (s + 2) s2 ... (1)
On substituting s = 0 in equation (1), we get,
– 2 = 0 + B(0 + 2) (0 – 1) + 0 + 0
⇒ – 2 = B(– 2)

∴ B =1
On substituting s = 1 in equation (1), we get,
5 – 2 = A(1) (1 + 2) (1 – 1) + B(1 + 2) (1 – 1) + C(1 – 1) (1)2 + D(1 + 2)(1)2
⇒ 3 = 3D

∴ D =1
On substituting s = – 2 in equation (1), we get,
5 (– 2) – 2 = A(– 2) (– 2 + 2) (– 2 – 1) + B(– 2 + 2) (– 2 – 1) + C(– 2 – 1) (– 2)2 + D(– 2 + 2)(–2)2
⇒ – 10 – 2 = C (– 2 – 1) (– 2)2
⇒ – 12 = C (– 3) (4)

∴ C =1
On equating s3 terms on both sides of equation (1), we get,
0=A+C+D
⇒ A=–C–D
=–1–1

∴ A = −2

Look for the SIA GROU P LOGO on the TITLE COVER before you buy
UNIT-1 (Laplace Transform) 1.15
On substituting the corresponding values of A, B, C and D in equation (1), we get,

5s − 2 2 1 1 1
∴ =– + 2 + +
2
s ( s + 2)( s − 1) s s s+2 s −1

On applying inverse Laplace transform on both sides, we get,

⎧⎪ 5s − 2 ⎫⎪ −1 ⎡ − 2 1 1 1 ⎤
L–1 ⎨ 2 ⎬= L ⎢ + 2+ +
⎪⎩ s ( s + 2)(s − 1) ⎪⎭ ⎣ s s s + 2 s − 1 ⎥⎦

⎧1 ⎫ ⎧1 ⎫ ⎧ 1 ⎫ ⎧ 1 ⎫
= – 2 L–1 ⎨ ⎬ + L–1 ⎨ 2 ⎬ + L–1 ⎨ ⎬ + L–1 ⎨ ⎬
⎩ ⎭
s ⎩s ⎭ ⎩ s + 2 ⎭ ⎩ s −1 ⎭

⎛ n ⎞
⎜Q L−1 ⎛⎜ 1 ⎞⎟ = t ⎟
= – 2(1) + t + e–2t + et ⎜ ⎝ s n +1 ⎠ n! ⎟
⎜ ⎟
⎜ −1 ⎛ 1 ⎞ at ⎟
⎜ L ⎜ ⎟ = e ⎟
⎝ ⎝s−a⎠ ⎠

= – 2 + t + et + e–2t

⎧⎪ 5s − 2 ⎫⎪
∴ L−1 ⎨ 2 t
⎬ = −2 + t + e + e
−2 t
⎪⎩ s ( s + 2)( s − 1) ⎪⎭

⎡ s ⎤
Q23. Find L–1 ⎢ 2 2 2 ⎥.
⎣ (s + 1) (s + 9) (s + 25) ⎦
Ans: The given function is,

⎡ s ⎤
L−1 ⎢ 2 ⎥
⎣ ( s + 1) ( s + 9) ( s + 25) ⎦
2 2

Consider,

1
( s + 1) ( s 2 + 9)
2

Taking partial fractions, we get,

1 As + B Cs + D
= 2 + 2 ... (A)
( s + 1) ( s + 9)
2 2
s +1 s +9

1 ( As + B) ( s 2 + 9) + (Cs + D ) (s 2 + 1)
⇒ =
( s 2 + 1) ( s 2 + 9) ( s 2 + 1) ( s 2 + 9)
1= (As + B) (s2 + 9) + (Cs + D) (s2 + 1)
⇒ 1= As3 + 9As + Bs2 + 9B + Cs3 + Cs + Ds2 + D
⇒ 1= (A + C)s3 + (B + D)s2 + (9A + C)s + (9B + D) ... (1)
On comparing the coefficients of s3, s2, s and constants in equation (1), we get,
0 =A+C ... (2)
0 =B+D ... (3)
0 = 9A + C ... (4)
1= 9B + D ... (5)
From equation (2), A = – C

SPECTRUM ALL-IN-ONE JOURNAL FOR ENGINEERING STUDENTS SIA GROUP


1.16 MATHEMATICS-II [JNTU-ANANTAPUR]
On substituting A = – C in equation (4), we get,
⇒ –9C + C = 0
⇒ –8C = 0

∴C = 0

⇒ ∴A= 0
From equation (3), B = –D
On substituting B = –D in equation (5), we get,
⇒ 1= –9D + D
1= – 8D

−1
∴ D=
8

1
∴ B=
8
On substituting the corresponding values in equation (A), we get,

1 1
0. s +
0.s −
1 8 + 8
∴ = 2
2 2
( s + 1) ( s + 9) ( s + 1) s + 9
2

1⎡ 1 1 ⎤
= − 2

8 ⎣ s + 1 s + 9 ⎥⎦
2

Consider,

s ⎡ 1 ⎤ s
=⎢ 2 ⎥. 2
( s + 1) ( s + 9) (s + 25)
2 2 2
⎢⎣ (s + 1)(s + 9) ⎥⎦ s + 25
2

1⎡ 1 1 ⎤ s
= − .
8 ⎢⎣ s 2 + 1 s 2 + 9 ⎥⎦ s 2 + 25

1⎡ s s ⎤
= ⎢ − ⎥ ... (B)
8 ⎣⎢ ( s 2 + 1) ( s 2 + 25) ( s 2 + 9) ( s 2 + 25) ⎦⎥

s As + B Cs + D
Consider, = + ... (C)
2 2
( s + 1) ( s + 25) s +1
2
s 2 + 25

s ( As + B ) ( s 2 + 25) + (Cs + D ) ( s 2 + 1)
=
( s 2 + 1) ( s 2 + 25) ( s 2 + 1) ( s 2 + 25)

⇒ s = (As + B) (s2 + 25) + (Cs + D) (s2 + 1)


⇒ s = As3 + 25 As + Bs2 + 25B + Cs3 + Cs + Ds2 + D
s = (A + C) s3 + (B + D)s2 + (25 A + C) s + (25 B + D) ... (6)
Comparing the coefficients of s3, s2, s and constants in equation (6), we get,
0 =A+C ... (7)
0 =B+D ... (8)
1 = 25A + C ... (9)
0 = 25B + D ... (10)

Look for the SIA GROU P LOGO on the TITLE COVER before you buy
UNIT-1 (Laplace Transform) 1.17
From equation (7), A = –C
On substituting A = –C in equation (9), we get,
⇒ 1= 25(–C) + C
⇒ 1= –24C
−1
∴C=
24

1
∴ A=
24
From equation (8), B = –D
On substituting B = –D in equation (10), we get,
⇒ 0 = 25(–D) + D
⇒ 0 = –24D

∴ D=0

∴B =0
On substituting corresponding values in equation (C), we get,

1 ⎛ −1⎞
s + 0 ⎜ ⎟s + 0
+⎝ 2 ⎠
s 24 24
∴ = 2
2 2
( s + 1) ( s + 25) s +1 s + 25

1 ⎡ s s ⎤
= − 2 ... (11)

24 ⎣ s + 1 s + 25 ⎥⎦
2

Consider,
s As + B Cs + D
= 2
+ ... (D)
2 2
( s + 9) ( s + 25) s +9 s 2 + 25

s ( As + B ) ( s 2 + 25) + (Cs + D ) ( s 2 + 9)
⇒ 2 2 =
( s + 9) ( s + 25) ( s 2 + 9) ( s 2 + 25)
s = (As + B) (s2 + 25) + (Cs + D) (s2 + 9)
⇒ s = As3 + 25 As + Bs2 + 25B + Cs3 + 9Cs + Ds2 + 9D
⇒ s = (A + C)s3 + (B + D)s2 + (25 A + 9C)s + (25 B + 9D) ... (12)
On comparing the coefficients of s3, s2, s and constants in equation (12), we get,
0 =A+C ... (13)
0 =B+D ... (14)
1 = 25A + 9C ... (15)
0 = 25B + 9D ... (16)
From equation (13), we get, A = – C
On substituting A = – C in equation (15), we get,
1= 25(– C) + 9C
⇒ 1 = – 16C
−1
∴C =
16
1
∴A =
16
From equation (14), we get, B = – D

SPECTRUM ALL-IN-ONE JOURNAL FOR ENGINEERING STUDENTS SIA GROUP


1.18 MATHEMATICS-II [JNTU-ANANTAPUR]
On substituting B = –D in equation (16), we get,
0 = 25B + 9D
⇒ 0 = 25(–D) + 9D
⇒ 0 = –16D
∴D = 0
∴B = 0
On substituting corresponding values in equation (D), we get,

1 ⎛ −1⎞
s + 0 ⎜ ⎟s + 0
+⎝ 2 ⎠
s 16
2 2 = 162
( s + 9) ( s + 25) s +9 s + 25

s 1 ⎡ s s ⎤
= − ... (17)
2
( s + 9) ( s + 25)2
16 ⎢⎣ s 2 + 9 s 2 + 25 ⎥⎦
On substituting equations (11) and (17) in equation (B), we get,
s 1⎡ 1 ⎛ s s ⎞ 1 ⎛ s s ⎞⎤
2 2 2 = ⎢ ⎜ 2 − 2 ⎟− ⎜ 2 − 2 ⎟⎥
( s + 1)( s + 9)( s + 25) 8 ⎣ 24 ⎝ s + 1 s + 25 ⎠ 16 ⎝ s + 9 s + 25 ⎠⎦
1 ⎡1 ⎛ s s ⎞ 1⎛ s s ⎞⎤
⎜ − ⎟− ⎜ − ⎟
8 × 8 ⎢⎣ 3 ⎝ s 2 + 1 s 2 + 25 ⎠ 2 ⎝ s 2 + 9 s 2 + 25 ⎠ ⎥⎦
=

1 ⎡ s s s s ⎤
= ⎢ − − + ⎥
64 ⎣⎢ 3( s 2 + 1) 3( s 2 + 25 ) 2 ( s 2 + 9 ) 2( s 2 + 25 ) ⎦⎥

1 ⎡ s s − 2s + 3s ⎤
= ⎢ 2 − + ⎥
64 ⎢⎣ 3( s + 1) 2( s + 9) 6( s 2 + 25) ⎥⎦
2

1 ⎡ s s s ⎤
= ⎢ − + ⎥
⎢⎣ 3( s + 1) 2( s + 9 ) 6 ( s + 25 ) ⎥⎦
2 2 2
64

s 1 ⎡ s s s ⎤
∴ = ⎢ − + ⎥
⎢⎣ 3( s + 1) 2( s + 9 ) 6 ( s + 25 ) ⎥⎦
2 2 2 2 2 2
( s + 1) ( s + 9 ) ( s + 25) 64
Taking inverse Laplace transform on both sides, we get,

−1 ⎧
⎪ s ⎫⎪ ⎧
−1 ⎪ 1 ⎛ s s s ⎞ ⎫⎪
∴ L ⎨ 2 ⎬ = L ⎨ ⎜⎜ − + ⎟⎬
⎪⎩ 64 ⎝ 3( s + 1) 2 ( s + 9 ) 6 ( s + 25 ) ⎟⎠ ⎪⎭
2 2 2
⎪⎩ ( s + 1) ( s + 9) ( s + 25) ⎪⎭
2 2

1 ⎡ 1 −1 ⎧⎪ s ⎫⎪ 1 −1 ⎧⎪ s ⎫⎪ 1 −1 ⎧⎪ s ⎫⎪⎤
= ⎢ L ⎨ 2 ⎬− L ⎨ 2 ⎬+ L ⎨ 2 ⎬⎥
64 ⎣⎢ 3 ⎪⎩ ( s + 1) ⎪⎭ 2 ⎪⎩ ( s + 3 ) ⎪⎭ 6
2
⎪⎩ ( s + 5 ) ⎪⎭⎦⎥
2

1 ⎡1 1 1 ⎤ ⎡ −1 ⎧ s ⎫ ⎤
= ⎢ 3 cos t − 2 cos 3t + 6 cos 5t ⎥ ⎢Q L ⎨ 2 2⎬
= cos at ⎥
64 ⎣ ⎦ ⎣ ⎩s + a ⎭ ⎦

1 ⎡ 2 cos t − 3 cos 3t + cos 5t ⎤


64 ⎢⎣ ⎥
=
6 ⎦
1
= [ 2 cos t − 3 cos 3t + cos 5t ]
384

⎧⎪ s ⎫⎪ 1
∴ L−1 ⎨ 2 ⎬= [ 2 cos t − 3 cos 3t + cos 5t ]
⎪⎩ ( s + 1)( s + 9)( s + 25) ⎪⎭ 384
2 2

Look for the SIA GROU P LOGO on the TITLE COVER before you buy
UNIT-1 (Laplace Transform) 1.19

(2s2 − 6s + 5)
Q24. Find the inverse Laplace transform of, .
(s3 − 6s 2 + 11s − 6)
Ans: The given function is,

(2s 2 − 6s + 5)
( s 3 − 6s 2 + 11s − 6)
Resolving (s3 – 6s2 + 11s – 6) into factors, we get,

1 1 –6 11 –6
0 1 –5 6
1 –5 6 0

i.e., s2 – 5s + 6
⇒ s3 – 6s2 + 11s – 6 = (s – 1) (s2 – 5s + 6)
=(s – 1) (s2 – 3s – 2s + 6)
= (s – 1) (s (s – 3) – 2(s – 3))
= (s – 1) (s – 2) (s – 3)

2s 2 − 6s + 5 2s 2 − 6s + 5
⇒ = ... (1)
s 3 − 6s 2 + 11s − 6 ( s − 1)(s − 2)( s − 3)
On applying partial fraction to equation (1), we get,

(2s 2 − 6s + 5) A B C
⇒ = + + ... (2)
( s − 1)(s − 2)(s − 3) ( s − 1) ( s − 2) ( s − 3)

(2 s 2 − 6s + 5) A( s − 2)( s − 3) + B( s − 1)( s − 3) + c(s − 1)( s − 2)


⇒ =
( s − 1)( s − 2)( s − 3) ( s − 1)( s − 2)( s − 3)

⇒ A(s – 2) (s – 3) + B(s – 1) (s – 3) + C(s – 1) (s – 2) = 2s2 – 6s + 5 ... (3)


On substituting s = 1 in equation (3) we get,
A(1 – 2) (1 – 3) + B(1 – 1) (1 – 3) + C(1 – 1)(1 – 2) = 2(1)2 – 6(1) + 5
⇒ 2A = 2 – 6 + 5
⇒ 2A = 1

1
∴ A=
2
On substituting s = 2 in equation (3), we get,
A(2 – 2) (2 – 3) + B(2 – 1) (2 – 3) + C(2 – 1) (2 – 2) = 2(2)2 – 6(2) + 5
⇒ –B = 8 – 12 + 5
⇒ –B = 1

∴ B = −1

On substituting s = 3 in equation (3), we get,


A(3 – 2) (3 – 3) + B(3 – 1) (3 – 3) + C(3 – 1) (3 – 2) = 2(3)2 – 6(3) + 5
⇒ 2C = 18 – 18 + 5
⇒ 2C = 5

∴ C = 5/ 2

SPECTRUM ALL-IN-ONE JOURNAL FOR ENGINEERING STUDENTS SIA GROUP


1.20 MATHEMATICS-II [JNTU-ANANTAPUR]
On substituting A, B, C values in equation (2), we get,

(2s 2 − 6s + 5) 1 1 5
∴ = − +
( s − 1)(s − 2)(s − 3) 2 ( s − 1) ( s − 2) 2( s − 3)
On applying inverse Laplace transform on both sides, we get,
⎡ (2 s 2 − 6s + 5) ⎤ −1 ⎡ 1 1 5 ⎤
L−1 ⎢ ⎥ = L ⎢ − + ⎥
⎣⎢ ( s − 1)( s − 2 )( s − 3) ⎦⎥ ⎣ 2( s − 1) ( s − 2) 2 ( s − 3) ⎦

−1 ⎛ 1 ⎞ −1 ⎛ 1 ⎞ −1 ⎛ 5 ⎞
= L ⎜⎜ ⎟⎟ − L ⎜⎜ ⎟⎟ + L ⎜⎜ ⎟⎟
⎝ 2(s − 1) ⎠ ⎝ ( s − 2) ⎠ ⎝ 2( s − 3) ⎠

1 −1 ⎛ 1 ⎞ −1 ⎛ 1 ⎞ 5 −1 ⎛ 1 ⎞ 1 t 5 ⎡ −1 ⎛ 1 ⎞ ⎤
= L ⎜ ⎟−L ⎜ ⎟+ L ⎜ ⎟ = e − e 2t + e 3t ⎢Q L ⎜ ⎟ = e at ⎥
2 ⎝ s −1⎠ ⎝ s−2⎠ 2 ⎝ s−3⎠ 2 2 ⎣ ⎝ s−a⎠ ⎦

⎡ ( 2 s 2 − 6 s + 5) ⎤ 1 t 5 3t
∴ L−1 ⎢ 2t
⎥ = e −e + e
⎢⎣ ( s − 1)( s − 2)( s − 3) ⎥⎦ 2 2

1.3 FIRST SHIFTING THEOREM


Q25. State and prove first shifting theorem.
Ans:
Statement
For answer refer Unit-I, Q5.
Proof: Given that,
L{f(t)} = f (s ) ... (1)


− st
∴ L{f(t)} = e f (t ) dt
0


− st at
∴ L{e f(t)}= e [ e f (t )]dt
at

0

∫e
−( s − a )t
= f (t ) dt ... (2)
0

On comparing equations (1) and (2), we get,


L{e at f (t )} = f ( s − a)
Hence, the statement is proved.
Q26. Prove that,
b
(i) L[eat sinh bt] =
(s − a)2 − b 2
s−a
(ii) L[eat cosh bt] = .
(s − a)2 − b 2
Ans: Model Paper-II, Q3

b
(i) L[eat sinh bt] =
(s − a)2 − b 2
Consider, L[sinh bt]


− st
Since, L[f(t)] = e . f (t ) dt
0

Look for the SIA GROU P LOGO on the TITLE COVER before you buy
UNIT-1 (Laplace Transform) 1.21
∞ s−a
(ii) L(eat cosh bt) =
⇒ L[sinh bt] = ∫
0
e −st . sinh bt dt (s − a)2 − b 2
Consider, L[cosh bt]


− st ⎛ebt − e −bt ⎞

− st
= ∫e
0



2
⎟dt


Since, L[f(t)] = e f (t ) dt
0


⎛ − at ⎞
⎜Q sinh(at ) = e − e
at


− st
⎜ 2 ⎟ ⇒ L[cosh bt] = e cosh bt dt
⎝ ⎠ 0


∞ ⎛ ebt + e −bt ⎞
=
1
2 ∫
(e − st .e bt − e − st e −bt ) dt
0

= e − st ⎜⎜
⎝ 2
⎟ dt


0

⎛ − at ⎞
⎜Q cosh at = e + e
at

1 ⎡ − ( s −b ) t ⎤
∞ ∞
⎜ ⎟
= ⎢ e
2⎢ ∫ ∫
dt − e −( s +b )t dt ⎥

⎝ 2 ⎠
⎣0 0 ⎦

1
⎡ ∞ ∞⎤
=
2 ∫
(e −st .ebt + e −st .e −bt ) dt
1 ⎢⎛⎜ e −( s −b ) t ⎞⎟ ⎛⎜ e −( s +b) t ⎞⎟ ⎥ 0
= −
2 ⎢⎜⎝ − (s − b) ⎟⎠ ⎜⎝ − (s + b) ⎟⎠ ⎥
⎣ 0 ⎦
1 ⎡ − ( s −b ) t ⎤
0 ∞ ∞

=
2⎢
⎢ e ∫ ∫
dt + e −( s +b ) t dt ⎥

⎣0 ⎦
1 ⎡⎛⎜ e −∞ − e 0 ⎞⎟ ⎛⎜ e −∞ − e 0 ⎞⎟⎤
0

= ⎢⎜ − ⎥
2 ⎣⎢⎝ − ( s − b) ⎟⎠ ⎜⎝ − ( s + b ) ⎟⎠⎦⎥
⎡ ∞ ∞⎤
1 ⎢⎛⎜ e −( s−b ) t ⎞⎟ ⎛⎜ e −( s+b )t ⎞⎟ ⎥
= +
2 ⎢⎜⎝ − ( s − b) ⎟⎠ ⎜⎝ − (s + b) ⎟⎠ ⎥
1 ⎡⎛ 0 − 1 ⎞ ⎛ 0 − 1 ⎞⎤ ⎣ 0 0⎦
= ⎢⎜ ⎟−⎜ ⎟⎥
2 ⎣⎜⎝ − ( s − b) ⎟⎠ ⎜⎝ − ( s + b) ⎟⎠⎦
1 ⎡ e −∞ − e 0 e −∞ − e 0 ⎤
= ⎢ + ⎥
1⎡ 1 1 ⎤ 2 ⎢⎣ − ( s − b) − (s + b) ⎥⎦
= −
2 ⎢⎣ s − b s + b ⎥⎦
1⎡ 1 1 ⎤
= +
1 ⎡ s + b − ( s − b) ⎤ 2 ⎢⎣ s − b s + b ⎥⎦
2 ⎢⎣ ( s + b)( s − b) ⎥⎦
=
1 ⎡ s + b + s −b ⎤
2 ⎢⎣ ( s − b)( s + b) ⎥⎦
=
1 ⎡ 2b ⎤
2 ⎢⎣ s 2 − b 2 ⎥⎦
=
1 ⎛ 2s ⎞
= ⎜ ⎟
2 ⎝ s 2 − b2 ⎠
b
⇒ L[sin h(bt)]= s
s − b2
2
⇒ L[cosh bt] =
s − b2
2

b
Now, L[eat sinh bt] = ( s − a)
( s − a) 2 − b 2 L[eat cosh bt] =
( s − a) 2 − b 2

(Q L[eat f(t)] = f (s – a)) [Q L[eat f(t)] = f (s – a)]

∴ L[e at sinh bt ] =
b (s − a)
∴ L[e at cosh bt ] =
(s − a)2 − b 2 (s − a) 2 − b 2
Hence proved. Hence proved.

SPECTRUM ALL-IN-ONE JOURNAL FOR ENGINEERING STUDENTS SIA GROUP


1.22 MATHEMATICS-II [JNTU-ANANTAPUR]

⎧ ⎛ 1 ⎞⎫
Q27. Evaluate L ⎨ e t ⎜ cos2t + sinh2t ⎟ ⎬ .
⎩ ⎝ 2 ⎠⎭

Ans: The given function is,

⎧ ⎛ 1 ⎞⎫
L ⎨e t ⎜ cos 2t + sinh 2t ⎟ ⎬ ... (1)
⎩ ⎝ 2 ⎠⎭

On simplifying equation (1), we get,

⎧ ⎛ 1 ⎞⎫ ⎧ 1 ⎫
L ⎨e t ⎜ cos 2t + sinh 2t ⎟ ⎬ = L ⎨e t cos 2t + et sinh 2t ⎬
⎩ ⎝ 2 ⎠⎭ ⎩ 2 ⎭

1
= L{e cos 2t} +
t
L{et sinh 2t} ... (2)
2

Consider L{et cos 2t},

Here, a = 1, b = 2

( s − 1) ⎡ (s − a) ⎤
⎢Q Le cos bt =
at
⇒ L{et cos 2t}= ⎥
2
( s − 1) + 2 2
⎣⎢ (s − a) 2 + b 2 ⎦⎥

s −1
= 2
s − 2s + 1 + 4

s −1
⇒ L{et cos 2t}= ... (3)
s − 2s + 5
2

Consider L{et sinh 2t},

2 ⎡ b ⎤
⎢Q L[e sinh bt ] =
t
⇒ L{et sinh 2t} = 2⎥
( s − 1) 2 − 2 2 ⎣⎢ ( s − a) − b ⎦⎥
2

2
=
s − 2s + 1 − 4
2

2
⇒ L{et sinh 2t] = 2 ... (4)
s − 2s − 3

On substituting equations (3) and (4) in equation (2), we get,

⎧ ⎛ 1 ⎞⎫ s −1 1 2
L ⎨e t ⎜ cos 2t + sinh 2t ⎟ ⎬ = 2 +
⎠⎭ s − 2 s + 5 s − 2s − 3
2 2
⎩ ⎝ 2

s −1 1
= 2
+ 2
s − 2s + 5 s − 2s − 3

⎧ ⎛ 1 ⎞⎫ s −1 1
∴ L⎨e t ⎜ cos 2t + sinh 2t ⎟⎬ = 2 + 2
⎩ ⎝ 2 ⎠⎭ s − 2 s + 5 s − 2s − 3

Look for the SIA GROU P LOGO on the TITLE COVER before you buy
UNIT-1 (Laplace Transform) 1.23
–3t
Q28. Find the Laplace transform of the function e (2 cos 5t – 3 sin 5t).
Ans: The given function is,
e–3t (2 cos 5t – 3 sin 5t)
The Laplace transform of above expression can be obtained as,
L[e–3t (2 cos 5t – 3 sin 5t) = L[e–3t (2 cos 5t)] – L[e–3t. (3 sin 5t)]
= 2L[e–3t.cos 5t] – 3 L[e–3t. sin 5t]

⎛ − at s+a ⎞
⎜Q L (e cos bt ) = ⎟
⎜ ( s + a) 2 + b 2 ⎟
⎡ s+3 ⎤ ⎡ 5 ⎤ ⎜ ⎟
=2⎢ ⎥ − 3⎢ ⎥ ⎜ L ( e − at sin bt ) =
b

⎣ ( s + 3) + 25 ⎦ ⎣ ( s + 3) + 25 ⎦
2 2
⎜ ( s + a) 2 + b 2 ⎟
⎝ ⎠

2( s + 3) 15
= −
( s + 3) + 25 ( s + 3)2 + 25
2

2 s + 6 − 15 2s − 9
= =
2
( s + 3) + 25 ( s + 3) 2 + 25

2s − 9
∴ L[e −3t (2 cos 5t − 3 sin 5t )] =
( s + 3) 2 + 25

1.4 TRANSFORMS OF DERIVATIVES AND INTEGRALS


Q29. Show that L{fn(t)} = sn f(s) – sn – 1f(0) – sn–2f' (0)..... fn– 1(0), where L{f (t)} = f (s). Model Paper-III, Q2
OR
Prove the theorem of derivatives on Laplace transforms.
Ans: Given that,

L[f(t)]= f (s)
And
L[f n(t)] = snf(s) – sn – 1 f(0) – sn – 2 f' (0) ... f n–1(0)
Consider,

L[ f ′(t )] = ∫e
0
–st f' (t) dt

= [f (t )e − st ]∞
0 – ∫ f (t) (– s)e
0
–st dt ⎡Q
⎢⎣ ∫ f ' (t ).dt = f (t )⎤⎥⎦


∫e
− st − st
= [e f (t )] 0 – ( − s ). f (t ) dt
0


= – f(0) + s e–st f (t) dt = –f (0) + s L[f (t)]
0

= s f (s) – f(0)
Similarly,
L[f'' (t)] = s2 f (s) – sf (0) – f' (0)

∴ L[fn (t)] = sn f (s) – sn–1f (0) – sn–2 f' (0).... fn–1(0)

SPECTRUM ALL-IN-ONE JOURNAL FOR ENGINEERING STUDENTS SIA GROUP


1.24 MATHEMATICS-II [JNTU-ANANTAPUR]
Q30. Find the Laplace transform of the following ⇒ L{2 a cos at – a2t sin at} = s2L{t sin at} – s(0.sin a
functions using the theorem on transforms of 0) – (sin a.0 + a(0) cos a(0))
derivatives. ⇒ L{2 a cos at} – L{a t sin at}= s2L{t sin at} – 0 – 0
2

(a) cos at ⇒ 2aL{cos at} – a2 L{t sin at}= s2L{t sin at}
⇒ L{t sin at} (s2 + a2) = 2a L{cos at}
(b) t sin at.
Ans: 2a s
⇒ L{t sin at} = .
s +a
2 2
s + a2
2
(a) cos at
⎛ s ⎞
The given function is, ⎜Q L{cos at } 2 ⎟
⎝ s + a2 ⎠
cos at 2 as
=
Let, (s + a 2 ) 2
2

f(t) = cos at 2as


∴ L{t sin at} =
Then, (s 2 + a 2 ) 2
f '(t)= – a sin at
⎧t ⎫ 1
Q31. Prove that L ⎪⎨ f(u) du ⎪⎬ = f(s), where L{f(t )} = f (s).
f ''(t)= – a2 cos at
⎪⎩ 0
∫ ⎪⎭ s
According to the theorem of derivatives, we have,
L{f n(t)} = sn L{f(t)} – sn – 1 f(0) – sn – 2 f '(0).... Ans: Given that,
∴ L{f''(t)} = s2 L{f(t)} – sf(0) – f '(0) L[f(t)] = f (s )
⇒ L{– a2 cos at} = s2 L{cos at} – s.cos a(0) – (– a sin a(0)) ⎡t ⎤ f (s)
⇒ 2 2
– a L{cos at } = s L{cos at} – s.1 – 0 ∫
L ⎢ f (u ) du ⎥ =
⎢0 ⎥ s
⎣ ⎦
⇒ 2
L{cost at} (s + a ) = s 2

Consider L.H.S,
s
⇒ L{cos at} = ⎡t ⎤ ∞ ⎡t ⎤
s2 + a2

L ⎢ f (u ) du ⎥ =
⎢ ⎥ ∫ ∫
e − st ⎢ f (u ) du ⎥ dt
⎢ ⎥
⎣0 ⎦ 0 ⎣0 ⎦
s
∴ L{cos at} = ⎛ ∞ ⎞
s + a2 ⎜Q L[ f (t )] = e −st f (t )dt ⎟

2
⎜ ⎟
(b) tsin at ⎝ 0 ⎠

The given function is, ⎡ e − st t ⎤ ∞
1
t sin at =⎢
⎢ −s ∫ ⎥
f ( u ) du +
⎥ s ∫
e − st f ( t ) dt
⎣ 0 ⎦0 0
Let,
⎡ d ⎛t ⎞ ⎤
⎢Q ⎜ f (u ).du ⎟ = f (t )⎥
f(t) = t sin at
⎢ dt ⎜
⎝0


∫ ⎥
Then, ⎣ ⎦

f '(t) = t(a cos at) + sin at(1) 1


=0+ L[f (t)]
⇒ f '(t) = sin at + at cos at s
And f ''(t) = [sin at + at cos at]1 ⎛ 0 ⎞
⎜Q
= a cos at + [at (– a sin at) + cos at.a] ⎜ ∫ f (u ) du = 0 ⎟⎟
⎝ 0 ⎠
= a cos at – a2t sin at + a cos at 1
⇒ f''(t) = 2 a cos at – a2t sin at = f (s)
s
According to the theorem of derivatives, t
f (s )
L{f n(t)} = sn L{f(t)} – sn–1 f(0) – sn – 2 f'(0) ...... ∫
∴ L f (u ) du =
0
s
∴ L{f''(t)} = s2 L{f(t)} – sf(0) – f '(0)
Hence proved.

Look for the SIA GROU P LOGO on the TITLE COVER before you buy
UNIT-1 (Laplace Transform) 1.25

⎡ t e− t sint ⎤
Q32. Find L ⎢
⎢0 ∫t
dt ⎥ .

⎣ ⎦

Ans: The given function is,

⎡ t e −t sin t ⎤
⎢∫
⎢⎣ 0 t
dt ⎥
⎥⎦

Consider, sin t

1 ⎛ a ⎞
⇒ L[sin t] = ⎜Q L[sin at ] = 2 ⎟
s +12
⎝ s + a2 ⎠

1 ⎛ b ⎞
⇒ L[e–t sin t] = ⎜Q L[e −at sin bt ] = ⎟
⎜ 2⎟
(s + a) + b ⎠
2
( s + 1) 2 + 1 ⎝

Then,

⎡ e −t sin t ⎤ ∞ ⎛ ∞ ⎞
⎜Q L ⎡ f (t ) ⎤ = f ( s)ds ⎟
L⎢
⎣⎢ t
⎥ =
⎦⎥
∫ L[e −t sin t ]ds ⎜ ⎢
⎣ t ⎦ s
⎥ ∫ ⎟
s ⎝ ⎠


ds
= ∫ ( s + 1)
s
2
+1


⎡ −1 ⎛ s + 1 ⎞⎤ ⎡ 1 1 ⎤
= ⎢tan ⎜
⎣ ⎝ 1 ⎠⎦ s
⎟⎥
⎣ ∫
⎢Q 2
x + a2
dx = tan −1 ( x / a ) ⎥
a ⎦

= tan–1 (∞) – tan–1 (s + 1)

⎛ π⎞
= π/2 – tan–1 (s +1) ⎜Q tan −1 ∞ = ⎟
⎝ 2⎠

⎡ e −t sin t ⎤ ⎛ −1 π −1 ⎞
⇒ L⎢ –1
⎥ = cot (s +1) ⎜Q cot θ = − tan θ ⎟
⎝ 2 ⎠
⎣⎢ t ⎦⎥

⎡t ⎤ 1
⎢0 ∫
Since, L ⎢ g (t ) dt ⎥ = g ( s ) , we get,
⎥ s
⎣ ⎦

⎡ t e −t sin t ⎤ 1
L⎢
⎢ ∫ t
⎥ dt = . cot −1 (s + 1)
⎥ s
⎣0 ⎦

⎡ t e −t sin t ⎤ 1
∴ L⎢
⎢0 ∫
t
dt ⎥ = ⋅ cot −1 ( s + 1)
⎥ s
⎣ ⎦

SPECTRUM ALL-IN-ONE JOURNAL FOR ENGINEERING STUDENTS SIA GROUP


1.26 MATHEMATICS-II [JNTU-ANANTAPUR]

1.5 UNIT STEP FUNCTION [e − st ]∞2


Q33. Define unit step function and find the Laplace =
−s
transform of unit step function.
Ans:
0 − e −2 s
Unit Step Function =
−s
For answer refer Unit-I, Q7.
Laplace Transform of Unit Step Function: From the definition
of Laplace transform, e −2 s
⇒ L{u(t – 2)} = ... (2)
s

∫e
− st
L{f(t)}= f (t ) dt From equation (1),
0


d2
L{t2u(t – 2)} = (–1)2 [L{u(t – 2)}]
∫e ds 2
− st
L{u(t – a)}= u (t − a ) at
0
dn
a ∞ [Q L{tn f(t)} = (−1)n f (s) ]
ds n
∫e ∫
− st
= .0dt + e − st .1dt
0 a
d 2 ⎡ e −2 s ⎤
= 0+
[ ] ∞
e − st a
= ⎢
ds 2 ⎢⎣ s ⎥⎦
⎥ [Q From equation (2)]
−s
− as
e −∞ − e − as − e d ⎡ s.( −2e −2 s ) − e −2 s .1 ⎤
= = = ⎢ ⎥
−s −s ds ⎣⎢ s2 ⎦⎥
e − as
=
s d ⎡ − 2se−2 s − e −2 s ⎤
= ⎢ ⎥
e − as ds ⎢⎣ s2 ⎥⎦
∴ L{u (t − a)} =
s
Q34. Evaluate L{t2 u(t – 2)}. d ⎡ e −2 s (2 s + 1) ⎤
= ( −1) ⎢ ⎥
Ans: The given Laplace function is, s2
ds ⎣⎢ ⎦⎥
L{t2u (t – 2)} ... (1)
Consider, L{u(t – 2)}
Here, ⎡ s 2 .{e −2 s .2 + (2s + 1)(−2e −2 s )} − e −2s (2s + 1)( 2s) ⎤
= (−1) ⎢ ⎥
⎣⎢ s4 ⎦⎥
⎧ 0, t < a
u(t – 2) = ⎨
⎩ 1, t > 2
−1
= [ 2 s 2e − 2 s − 4 s 3e − 2s − 2 s 2 e − 2s − 4 s 2 e − 2s − 2 se −2 s ]
∞ s4

− st
∴ L{u(t – 2)}= e u (t − 2)dt
0 e −2 s
∞ =– [ −4 s 3 − 4 s 2 − 2 s ]
s4

[Q L{f (t)} = e −st . f (t ) dt ]
0
2 ∞ e −2 s
= (4 s 3 + 4 s 2 + 2s )
∫ ∫
− st − st
= e u (t − 2 ) + e u (t − 2) dt s4
0 2
2 ∞
2 s.e −2 s
( 2 s 2 + 2 s + 1)

= e − st .0 + e − st .1dt
0
∫2
=
s 4


2e −2 s
∴ L{t 2 u (t − 2)} = (1 + 2s + 2s 2 )

− st
= 0+ e .dt
s3
2

Look for the SIA GROU P LOGO on the TITLE COVER before you buy
UNIT-1 (Laplace Transform) 1.27
1.6 SECOND SHIFTING THEOREM Upper Limit
When t = ∞; x = ∞
Q35. State and prove second shifting theorem. On substituting the corresponding values in
equation (4), we get,
Ans:


Statement −s ( x+ a )
f ( x ) dx
L[g(t)] = e
For answer refer Unit-I, Q8. 0

Proof ∞


− sx − sa
= e .e f ( x ) dx
Given that,
0

L[f(t)] = f (s) ... (1) ∞

∫e
− sa − sx
=e f ( x) dx
And 0

⎧ f (t − a ) ; t > a = e − sa L[ f (t )] [Q From equation (3)]


g(t) = ⎨ ... (2)
⎩0 ;t < a = e − sa . f ( s ) [Q From equation (1)]
According to the definition of Laplace transform,
∴ L[ g (t )] = e − sa f ( s )
∞ Hence, the statement is proved.
L[f(t)] = ∫ e −st . f (t ) dt ... (3) Q36. Find L–1 [e–2s/(s2 + 4s + 5)].
0 Ans: The given function is,

∞ ⎧⎪ e −2 s ⎫⎪
L–1 ⎨ 2 ⎬ ... (1)
∫e
− st
∴ L[g(t)] = .g (t ) dt ⎪⎩ s + 4 s + 5 ⎪⎭
0
Equation (1) can also be written as,
∞ ∞ ⎧⎪ e −2 s ⎫⎪ −1 ⎧
⎪ e −2 s ⎫⎪
= ∫
0

e − st .g (t ) dt + e − st .g (t ) dt
a
L–1 ⎨ 2 ⎬= L ⎨
⎪⎩ s + 4 s + 5 ⎪⎭ 2⎬
⎪⎩ (s + 2) + 1 ⎪⎭
2 ... (2)

[Q From equation (2)] ⎧⎪ 1 ⎫⎪


Consider, L–1 ⎨ 2⎬
⎪⎩ ( s + 2) + 1 ⎪⎭
2
a ∞

= ∫ ∫
e −st .0 dt + e −st . f (t − a ) dt
⎧⎪ 1 ⎫⎪
0 a ∴ L–1 ⎨ 2⎬
= e–2tsint
⎪⎩ ( s + 2) + 1 ⎪⎭
2


⎛ ⎞

− st b
= 0 + e f (t − a ) dt ⎜Q L[e −at sin bt ] = ⎟
⎜ 2⎟
(s + a) + b ⎠
2
a ⎝
According to Second shifting theorem,

∫e
− st
∴ L{g(t)] = f (t − a ) dt ... (4) ⎧⎪ e −2 s ⎫⎪
L–1 ⎨ ⎬ = f(t – 2) H(t – 2)
a ⎪⎩ ( s + 2) + 1 ⎪⎭
2

Let, [Q L−1[e − as f (s ) = f (t − a) H (t − a) ]
t – a=x ⇒ t=x +a = [e–2(t – 2).sin(t – 2)]H(t – 2)
[Q f(t) = e–2t sin t]
dt – 0 = dx
= e 4−2t . sin(t − 2) H (t − 2)
⇒ dt = dx
Lower Limit ⎧⎪ e −2 s ⎫⎪
∴ L−1 ⎨ ⎬=e
4− 2t
. sin(t − 2).H (t − 2)
When t = a ; x = 0 ⎪⎩ ( s + 2) + 1 ⎪⎭
2

SPECTRUM ALL-IN-ONE JOURNAL FOR ENGINEERING STUDENTS SIA GROUP


1.28 MATHEMATICS-II [JNTU-ANANTAPUR]
On substituting corresponding values in equation (2),
⎧ ⎛ 2π ⎞ 2π
⎪⎪cos⎜ t − 3 ⎟, t > 3 we get,
⎝ ⎠
Q37. Find L[g(t)] where, g(t) = ⎨ 2π . ∞ −s⎛ u + 2 π ⎞
⎪ 0, t< ⎜ ⎟
⎩⎪ ∫e ⎝ 3 ⎠
3 L[g(t)] = cosu.du
0
Ans: The given function is,
∞ 2π
− s
∫e
− su 3
= .e cosu.du
⎧ ⎛ 2π ⎞ 2π
⎪⎪cos ⎜⎝ t − ⎟⎠ , t > 0
3 3
g(t) = ⎨ ... (1) −2 π ∞ −2 π
⎪ 0, 2π s s
⎪⎩
t<
3
=e ∫
3 . e − su . cos u. du

0
=e 3 .L[cos u ]

On applying Laplace transform, we get, ⎛ ∞ ⎞


⎜Q e − su cos t dt = L[cos t ]⎟
∞ ⎜ ∫ ⎟
⎝ 0 ⎠
∫e
− st
L[g(t)] = g (t ) dt

0 − .s ⎛ s ⎞
=e 3
⎜ 2 ⎟
2π 3 ∞ ⎝ s +1 ⎠
∫e ∫e
− st − st
= g (t )dt + g (t )dt
⎛ s ⎞
0 2π 3 ⎜Q L{cos at} = 2 ⎟
⎝ s + a2 ⎠
2π 3 ∞
⎛ 2π ⎞ −2 π
= ∫ e − st (0)dt + ∫
2π 3
e −st cos ⎜ t −

⎟dt
3 ⎠ ∴L[ g (t )] = e 3
.s ⎛
⎜ 2
s ⎞

0
⎝ s +1⎠

⎛ 2π ⎞ 9s 2 − 12s + 5
=0+ ∫
2π 3
e − st cos⎜ t − ⎟dt
⎝ 3 ⎠
Q38. If L[f(t)] =
(s − 1) 3
. Find L[f(3t)] using

change of scale property.


Ans: The Laplace transform of f(3t) is obtained as,

⎛ 2π ⎞
∫e
− st
= cos⎜ t − ⎟dt ... (2) 1 ⎛s⎞ ⎛ 1 ⎛ s ⎞⎞
⎝ 3 ⎠ L[f (3t)] = f⎜ ⎟ ⎜⎜Q L[ f ( at )] = f ⎜ ⎟ ⎟⎟
2π 3 3 ⎝3⎠ ⎝ a ⎝ a ⎠⎠

Let,
⎡ ⎛ s ⎞2 ⎛s⎞ ⎤
⎢ 9⎜ ⎟ − 12 ⎜ ⎟ + 5 ⎥
2π 1 ⎢ ⎝3⎠ ⎝3⎠ ⎥
t– =u = ⎢ 3 ⎥
3 3 ⎛s ⎞
⎢ ⎜ − 1⎟ ⎥
⎢⎣ ⎝3 ⎠ ⎥⎦

⇒ t=u+ ⎡ 9 × s2 s ⎤
− 12. + 5 ⎥
3 1⎢ 9 3
= ⎢ ⎥
⇒ dt = du 3⎢ ( s − 3) 3 ⎥
⎢⎣ 27 ⎥⎦
For Lower Limits
1 ⎡ s 2 − 4s + 5 ⎤
= ⎢ ⎥ × 27
2π 2π 2π 3 ⎣⎢ ( s − 3) 3 ⎦⎥
For t = ,u= − =0
3 3 3
9( s 2 − 4 s + 5)
=
For Upper Limits ( s − 3) 3

2π 9( s 2 − 4 s + 5)
t = ∞, u = ∞ – = ∞. ∴ L[ f (3t )] =
3 ( s − 3) 3

Look for the SIA GROU P LOGO on the TITLE COVER before you buy
UNIT-1 (Laplace Transform) 1.29
Q39. Find L[e–3t sinh 3t] using change of scale
property.
1.7 DIRAC’S DELTA FUNCTION
Q40. Define Dirac delta function or the unit impulse
Ans: The given function is, function. Also obtain its Laplace transform.
L[e–3t sinh 3t] Ans:
Dirac Delta Function
Let, f(t) = e–t sinht For answer refer Unit-I, Q9.
Laplace Transform of Dirac Delta Function: The Laplace
1 ⎡ a ⎤
⎢Q L[sinh at ] = s 2 − a 2 ⎥
L[sinh t] = transform of Dirac delta function is given by,
s − 12
2
⎣ ⎦
L{δ(t – a)} = L ⎡⎢ Lt f ε (t − a ) ⎤⎥
⎣ε→0 ⎦
1
= 2 = Lt L{ f ε (t − a)}
s −1 ε →0

⎡∞ ⎤
L[e–t sinh t] =
1
( s + 1) 2 − 1
= Lt
ε →0 ⎢ ∫
⎢ e −st f ε (t − a )dt ⎥

⎣0 ⎦
⎛ ∞ ⎞
⎜Q L{ f (t )} = e −st . f (t )dt ⎟


t
⎜Q L[e −a sinh bt =
b ⎞

2⎟
⎜ ∫ ⎟
⎝ (s + a) − b ⎠
2 ⎝ 0 ⎠
⎡a a+ε ∞ ⎤
1 = Lt ⎢ ∫ e − st f ε (t − a ) dt + ∫ e − st f ε (t − a )dt + ∫ e − st f ε (t − a) dt ⎥
L[e– t sinh t] = 2 ε →0 ⎢ ⎥
⎣0 a +ε ⎦
s + 1 + 2s − 1
2 a

⎛ ⎧0 , t<a ⎞
⎜ ⎟
1 ⎜Q f ε (t − a ) = ⎪⎨ 1 , a ≤ t ≤ a + ε⎟
= ⎜ ⎟
s + 2s + 1 − 1
2
⎜ ⎪ε ⎟
⎝ ⎪⎩ 0 , t>a ⎠
1 ⎡a a+ ε ∞ ⎤
= − st 1
∫ ∫ ∫
− st − st
s + 2s
2
= Lt ⎢ e .0dt + e . dt + e .0dt ⎥
ε →0 ⎢ ε ⎥
⎣0 a a+ ε ⎦
From the change of scale property,
⎡ 1 a+ε ⎤

⎡ ⎤
= Lt ⎢
ε →0 ⎢ ε ∫
e − st dt ⎥

⎣ 0 ⎦
⎢ ⎥
∴ L[e–3t sinh 3t] =
1 ⎢
3 ⎢ ⎛ s ⎞2
1
⎛ s ⎞⎥

= Lt ⎢
[ ]
⎡ 1 e −st a +ε ⎤
a ⎥
⎢ ⎜ ⎟ + 2⎜ ⎟ ⎥ ε →0 ⎢ ε −s ⎥
⎣ ⎦
⎣⎢ ⎝ 3 ⎠ ⎝ 3 ⎠ ⎦⎥
⎡ e − s ( a +ε ) − e − sa ⎤
⎛ 1 ⎛ s ⎞⎞ = Lt ⎢ ⎥
⎜⎜Q L[ f (at )] = f ⎜ ⎟ ⎟⎟ ε →0 ⎢
⎣ − εs ⎦⎥
⎝ a ⎝ a ⎠⎠
= Lt ⎢
(
⎡ e − sa e − sε − 1 ⎤

)
⎡ ⎤ ⎡ ⎤ ε →0 ⎢
⎣ − εs ⎥⎦
⎢ ⎥ ⎢ 1 ⎥
=
1

3 ⎢s 2
1
⎥ =
2s ⎥
1
⎢ 2 ⎥
3 ⎢ s + 6s ⎥ − sa (
⎡ 1 − e − sε ⎤
⎢ ⎥
)
= e εLt
⎢⎣ 9 + 3 ⎥⎦ ⎢⎣ 9 ⎥⎦ ⎣ εs ⎥⎦
→0 ⎢

On applying L.Hospital’s rule, we get,


1 ⎡ 9 ⎤ − sa ⎡ − (−ε)e − sε ⎤
=
3 ⎢⎣ s 2 + 6 s ⎥⎦ L{δ(t – a)} = e . Lt ⎢ ⎥
ε →0 ⎢
⎣ ε ⎥⎦
− sa
3 = e . εLt e − sε
= →0
s 2 + 6s
= e − sa .1
3 = e–sa
∴ L[ e −3t sinh 3t ] =
∴ L{δ(t − a )} = e − sa
2
s + 6s

SPECTRUM ALL-IN-ONE JOURNAL FOR ENGINEERING STUDENTS SIA GROUP


1.30 MATHEMATICS-II [JNTU-ANANTAPUR]
1.8 CONVOLUTION THEOREM Limits
For t = u ⇒ v = 0
Q41. State and prove convolution theorem.
For t = ∞ ⇒ v = ∞
Ans:
∞ ⎧⎪∞ ⎫⎪
Statement: Consider two functions f(t) and g(t) for t > 0. If
f (s ) and g (s ) are Laplace transforms of these functions then,
L{y(t)} = ∫
0 ⎪⎩ 0

e − su f (u )⎨ e − sv g (v )dv ⎬du
⎪⎭

∫e
− su
L{ f (t)*g(t)} = f ( s)· g ( s) = f (u ){ g ( s )}du
0
Proof ∞


− su
Let, y(t) = f(t) * g(t) = g ( s ) e f (u ) du = g ( s )· f ( s )
o
t
∴ L{ y(t )} = f ( s).g ( s)
Then, y(t) = ∫ f (u ) g (t − u ) du then,
0 Q42. Using Laplace transform solve,
Taking Laplace transform, we get,
t
∞ ⎧t
⎪ ⎫⎪ y(t) = 1 – e–t + ∫ y(t − u) sin u du.
∫e ∫
− st
L{y(t)}= ⎨ f (u ) g (t − u )du ⎬dt 0
⎪⎩ 0 ⎪⎭
0 Ans: Given function is,
∞ t
t
∫∫e
− st
f (u ) g (t − u ) du dt

= −t
y(t) = 1 − e + y (t − u ) sin u du ... (1)
0 0
0
On changing the order of integration from figure, we get,
From the definition of convolution,
u For y(t) = f(t) × g(t),
t

u=t
y(t) = ∫ f (u ) g (t − u ) du ... (2)
t=∞ 0
t=u
R On comparing equation (2) with equation (1), we get,
O t y(t) = 1 – e–t + y(t) × sin t
u=0
Taking Laplace transform, we get,
Figure L{y(t)} = L{1} – L{e–t} + L{y(t) × sint}
Where, 1 1
= − + L{ y (t )}.L{sin t}
R – Region of integration s s +1
Lt t → u to ∞
⎧ 1 ⎫
Lt u → 0 to ∞ ⎪⎪Q L(1) = s ⎪⎪
⎨ ⎬
∞∞ ⎪ L[e − at ] = 1 ⎪
⎪⎩ s + a ⎪⎭
∫∫e
− st
⇒ L{y(t)} = f (u ) g (t − u ) dt du
0 u
1 1
∞∞ ⇒ L{y(t)} = + L{ y (t )}. 2
s ( s + 1) s +1
∫∫e
− st + su − su
= f (u ) g (t − u ) du dt
0 u
⎡ ⎡1 ⎤ 1 ⎤
⎢Q L ⎢ sin at ⎥ = 2 ⎥
⎣ ⎣a ⎦ s + a2 ⎦
∞ ⎧⎪∞ ⎫⎪
∫ ∫
− su − s (t −u )
= e f ( u ) ⎨ e g (t − u )dt ⎬ du
⎪⎩ 4 ⎪⎭ 1 1
0 ⇒ L{ y (t )} − L{ y (t )}. =
s +12 s (s + 1)
Let,
t–u=v ⎡ s 2 + 1 − 1⎤ 1
⇒ L{ y (t )}⎢ 2 ⎥ =
⎢⎣ s + 1 ⎥⎦ s (s + 1)
⇒ dt = dv

Look for the SIA GROU P LOGO on the TITLE COVER before you buy
UNIT-1 (Laplace Transform) 1.31

s2 1
⇒ L{ y (t )}. =
s +1
2
s (s + 1)

1 s2 +1
⇒ L{y(t)} = ×
s (s + 1) s2

s2 +1
⇒ L{y(t)} =
s 3 ( s + 1)

⎧ 2
−1 ⎪ s + 1 ⎪

∴ y(t) = L ⎨ 3 ⎬
⎪⎩ s (s + 1) ⎪⎭

−1 ⎧
⎪ 1 1 ⎫⎪
= L ⎨ + 3 ⎬
⎪⎩ s( s + 1) s ( s + 1) ⎪⎭

⎧1 1 ⎫ −1 ⎧⎪ 1 ⎫⎪
= L−1 ⎨ − ⎬+ L ⎨ 3 ⎬
⎩ s s + 1⎭ ⎪⎩ s ( s + 1) ⎪⎭

⎧1 ⎫ ⎧ 1 ⎫ −1 ⎧⎪ 1 ⎫⎪
= L−1 ⎨ ⎬ − L−1 ⎨ ⎬+ L ⎨ 3 ⎬
⎩s ⎭ ⎩ s + 1⎭ ⎪⎩ s (s + 1) ⎪⎭

⎧⎪ 1 ⎫⎪ ⎧1⎫ ⎧ 1 ⎫
= 1 – et + L–1 ⎨ 3 ⎬ = 1 − e t + L−1 ⎨ 3 ⎬.L−1 ⎨ ⎬
⎪⎩ s (s + 1) ⎪⎭ ⎩s ⎭ ⎩ s + 1⎭

⎡ −1 ⎡ 1 ⎤ t n ⎤
⎢Q L ⎢ n +1 ⎥ = ⎥
⎢ ⎣ s ⎦ n! ⎥
⎛ t2 ⎞
= 1 − e t + ⎜ ⎟(e −t ) ⎢ − ⎡ 1 ⎤ ⎥
⎜2⎟ ⎢ L 1⎢ = e − at

⎝ ⎠ ⎥
⎣⎢ ⎣s + a⎦ ⎦⎥

⎛ t2 ⎞ −t
∴ y (t ) = 1 − e t + ⎜ ⎟( e )
⎜2 ⎟
⎝ ⎠
Q43. Using convolution theorem find, L–1[1/(s2 – 1) (s2 + 25)].
Ans: The given function is,
⎡ 1 ⎤
L−1 ⎢ 2 ⎥
⎣⎢ ( s − 1)( s + 25) ⎦⎥
2

1
Let, f (s ) = 2
s −1
⎧ 1 ⎫
then, f (t) = L−1 ⎨ 2 ⎬ = sin ht
⎩ s − 1⎭
1
Let, g (s ) = 2
s + 25
⎧ 1 ⎫ sin 5t
then, g(t) = L−1 ⎨ 2 ⎬=
⎩ s + 25 ⎭ 5

1
⇒ f ( s) g ( s) =
( s 2 − 1)(s 2 + 25)

SPECTRUM ALL-IN-ONE JOURNAL FOR ENGINEERING STUDENTS SIA GROUP


1.32 MATHEMATICS-II [JNTU-ANANTAPUR]
On applying Laplace transform on both sides, we get,

−1 ⎧
⎪ 1 ⎫⎪ −1 ⎧
⎪ 1 1 ⎫⎪
⇒ L−1{ f ( s) g ( s )} = L ⎨ 2 ⎬ = L ⎨ 2 −1
⎬ (Q L { f ( s) g (s )} = f (t ) * g (t ))
⎪⎩ ( s − 1)( s + 25 ) ⎪⎭
2
⎪⎩ ( s − 1) ( s + 25 ) ⎪⎭
2

= f (t) * g(t)

t ⎛ t ⎞
sin 5(t − u ) ⎜Q f (t ) * g (t ) = f (t ) g (t − u)du ⎟
= ∫ sin ht
5
du ⎜ ∫ ⎟
0 ⎝ 0 ⎠

On multiplying and dividing by 2, we get,

t
−1 ⎧
⎪ 1 ⎫⎪ 2
⇒ L ⎨
⎪⎩ ( s − 1)( s + 25) ⎪⎭
2 2
0

⎬ = 5 × 2 sin ht sin 5( t − u ).du

t
1
=
10 ∫
2 sin ht sin 5(t − u ).du
0
[Q 2 sin A sin B = cos(A – B) – cos(A + B)]

t
1
=
10 ∫
cos( ht − (5t − 5u )) − cos( ht + (5t − 5u )) du
0

t
1
=
10 ∫ cos( ht − 5t + 5u ) − cos( ht + 5t − 5u ) du
0

t
1 ⎡ sin(ht − 5t + 5u ) sin(ht + 5t − 5u ) ⎤
+
10 ⎢⎣ ⎥
=
5 5 ⎦0

1 ⎡ sin( ht − 5t + 5t ) sin( ht + 5t − 5t ) sin( ht − 5t + 0) sin( ht + 5t − 0) ⎤


+ − −
10 ⎢⎣ ⎥
=
5 5 5 5 ⎦

1 ⎧1 ⎫
= ⎨ (sin ht + sin ht − sin( ht − 5 t ) − sin( ht + 5t )) ⎬
10 ⎩ 5 ⎭

1
= {2 sin ht − [sin ( ht − 5t ) + sin( ht + 5t )]}
50

1
= {2 sin ht − ( 2 sin ht cos 5t )} (Q 2 sin A cos B = sin(x + y) + sin(x – y))
50

2 sin ht
= {1 – cos 5t}
50

sin ht
= {1 – cos 5t}
25

⎧⎪ 1 ⎫⎪ sin ht
∴ L−1 ⎨ 2 ⎬= {1 − cos 5t}
⎪⎩ ( s − 1)( s + 25) ⎪⎭
2 25

Look for the SIA GROU P LOGO on the TITLE COVER before you buy
UNIT-1 (Laplace Transform) 1.33

⎡ s+3 ⎤
Q44. Find the inverse Laplace transformation of ⎢ .
2 2⎥
⎣ (s + 6s + 13) ⎦
s+3
Ans: The given function is,
( s + 6 s + 13) 2
2

–1
⎡ s+3 ⎤ –1
⎡ s+3 ⎤
L ⎢ 2 2 ⎥ =L ⎢ ⎥
⎣⎢ ( s + 6 s + 13) ⎦⎥ ⎣⎢ [( s + 3) + 4][( s + 3) + 4] ⎦⎥
2 2

s+3
Let, f (s) =
( s + 3) 2 + 2 2

⎡ s+3 ⎤ –3t ⎡ −1 ⎛ s+a ⎞ ⎤


⇒ f(t) = L−1 ⎢ ⎢Q L ⎜⎜ ⎟ = ( e − at cos bt ) ⎥
2⎥=e cos 2t ⎟
⎣⎢ ( s + 3) + 2 ⎦⎥
2
⎣⎢ ⎝ ( s + a)2 + b 2 ⎠ ⎦⎥
1
And g(s) =
( s + 3) 2 + 2 2

⎛ ⎞ ⎡ −1 ⎛ 1 ⎞ bt ⎤
–1 ⎜ 1 ⎟ = e −3t sin 2t ⎢Q L ⎜⎜ ⎟ = e − at sin ⎥
⇒ g(t) = L ⎜ ( s + 3) 2 + 2 2 ⎟ ⎟
⎝ ⎠ 2 ⎣⎢ ⎝ ( s + a ) 2
+ b2 ⎠ b ⎦⎥

From convolution theorem, we get,


⎡ s+3 ⎤ t t
–1
L ⎢ 2 2⎥ =
⎣⎢ ( s + 6 s + 13) ⎦⎥
∫ f (u ) g (t − u ) du –1
(Q L [ f (s ) g (s )] = f(t) * g(t) = ∫ f (u ) g (t − u )du )
0
0


sin 2(t − u )
∫e
− 3u
= cos 2u.e −3(t −u ) du
2
0
t
1
=
2 ∫
e − 3u cos 2u e − 3t e 3u sin( 2t − 2u ) du
0
[Q e
–3u 3u 0
.e = e = 1]

t
1 − 3t
=
2
e ∫
cos 2u sin( 2t − 2u ) du
0
On multiply and divide with 2, we get,
t
1 e −3 t
=
2 2 ∫ 2 cos 2u sin( 2t − 2u ) du
0

1 − 3t t
=
4
e ∫
[sin( 2u + 2t − 2u ) − sin( 2u − 2 t + 2u )]du [Q 2 cos A sin B = sin (A + B) – sin (A – B)]
0

e − 3t ⎡ ⎤
t
e −3t
t t
=
4 ∫
[sin 2t − sin( 4u − 2t )]du =
4 ⎢ ∫ ∫
⎢ sin 2t du − sin( 4u − 2t ) du ⎥

0 ⎣0 0 ⎦

e −3t ⎡ ⎡ − cos( 4t − 2t ) ⎤ ⎤ e −3t ⎡


t t
⎡ cos( 4t − 2t ) cos(0 − 2t ) ⎤ ⎤
⎢ − ∫ ⎥ ⎥= ⎢sin 2t[t ]o + ⎢
t
= 4 ⎢ sin 2t du ⎢ − ⎥⎥
⎣ 0
⎣ 4 ⎦ 0⎦⎥ 4 ⎣ ⎣ 4 4 ⎦⎦

e −3t ⎡ ⎡ cos 2t cos 2t ⎤ ⎤ e −3t


⎢t sin 2t + ⎢ − ⎥=
4 ⎥⎦ ⎦
= [t sin 2t ]
4 ⎣ ⎣ 4 4
⎡ s+3 ⎤ e −3t
∴ L−1 ⎢ 2 2⎥
= t sin 2t
⎣⎢ ( s + 6 s + 13) ⎦⎥ 4

SPECTRUM ALL-IN-ONE JOURNAL FOR ENGINEERING STUDENTS SIA GROUP


1.34 MATHEMATICS-II [JNTU-ANANTAPUR]

1.9 LAPLACE TRANSFORM OF PERIODIC FUNCTION


T
1
∫e
− st
Q45. If f(t) is a periodic function with period T, prove that L{f(t)} = − sT
f(t) dt .
1− e 0
Ans: According to the definition of Laplace transform,

∫e
− st
L[f(t)] = f ( t ) dt
0

T 2T 3T

∫ e − st f ( t ) dt + ∫ e − st f ( t ) dt + ∫e
− st
= f ( t ) dt + … ... (1)
0 T 2T

As f(t) is a periodic function with period T, substituting t= u, t = u + T, t = u + 2T, … respectively, in equation (1), we get,

T T T

L{f(t)} = ∫
0

0

e − su f (u ) du + e − s ( u + T ) f ( u + T ) du + e − s ( u + 2T ) f ( u + 2T ) du + …
0

∴ f(u) = f(u + T) = f(u + 2T) = …


T T T

∫e ∫e ∫e
− su − sT − su − 2 sT − su
L{f(t)} = f (u ) du + e f (u ) du + e f (u ) du + …
0 0 0


− sT
= (1 + e + e − 2 sT + …) e − su f (u ) du
0

T
1 ⎛ 1 ⎞
∫e
− st −x −2 x
= f (t ) dt ⎜Q t = u and 1 + e + e ..... = ⎟
1 − e − sT 0
⎝ 1 − e−x ⎠

T
1
∫e
− st
∴ L{ f (t )} = f (t )dt
1 − e − sT 0
Hence proved.
Q46. Find L[f(t)] where f(t) is given by,
f(t) = t, 0 < t < b
= 2b – t, b < t < 2b, where 2b is the period of f(t)
Ans: Given that,
f(t) = t, 0 < t < b
= 2b – t, b < t < 2b ... (1)
Where, 2b is the period of f(t)
Laplace transform of a periodic function f(t) with a period ‘T’ is given by,
T

∫e
− st
f (t ).dt
L{ f (t )} = 0
1 − e − sT
2b
1
∫e
− st
∴ L{f(t)} = − 2 bs
f ( t ) dt (Q T = 2b)
1− e 0

1 ⎡b 2b ⎤
=
1 − e − 2 bs ∫
⎢ e − st f (t )dt + e − st f (t ) dt ⎥
⎢0 ⎥ ∫
⎣ b ⎦

Look for the SIA GROU P LOGO on the TITLE COVER before you buy
UNIT-1 (Laplace Transform) 1.35

1 ⎡ b − st 2b ⎤
=
1 − e −2 bs ∫
⎢⎣ 0 b

⎢ e tdt + e − st (2b − t )dt ⎥
⎥⎦
[Q From equation (1)]

⎧⎡ b ⎫
⎞ ⎤ ⎡⎢ ⎞ ⎤⎪
b 2b 2b
1 ⎪⎢ ⎛d ⎛ d
∫ ∫ ∫ ∫ ∫ ∫
− st − st − st − st
= ⎨ t e dt − ⎜ (t ) e dt ⎟dt ⎥ + ( 2b − t ) e .dt − ⎜ ( 2b − t ) e dt ⎟ dt ⎥ ⎬
1 − e − 2 bs ⎪⎩ ⎢⎣ 0 ⎝ dt ⎠ ⎥⎦ ⎢⎣ ⎝ dt ⎠ ⎥⎦ ⎪
0 b b ⎭

⎧⎡ ⎛ − st ⎞ 1 −st ⎤ ⎡
b b
⎛ e −st ⎞
2b 2b
− st ⎤⎫
1 ⎪⎢ ⎜ e ⎟ − ( −1) ( e ) dt ⎥ ⎪⎬
=
1 − e −2bs
⎨ t⎜
⎪⎩⎢⎣ ⎝ − s
⎟ +

⎠0 s 0

e dt ⎥ + ⎢ ( 2b − t )⎜
⎥ ⎢ ⎜ −s


⎠b
∫ −s ⎥⎪
⎦ ⎣ b ⎦⎭

⎧⎛ ⎛ −st b
⎞ 1 ⎛ e −st ⎞ ⎞⎟ ⎡
b
⎛ − st ⎞
2b
⎛ − st ⎞
2b ⎤ ⎫
1 ⎪⎜ ⎜ e ⎟ + ⎜ ⎟ + ⎢( 2 b − t ) ⎜ e ⎟ −1⎜e ⎟ ⎥ ⎪⎬
= ⎨⎜ t ⎜ ⎟ ⎜ ⎟
⎟ ⎟ ⎢ ⎜ −s ⎟
1 − e −2 bs ⎪⎩⎜⎝ ⎝ − s ⎠0 s ⎝ − s ⎠0 ⎠ ⎣ ⎝ ⎠b s ⎜⎝ − s ⎟
⎠b ⎥⎪
⎦⎭

⎧⎡ − st b b
⎤ ⎡ e − st ⎤ ⎡ e − st ⎤
2b
⎡ e − st ⎤ ⎡ e − st ⎤
2b 2b ⎫
1 ⎪ e ⎪
= − 2bs ⎨⎢t ⎥ + ⎢ 2 ⎥ + ⎢2b ⎥ + ⎢t ⎥ +⎢ 2 ⎥ ⎬
1− e ⎪⎩⎢⎣ − s ⎥⎦ 0 ⎢⎣ − s ⎥⎦ 0 ⎢⎣ − s ⎥⎦ b ⎢⎣ s ⎥⎦ b ⎢⎣ s ⎥⎦ b ⎪⎭

1 ⎧⎪⎡ b.e − sb ⎤ ⎡ e − sb e 0 ⎤ ⎡ e −2bs e −bs ⎤ ⎡ 2be −2bs be − sb ⎤ ⎡ e − s 2 b e −bs ⎤ ⎫⎪


= ⎨⎢ − 0⎥ + ⎢ 2 − ⎥ + ⎢ 2b − 2b ⎥+⎢ − ⎥+⎢ − 2 ⎥⎬
1 − e −2bs ⎪⎩⎣⎢ − s ⎥⎦ ⎣⎢ − s − s 2 ⎦⎥ ⎣⎢ −s − s ⎦⎥ ⎣⎢ s s ⎦⎥ ⎣⎢ s 2 s ⎦⎥ ⎪⎭

1 ⎪⎧ − e −bs e −bs 1 e −2bs e −bs 2be −2bs e −bs e −2 bs e −bs ⎫⎪


= ⎨ b − + − 2b + 2b + − b + − 2 ⎬
1 − e −2 bs ⎪⎩ s s2 s2 s s s s s2 s ⎪⎭

1 ⎡1 − bs − 2bs 1 ⎤
= − 2bs ⎢ 2 (− e + 1 + e − e −bs ) + ( −be −bs − 2be − 2bs + 2be −bs + 2be −2bs − be −bs ) ⎥
1− e ⎣s s ⎦

1 ⎡ − 2e −bs + 1 + e −2bs ⎤ 1
= ⎢ + 0⎥ = 2 (1 − 2e −bs + e −2bs )
1 − e −2bs ⎢⎣ s 2
⎥⎦ s (1 − e −2bs )
1
= (1 − e −bs ) 2 [Q a2 – 2ab + b2 = (a – b)2]
s (1 − e −2bs )
2

(1 − e −bs )(1 − e −bs )


= [Q (a – b)2 = (a – b) (a – b); (a2 – b2) = (a + b) (a – b)]
s 2 (1 + e −bs )(1 − e −bs )

1 − e − bs 1 ⎡ 1 − e − bs ⎤
= = ⎢ ⎥
s 2 (1 + e −bs ) s 2 ⎢⎣1 + e −bs ⎥⎦

⎛ 1 + e − x e − x / 2 (e x / 2 − e − x / 2 ) ⎞
1 ⎡ e −bs / 2 (e bs / 2 − e − bs / 2 ) ⎤ ⎜Q ⎟
= ⎢ ⎥ ⎜ 1 + e − x = e − x / 2 (e x / 2 + e x / 2 ) ⎟
s 2 ⎣⎢ e −bs / 2 (e bs / 2 + e −bs / 2 ) ⎦⎥ ⎝ ⎠

1 ⎡ e bs / 2 − e − bs / 2 ⎤
= ⎢ bs / 2 ⎥
s2 ⎢⎣ e + e −bs / 2 ⎥⎦

⎛ −x ⎞
⎜Q tan hx = e − e
x
1 ⎛ bs ⎞ ⎟
= tan h⎜ ⎟ ⎜ ⎟
s2 ⎝ 2⎠ ⎝ e x + e−x ⎠

1 ⎛ bs ⎞
∴ L[ f (t )] = 2
tan h⎜ ⎟
s ⎝ 2⎠

SPECTRUM ALL-IN-ONE JOURNAL FOR ENGINEERING STUDENTS SIA GROUP


1.36 MATHEMATICS-II [JNTU-ANANTAPUR]
Q47. Find the Laplace transform of periodic function f(t) with period T,
4Et T 4Et T
where f(t) = − E, 0 ≤ t ≤ = 3E − , ≤ t ≤ T. Model Paper-III, Q3
T 2 T 2
Ans: Given that,
f(t) is a periodic function of period T and is given by,

⎧ 4 Et T
⎪ T −E ; 0≤t ≤ 2
f(t) = ⎨
4 Et T
⎪3E − ; ≤t ≤T
⎩ T 2
Laplace transform of a periodic function f(t) with period ‘T’ is given by,
T
1
∫e
− st
L{f(t)} = f (t ) dt
1 − e −sT
0

T /2 T
1
∫ e − st f (t ) dt + ∫e
− st
= − sT
f (t ) dt
1− e 0 T /2

⎡T / 2 T
4 Et ⎞ ⎤
1 − st ⎡ 4 Et ⎤ − st ⎛
= ⎢
(1 − e − sT ) ⎢⎣ 0
e ∫ ⎢ T

− E ⎥

dt + e ⎜

3E − ∫⎟dt ⎥
T ⎠ ⎥
T/2 ⎦

⎡⎧ E T / 2 T /2 ⎫⎪⎤ ⎧⎪ T T ⎫⎪
1 ⎢⎪⎨ 4 4E
=
1 − e − sT ⎢⎪ T ∫ ∫
t.e − st dt − E e − st dt ⎬⎥ + ⎨3E e − st dt −
⎪⎭⎥⎦ ⎪⎩ T / 2 T ∫
te − st dt ⎬
⎪⎭

⎣⎩ 0 0 T /2

⎡ − st
T /2 T /2 T T ⎤
⎢ 4 E ⎛⎜ − te
1 e − st ⎞

⎛ e − st ⎞ ⎛ 3Ee − st ⎞ ⎛ − st
⎟ − 4 E ⎜ − te − e
− st ⎞ ⎥
= − sT ⎢ T ⎜
− 2 + E⎜ ⎟ +⎜ ⎟
(1 − e ) ⎢ ⎟ ⎜ s ⎟ ⎜ −s ⎟T T ⎜⎝ s2 ⎟T ⎥
⎝ s s ⎠0 ⎝ ⎠0 ⎝ ⎠ s ⎠ ⎥
⎣ 2 2⎦

⎡ ⎡⎛ − T − sT / 2 ⎞ ⎤ ⎡ ⎛ − T −sT / 2 ⎞⎤
⎢ 4E ⎢⎜ 2 e e − sT / 2 ⎟⎟ ⎛ ⎞⎥ ⎛ e − sT / 2 1 ⎞ 3E −sT 4 E ⎢⎛⎜ Te − sT e −sT ⎞ ⎜ 2 e e −sT / 2 ⎟⎥
=
1

(1 − e − sT ) ⎢ T
⎢ ⎜ −
1
−⎜0 − 2 ⎟⎥ + E ⎜
⎜ s − ⎟ −
s ⎟⎠ s
(e − e − sT / 2
)− ⎢ −
T ⎢⎜⎝
− 2 ⎟ −⎜
⎟ ⎜ − ⎟⎥
⎢⎜⎜ s s2 ⎟ ⎝

s ⎠⎥ ⎝ s s ⎠ ⎜ s s2 ⎟⎥

⎢⎣ ⎣⎢⎝ ⎠ ⎦⎥ ⎢⎣ ⎝ ⎠ ⎦⎥

1 ⎡ 2 E − sT / 2 4 E − sT / 2 4 E E − sT / 2 E 3E − sT 3E − sT / 2 4 E − sT 4 E − sT 2 E − sT / 2 4 E − sT / 2 ⎤
= ⎢− e − 2e + 2+ e − − e + e + e + 2e − e − 2e ⎥
1 − e − sT ⎣ s Ts Ts s s s s s Ts s Ts ⎦

=
1 ⎡E
− sT ⎢ s
(1 − e ) ⎣
( 4E
) ( ⎤
− 2e − sT / 2 + e − sT / 2 − 1 + 3e − sT / 2 − 3e − sT + 4e − sT − 2e − sT / 2 + 2 − e − sT / 2 + 1 + e − sT − e − sT / 2 ⎥ )
Ts ⎦

1 ⎡ E − sT 4E ⎤
= − sT ⎢ s
(e − 1) + 2 (e − sT + 1 − 2e − sT / 2 )⎥
(1 − e ) ⎣ Ts ⎦

1 ⎡− E 4E ⎤
= − sT ⎢
(1 − e − sT ) + 2 (1 − e − sT / 2 ) 2 ⎥ [Q a2 – 2ab + b2 = (a – b)2]
1− e ⎣ s Ts ⎦

=
− E ⎛⎜ 1 − e −sT (
⎞ 4 E 1 − e −sT / 2
⎟+ ) 2

s ⎜⎝ 1 − e −sT ⎟ Ts 2 1 − e −sT

−E 4E (1 − e− sT /2 )2
= × 1 + [Q a2 – b2 = (a – b) (a + b)]
s Ts2 (1 − e−sT /2 )(1 + e−sT /2 )

Look for the SIA GROU P LOGO on the TITLE COVER before you buy
UNIT-1 (Laplace Transform) 1.37

− E 4 E ⎛⎜ 1 − e − sT / 2 ⎞⎟ − E 4E ⎛ 1 − e −sT / 4 .e − sT / 4 ⎞
= + 2 = + 2⎜ ⎟
s Ts ⎜⎝ 1 + e −sT / 2 ⎟⎠ s Ts ⎜⎝ 1 + e − sT / 4 .e −sT / 4 ⎟⎠

⎛ − sT / 4 ⎞
⎜1− e ⎟
− E 4E ⎜ ⎟ − E + 4 E ⎛⎜ e − e − sT / 4 ⎞⎟
sT / 4
+ 2⎜ e sT / 4
= ⎟ = s Ts 2 ⎜ e sT / 4 + e −sT / 4 ⎟
s Ts e − sT / 4 ⎝ ⎠
⎜⎜ 1 + sT / 4 ⎟⎟
⎝ e ⎠

− E 4E ⎛ sT ⎞ ⎛ e x − e− x ⎞
+ 2 tanh⎜ ⎟ ⎜Q = ⎟
= ⎜ e x + e − x tanh x ⎟
s Ts ⎝ 4 ⎠ ⎝ ⎠

E 4E ⎛ sT ⎞
∴ L{ f (t )} = − + tanh⎜ ⎟
s Ts 2 ⎝ 4 ⎠

1.10 DIFFERENTIATION AND INTEGRATION OF TRANSFORM


dn
Q48. Show that L{tn(f(t))} = (–1)n { f (s)} where, n = 1, 2, 3,.....
dsn
OR

dn
Prove that L{tn f(t)} = (–1)n { f (s)} where, n = 1, 2, 3,.....
dsn
Ans: Given that,

dn
L[tn f (t)] = (–1)n { f (s)} ... (1)
ds n
Where, n = 1, 2, 3,.....
From the definition of Laplace transform,

f (s)= ∫e
0
–st f(t) dt

= L[ f(t)]
On differentiating on both sides with respect to s, we get,

d d

ds
f (s ) =
ds ∫e
0
–st f(t)dt

∞ ∞
d –st
= ∫
0
ds
e f(t)dt = ∫ – te
0
–st f(t)dt


= – e–st [t f(t)]dt = – L[t f(t)]
0

d
⇒ L[t f (t)] = − f (s) ... (2)
ds
By mathematical induction, equation (2) can be written as,
dn f
L[tn f(t)] = (–1)n (s)
ds n
Hence proved.

SPECTRUM ALL-IN-ONE JOURNAL FOR ENGINEERING STUDENTS SIA GROUP


1.38 MATHEMATICS-II [JNTU-ANANTAPUR]
Q49. Find L[t sin 3t cos 2t]. Consider,

Ans: Consider, t sin 3t cos 2t 1 1


L[sin t] = =
s 2 + 12 s2 +1
On multiplying and dividing by 2, we get,
d ⎡ 1 ⎤
ds ⎢⎣ s 2 +1 ⎥⎦
t L[t sin t] = (–1)
t sin3t cos2t = [2 sin 3t cos 2t]
2
⎡ ( s 2 + 1)(0) − 2s ⎤ (−1)(−2 s)
= (−1) ⎢ ⎥=
⎥⎦ ( s + 1)
2 2
⎢⎣ ( s + 1)
2 2
t
= [sin(3t + 2t) + sin(3t – 2t)]
2 2s
[Q sin(A + B) + sin(A – B) = 2 sin A cos B] = ... (3)
( s 2 + 1) 2
t On substituting equations (2) and (3), in equation (1),
= [sin 5t + sint]
2 we get,
1 ⎡ 10s 2s ⎤
1 ∴ L[t sin 3t cos 2t] = ⎢ + ⎥
⇒ t sin3t cos2t = [t sin 5t + t sin t] 2 ⎣ ( s 2 + 52 ) 2 ( s 2 + 1) 2 ⎦
2
5s s
Applying Laplace transform on both sides, we get, = + 2
( s + 25)
2 2
(s + 1) 2
5s s
⎡1 ⎤ ∴ L[t sin 3t cos 2t ] = +
L[t sin 3t cos 2t] = L ⎢ [t sin 5t + t sin t ⎥ ( s 2 + 252 ) ( s 2 + 1) 2
⎣2 ⎦
Q50. Find the Laplace transform of te–t sin 2t.
1
= [L(t sin 5t) + L(t sin t)] ... (1) Ans: The given function is,
2 te–t sin 2t
Consider, The Laplace transform of te–t sin 2t is given by,
d
5 a L{te–t sin 2t} = (–1) L{e −t sin 2t}
L[sin 5t] = [Q L(sin at) = ] ds
s 2 + 52 s2 + a2
⎡ n d
n ⎤
⎢Q L{t f (t )} = (−1)
n
n
[ f (s )]⎥
d ⎛ 5 ⎞ ⎢⎣ ds ⎥⎦
∴ L[t sin 5t] = (–1) ⎜ ⎟
ds ⎝ s 2 + 5 2 ⎠ d 2
= (–1)
n d s ( s + 1) 2 + 2 2
d
[Q L[tn f(t)] = (– 1)n (f(s)), for n = 1, 2, 3, .... ]
ds n ⎡ ⎤
at
{
⎢QL e sin bt = }
b
2⎥
(s − a) + b ⎦
2

d ⎛ 1 ⎞
= ( −1)(5) ⎜ ⎟
= (–1)
d ⎡ 2 ⎤
ds ⎝ s 2 + 5 2 ⎠ ⎢
ds ⎣ s + 1 + 2 s + 4 ⎥⎦
2

⎡ ( s 2 + 52 )(0) − (1)(2s) ⎤ d ⎡ 2 ⎤
= (–1) ds ⎢ 2 ⎥
= − 5⎢ ⎥ ⎣ s + 2s + 5 ⎦
⎣⎢ (s 2 + 52 )2 ⎦⎥
⎛ ′ ⎞
0 − 2( 2 s + 2) ⎜Q ⎛⎜ u ⎞⎟ = vu ′ − uv′ ⎟
⎛ ′ ⎞ = (–1) ⎜ ⎝v⎠ ⎟
⎜Q ⎛ u ⎞ vu′ − uv′ ⎟ ( s 2 + 2 s + 5) 2 v 2

⎜ ⎜ ⎟ = ⎝ ⎠
⎝v⎠ v2 ⎟
⎝ ⎠ 2( 2 s + 2 )
=
( s + 2 s + 5) 2
2

⎡ − 2s ⎤
= − 5⎢ 2 4(s + 1)
2 2⎥ =
⎣ (s + 5 ) ⎦ ( s 2 + 2s + 5) 2

10s 4( s + 1)
= ... (2) ∴L{te −t sin 2t} =
( s + 52 ) 2
2 ( s + 2s + 5) 2
2

Look for the SIA GROU P LOGO on the TITLE COVER before you buy
UNIT-1 (Laplace Transform) 1.39
∞ ∞
⎡[f(t)]⎤
Q51. If L[f(t)] = f (s), then prove that L⎢
⎣ t ⎦
⎥ = ∫
s
f(s)ds Q52. Using Laplace transform evaluate, e – at sin 2 t/t dt .

0

provided Lim f(t) exists. Ans: The given function is,


t→0 t

sin 2 t
Ans: Given that,

0
e − at .
t
dt
L{f (t)} = f (s)


sin 2 t
⎧ f (t ) ⎫ Given integral is exactly the Laplace of with s = a.
∫ f ( s)ds for
f(t) t
And L ⎨ ⎬ = Lt exists
⎩ t ⎭ s
t →0 t

⎧ sin 2 t ⎫ e − at . sin 2 t
From the definition of Laplace transform, L⎨ ⎬
⎩ t ⎭ s=a 0
=
t ∫ dt ... (1)
∞ s=a

∫e
− st
L{f (t)} = f (s ) = f (t ) dt Consider, sin2 t
0
Integrating on both sides with respect to ‘s’, we get, ⎧1 − cos 2t ⎫
⇒ L[sin2t] = L ⎨ ⎬
∞ ⎡∞
⎩ 2 ⎭
∞ ⎤
∫ ⎢⎢∫ e
− st
f (t )dt ⎥ ds
∴ ∫ f ( s ) ds =
⎣0



⎜Q sin θ =
2 1 − cos 2θ ⎞

s s
⎝ 2 ⎠
∞∞
1 1
L{}
1 − L{cos 2t}
= ∫∫
0 s
e − st f (t ) ds dt =
2 2
[Changing the order of integration] 1 1 1 s
= . − .
2 s 2 s2 + 4
∞ ⎡∞ ⎤
= ∫ ∫
f (t ) ⎢ e − st ds ⎥ dt
⎢s ⎥
⎛ 1
⎜Q L{1} = , L{cos 2t} = 2
s ⎞

0 ⎣ ⎦ ⎝ s s + a2 ⎠
[Q f (t) is independent of s]
1 ⎡1 s ⎤
= −

⎡e − st ⎤

2 ⎢⎣ s s 2 + 4 ⎥⎦
= ∫ f (t )⎢⎢⎣ − t ⎥⎥⎦ s
dt

0
⎧⎪ sin 2 t ⎫⎪ 1⎛1 s ⎞

L⎨
⎪⎩ t ⎪⎭
⎬= ∫ 2 ⎜⎝ s − s 2 ⎟ ds
+4⎠
⎡ 1 ⎤ s
∫ f (t ) ⎢⎣− t (e
−∞
= − e − st ) ⎥ dt
⎦ ⎛ ∞ ⎞
⎜Q L ⎡ f (t ) ⎤ = f (s )ds ⎟
0

∞ ⎜ ⎢
⎣ t ⎦ s
⎥ ∫ ⎟
⎡ 1 ⎤ ⎝ ⎠
= ∫ f (t )⎢− (0 − e − st )⎥ dt
⎣ t ⎦ ∞
0
⎧⎪ sin 2 t ⎫⎪ 1 ⎡ 1 s ⎤

⎡ e − st ⎤
L⎨ ⎬=
⎪⎩ t ⎪⎭ 2 ⎣ s ⎢ ∫
ds − 2 ds
s + 4 ⎥⎦

s
= f (t ) ⎢ ⎥ dt
0
⎢⎣ t ⎥⎦ ∞
1 ⎡1 2s ⎤

f (t ) ⎧ f (t ) ⎫
=
2 ⎣s ∫
⎢ ds − ds ⎥
2( s + 4) ⎦
2

∫ e − st
s
= dt = L ⎨ ⎬
t ⎩ t ⎭
1⎡ 1 ⎤
0 ∞ ∞
1 2s
⎧ f (t ) ⎫

= ⎢
2⎢ s ∫
ds −
2 s +4 ⎥
2
ds ⎥ ∫
⎣s ⎦
∫ f (s)ds
s
∴ L⎨ ⎬=
⎩ t ⎭
s ⎛ 2s ⎞
Hence proved.
⎜Q
⎝ ∫s 2
+a 2
ds = log(s 2 + a 2 ) ⎟

SPECTRUM ALL-IN-ONE JOURNAL FOR ENGINEERING STUDENTS SIA GROUP


1.40 MATHEMATICS-II [JNTU-ANANTAPUR]
∞ ∞
⎡1 1 ⎤ 1 ⎧ 2s 2s ⎫
= ⎢ log( s) − log(s + 4)⎥

2
= ⎨ − ⎬ ds
⎣2 4 ⎦s 2 ⎩ s2 + a2 s2 + b2 ⎭
s

[
= log( s)1/ 2 − log( s 2 + 4)1 4 ] ∞
s
1 ⎧⎪ ⎫⎪
∞ ∞
2s 2s
⎡ s ⎤
∞ = ⎨ 2
2⎪ s +a 2∫ds −
s +b
2 2 ∫
ds ⎬
⎪⎭
= log ⎢ 2 14⎥ ⎩s 0
⎢⎣ (s + 4) ⎥⎦ s
⎡ ⎤ ∞
= –log ⎢ 2
s
14⎥ =
1
[
log(s 2 + a 2 ) − log( s 2 + b 2 ) ] s
⎣⎢ (s + 4) ⎦⎥ 2

⎡ ( s 2 + 4)1 4 ⎤ 1 ⎡ s2 + 4 ⎤ ∞
⎢ log ⎢ 2 ⎥ 1 ⎡ ⎪⎧ s 2 + a 2 ⎪⎫⎤
2 14 ⎥ = 4
= log
⎢⎣ s ⎥⎦ = ⎢log ⎨ 2 ⎬⎥
⎣⎢ (s ) ⎦⎥ 2 ⎢⎣ ⎪⎩ s + b 2 ⎪⎭⎥⎦
s


⎧⎪ sin 2 t ⎫⎪ sin 2 t ⎡ s2 + 4 ⎤
1 1 ⎧⎪ ⎛ ∞ ⎞ ⎛ s2 + a2 ⎞ ⎫⎪
⇒ L⎨
⎪⎩ t ⎪⎭
⎬ = ∫ e −at .
t
dt = log ⎢ 2 ⎥
4 ⎣⎢ s ⎦⎥
= ⎨log ⎜ ⎟ − log ⎜⎜ 2
2 ⎪⎩ ⎝ ∞ ⎠
⎟⎬
⎟⎪
⎝ s +b
2
0 ⎠⎭
For s = a,

sin 2 t 1 ⎪⎧ a 2 + 4 ⎫⎪ 1 ⎧⎪ ⎛ s 2 + a 2 ⎞ ⎫⎪
⎜ ⎟
∫ −
− at
∴ e dt = log ⎨ 2 ⎬ = ⎨ log(1) log ⎜ s 2 + b 2 ⎟ ⎬⎪
t 4 ⎪⎩ a ⎪⎭ 2 ⎪⎩ ⎝ ⎠⎭
0

Q53. Using Laplace transform, evaluate


1 ⎛ s2 + a2 ⎞

(cosat − cosbt) = − log⎜ 2 ⎟ (Q log(1) = 0)
2 ⎜⎝ s + b 2 ⎟

0
t
. dt. ⎠

Ans: The given function is, ∞


⎧ cos at − cos bt ⎫ 1 ⎪⎧ s 2 + b 2 ⎫⎪

(cosat-cosbt)
⇒ ∫ e − st ⎨
⎩ t
⎬dt = log ⎨ 2
⎭ 2 ⎪⎩ s + a 2 ⎪⎭

∫ t
. dt 0

0
Let, s = 0,
Consider, cos at – cos bt
∴ L{cos at – cos bt} = L{cos at} – L{cos bt} ∞
⎧ cos at − cos bt ⎫ 1 ⎧⎪ 0 + b 2 ⎫⎪
=
s

s ⇒ ∫ e0 ⎨
⎩ t


dt =
2
log ⎨ ⎬
⎪⎩ 0 + a 2 ⎪⎭
s +a2 2
s +b
2 2 0

⎛ s ⎞
⎜Q L{cos at} = 2 ⎟ ∞
⎧ cos at − cos bt ⎫ 1 ⎧⎪ ⎛ b ⎞
2⎫

⎝ s + a2 ⎠

0

⎩ t
⎬dt = ⎨log⎜ ⎟
⎭ 2 ⎪⎩ ⎝ a ⎠

⎪⎭

⎧ cos at − cos bt ⎫ ⎧ s s⎫
L⎨
⎩ t
⎬ =

∫ ⎨⎩ s 2
+a 2
− 2 ⎬ ds
s +b ⎭ 2
s 1⎧ ⎛ b ⎞⎫
= ⎨2 log ⎜ ⎟ ⎬
2⎩ ⎝ a ⎠⎭
⎛ ∞ ⎞
⎜Q L ⎡ f (t ) ⎤ = f (s )ds ⎟
⎜ ⎢ t ⎥
⎣ ⎦ s∫ ⎟
⎝ ⎠ ⎛b⎞
= log ⎜ ⎟
On multiplying and dividing by 2, we get, ⎝a⎠

2 ⎧ s s ⎫ ∞
= ∫
⎨ 2
2 ⎩s + a 2
− 2 ⎬ ds
s + b2 ⎭ ∴ ∫
cos at − cos bt ⎛b⎞
dt = log⎜ ⎟
s
0
t ⎝a⎠

Look for the SIA GROU P LOGO on the TITLE COVER before you buy
UNIT-1 (Laplace Transform) 1.41
1.11 APPLICATION OF LAPLACE TRANSFORMS TO ORDINARY DIFFERENTIAL EQUATIONS OF
FIRST AND SECOND ORDER
Q54. Using Laplace transform, solve (D2 + 4D + 5)y = 5, given that y(0) = 0, y"(0 ) = 0. Model Paper-I, Q3

Ans: The given differential equation is,


(D2 + 4D + 5)y = 5
y(0) = 0, y"(0) = 0 ... (1)
⇒ D2y + 4Dy + 5y = 5

d 2 y 4dy ⎛ dy ⎞
+ + 5y = 5 ⎜Q D = ⎟
dx 2 dx ⎝ dx ⎠
⇒ y'' + 4y' + 5y = 5
Applying Laplace transform on both sides, we get,
L{y'' + 4y' + 5y} = L{5}
L{y''} + 4L{y'} + 5L{y} = L{5}

5
s 2 L{ y} − sy(0) − y ' (0) + 4{sL{ y} − y (0)} + 5L{ y} = [Q L{yn} = sn L{y} – sn–1 y(0) – sn – 2 y'(0)....]
s
5
(s2 + 4s + 5)L{y} – (s + 4)y(0) – y'(0) =
s
Since y(0) = 0, y'(0) = 0, the above equation becomes,

5
(s2 + 4s + 5)L{y} – (s + 4)0 – 0 =
s

5
L{y} = 2
s ( s + 4 s + 5)
The roots of s2 + 4s + 5 are,

− 4 ± 16 − 4 × 5 − 4 ± − 4 −4 ± 2i 2(−2 ± i )
s= = = =
2 2 2 2
= –2 ± i
= –2 + i, – 2 – i

5 A B C
⇒ L{y} = = + + ... (1)
s ( s + ( 2 − i ))( s + ( 2 + i )) s s + (2 − i ) s + (2 + i )

5 A( s + ( 2 − i ))( s + ( 2 + i )) + Bs ( s + (2 + i )) + C ( s )( s + (2 − i ))
=
s( s + (2 − i ))(s + (2 + i )) s ( s + ( 2 − i ))( s + (2 + i ))

5 = A(s + (2 – i))(s + (2 + i)) + Bs(s + (2 + i)) + C(s) (s + (2 – i)) ... (2)


On substituting s= 0 in equation (2), we get,
⇒ 5 = A(2 – i) (2 + i)
⇒ 5 = A[4 – (i2)]
⇒ 5 = A[5]

∴ A =1

SPECTRUM ALL-IN-ONE JOURNAL FOR ENGINEERING STUDENTS SIA GROUP


1.42 MATHEMATICS-II [JNTU-ANANTAPUR]
On substituting s = –(2 – i) in equation (2), we get,
5 = B(– (2 – i)) (– 2 – i + 2 + i)
⇒ 5 = B(–2 + i) (2i)
⇒ 5 = B(– 4i + 2i2)
⇒ 5 = B(– 4i – 2)

−5
∴B =
4i + 2
On substituting s= – (2 + i) in equation (2), we get,
5 = C(– 2 – i)(– 2 – i + 2 – i)
⇒ 5 = C(– 2 – i)(– 2i)
⇒ 5 = C(2 + i)(2i)
⇒ 5 = C(4i + 2i2)
⇒ 5 = C [4i – 2]

5
∴C =
4i − 2
On substituting the corresponding values of A, B, C in equation (1), we get,

1 5 ⎡ 1 ⎤ 5 ⎡ 1 ⎤
∴ L–1{L(y)} = − ⎢ ⎥+ ⎢ ⎥
s ( 4i + 2 ) ⎣ ( s + ( 2 − i )) ⎦ 4i − 2 ⎣ ( s + ( 2 + i )) ⎦

On applying inverse Laplace transform on both sides, we get,

⎡1 5 ⎛ 1 ⎞ 5 ⎛ 1 ⎞⎤
L–1{L(y)} = L–1 ⎢ − ⎜⎜ ⎟⎟ + ⎜⎜ ⎟⎥
⎢⎣ s ( 4i + 2) ⎝ ( s + ( 2 − i )) ⎠ ( 4i − 2) ⎝ s + ( 2 + i ) ⎟⎠ ⎥⎦

5 5 ⎛ ⎧ 1 ⎫ − at ⎞
y = 1− e − ( 2 −i )t + e − (2 +i )t ⎜⎜Q L−1 ⎨ ⎬ = e ⎟⎟
4i + 2 4−2 ⎝ ⎩ s + a ⎭ ⎠

5 − ( 2 − i )t 5 − ( 2 + i )t
∴ y = 1− e + e
4i + 2 4i − 2
Q55. Using Laplace transform, solve (D2 + 2D – 3)y = sin x, y(0) = y'(0) = 0.
Ans: Given that,
(D2 + 2D – 3)y = sin x

d2y dy ⎛ dy ⎞
⇒ +2 − 3 y = sin x ⎜Q D = ⎟
dx 2 dx ⎝ dx ⎠

y'' + 2y' – 3y = sin x ... (1)


On applying Laplace transform on both sides of equation (1), we get,
L{y'' + 2y' – 3y} = L{sin x}
⇒ L{y''} + 2L{y'} – 3L{y} = L{sin x}
⇒ {s2 L(y) – s(y(0)) – y'(0)} + 2{s L(y) – y(0)} – 3 L{y} = L{sin x} {Q L{yn} = sn L{y} – sn – 1 y{0} – sn – 2 y'(0) ... ... (2)

Look for the SIA GROU P LOGO on the TITLE COVER before you buy
UNIT-1 (Laplace Transform) 1.43
Since, y(0) = 0 and y'(0) = 0, equation (2) becomes,
⇒ [s2 L{y} – s(0) – 0] + 2[s L(y) – 0] – 3 L(y) = L{sin x}
⇒ [s2 L(y)] + 2[s L(y)] – 3 L(y) = L{sin x}

1 ⎧⎪ 1 ⎫⎪
⇒ L(y)[s2 + 2s – 3] = ⎨Q L (sin x ) = ⎬
1 + s2 ⎪⎩ 1 + s 2 ⎪⎭

1
⇒ L(y) =
(1 + s )( s 2 + 2 s − 3)
2

1
⇒ L(y) = 2 2
(1 + s )( s + 3s − s − 3)

1
=
(1 + s )(s ( s + 3) − 1( s + 3))
2

1
L(y) = 2 ... (3)
(1 + s )( s + 3)( s − 1)
On applying partial fractions to equation (3), we get,

1 As + B C D
= + + ... (4)
(1 + s 2 )( s + 3)( s − 1) ( s 2 + 1) ( s + 3) ( s − 1)

1 = (As + B)(s + 3)(s – 1) + C(s2 + 1)(s – 1) + D(s2 + 1)(s + 3)


= (As + B)(s2 + 3s – s – 3) + C(s3 – s2 + s – 1) + D(s3 + s + 3s2 + 3)
= (As + B)(s2 + 2s – 3) + C(s3 – s2 + s – 1) + D(s3 + 3s2 + s + 3)
= As3 + Bs2 + 2As2 + 2Bs – 3As – 3B + Cs3 – Cs2 + Cs – C + Ds3 + 3Ds2 + Ds + 3D
⇒ 1 = s3(A + C + D) + s2(B + 2A – C + 3D) + s(2B – 3A + C + D) + (– 3B – C + 3D) ... (5)
On comparing the coefficients of s3, s2, s and constant terms on both sides of equation (4), we get,
A + C + D =0 ... (6)
2A + B – C + 3D = 0 ... (7)
– 3A + 2B + C + D = 0 ... (8)
– 3B – C + 3D = 1 ... (9)
Solving equations (5) and (8), we get,
A+C+D=0
– 3B – C + 3D = 1
A − 3B + 4 D = 1 ... (10)
Solving equations (6) and (7), we get,
2A + B – C + 3D = 0
– 3A + 2B + C + D = 0
− A + 3B + 4D = 0 ... (11)

On solving equations (9) and (10), we get,


A – 3B + 4D = 1
– A + 3B + 4D = 0
8D = 1

1
∴D = ... (12)
8
SPECTRUM ALL-IN-ONE JOURNAL FOR ENGINEERING STUDENTS SIA GROUP
1.44 MATHEMATICS-II [JNTU-ANANTAPUR]
On substituting equation (11) in equation (10), we get,
⎛1⎞
– A + 3B + 4 ⎜ ⎟ = 0
⎝8⎠
−1
⇒ – A + 3B =
2
−1
⇒ – (A – 3B) =
2
1
⇒ A − 3B =
2
⇒ 2 A − 6B = 1 ... (13)
From equation (8), we get,
C = – 3B + 3D – 1 ... (14)
On substituting equation (14) in equation (6), we get,
2A + B – (– 3B + 3D – 1) + 3D = 0
⇒ 2A + B + 3B – 3D + 1 + 3D = 0
2A + 4B = –1 ... (15)
Solving equations (13) and (15), we get,
2A – 6B = 1
2A + 4B = –1
– – +
–10B = 2

−2 −1
B = =
10 5

−1
∴ B= ... (16)
5
On substituting the value of B in equation (12), we get,
⎛ −1⎞ 1
A–3⎜ ⎟ =
⎝ 5 ⎠ 2

1 3
⇒ A= –
2 5
5−6
=
10

−1
∴A= ... (17)
10
On substituting the value of ‘A’ and ‘D’in equation (5), we get,
−1 1
+ C + =0
10 8

1 1
C= −
10 8

4−5
=
40

−1
∴C = ... (18)
40

Look for the SIA GROU P LOGO on the TITLE COVER before you buy
UNIT-1 (Laplace Transform) 1.45
On substituting the values of A, B, C and D in the equation (4), we get,

⎛ −1 ⎞ 1 −1 1
⎜ ⎟s −
⎝ 10 ⎠ 5 + 40 + 8
L(y) =
s2 +1 s + 3 s −1

− 1 ⎧⎪ s ⎫⎪ ⎛ − 1 ⎞⎧⎪ 1 ⎫⎪ ⎛ − 1 ⎞⎧ 1 ⎫ 1 ⎧ 1 ⎫
= ⎨ ⎬ + ⎜ ⎟⎨ ⎬ + ⎜ ⎟⎨ ⎬+ ⎨ ⎬ ... (19)
10 ⎪⎩ s 2 + 1 ⎪⎭ ⎝ 5 ⎠⎪⎩ s 2 + 1 ⎪⎭ ⎝ 40 ⎠⎩ s + 3 ⎭ 8 ⎩ s − 1 ⎭

By applying inverse Laplace transform on both sides of equation (19), we get,


⎧⎛ − 1 ⎞⎛ s ⎞ ⎛ 1 ⎞⎛ 1 ⎞ 1 ⎛ 1 ⎞ 1 ⎛ 1 ⎞⎫
L–1{L(y)} = L−1 ⎨⎜ ⎟⎜ 2 ⎟ − ⎜ ⎟⎜ 2 ⎟ − ⎜ ⎟+ ⎜ ⎟⎬
⎩⎝ 10 ⎠⎝ s + 1 ⎠ ⎝ 5 ⎠⎝ s + 1 ⎠ 40 ⎝ s + 3 ⎠ 8 ⎝ s − 1 ⎠⎭
− 1 −1 ⎧⎪ s ⎫⎪ 1 −1 ⎧⎪ 1 ⎫⎪ 1 −1 ⎧ 1 ⎫ 1 −1 ⎧ 1 ⎫
y= L ⎨ 2 ⎬− L ⎨ 2 ⎬− L ⎨ ⎬+ L ⎨ ⎬ ... (20)
10 ⎪⎩ s + 1 ⎪⎭ 5 ⎪⎩ s + 1 ⎪⎭ 40 ⎩ s + 3⎭ 8 ⎩ s − 1⎭

s
Q L(cos t) =
s2 + 1
−1 1 1 −3t 1 t 1
y= {cos t} − {sin t} − e + e L(sin t) = 2
10 5 40 8 s +1
1
L(e–3t) = and
s+3
1
L(et) =
s −1
d2 x
Q56. Solve the differential equation + 9x = sin t using Laplace transforms given that x(0) = 1, x'(0) = 0.
dx 2
Ans: The given differential equation is,

d 2x
+ 9 x = sin t
dt 2

⎛ dx ⎞
⇒ x'' + 9x = sin t ⎜Q = x′ ⎟
⎝ dt ⎠
Taking Laplace transform on both sides, we get,
L[x'' + 9x] = L[sin t]

1 ⎛ a ⎞
L[x''] + 9L[x] = ⎜Q L(sin at ) = 2 ⎟
2
s +1 ⎝ s +1⎠

1
s2L[x] – s.x(0) – x'(0) + 9L[x] = 2 (Q L[ y n ] = s n L[ y ] − s n−1 y (0) − s n −2 y ′(0)....) ... (1)
s +1
Since, x(0) = 1, x'(0) = 0, equation (1) becomes,
1
s2 L[x] – s(1) – 0 + 9 L[x] = 2
s +1
1
⇒ s2L[x] – s + 9 L[x] = 2
s +1
1
⇒ s2L[x] + 9 L[x] = +s
s2 +1
1
⇒ L[x] (s2 + 9) = 2 +s
s +1
1 s
L[x] = + 2 ... (2)
( s + 1)( s + 9) (s + 9)
2 2

SPECTRUM ALL-IN-ONE JOURNAL FOR ENGINEERING STUDENTS SIA GROUP


1.46 MATHEMATICS-II [JNTU-ANANTAPUR]

1
Consider,
( s 2 + 1)( s 2 + 9 )
By using partial fractions, we get,
1 As + B Cs + D
= 2 + 2
( s + 1)( s 2 + 9)
2
s +1 s +9
⇒ 1 = (As + B) (s2 + 9) + (Cs + D) (s2 + 1)
= As3 + 9As + Bs2 + 9B + Cs3 + Cs + Ds2 + D
1 = (A + C) s3 + (B + D)s2 + (9A + C) s + (9B + D)
Comparing s3, s2, s and constant terms on both sides, we get,
0=A +C ... (3)
0=B+D ... (4)
0 = 9A + C ... (5)
1 = 9B + D ... (6)
From equation (3), we get,
A=–C ... (7)
On substituting equation (7) in equation (5), we get,
0 = 9(– C) + C
⇒ 0 = – 9C + C
⇒ 0 = – 8C

∴C = 0

∴A= 0
From equation (4), we get,
B =–D ... (8)
On substituting equation (8) in equation (6), we get,
1= 9(– D) + D
⇒ 1= – 9D + D
⇒ 1= – 8 D

1
∴D = −
8

1
∴B =
8
On substituting the corresponding values in equation (2), we get,
1 s
L[x] = + 2
( s + 1)( s + 9) s + 9
2 2

⎡ 1 1⎤
⎢ 0s + 8 0s − 8 ⎥ s
=⎢ 2 + 2 ⎥+ 2
⎢ s +1 s + 9 ⎥ s + 9
⎣ ⎦

1 1 1 1 s
= . − . +
8 s2 + 1 8 s2 + 9 s2 + 9

1⎡ 1 1 ⎤ s
. − +
8 ⎢⎣ s 2 + 1 s 2 + 32 ⎥⎦ s 2 + 32
=

Look for the SIA GROU P LOGO on the TITLE COVER before you buy
UNIT-1 (Laplace Transform) 1.47
Applying inverse Laplace transform, we get,

−1 ⎧ 1 ⎡ 1 1 ⎤ s ⎫
L–1 {L[x]} = L ⎨ ⎢ 2 − 2 2⎥
+ 2 2⎬
⎩8 ⎣ s +1 s + 3 ⎦ s + 3 ⎭

1 ⎡ −1 ⎧ 1 ⎫ −1 ⎧ 1 ⎫ −1 ⎧ s ⎫⎤
L ⎨ ⎬−L ⎨ 2 2⎬
+L ⎨ 2 2 ⎬⎥
8 ⎢⎣ ⎩ s 2 + 1 ⎭
x=
⎩s + 3 ⎭ ⎩ s + 3 ⎭⎦

⎛ −1 ⎧ 1 ⎫ 1 ⎞
⎜Q L ⎨ 2 2 ⎬ = sin at⎟
1⎛ 1 ⎞ ⎜ ⎩s + a ⎭ a ⎟
= ⎜ sin t − sin 3t ⎟ + cos 3t ⎜ ⎟
8⎝ 3 ⎠ ⎧ s ⎫
⎜ L−1 ⎨ 2 2⎬
= cos at ⎟
⎝ ⎩s + a ⎭ ⎠

1⎛ 1 ⎞
∴ x = ⎜ sin t − sin 3t ⎟ + cos 3t
8⎝ 3 ⎠

Q57. Solve the following differential equation using the Laplace transforms,

d2 y 2dy
+ + 2y = 5 sint and y(0) = y'(0) = 0.
dt 2 dt
Ans: Given that,

d 2 y 2dy
+ + 2 y = 5 sin t and y(0) = y'(0) = 0
dt 2 dt

⎛ dy ⎞
⇒ y'' + 2y' + 2y = 5 sin t ⎜Q = y′ ⎟
⎝ dt ⎠

Applying Laplace transform on both sides, we get,


L{y'' + 2y' + 2y} = L{5 sin t}
L{y''} + 2L{y'} + 2L{y} = 5L{sin t}
[s2 y(s) – sy(0) – y'(0)] + 2[sy(s) – y(0)] + 2y(s) = 5L{sin t} [ Q L{yn} = snL{y} – sn–1.y{0} – sn–2.y1{0} .....] ... (1)
Since, y(0) = 0 and y'(0) = 0, equation (1) becomes,
[s2y(s) – 0 – 0] + 2[sy(s) – 0] + 2y(s) = 5L{sin t}
⇒ s2 y(s) + 2sy(s) + 2y(s) = 5 L{sin t}

1 ⎡ a ⎤
⇒ y(s) (s2 + 2s + 2) = 5 × ⎢Q L{sin at} = s 2 + a 2 ⎥
2
s +1 ⎣ ⎦

5
⇒ y(s) =
( s + 1)(s 2 + 2s + 2)
2

By using partial fractions, we get,

5 As + B Cs + D
y(s) = = + ... (2)
( s + 1)(s + 2s + 2)
2 2
s +1
2
s + 2s + 2
2

⇒ 5 = (As + B) (s2 + 2s + 2) + (Cs + D) (s2 + 1)


⇒ 5 = As3 + 2As2 + 2As + Bs2 + 2Bs + 2B + Cs3 + Cs + Ds2 + D
⇒ 5 = s3(A + C) + S2(2A + B + D) + s(2A + 2B + C) + (2B + D) ... (3)

SPECTRUM ALL-IN-ONE JOURNAL FOR ENGINEERING STUDENTS SIA GROUP


1.48 MATHEMATICS-II [JNTU-ANANTAPUR]
3 2
Thus, comparing the coefficients of s , s , s and constant from equation (3), we get,
A+C=0 ... (4)
2A + B + D = 0 ... (5)
2A + 2B + C = 0 ... (6)
2B + D = 5 ... (7)
From equation (4),
A=–C ... (8)
On substituting equation (8) in equations (5) and (6), we get,
2(– C) + B + D = 0
⇒ – 2C + B + D = 0 ... (9)
And,
2(– C) + 2B + C = 0
⇒ – 2C + 2B + C = 0
⇒ – C + 2B = 0
⇒ C = 2B ... (10)
On substituting equation (10) in equation (9), we get,
– 2(2B) + B + D = 0
⇒ – 4B + B + D = 0
⇒ – 3B + D = 0 ... (11)
On solving equations (7) and (11), we get,

2B+D=5
–3B+D=0
+ – –
5B=5

∴B = 1

On substituting equation (12) in equation (7), we get,


2(1) + D = 5
⇒ D=5–2

∴D = 3

Thus, from equation (10), we get,


C = 2B = 2(1) = 2

∴C = 2

From equation (8), we get,


A =–C=–2

∴ A = −2

On substituting corresponding values in equation (2), we get,

− 2s + 1 2s + 3
y(s) = +
s2 +1 s 2 + 2s + 2

Look for the SIA GROU P LOGO on the TITLE COVER before you buy
UNIT-1 (Laplace Transform) 1.49
On applying inverse Laplace on both sides, we get,

−1 ⎡ − 2 s + 1 2s + 3 ⎤
⇒ L–1[y(s)] = L ⎢ 2 + 2 ⎥
⎣ s + 1 s + 2s + 2 ⎦

−1 ⎡ − 2 s + 1 ⎤ −1 ⎡ 2s + 3 ⎤
⇒ y(t) = L ⎢ 2 ⎥+L ⎢ 2 ⎥
⎣ s +1 ⎦ ⎣ s + 2s + 2 ⎦

−1 ⎡ − 2 s ⎤ −1 ⎡ 1 ⎤ −1 ⎡ 2s ⎤ −1 ⎡ 3 ⎤
⇒ y(t) = L ⎢ 2 ⎥+L ⎢ 2 ⎥+ L ⎢ 2 ⎥+ L ⎢ 2 ⎥
⎣ s + 1⎦ ⎣ s + 1⎦ ⎣ s + 2s + 2 ⎦ ⎣ s + 2s + 2 ⎦

−1 ⎡ s ⎤ −1 ⎡ 1 ⎤ −1 ⎡ s ⎤ −1 ⎡ 1 ⎤
= − 2 L ⎢ 2 2 ⎥ + L ⎢ 2 2 ⎥ + 2L ⎢ ⎥ + 3L ⎢ 2⎥
⎣ s +1 ⎦ ⎣s +1 ⎦ ⎣⎢ ( s + 1) + 1 ⎦⎥
2
⎣⎢ ( s + 1) + 1 ⎦⎥
2

−1 ⎡ s ⎤ −1 ⎡ 1 ⎤ −1
⎡ s + 1−1 ⎤ −1
⎡ 1 ⎤
= − 2L ⎢ 2 2 ⎥ + L ⎢ 2 2 ⎥ + 2L ⎢ ⎥ + 3 L ⎢ 2⎥
⎣ s +1 ⎦ ⎣ s +1 ⎦ ⎣⎢ (s + 1) + 1 ⎦⎥
2 2
⎣⎢ (s + 1) + 1 ⎦⎥
2

−1 ⎡ ⎤s −1 ⎡ 1 ⎤ ⎡ −1 ⎡ s +1 ⎤ ⎡ 1 ⎤⎤ −1 ⎡ 1 ⎤
= − 2L ⎢ 2 ⎥ + L ⎢ 2 2 ⎥ + 2⎢ L ⎢
2 2 2⎥
− L−1 ⎢ 2 2⎥
⎥ + 3L ⎢ 2 2⎥
⎣ s +1 ⎦ ⎣ s +1 ⎦ ⎣⎢ ⎢⎣ ( s + 1) + 1 ⎥⎦ ⎢⎣ ( s + 1) + 1 ⎥⎦ ⎦⎥ ⎢⎣ ( s + 1) + 1 ⎥⎦
⇒ y(t) = –2 cos t + sin t + 2 [e–t cos t – e–t sin t] + 3 e–t sin t

⎡ −1 ⎡ s ⎤ ⎡ a ⎤ ⎡ 1 ⎤ 1 − at ⎤
⎢Q L ⎢ 2 2⎥
= cos at , L−1 ⎢ 2 2⎥
= sin at , L−1 ⎢ 2⎥
= e . sin bt , ⎥
⎢ ⎣s + a ⎦ ⎣s + a ⎦ ⎣⎢ ( s + c ) + b ⎦⎥ b
2

⎢ ⎥
⎢ ⎡ s+a ⎤ ⎥
⎢ L−1 ⎢ 2⎥
= e − at cos bt ⎥
⎣ ⎢
⎣ ( s + a ) 2
+ b ⎥
⎦ ⎦
⇒ y(t) = – 2cos t + sin t + 2e–t cos t – 2e–t sin t + 3e–t sin t
⇒ y(t) = – 2 cos t + sin t + 2e–t cos t + e–t sin t

∴ y (t ) = 2 cos t (e −t − 1) + sin t ( e −t + 1)
Q58. Using Laplace transform solve (D3 – D2 + 4D – 4)y = 68 exsin2x, y = 1, Dy = –19, D2y = –37 at x = 0.
Ans: Given differential equation is,
(D3 – D2 + 4D – 4)y = 68 ex sin 2x

d3y d2y dy ⎛ dy ⎞
− +4 − 4 y = 68e x sin 2 x ⎜ Q D = ⎟
dx 3
dx 2 dx ⎝ dx ⎠
y''' – y'' + 4y' – 4y = 68 ex sin 2x
Applying Laplace transform on both sides, we get,
⇒ L{y''' – y'' + 4y' – 4y} = L{68 ex sin 2x}
⇒ L{y'''} – L{y''} + 4{y'} – 4{y}= 68 L {ex sin 2x}
⇒ [s3 L{y} – s2y(0) – sy'(0) – y''(0)] – [s2 L{y} – sy(0) – y'(0)] + 4[sL{y} – y(0)] – 4L{y} = 68 L{ex sin 2x}
(Q L{ y n } = s n L{ y} − s n −1 y (0) − s n−2 y ′(0)...)

136
⇒ s3 L{y} – s2 y(0) – sy'(0) – y''(0) – s2 L{y} + sy(0) + y'(0) + 4sL{y} – 4y(0) – 4L{y} =
( s − 1) 2 + 22


{
⎢Q L e sin bt =
at
} b ⎤

( s − a) 2 + b 2 ⎦⎥
⎣⎢

136
⇒ L{y}(s3 – s2 + 4s – 4) –y''(0) – y'(0)(s – 1) + y(0)(– s2 + s – 4) =
( s − 1) 2 + 4

SPECTRUM ALL-IN-ONE JOURNAL FOR ENGINEERING STUDENTS SIA GROUP


1.50 MATHEMATICS-II [JNTU-ANANTAPUR]
On substituting the given conditions, we get,

136
L{y}(s3 – s2 + 4s – 4) – (– 37) – (– 19) (s –1) + (–s2 + s – 4) =
( s − 1) 2 + 4

136
⇒ y(s) (s3 – s2 + 4s – 4) + 37 + 19 s – 19 – s2 + s – 4 = 2
s − 2s + 1 + 4

136
⇒ y(s) (s3 – s2 + 4s – 4) – s2 + 20s + 14 = 2
s − 2s + 5

136
⇒ y(s) (s3 – s2 + 4s – 4) = + s2 – 20 s – 14
s 2 − 2s + 5

136 + ( s 2 − 20s − 14)( s 2 − 2s + 5)


⇒ y(s) (s3 – s2 + 4s – 4) =
s 2 − 2s + 5

136 + s 4 − 2s 3 + 5s 2 − 20s 3 + 40s 2 − 100s − 14s 2 + 28s − 70


⇒ y(s) (s3 – s2 + 4s – 4) =
s 2 − 2s + 5

s 4 − 22 s 3 + 31s 2 − 72s + 66
⇒ y(s) =
( s 2 − 2s + 5)(s 3 − s 2 + 4 s − 4)

Consider,
s3 – s2 + 4s – 4

1 1 −1 4 −4
0 1 0 4

1 0 4 0

∴ s3 – s2 + 4s – 4 = (s – 1) (s2 + 4)

s 4 − 22 s 3 + 31s 2 − 72 s + 66
∴ y(s) =
( s − 1)( s 2 + 4)( s 2 − 2 s + 5)

Taking partial fractions of above equation, we get,

s 4 − 22 s 3 + 31s 2 − 72 s + 66 A Bs + C Ds + E
y(s) = = + 2 + 2 ... (1)
( s − 1)( s + 4)( s − 2 s + 5)
2 2
s − 1 s + 4 s − 2s + 5

⇒ s4 – 22s3 + 31s2 – 72s + 66 = A (s2 + 4)(s2 – 2s + 5) + (Bs + C)(s – 1)(s2 – 2s + 5) + (Ds + E) (s – 1)(s2 + 4)
⇒ s4 – 22s3 + 31s2 – 72s + 66 = A (s4 – 2s3 + 9s2 – 8s + 20) + (Bs + C)(s3 – 3s2 + 7s – 5) + (Ds + E) (s3 – s2 + 4s – 4) ... (2)
On substituting, s = 1 in equation (2), we get,
1 – 22 + 31 – 72 + 66 = A(1 – 2 + 9 – 8 + 20)
⇒ 4 = A(20)

1
∴A =
5

On equating the coefficients of s4, s 3, s2 and s, we get,

Look for the SIA GROU P LOGO on the TITLE COVER before you buy
UNIT-1 (Laplace Transform) 1.51
4
s -terms On substituting equation (3) in equation (5), we get,
1= A + B + D
⎛4 ⎞ 146
⇒ B+D=1–A ⇒ 7⎜ − D ⎟ –3C + 4D – E =
⎝5 ⎠ 5
1
⇒ B+D=1–
5 28 146
⇒ –7D – 3C + 4D – E =
5 5
4
⇒ B+D=
5 146 28
⇒ –3C – 3D – E = –
5 5
4
⇒ B= –D ... (3)
5 118
⇒ – 3C – 3D – E =
3
s -terms 5
⇒ – 22 = –2A – 3B + C – D + E
−118
⇒ 3C + 3D + E = ... (8)
⎛1⎞ 5
– 22 = –2 ⎜ ⎟ (– 3B + C) + (– D + E)
⎝5⎠ On substituting equation (3) in equation (6), we get,
−2 ⎛4 ⎞ −352
⇒ – 22 = – 3B + C – D + E –5 ⎜ − D ⎟ + 7C – 4D + 4E =
5 ⎝5 ⎠ 5
108
⇒ – 3B + C – D + E = – ... (4) −352
5 ⇒ – 4 + 5D + 7C – 4D + 4E =
5
s2-terms
31 = 9A + (7B – 3C) + (4D – E) 352
⇒ 7C + D + 4E = – +4
5
⎛1⎞
⇒ 31 = 9 ⎜ ⎟ + 7B – 3C + 4D – E
⎝5⎠ −332
⇒ 7C + D + 4E = ... (9)
9 5
⇒ 31 – = 7B – 3C + 4D – E Solving equation (7) and equation (8), we get,
5
146 −96
7B – 3C + 4D – E = ... (5) C + 2D + E =
5 5
s-terms
– 72 = – 8A + (– 5B + 7C) + (– 4D + 4E) −118
3C + 3D + E =
5
−8
⇒ – 72 = – 5B + 7C – 4D + 4E
5
(–) (–) (–)
⎛ 1⎞
⎜Q A = ⎟
⎝ 5⎠ 22
– 2C – D = ... (10)
−352 5
⇒ – 5B + 7C – 4D + 4E = ... (6)
5
On substituting equation (3) in equation (4), we get, Multiplying equation (8) with ‘4’ and subtracting with
equation (9), we get,
⎛4 ⎞ −108
− 3⎜ − D ⎟ + C − D + E = 472
⎝5 ⎠ 5 12C + 12 D + 4 E = –
5
−12 −108 −332
⇒ + 3D + C − D + E = 7C + D + 4 E =
5 5 5

−108 12 (–) (–) (–)


⇒ C + 2D + E = +
5 5

−96 140
⇒ C + 2D + E = ... (7) 5C + 11 D = – ... (11)
5 5

SPECTRUM ALL-IN-ONE JOURNAL FOR ENGINEERING STUDENTS SIA GROUP


1.52 MATHEMATICS-II [JNTU-ANANTAPUR]
Multiplying equation (10) with equation (11) and adding with equation (11), we get,

242
– 22C – 11D =
5

140
5C + 11 D = –
5

102
– 17 C =
5

−6
∴C = ... (12)
5

On substituting equation (12) in equation (10), we get,

⎛−6⎞ 22
– 2⎜ ⎟− D =
⎝ 5 ⎠ 5

12 22
⇒ D= −
5 5

∴D = −2 ... (13)

On substituting equations (12) and (13) in equation (7), we get,

−6 −96
+ 2(−2) + E =
5 5

−96 6
⇒ E= + +4
5 5

∴ E = −14

On substituting ‘D’ value in equation (3), we get,

4
B= – (–2)
5

14
B=
5

On substituting A, B, C, D and E values in equation (1), we get,

14 ⎛ 6⎞
s +⎜− ⎟
1
+
5 ⎝ 5 ⎠ + (−2) s + (−14)
y(s) =
5(s − 1) 5(s + 4)
2
s 2 − 2s + 5

1 14 s − 6 ( 2 s + 14)
= + −
5( s − 1) 5( s 2 + 4) s 2 − 2 s + 5

Look for the SIA GROU P LOGO on the TITLE COVER before you buy
UNIT-1 (Laplace Transform) 1.53
Taking Laplace inverse on both sides, we get,

⎧ 1 14s − 6 2s + 14 ⎫
L–1 {y(s)} = L−1 ⎨ _ − 2 ⎬
⎩ 5( s − 1) 5( s + 4) s − 2s + 5 ⎭
2

1 −1 ⎧ 1 ⎫ 14 −1 ⎧ s ⎫ 6 −1 ⎧ 1 ⎫ −1 ⎧ s ⎫ −1 ⎧ 1 ⎫
y(t) = L ⎨ ⎬+ L ⎨ 2 ⎬− L ⎨ 2 ⎬ − 2L ⎨ 2 ⎬ − 14 L ⎨ 2 ⎬
5 ⎩ s − 1⎭ 5 ⎩s + 4⎭ 5 ⎩s + 4⎭ ⎩ s − 2s + 5 ⎭ ⎩ s − 2s + 5 ⎭

1 x 14 6 1 ⎧ s −1+ 1 ⎫ −1 ⎧ 1 ⎫
= e + cos 2 x − × sin 2 x − 2 L−1 ⎨ ⎬ − 14 L ⎨ ⎬
⎩ ( s − 1) + 4 ⎭ ⎩ ( s − 1) + 4 ⎭
2 2
5 5 5 2

⎛ ⎧ 1 ⎫ at −1 ⎧ s ⎫ ⎧ a ⎫ ⎞
⎜⎜Q L−1 ⎨ ⎬ = e ,L ⎨ 2 2⎬
= cos at , L−1 ⎨ 2 2⎬
= sin at ⎟⎟
⎝ ⎩ s − a ⎭ ⎩s + a ⎭ ⎩s + a ⎭ ⎠

1 x 14 3 ⎧ s −1 ⎫ ⎧ 1 ⎫ ⎧ 1 ⎫
= e + cos 2 x − sin 2 x − 2 L−1 ⎨ 2⎬
− 2 L−1 ⎨ 2⎬
− 14L−1 ⎨ 2⎬
⎩ ( s − 1) + 2 ⎭ ⎩ ( s − 1) + 2 ⎭ ⎩ ( s − 1) + 2 ⎭
2 2 2
5 5 5

1 x 14 3 1 1
= e + cos 2 x − sin 2 x − 2e x cos 2 x − 2 × e x sin 2 x − 14 × e x sin 2 x
5 5 5 2 2

⎛ ⎧ s−a ⎫ at ⎧ 1 ⎫ 1 at ⎞
⎜Q L−1 ⎨ = e cos bt , L−1 ⎨ = e sin bt ⎟
⎜ 2⎬ 2⎬ ⎟
⎩ ( s − a) + b ⎭ ⎩ ( s − a) + b ⎭ b
2 2
⎝ ⎠

e x 14 3
= + cos 2 x − sin 2 x − 2e x cos 2 x − e x sin 2 x − 7e x sin 2 x
5 5 5

e x 14 3
= + cos 2 x − sin 2 x − 2e x cos 2 x − 8e x sin 2 x
5 5 5

ex ⎛ 14 ⎞ ⎛3 ⎞
= + cos 2 x⎜ − 2e x ⎟ − sin 2 x⎜ + 8e x ⎟
5 ⎝ 5 ⎠ ⎝5 ⎠

ex ⎛ 14 ⎞ ⎛3 ⎞
∴ y (t ) = + cos 2 x⎜ − 2e x ⎟ − sin 2 x ⎜ + 8e x ⎟
5 ⎝ 5 ⎠ ⎝5 ⎠

Q59. Solve, y'' – 3y' + 2y = 4t + e3t when, y(0) = 1, y'(0) = – 1.


Ans: Given that,
y''– 3y' + 2y = 4t + e3t
On applying Laplace transform the above equation becomes,
L[y'' – 3y' + 2y] = L[4t + e3t]
⇒ L[y"] – 3L[y'] + 2L[y] = 4L[t] + L[e3t]

⎡Q L{ y n } = s n L{ y} − s n −1 y (0) − s n − 2 y ' (0).....⎤


4 1 ⎢ ⎥
⇒ [s2L[y] – sy(0) – y'(0)] – 3[sL[y] – y(0)] + 2L[y] = + ⎢Q L{e at } = 1 ⎥
s 2
s −3 ⎢⎣ ⎥⎦
s−a

SPECTRUM ALL-IN-ONE JOURNAL FOR ENGINEERING STUDENTS SIA GROUP


1.54 MATHEMATICS-II [JNTU-ANANTAPUR]
Since, given y(0) = 1 and y'(0) = –1

4 1
[s2L[y] – s(1) – (–1) – 3[sL[y] – 1] + 2L[y] = +
s 2
s−3

4 1
s2L[y] – s + 1 – 3sL[y] + 3 + 2L[y] = +
s2 s−3

4 1
⇒ (s2 – 3s + 2)L[y] – s + 1 + 3 = +
s 2 s −3

4 1
⇒ (s2 – 3s + 2)L[y] = 2 + +s–4
s s −3

4( s − 3) + 1( s 2 ) + ( s − 4)( s 2 )( s − 3)
⇒ (s – 1)(s – 2)L[y] =
s 2 ( s − 3)

s 4 − 7 s 3 + 13s 2 + 4s − 12
⇒ L[y] =
s 2 ( s − 1)(s − 2)( s − 3)
By using partial fractions, we get,

s 4 − 7s 3 + 13s 2 + 4s − 12 A B C D E
L[y] = = + 2+ + + ... (1)
2
s (s − 1)(s − 2)(s − 3) s s s −1 s − 2 s − 3
⇒ s4 – 7s3 + 13s2 + 4s – 12 = A s(s – 1) (s – 2) (s – 3) + B(s – 1) (s – 2) (s – 3) + C s2 (s – 2) (s – 3) +
D s2(s – 1) (s – 3) + E s2(s – 1) (s – 2) ... (2)
On substituting s = 0 in equation (2), we get,
0 – 0 + 0 + 0 – 12 = 0 + B (0 – 1) (0 – 2) (0 – 3) + 0 + 0 + 0
– 12 = B(– 6)

∴B = 2
On substituting s = 1 in equation (2), we get,
14 – 7(1)3 + 13(1)2 + 4(1) – 12= 0 + 0 + C(1)2 (1 – 2) (1 – 3) + 0 + 0
1 – 7 + 13 + 4 – 12 = C(– 1) (– 2)
– 1 = 2C

−1
∴C =
2

On substituting s = 2 in equation (2), we get,


24 – 7(2)3 + 13(2)2 + 4(2) – 12 = 0 + 0 + 0 D(2)2 (2 – 1)(2 – 3) + 0
8 = D(4)(1)(–1)

−8
D=
4
D = –2
On substituting s = 3 in equation (2), we get,
34 – 7(3)3 + 13(3)2 + 4(3) – 12 = 0 + 0 + 0+ 0 + E(3)2 (3 – 1) (3 – 2)
9 = E(9) (2) (1)

1
∴E =
2

Look for the SIA GROU P LOGO on the TITLE COVER before you buy
UNIT-1 (Laplace Transform) 1.55
Comparing ‘s4’ terms on both sides of equation (2), we get,
1=A+C+D+E
⎛ −1 ⎞ 1
1 = A + ⎜ ⎟ + (−2) +
⎝ 2 ⎠ 2
1=A–2

⇒ ∴A= 3
On substituting corresponding values in equation (1), we get,
−1 1
3 2 2 −2
L[y] = + 2 + + + 2
s s s −1 s − 2 s − 3
Applying inverse Laplace transform on both sides we get,
⎧3 2 1 1 2 1 1 ⎫
L−1{L[ y ]} = L–1 ⎨ + 2 − − + ⎬
⎩ s s 2 s − 1 s − 2 2 s − 3⎭

⎡1 ⎤ ⎡ 1 ⎤ 1 –1 ⎡ 1 ⎤ ⎡ 1 ⎤ 1 –1 ⎡ 1 ⎤
y = 3L−1 ⎢ ⎥ + 2L−1 ⎢ 2 ⎥ – L ⎢ ⎥ – 2 L–1 ⎢ ⎥ + L ⎢ ⎥
⎣s⎦ ⎣s ⎦ 2 ⎣ s −1 ⎦ ⎣s − 2⎦ 2 ⎣ s − 3⎦

1 t 1 ⎛ ⎡ 1 ⎤ at ⎞
= 3(1) + 2(t) – e − 2e 2t + e 3t ⎜⎜Q L−1 ⎢ ⎥ = e ⎟⎟
2 2 ⎝ ⎣ s − a ⎦ ⎠

1 1
∴ y = 3 + 2t − e t − 2e 2t + e 3t
2 2
d2 x dx 2t
Q60. Solve the following differential equation using the Laplace transforms 2 – 2 dt + x = e with
dt
dx
x(0) = 2, = –1 at t = 0.
dt
Ans: Given differential equation is,

d 2x dx
2
− 2 + x = e2t
dt dt

⎛ dx ⎞
⇒ x'' – 2x' + x = e2t ⎜Q = x′ ⎟
⎝ dt ⎠
Applying Laplace transform on both sides, we get,
L[x'' – 2x' + x] = L[e2t]

1 ⎛ 1 ⎞
⎜Q L[ e ] =
at
⇒ L[x''] – 2L[x'] + L[x] = ⎟
s−2 ⎝ s − a⎠

1
⇒ [s2L[x] – s x(0) – x'(0)] – 2(s L[x] – x(0)) + L[x] = ... (1)
s−2
[Q L[yn] = snL{y} – sn – 1. y(0) – sn–2. y'(0)...]
dx
Since, x(0) = 2 and x' = = – 1 equation (1) becomes,
dt

1
[s2 L[x] – s(2) – (–1)] – 2(sL[x] – 2) + L[x] =
s−2

1
⇒ s2L[x] – 2s + 1 – 2sL[x] + 4 + L[x] =
s−2

SPECTRUM ALL-IN-ONE JOURNAL FOR ENGINEERING STUDENTS SIA GROUP


1.56 MATHEMATICS-II [JNTU-ANANTAPUR]

1
⇒ (s2 – 2s + 1)L[x] – 2s + 1 + 4 =
s−2

1 1 + (2s − 5)(s − 2)
⇒ (s2 – 2s + 1)L[x] = + 2s – 5 =
( s − 2) ( s − 2)

1 + (2s 2 − 4s − 5s + 10)
=
( s − 2)

1 + 2s 2 − 9s + 10
⇒ (s – 1)2 L[x] =
( s − 2)

2s 2 − 9s + 11
L[x] =
( s − 1) 2 (s − 2)
On applying partial fractions, we get,
2s 2 − 9s + 11 A B C
L[x] = = + + ... (2)
( s − 1) (s − 2) ( s − 1) ( s − 1)
2 2
( s − 2)
⇒ A(s – 1) (s – 2) + B(s – 2) + C(s – 1)2 = 2s2 – 9s + 11 ... (3)
On substituting s = 1 in equation (3), we get,
A(1 – 1) (1 – 2) + B(1 – 2) + C(1 – 1)2 = 2(1)2 – 9(1) + 11
⇒ – B = 2 – 9 + 11
⇒ – B =4
∴ B = −4
On substituting s = 2 in equation (3), we get,
A(2 – 1) (2 – 2) + B(2 – 2) + C(2 – 1)2 = 2(2)2 – 9 (2) + 11
C = 2(4) – 18 + 11
= 8 – 18 + 11
=1
∴ C =1
On comparing coefficient of s2 in equation (3), we get,
A + C =2
A + 1 =2
A =2–1=1
∴ A =1
On substituting A, B, C in equation (2), we get,
1 4 1
L(x) = − +
( s − 1) ( s − 1) 2
( s − 2)
On applying inverse Laplace transform on both sides, we get,
⎡ 1 4 1 ⎤
⇒ L–1 L{x} = L–1 ⎢ ( s − 1) − + ⎥
(s − 2) ⎥⎦
⎢⎣ ( s − 1) 2

⎛ 1 ⎞ ⎛ 1 ⎞ −1 ⎛ 1 ⎞
⎜ ⎟
x = L–1 ⎜ ⎟ – 4 L–1
⎝ s −1 ⎠ ⎜ ( s − 1) 2 ⎟ + L ⎜⎝ s − 2 ⎟⎠
⎝ ⎠

⎛ 1 ⎞ ⎛ 1 ⎞
x = et – 4 t.et + e2t [Q L–1 ⎜ ⎟ = eat; L–1 ⎜ ⎟
⎝ s−a⎠ ⎜ (s − a) 2 ⎟ = t eat]
⎝ ⎠

∴ x = e t − 4t e t + e 2 t

Look for the SIA GROU P LOGO on the TITLE COVER before you buy
UNIT-2 (Fourier Series) 2.1

UNIT FOURIER SERIES

2
PART-A
SHORT QUESTIONS WITH SOLUTIONS
Q1. Define a periodic function.
Ans: A function f (x) defined for all real numbers is said to be periodic if there is a real number T < 0 such that,
f (x + T) = f (x) for all x ... (1)
And the number T is called its period.
∴ From equation (1),
f(x ± kT) = f (x) for all x, and
k = 1, 2, 3, ... i.e., 2T, 3T, 4T etc., are also periods of f (x).
Q2. Give the Euler’s formulae for Fourier series. Model Paper-I, Q1(c)

Ans: The Fourier series expansion of function f(x) in the interval k ≤ x ≤ k + 2π is given by,

a0 ∞
f(x) = +
2 n =1 ∑
(an cos nx + bn sin nx)

Where,
k +2 π
1
a0 =
π ∫ f ( x)dx
k

k +2 π
1
an=
π ∫ f ( x) cos nx dx
k

k + 2π
1
bn=
π ∫ f ( x) sin nx dx
k

The above Fourier coefficients a0, an and bn are referred to as Euler’s formulae.
Q3. Give the Dirichlet conditions for Fourier expansion. Model Paper-II, Q1(c)

Ans:
Dirichlet’s Conditions
1. f (x) is uniformly bounded on the fundamental interval (–π, π) of period 2π, that is, | f (x)| < M for all –π < x < π where M is
a constant.
2. f (x) has no more than a finite number of finite discontinuities in (–π, π).
3. f (x) has a finite number of strict maxima or minima in (–π, π).

SPECTRUM ALL-IN-ONE JOURNAL FOR ENGINEERING STUDENTS SIA GROUP


2.2 MATHEMATICS-II [JNTU-ANANTAPUR]
Q4. 2
π as a Fourier series.
Expand f(x) = x , 0 < x < 2π
Ans: Given that,
f(x) = x2
The Fourier series is given as,

a0
f(x) = +
2 n =1 ∑
(an cos nx + bn sin nx) ... (1)


1
a0 =
π ∫ f ( x)dx
0

2π 2π
1 1 ⎡ x3 ⎤ 1 1 8π 2
∫ x 2 dx = π − = π
3 3 3
⇒ a0 = ⎢ ⎥ = [( 2 ) 0 ] ( 8 ) =
π π ⎣⎢ 3 ⎦⎥ 3π 3π 3
0 0


1
an =
π ∫ f ( x) cos nxdx
0

2π 2π
1 1 ⎡ 2 sin nx ⎛ − cos nx ⎞ ⎛ − sin nx ⎞⎤
∫ − 2 x⎜ ⎟ + 2⎜ ⎟⎥
2
⇒ an = x cos nx.dx = ⎢ x
π π⎣ n ⎝ n 2
⎠ ⎝ n 3 ⎠⎦ 0
0

1 ⎡⎛ 4π ⎞ ⎤
= ⎢⎜ 0 + 2 cos 2nπ − 0 ⎟ − (0 + 0 − 0) ⎥
π ⎣⎝ n ⎠ ⎦
an = 4/n2
2π 2π
1 1 ⎡ 2 ( − cos nx ) (sin nx) 2 cos nx ⎤
bn =
π ∫
0
x 2 sin nx.dx =
π ⎢⎣
x
n
+ 2x
n2
+
n3 ⎦ 0

1 ⎡⎛⎜ − 4π 2 cos 2nπ 2 cos 2nπ ⎞⎟ ⎛ 2 ⎞⎤


= ⎢⎜ +0+ − ⎜0 + 0 + 3 ⎟⎥
π ⎣⎢⎝ 3 ⎟
n n ⎠ ⎝ n ⎠⎦⎥

1 ⎡⎛⎜ − 4 π 2 2 ⎞ ⎤ −4π
= ⎢⎜ + 0 + 3 ⎟ − (0 + 0 + 2 n 3 )⎥ =
π ⎣⎢⎝ n n ⎠⎟ ⎥⎦ n
On substituting a0, an, bn values in equation (1), we get,
∞ ∞
4 2 4 4π
∴ x2 =
3
π + ∑n
n =1
2
cos nx − ∑
n =1
n
sin nx

Q5. Define even and odd function. Model Paper-III, Q1(c)

Ans:
Even Function: A function f(x) is an even function if f(–x) = f(x)
Examples: cos x, sec x, x2
Odd Function: A function f(x) is an odd function if f(–x) = –f(x).
Examples: sin x, tan x, x3
Q6. Give the Fourier series expansion for even periodic functions. Model Paper-II, Q1(d)

Ans: Generally, a periodic function f(x) defined in the interval (–π, π) is represented by,
∞ ∞
a0
f(x) =
2
+ ∑ n =1
an cos nx + ∑b
n =1
n sin nx ... (1)

Look for the SIA GROU P LOGO on the TITLE COVER before you buy
UNIT-2 (Fourier Series) 2.3
Where, Ans: A periodic function f(x) defined in the interval (–π, π) is
represented by,
π
1
a0 =
π ∫ f ( x ) dx
−π
... (2)
f(x) =
a0


+ an cosnx + bn sin nx

∑ ... (1)
2 n=1 n=1
π
1 Where,
an =
π ∫ f ( x) cos nx dx
−π
... (3)
π
1
π
a0 =
π ∫ f ( x ) dx
−π
... (2)
1
bn =
π ∫ f ( x) sin nx dx
−π
... (4)
π
1
When f(x) is an even function,
an =
π ∫ f ( x) cos nx dx
−π
... (3)

From equation (2), we get,


π
1
∫ f ( x) sin nx dx
π
1 bn = ... (4)
a0 =
π ∫
−π
f ( x) dx π
−π

When f(x) is an odd function,


π From equation (2), we get,
2
⇒ a0 =
π ∫ f ( x) dx 1
π

∫ f ( x)dx = 0
0
a0 = (Q f(x) is odd)
As ‘cos nx’ is an even function, f(x) cos nx is also an π
−π
even function.
As ‘cos nx’ is an even function, here f(x) cos nx becomes
∴ From equation (3), we get, an odd function
π ∴ From equation (3),
1
an =
π ∫ f ( x) cos nx dx 1
π
−π
an =
π ∫ f ( x) cos nx dx = 0
−π
π
2
⇒ an =
π ∫ f ( x) cos nx dx
0
sin nx is an odd function and f(x) sin nx is an even
function.
Since, sin nx is an odd function and f(x) sin nx also ∴ From equation (4),
beomes an odd function π
1
∴ From equation (4), bn =
π ∫ f ( x) sin nx dx
−π
π
1
bn =
π −π∫f ( x ) sin x dx = 0
2
π

⇒ bn =
π ∫ f ( x) sin nx dx
Therefore, it can be concluded that if a function f(x) is 0
even in the interval (–π, π), its Fourier series expansion Therefore, it can be concluded that if a function f(x) is
consists of only cosine terms. odd in the interval (–π, π), its Fourier series expansion consists

of only sine terms.
∑a
a0
∴ f(x) = + n cos nx ∞
2 n=1
∴ f(x) = ∑b
n =1
n sin nx
Where,
Where,
π
2
a0 =
π ∫ f ( x)dx 2
π

0 bn =
π ∫ f ( x) sin nx dx
0
π
2 Q8. Define the Fourier series for functions having
an =
π ∫ f ( x) cos nx . dx
0
period ‘2l’.
Ans: Consider a function f(x), which is periodic in the interval
Q7. Give the Fourier series expansion for odd (k, k + 2l) and period ‘2l’. Then, the fourier series expansion of
periodic functions. Model Paper-I, Q1(d) f(x) is represented as,

SPECTRUM ALL-IN-ONE JOURNAL FOR ENGINEERING STUDENTS SIA GROUP


2.4 MATHEMATICS-II [JNTU-ANANTAPUR]
∞ ∞
nπx nπx
∑ ∑b sin
a0
f(x) = + an cos + n
2 n =1
l n=1
l

Where,
k + 2l
1
a0 =
l ∫ f ( x)dx
k

k + 2l
1 nπx
an =
l ∫
k
f ( x ) cos
l
dx

k + 2l
1 nπx
bn =
l ∫
k
f ( x ) sin
l
dx

Q9. Define half range Fourier sine series of f(x).


Ans: The half-range Fourier sine series of f(x) is defiend in the interval (0, π) is represented as,

f(x) = ∑ b sin nx
n =1
n

π
2
Where, bn =
π ∫ f ( x) sin nx dx
0

Q10. Define half-range Fourier cosine series of f(x). Model Paper-III, Q1(d)

Ans:
The half-range Fourier cosine series of f(x) defined in the interval (0, π) is represented as,

∑a
a0
f(x)= + n cos nx
2 n=1

Where,
π
2
a0 =
π ∫ f ( x)dx
0
and

π
2
an =
π ∫ f ( x) cos nx dx
0

Look for the SIA GROU P LOGO on the TITLE COVER before you buy
UNIT-2 (Fourier Series) 2.5

PART-B
ESSAY QUESTIONS WITH SOLUTIONS
2.1 DETERMINATION OF FOURIER COEFFICIENTS
π , π ).
Q11. Express f(x) = x as a Fourier series in (–π
Ans:
Given expression is,
f(x) = x in (–π,π)
The Fourier series representation of f(x) in (–π, π) is given by,
∞ ∞

∑b
a0
f(x) =
2
+ ∑an =1
n cos nx +
n =1
n sin nx ... (1)

π π π
1 1 1 ⎡ x2 ⎤
Where, a0 =
π ∫
−π
f(x) dx =
π ∫
−π
x.dx = ⎢ ⎥
π ⎣⎢ 2 ⎦⎥
−π

1 1 2
= [(π)2 – (–π)2] = [π – π2]
2π 2π

∴ a0 = 0

π
1
an =
π ∫ x. cos nx dx
−π

=
1
π
[∫ ∫ (∫
x cos nx dx – 1 cos nx.dx dx
π

–π
) ]
π
1 ⎡ x. sin nx sin nx ⎤
= ⎢
π ⎣ n

n ∫
dx ⎥
⎦ −π
π
1 ⎡ x. sin nx ( − cos nx ) ⎤

π ⎢⎣ n ⎥
=
n2 ⎦ −π

π
1 ⎡ x. sin nx cos nx ⎤
= ⎢ + ⎥
π ⎣ n n 2 ⎦ −π

1 ⎧ π sin nπ cos nπ ⎡ − π sin n(− π) cos n(− π) ⎤ ⎫


= ⎨ + −⎢ + ⎥⎬
π ⎩ n n2 ⎣ n n2 ⎦⎭

1 ⎧ π sin nπ cos nπ ⎡ π sin nπ cos nπ ⎤ ⎫


⎨ + −⎢ + ⎬
n 2 ⎥⎦ ⎭
=
π ⎩ n n2 ⎣ n

1 ⎧ π sin nπ cos nπ π sin nπ cos nπ ⎫


= ⎨ + − − ⎬
π ⎩ n n2 n n2 ⎭

1
= {0} = 0
π

∴ an = 0

SPECTRUM ALL-IN-ONE JOURNAL FOR ENGINEERING STUDENTS SIA GROUP


2.6 MATHEMATICS-II [JNTU-ANANTAPUR]
π π
1 1
bn =
π
−π
∫ f(x). sin nx dx =
π ∫ x.sin nx dx
−π

1 ⎡ ⎧⎪ ⎛ ⎞ ⎫⎪

π ⎢ ∫
= ⎢ x sin nx dx − ⎨ 1⎜ sin nx.dx ⎟dx ⎬⎥
⎪⎩ ⎝ ⎠ ⎪⎭⎥ ∫ ∫
⎣ ⎦ −π
π
1 ⎡ x.(− cos nx) ( − cos nx ) ⎤
=
π ⎢⎣ n

n ∫ dx ⎥
⎦ −π
π
1 ⎡ − x. cos nx sin nx ⎤
+ 2 ⎥
π ⎢⎣
=
n n ⎦ −π

1 ⎧ − π cos nπ sin nπ ⎡ − (− π) cos n (− π) sin n( − π) ⎤ ⎫


= ⎨ + −⎢ + ⎥⎬
π ⎩ n n2 ⎣ n n2 ⎦⎭

1 ⎧ − π cos nπ sin nπ ⎡ π cos nπ sin nπ ⎤ ⎫


= ⎨ + 2 −⎢ − 2 ⎥⎬
π ⎩ n n ⎣ n n ⎦⎭

1 ⎧ − π cos nπ sin nπ π cos nπ sin nπ ⎫


= ⎨ + − + 2 ⎬
π ⎩ n n2 n n ⎭

1 ⎧ − 2π cos nπ 2 sin nπ ⎫
= ⎨ + ⎬
π ⎩ n n2 ⎭

− 2 cos nπ 2 sin nπ
= + (Here, sin nπ = 0)
n πn 2

−2 −2
= cos nπ = (–1)n For n = 1, 2, 3...
n n
(Q cos nπ = (–1)n for n = 1, 2, 3...)
−2
∴ bn = (−1) n
n
On substituting the values of a0, an and bn in equation (1), we get,
∞ ∞
−2 ⎡ (−1) n ⎤
f(x) = ∑ ⎡⎢⎣ n
n =1

(−1) n sin nx ⎥ = –2
⎦ n =1
∑ ⎢⎢⎣ n
sin nx ⎥
⎥⎦

⎡ 1 1 ⎤
∴ f ( x) = −2⎢− sin x + sin 2 x − sin 3x + ...⎥
⎣ 2 3 ⎦


1
π , π], deduce the value of
Q12. Obtain the Fourier series for the function, f(x) = x + x in [–π 2
∑n
n =1
2
. Model Paper-III, Q4

Ans: The given function is,


f (x) = x + x2 in (–π, π)
From the definition of trigonometric Fourier series,

∞ ∞

∑ ∑b
a0
f(x) = + an cos nx + n sin nx ... (1)
2 n =1 n =1

Look for the SIA GROU P LOGO on the TITLE COVER before you buy
UNIT-2 (Fourier Series) 2.7

π π π
1 1 1 ⎡ x2 x3 ⎤
Where, a0 =
π ∫ f ( x ) dx =
π ∫
−π
( x + x 2 ) dx = ⎢ + ⎥
π ⎣⎢ 2 3 ⎥⎦
−π
−π

1 ⎡ π 2 π 3 ⎛ ( −π ) 2 ( −π )3 ⎞ ⎤ 1 ⎡ π 2 π3 π 2 π 3 ⎤
⎢ + − + ⎥ = ⎢ + − + ⎥
3 ⎝⎜ 2 3 ⎠⎟ ⎦⎥
=
π ⎣⎢ 2 π ⎢⎣ 2 3 2 3 ⎥⎦

1 ⎡ 2π3 ⎤ 2π2
= ⎢ ⎥ =
π ⎣⎢ 3 ⎦⎥ 3

2π 2
∴ a0 =
3

1 ⎡⎡ sin nx ⎤
π π π
1 sin nx ⎤
an =
π ∫ ( x + x 2 ) cos nx.dx = ⎢ ⎢ ( x + x 2 ).
π ⎢⎣ ⎥
n ⎦ −π
− ∫
(1 + 2 x).
n
.dx ⎥

−π ⎣ −π ⎦

⎡ ⎛⎡ ⎤
(− cos nx) ⎤ ⎞⎟⎥
π −π π
1 ⎢⎡ 2 sin nx ⎤ ⎜⎢ + ⎛ − cos nx ⎞ ⎡ ⎛ d ⎞ ⎤
=
π ⎢⎢⎣
( x + x ). −
n ⎥⎦ −π ⎜⎜ ⎢
(1 2 x )⎜ ⎟
⎝ n 2 ⎠ −π − π
− ( 2)
n2 ∫ .dx ⎥
⎥ ⎟⎟⎥ ⎣
∫ ∫ ∫ ⎝ dx ∫
⎢Q uvdx = u vdx − ⎜ u vdx ⎟dx ⎥
⎠ ⎦
⎣ ⎝⎣ ⎦ ⎠⎦

⎡ π ⎛⎡ π π ⎤
1 ⎢⎡ 2 sin nx ⎤ ⎛ − cos nx ⎞ ⎤ ⎛ sin nx ⎞ ⎥
= π ⎢ ⎣⎢ +
( x x ). – ⎜⎢ +
n ⎦⎥ −π ⎜⎝ ⎣
(1 2 x )
⎝⎜ n 2 ⎠⎟ ⎥⎦ −π ⎝⎜
+ 2. ⎟
n3 ⎠ – π ⎥
⎣ ⎦

π π π
1 ⎡⎡ sin nx ⎤ ⎡ cos nx ⎤ ⎡ sin nx ⎤ ⎤
= ⎢ ⎢( x + x 2 ). ⎥ + ⎢(1 + 2 x). 2 ⎥ − ⎢2. 3 ⎥ ⎥
π ⎣⎢ ⎣ n ⎦ −π ⎣ n ⎦ −π ⎣ n ⎦ − π ⎦⎥

1⎡ sin nπ cos nπ sin nπ sin n(−π) −(1 + 2( −π)). cos n(−π) + 2 sin n( −π ) ⎤
= ⎢ (π + π) 2 . + (1 + 2π). 2 − 2. 3 − (−π + (−π) 2 ). ⎥
π⎣ n n n n n2 n3 ⎦

1⎡ (−1) n cos nπ ⎤
= ⎢0 + (1 + 2π) 2 + 0 − 0 − (1 − 2π). 2 − 0⎥ [Q sin nπ = 0]
π ⎢⎣ n n ⎥⎦

1⎡ (−1) n (−1) n ⎤
= ⎢(1 + 2π) 2 − (1 − 2π). 2 ⎥ [Q cos nπ = (–1)n]
π ⎣⎢ n n ⎦⎥

1 ⎡ (−1) n 2π.(−1) n (−1) n 2π(−1) n ⎤ 1 ⎡ (−1)n ⎤


= ⎢ + − 2 + ⎥ = ⎢4π 2 ⎥
π ⎢⎣ n 2 n2 n n 2 ⎥⎦ π ⎢⎣ n ⎥⎦

4(−1) n
=
n2

4(−1) n
∴ an =
n2

−4
∴ a1 =
(1) 2

SPECTRUM ALL-IN-ONE JOURNAL FOR ENGINEERING STUDENTS SIA GROUP


2.8 MATHEMATICS-II [JNTU-ANANTAPUR]

4
a2 =
(2) 2

−4
a3 =
(3) 2

4
a4 = and so on.
(4) 2

π
1
∫ (x + x
2
And bn = ). sin nx.dx
π
−π

1 ⎡⎡ − cos nx ⎤
π π
⎛ − cos nx ⎞⎤
= ⎢ ⎢ ( x + x 2 )⎜
π ⎢⎣ ⎝ n ⎠⎦ − π ∫
⎟⎥ − (1 + 2 x ).
n
.dx ⎥

⎣ −π ⎦

1 ⎢⎡ ⎡ ⎡⎡ sin nx ⎤ ⎤
π π π
⎛ − cos nx ⎞ ⎤ sin nx ⎤
=
π ⎢⎣ ⎢ (x + x2 ) ⎜
⎝ ⎟ ⎥
n ⎠ ⎦ −π ⎢ ⎣ ∫
+ ⎢ ⎢ (1 + 2 x ). 2 ⎥ − (2). 2 .dx ⎥ ⎥
n ⎦ −π – π n ⎥⎥
⎣ ⎣ ⎦⎦

π π
1⎡ 2 ⎛ − cos nx ⎞ ⎤ ⎡ sin nx ⎤ ⎡ ⎛ − cos nx ⎞ π ⎤
= ⎢( x + x ) ⎜ ⎟ ⎥ + ⎢(1 + 2 x ). 2 ⎥ − ⎢ 2⎜ ⎟ ⎥
π⎣ ⎝ n ⎠⎦ − π ⎣ n ⎦ − π ⎢⎣ ⎝ n 3 ⎠ − π ⎥⎦
π
1⎡ 2 ⎛ − cos nx ⎞ sin nx 2 cos nx ⎤
= ⎢( x + x )⎜ ⎟ + (1 + 2 x ). 2 + ⎥
π⎣ ⎝ n ⎠ n n 3 ⎦ −π

⎡ 2 ⎛ − cos n π ⎞ sin nπ cos nπ ⎤


⎢ ( π + π ) ⎜⎝ ⎟⎠ + (1 + 2 π ). 2
+2 3
( −π + ( −π ) 2 ) ⎥
1 n n n
= ⎢ ⎥
π ⎢ ⎛ − cos n ( −π ) ⎞ sin n ( −π ) 2 cos n ( −π ) ⎥
⎢ ⎜⎝ ⎟⎠ − (1 + 2( −π )). − ⎥
⎣ n n2 n3 ⎦

1⎡ (−1)n ( −1) n (−1)n (−1)n ⎤


⎢(π + π )( −1) + (1 + 2π ).0 + 2. 3 − (π 2 − π)( −1) − (1 − 2π).0 − 2. 3 ⎥
2
=
π ⎣⎢ n n n n ⎥⎦

1⎡ 2 ( −1)
n
2(−1)n (−1)n 2(−1)n ⎤
= π⎢ − ( π + π ) + (0) + + ( π 2
− π ) – (0) – ⎥
⎣⎢ n n3 n n3 ⎦⎥

1 ⎡ ( −1)n ( −1)n (−1)n ( −1)n ⎤ 1 ⎡ (−1) n ⎤ − 2(−1) n


= ⎢ −π − π2 + π2 −π ⎥ = ⎢− 2π ⎥ =
π ⎢⎣ n n n n ⎥⎦ π ⎢⎣ n ⎥⎦ n

− 2(−1) n
∴ bn =
n

2 −2
∴ b1 = ; b2 =
1 2

2 −2
b3 = ; b4 = and so on.
3 4

Look for the SIA GROU P LOGO on the TITLE COVER before you buy
UNIT-2 (Fourier Series) 2.9
On substituting the corresponding values in equation (1), we get,

∞ ∞
2π 2 4(−1) n − 2(−1) n
f (x) =
3(2)
+ ∑
n =1 n2
. cos nx + ∑
n =1
n
. sin nx

π2 ⎡ ( −1) 1 ( −1) (1) ⎤


f (x)= + 4 ⎢ 2 cos( x) + 2
.cos 2 x + 2
.cos(3x ) + 2
.cos 4 x + ...⎥ +
3 ⎣ (1) (2) (3) (4) ⎦
⎡ (2) 2 2 2 ⎤
⎢⎣ 1 .sin x − 2 .sin 2 x + 3 sin 3 x − 4 .sin 4 x + ....⎥⎦

π2 ⎡ cos 2 x cos 3 x cos 4 x ⎤ ⎡ sin 2 x sin 3 x sin 4 x ⎤


x+ x =2 − 4 ⎢ cos x − 2
+ 2
− 2
+ .....⎥ + 2 ⎢ sin x − + − + ....⎥ ... (2)
3 ⎣ 2 3 4 ⎦ ⎣ 2 3 4 ⎦
On substituting x = π in equation (2), we get,
π2 ⎡ cos2π cos3π cos4π ⎤ ⎡ sin 2π ⎤
π + π2 = − 4 ⎢cos π − 2 + 2 − 2 + ....⎥ + 2 ⎢sin π − + ....⎥
3 ⎣ 2 3 4 ⎦ ⎣ 2 ⎦

π2 ⎡ 1 1 1 ⎤
⇒ π + π2 = − 4 ⎢ −1 − 2 − 2 − 2 + ...⎥ + 2[0 − 0 + 0....]
3 ⎣ 2 3 4 ⎦

π2 ⎛ 1 1 1 ⎞
⇒ π + π2 – = − 4⎜ − 1 − 2 − 2 − 2 + .... ⎟
3 ⎝ 2 3 4 ⎠

⎛ 1⎞ ⎛ 1 1 1 ⎞
⇒ π + π2 ⎜ 1 − ⎟ = 4⎜1 + 2 + 2 + 2 + .... ⎟
⎝ 3 ⎠ ⎝ 2 3 4 ⎠

π 2π 2 1 1 1
⇒ + = 1 + 2 + 2 + 2 + .....
4 4×3 2 3 4

∑n
π π2 1
⇒ + = 2
4 6 n =1


π π2
∑n
1
∴ 2
= +
n =1
4 6

π
Q13. Expand f(x) = ex, – π < x < π as a Fourier series. Derive a series for .
sinh π
Ans: Given function is, f(x) = ex, – π < x < π
The Fourier series expansion of f(x) is given as,
∞ ∞
a0
f(x) =
2
+ ∑a
n =1
n cos nx + ∑b
n =1
n sin nx ... (1)

π
1
Where, a0 =
π −π ∫
f ( x) dx

π
1 1 x π 1
= ∫
π −π
e x dx = [e ]−π = [e π − e − π ]
π π

SPECTRUM ALL-IN-ONE JOURNAL FOR ENGINEERING STUDENTS SIA GROUP


2.10 MATHEMATICS-II [JNTU-ANANTAPUR]
On multiplying and dividing R.H.S by 2, we get,

1 2 π 2 ⎡ e π − e −π ⎤
a0 = . [e − e − π ] = ⎢ ⎥
π 2 π ⎢⎣ 2 ⎥⎦

2 ⎡ eπ − e−π ⎤
= . sinh π ⎢Q sinh π = ⎥
π ⎣ 2 ⎦

2
∴ a0 = sinh π
π
π
1
an =
π ∫ f ( x) cos nxdx
−π

π
1
=
π −π ∫
e x cos nxdx

∫e
x
Consider, cos nxdx = In

⎡d ⎤
∫ ∫
In = cos nx e dx − ⎢ (cos nx ). e dx ⎥ dx ∫
x x

⎣ dx ⎦


In = cos nx.e − − n sin nx.e dx
x x


In = e cos nx + n e sin nxdx
x x

⎡ ⎛d ⎞ ⎤
⇒ In = e x cos nx + n ⎢sin nx e x dx − ⎜ ∫ ∫ ⎝ dx (sin nx )∫ e dx ⎟⎠ dx ⎥⎦
x




In = e x cos nx + n[sin nx.e x − n cos nxe x dx ]

In = e cos nx + n[e sin nx − n e cos nxdx] ∫


x x x

In = e cos nx + n.e sin nx − n e cos nxdx ∫


x x 2 x

⇒ In = ex(cos nx + nsin nx) – n2In


⇒ In + n2In = ex(cos nx + nsin nx)
⇒ In(1 + n2) = ex(cos nx + nsin nx)
ex
⇒ In = (cos nx + nsin nx)
1 + n2
ex
⇒ ∫ e x cos nx dx =
1+ n2
(conx + nsin nx) ... (2)

π
1
∴ an =
π −π ∫
e x cos nxdx

π
1 ⎡ ex ⎤
= ⎢ (cos nx + n sin nx )⎥ [Q From equation (2)]
π ⎢⎣1 + n 2
⎥⎦ − π

Look for the SIA GROU P LOGO on the TITLE COVER before you buy
UNIT-2 (Fourier Series) 2.11

1
= [( e π (cos nπ + n sin nπ )) − ( e − π (cos( − n π) + n sin( − nπ ))]
π (1 + n 2 )
1
= [ e π (cos n π + 0) − e − π (cos( − nπ ) + 0 )]
π (1 + n 2 )

1
= [e π cos nπ − e − π cos nπ] [Q cos(– θ) = cos θ]
π(1 + n ) 2

cos nπ(e π − e − π )
= ... (3)
π(1 + n 2 )
On multiplying and dividing the equation (3) by 2 on R.H.S, we get,

cos nπ (e π − e − π )
an = 2. .
π(1 + n 2 ) 2

2(−1) n
= sinh π
π(1 + n 2 )

2(−1) n sinh π
∴ an =
π(1 + n 2 )
π
1
∴ bn =
π ∫ f ( x) sin nxdx
−π

π
1
∫e
x
= sin nxdx
π
−π

∫e
x
Consider, sin nxdx = In

⎡d ⎤
∫ ∫
In = sin nx e dx − ⎢ (sin nx ). e dx ⎥ dx ∫
x x
⎣ dx ⎦


x x
⇒ In = sin nx.e − n cos nx.e dx


x x
⇒ In = e . sin nx − n e cos nxdx

⎡ ⎡d ⎤ ⎤

⎣ ⎣ dx ∫ ∫
In = e x sin nx − n ⎢cos nx e x dx − ⎢ (cos nx ) e x dx ⎥ dx ⎥
⎦ ⎦ ∫
⇒ In = e x sin nx − n[cos nx .e x − − n sin nx.e x dx ]∫

x x x
⇒ In = e sin nx − n[e cos nx + n e sin nxdx]


x x 2 x
⇒ In = e sin nx − ne cos nx − n e sin nxdx

⇒ In = e x (sin nx − n cos nx) − n 2 I n


⇒ In + n2In = ex(sin nx – ncos nx)
⇒ In(1 + n2) = ex(sin nx – ncos nx)

ex
⇒ In = (sin nx − n cos nx )
1 + n2

SPECTRUM ALL-IN-ONE JOURNAL FOR ENGINEERING STUDENTS SIA GROUP


2.12 MATHEMATICS-II [JNTU-ANANTAPUR]

ex
∫ e x sin nxdx =
1 + n2
(sin nx – ncosnx) ... (4)

π
1
∴ bn =
π −π ∫
e x sin nxdx

π
1 ⎡ ex ⎤
= ⎢ (sin nx − n cos nx )⎥ [Q From equation (3)]
π ⎢⎣1 + n 2
⎥⎦ − π

1
= [e π (sin nπ − n cos nπ) − e − π (sin( − nπ) − n cos( − nπ))]
π(1 + n 2 )

1
= [eπ(0 – ncosnπ) – e– π(0 – ncosnπ)] [Q cos(–θ) = cos θ]
π(1 + n 2 )

1
= [ − ne π cos nπ + e − π n cos nπ]
π(1 + n 2 )

n cos nπ
= [− e π + e − π ] ... (5)
π(1 + n 2 )
On multiplying and dividing equation (5) by 2 on R.H.S, we get,

⎡ eπ − e−π ⎤
⎢Q sinh π = ⎥
n cos nπ [e − π − e π ] 2n( −1) n ( −1)
⎢ 2 ⎥
bn = × 2 = sinh π
π(1 + n 2 ) 2 π(1 + n 2 ) ⎢ e−π − eπ ⎥
⎢⇒ − sinh π = ⎥
⎣ 2 ⎦

2n( −1) n+1 sinh π


∴ bn =
π(1 + n 2 )

On substituting the values of a0, an and bn in equation (1), we get,


2
sinh π ∞ ∞
π 2 sinh π ( −1) n 2 sinh π n(−1) n +1
f(x) =
2
+
π ∑ 1+ n
n =1
2
cos nx +
π ∑n =1 1+ n 2
sin nx

∞ ∞
sinh π 2 sinh π ⎡ ( −1) n n (−1) n +1 ⎤
ex =
π
+
π
⎢ ∑
⎣⎢ n =1 1 + n
2
cos nx + ∑
n =1 1+ n2
sin nx ⎥
⎦⎥
∴ The required Fourier series is,

2 sinh π ⎡1 ∞
( −1) n

n( −1) n +1 ⎤
ex =
π
⎢ + ∑ 1+ n 2
cos nx + ∑ 1+ n 2
sin nx ⎥ ... (6)
⎣⎢ 2 n =1 n =1 ⎦⎥

π
Deriving the Series for : On substituting x = 0 in equation (6), we get,
sinhπ

2 sinh π ⎡ 1 ∞
(−1) n ∞
n( −1) n +1 ⎤
e0 = ⎢ +
π ⎢⎣ 2 ∑
n =1 1 + n
2
cos 0 + ∑
n =1 1 + n
2
sin 0⎥
⎥⎦

2 sinh π ⎡ 1 ∞
(−1) n ∞
n( −1) n +1 ⎤
⇒ 1= ⎢ +
π ⎣⎢ 2 ∑1+ n
n =1
2
.1 + ∑
n =1 1 + n
2
.0⎥
⎦⎥

Look for the SIA GROU P LOGO on the TITLE COVER before you buy
UNIT-2 (Fourier Series) 2.13

π ⎡1 ∞
( − 1) n ⎤

sinh π
= 2⎢ 2 +
⎢⎣
∑ 1+ n
n =1
2
+ 0⎥
⎥⎦

π ⎡ 1 (−1)1 (−1) 2 (−1) 3 (−1) 4 ⎤


⇒ = 2⎢ + + + + + .....⎥
sinh π ⎣⎢ 2 1 + 1 1 + 2
2 2
1+ 3 2
1+ 4 2
⎦⎥

π ⎡ 1 (−1) 1 (−1) 1 ⎤
⇒ = 2⎢ + + + + + ...⎥
sinh π ⎣ 2 1+1 1+ 2 2
1+ 3 2
1+ 4 2 ⎦

π ⎡1 1 1 1 1 ⎤
⇒ = 2⎢ − + − + + .....⎥
sinh π ⎣2 2 1+ 2 1+ 3 1+ 4
2 2 2

π ⎡ 1 1 1 ⎤
∴ = 2⎢ 2
– 2
+ 2
– ......⎥
sinhπ ⎣1+ 2 1+ 3 1+ 4 ⎦
π , π ).
Q14. If f(x) = cosh ax, expand f(x) as a Fourier Series in (–π
Ans: Given function is,
f (x) = cosh ax in (–π,π)
The Fourier series representation of f(x) is given by,

a0 ∞ ∞
f(x) = +
2 n =1 ∑
an cos nx +
n =1
bn sin nx ∑ ... (1)

π π
1 1
Where, a0 =
π ∫
−π
f ( x) dx =
π ∫ cosh ax dx
−π

π
⎛ e ax + e − ax ⎞ ⎛ − ax ⎞
⎜Q cosh ax = e + e
ax
1 ⎟
=
π ∫ ⎜

− π⎝
2
⎟ dx



⎝ 2 ⎟

1 ⎡ ax ⎤
π π π
1
=
2π ∫ (e ax + e −ax ) dx =
2π ⎢ ∫
⎢ e dx + e − ax dx ⎥
∫ ⎥
−π ⎣ −π −π ⎦
⎡⎛ ax ⎞ π ⎛ − ax ⎞ π ⎤
1 ⎡⎡ e e ⎤ ⎡ a −aπ eaπ ⎤⎤
aπ − aπ
1 ⎢⎜ e ⎟ ⎜ e ⎟ ⎥
+ ⎢ − + − ⎥
2π ⎢⎜⎝ a ⎟⎠ −π ⎜⎝ − a ⎟⎠ −π ⎥ 2π ⎣ ⎢⎣ a a ⎥⎦ ⎢⎣ −a −a ⎥⎦⎦
= =
⎣ ⎦
aπ − aπ − aπ aπ
1 ⎧⎡ e

e− aπ ⎤ ⎡ −e− aπ eaπ ⎤⎫ 1 ⎡ e − e – e + e ⎤
− + + ⎥⎬ = ⎢
2π ⎨⎢⎣ a a ⎥⎦ ⎢⎣ a a ⎥⎦
=
⎩ a ⎦ ⎭ 2π ⎣ a a a

1 ⎡ 2e aπ 2e – aπ ⎤
= ⎢ – ⎥
2π ⎢⎣ a a ⎥⎦

1 2
= [ e aπ – e – aπ ] = sinh aπ
aπ aπ

2
∴ a0 = sinh aπ

π
1
∴ an =
π ∫ f ( x) cos nx dx
−π

SPECTRUM ALL-IN-ONE JOURNAL FOR ENGINEERING STUDENTS SIA GROUP


2.14 MATHEMATICS-II [JNTU-ANANTAPUR]
π π ⎛ − ax ⎞
⎜Q cosh ax = e + e
ax
1 1 ⎛ e ax + e −ax ⎞ ⎟
=
π ∫
−π
cosh ax cos nx dx =
π ∫ ⎜

− π⎝
2
⎟ cosnx dx



⎝ 2 ⎟

1 ⎡⎢ ax ⎤
π π

=
2π ⎢ ∫ ∫
e cos nx dx + e − ax cos nx dx ⎥

⎣ −π −π ⎦

⎧⎡ ax ⎤
π
⎡ e − ax ⎤ ⎫⎪
π
1 ⎪ e
= ⎨⎢ 2 ( a cos nx + n sin nx ) ⎥ + ⎢ 2 ( − a cos nx + n sin nx ) ⎥ ⎬
2π ⎪⎩⎣⎢ a + n
2
⎦⎥ − π ⎣⎢ a + n
2
⎦⎥ −π ⎪⎭

1 ⎧⎪⎡ e aπ ⎤ ⎡ e − aπ ⎤
= ⎨⎢ 2 ( a cos n π + n sin n π) ⎥−⎢ 2 ( a cos n( − π) + n sin n ( − π) ⎥ +
2π ⎪⎩⎢⎣ a + n 2
⎥⎦ ⎢⎣ a + n
2
⎥⎦

⎡ e − aπ ⎤ ⎡ e aπ ⎤ ⎫⎪
⎢ 2 ( − a cos n π + n sin n π ) ⎥−⎢ 2 [ − a cos n( − π) + n sin n ( − π)]⎥ ⎬
⎢⎣ a + n ⎥⎦ ⎢⎣ a + n
2 2
⎥⎦ ⎪⎭

1 ⎧⎪⎡ ae aπ cos nπ ne aπ sin nπ ae − aπ cos nπ ne − aπ sin nπ ⎤


= ⎨⎢ 2 + − + ⎥ +
2π ⎪⎩⎢⎣ a + n
2
a2 + n2 a2 + n2 a 2 + n 2 ⎥⎦

⎡ − ae − aπ cos nπ ne − aπ sin nπ ae aπ cos nπ ne aπ sin nπ ⎤ ⎫⎪


⎢ + − + ⎥⎬ [Q sin nπ = 0]
⎢⎣ a + n a2 + n2 a2 + n2 a 2 + n 2 ⎥⎦ ⎪⎭
2 2

1 ⎧⎪ ae aπ cos nπ ae − aπ cos nπ ae − aπ cos nπ ae aπ cos nπ ⎫⎪


= ⎨ 2 − − + ⎬ [Q sin nπ = 0]
2π ⎪⎩ a + n 2 a2 + n2 a2 + n2 a 2 + n 2 ⎪⎭

1 ⎧⎪ 2ae aπ cos nπ 2ae − aπ cos nπ ⎫⎪


= ⎨ − ⎬
2π ⎪⎩ a 2 + n 2 a 2 + n 2 ⎪⎭

1 ⎧⎪ 2ae aπ cos nπ − 2ae −aπ cos nπ ⎫⎪ 2 a cos nπ ⎧⎪ e aπ − e − aπ ⎫⎪


= ⎨ ⎬= ⎨ ⎬
2π ⎪⎩ a +n
2 2
⎪⎭ π( a 2 + n 2 ) ⎪⎩ 2 ⎪⎭

2 a cos n π ⎛ aπ − aπ ⎞
= × sinh aπ ⎜Q sinh aπ = e − e ⎟
π(a 2 + n 2 ) ⎜ 2 ⎟
⎝ ⎠
2a sinh aπ cos nπ
=
π (a 2 + n 2 )

2a sinh aπ cos nπ
∴ an =
π(a 2 + n 2 )
π π
1 1
bn =
π ∫ f ( x ) sin nx dx =
π ∫ cosh ax sin nx dx
−π
−π

π ⎛ − ax ⎞
⎜Q cosh ax = e + e
ax
1 ⎛ e ax + e −ax ⎞ ⎟
=
π ∫ ⎜

− π⎝
2
⎟ sin nx dx



⎝ 2 ⎟

1 ⎡⎢ ax ⎤
π π
⎡ e ax ⎤
∫ ∫
e − ax sin nx dx ⎥ ∫ = (a sin nx − n cos nx)⎥
ax
= e sin nx dx + ⎢ Q e sin nxdx
2π ⎢ ⎥ ⎣⎢ a +b
2 2
⎦⎥
⎣− π −π ⎦

Look for the SIA GROU P LOGO on the TITLE COVER before you buy
UNIT-2 (Fourier Series) 2.15

⎧⎡ e ax ⎤
π
⎡ e −ax ⎤ ⎫⎪
π
1 ⎪
= ⎨⎢ 2 ( a sin nx − n cos nx ) ⎥ + ⎢ 2 ( − a sin nx − n cos nx ) ⎥ ⎬
⎪⎩⎣⎢ a + b ⎦⎥ −π ⎣⎢ a + b
2 2
2π ⎦⎥ − π ⎪⎭

1 ⎧⎪⎡ e aπ e − aπ ⎤
= ⎨⎢ 2 ( a sin n π − n cos n π ) − ( a sin n ( − π) − n cos n( − π) ⎥ +
2π ⎪⎩⎣⎢ a + b
2
a +b
2 2
⎦⎥
⎡ e − aπ e aπ ⎤ ⎫⎪
⎢ 2 ( − a sin nπ − n cos nπ) − 2 [ − a sin n ( − π) − n cos n ( − π)]⎥ ⎬
⎣⎢ a + b a +b
2 2
⎦⎥ ⎪⎭

1 ⎧⎪⎡ e aπ e − aπ ⎤ ⎫⎪
= ⎨⎢ 2 ( a sin π − n cos n π − a sin n π + n cos n π ) + ( a sin n π + n cos n π − a sin n π − n cos n π) ⎥⎬
2π ⎪⎩⎣⎢ a + b
2 a 2 + b2 ⎦⎥ ⎪⎭
1
= {0} = 0

∴ bn = 0
On substituting a0, an and bn values in equation (1), we get,
2 ∞

2a sinh aπ cos nπ
f(x) =
aπ sinh aπ
2
+ ∑
π(a 2 + n 2 )
cosnx +
n =1
( 0) ∑
n =1


sinh aπ 2a cos(nπ ) sinh aπ cos nx
∴ f ( x) =

+ ∑
n =1
π (a 2 + n 2 )

⎧⎪ 0 for −π≤x≤0
Q15. Find the Fourier series to represent the function f(x) given, f(x) = ⎨ 2 .
⎪⎩ x for 0≤x≤π
Ans: Given equation is,
⎧⎪ 0 for − π ≤ x ≤ 0
f ( x) = ⎨ 2
⎪⎩ x for 0 ≤ x ≤ π
The Fourier series representation of f (x) in the interval [–π, π] is given by,
∞ ∞

∑a ∑b
a0
f (x) = + n cos nx + n sin nx ... (1)
2 n =1 n =1

1⎡ ⎤
π 0 π
1
Where, a0 =
π ∫ f ( x) dx =
π⎢ ∫ ∫
⎢ f ( x ) dx + f ( x ) dx ⎥

−π ⎣−π 0 ⎦

1⎡
0 π ⎤ 1 ⎡ ⎛ x3 ⎞π ⎤ 1 3 π3
∫ ∫
= ⎢ (0) dx + x dx ⎥ = ⎢0 + ⎜⎜ ⎟⎟ ⎥ =
2
[ π − 0] =
π⎢ ⎥ π ⎢ ⎝ 3 ⎠ 0 ⎥ 3π 3π
⎣− π 0 ⎦ ⎣ ⎦

π2
∴ a0 =
3
Similarly,
π
1
an =
π
−π
∫ f ( x) cos nx dx
1⎡ ⎤
0 π

∫ ∫
= ⎢ f ( x ) cos nx dx + f ( x ) cos nx dx ⎥
π⎢ ⎥
⎣−π 0 ⎦

SPECTRUM ALL-IN-ONE JOURNAL FOR ENGINEERING STUDENTS SIA GROUP


2.16 MATHEMATICS-II [JNTU-ANANTAPUR]

1⎡ ⎤
0 π

= ⎢
π⎢
( 0) ∫
cos nx dx + ∫
x 2 cos nx dx ⎥

⎣− π 0 ⎦

∫( ∫ )
π
1 ⎡ 0 + x 2 cosnx dx − 2 x . cosnx dx dx ⎤
=
π ⎢⎣ ∫ ⎥⎦ 0

π
1 ⎡ 2 sin nx sin nx ⎤
= ⎢x
π⎣ n
−2 x
n ⎥⎦ 0 ∫
π
1 ⎡ x 2 sin nx 2 ⎤
= ⎢
π ⎣⎢ n

n
x ⋅ sin nx dx ⎥ ∫
⎥⎦ 0

[∫ ]
π
1 ⎡ x 2 sin nx 2 ⎤ ⎡ ⎡d ⎤ ⎤
= ⎢
π ⎣⎢ n
− x sin nx dx − (1 sin nx dx) dx ⎥
n ⎦⎥ 0
∫ ∫ ⎣
∫ ∫ ∫
⎣ dx ∫
⎢Q uvdx = u vdx − ⎢ u vdx ⎥ dx ⎥
⎦ ⎦

π
1 ⎧⎪ x 2 sin nx 2 ⎡ (− cos nx ) cos nx ⎤ ⎫⎪
= ⎨
π ⎪⎩ n
− ⎢x
n⎣ n
+
n
dx ⎥ ⎬
⎦ ⎪⎭0 ∫
π
1 ⎧⎪ x 2 sin nx 2 ⎡ − x cos nx sin nx ⎤ ⎫⎪
= ⎨ − ⎢ + 2 ⎥⎬
π ⎪⎩ n n⎣ n n ⎦ ⎪⎭ 0

π
1 ⎧⎪ x 2 sin nx 2 x cos nx 2 sin nx ⎫⎪
= ⎨ + − ⎬
π ⎪⎩ n n2 n 3 ⎪⎭ 0

1 ⎡ ⎧⎪ π 2 sin n π 2 πcos n π 2sin n π ⎫⎪ ⎤


=
π
⎢⎨ + − ⎬ − { 0} ⎥
⎢⎣ ⎩⎪ n n2 n 3 ⎭⎪ ⎥⎦

π sin nπ 2 cos nπ 2 sin nπ


= + −
n n2 πn 3
2
= 0+ ( −1) n − 0 [Q sin nπ = 0 and cos nπ = (–1)n]
n2
2
= 2
( −1) n
n

2
∴ an = 2
( −1) n
n
Similarly,
π
1
bn =
π
−π
∫ f ( x) sin nx dx

1⎡ ⎤
0 π

= ⎢
π⎢
f ( ∫
x ) sin nx dx + f ( x ) sin ∫
nx dx ⎥

⎣ − π 0 ⎦

1⎡ ⎤
0 π

= ⎢
π⎢ ∫
(0) sin nx dx + x 2 sin nx dx ⎥
⎥ ∫
⎣− π 0 ⎦

Look for the SIA GROU P LOGO on the TITLE COVER before you buy
UNIT-2 (Fourier Series) 2.17

1⎡ ⎤
π
1⎡ 2 ⎤
π


= ⎢0 + x sin nx dx ⎥ = ⎢ x sin nx dx ⎥ ∫
2
π⎢ ⎥ π⎢ ⎥
⎣ 0 ⎦ ⎣0 ⎦

=
π
[
1 2
∫ ∫ ∫
x ⋅ sin nx dx − ( 2 x sin nx dx ) dx
π

0
]
π
1⎡ 2 (− cos nx) ⎤
= ⎢
π⎣ ∫
x ⋅ sin nx dx − 2 x
n ∫dx ⎥
⎦0

π
1 ⎡ 2 (− cos nx) 2 ⎤
= ⎢x
π⎣ n
+
n
x ⋅ cos nx dx ⎥
⎦0∫
π
1 ⎪⎧ x 2 cos nx 2 ⎡ ⎫⎪
= π ⎨− + x ⋅ cos nx dx − (1 ⋅ cos nx dx ) dx ⎤ ⎬
∫ ∫ ∫
⎪⎩ n n ⎢⎣ ⎥⎦ ⎪
⎭ 0

π
1 ⎪⎧ − x 2 cos nx 2 ⎡ sin nx sin nx ⎤ ⎪⎫
= ⎨
π ⎩⎪ n
+ ⎢x ⋅
n⎣ n

n
dx ⎥ ⎬
⎦ ⎭⎪ 0 ∫
π
1 ⎧⎪ − x 2 cos nx 2 ⎡ x sin nx (− cos nx) ⎤ ⎫⎪
= ⎨ + ⎢ − ⎥ ⎬⎪
π ⎪⎩ n n⎣ n n2 ⎦⎭ 0

π
1 ⎧⎪ x 2 cos nx 2 ⎡ x sin nx cos nx ⎤ ⎫⎪
= ⎨− + ⎢ + ⎥⎬
π ⎪⎩ n n⎣ n n 2 ⎦ ⎪⎭ 0

π
1 ⎧⎪ x 2 cos nx 2 x sin nx 2 cos nx ⎫⎪
= ⎨− + + ⎬
π ⎪⎩ n n2 n 3 ⎪⎭ 0

1 ⎧⎪ − π 2 cos nπ 2π sin nπ 2 cos nπ 2 cos 0 ⎫⎪


= ⎨ + + − ⎬
π ⎪⎩ n n2 n3 n3 ⎪⎭

⎧⎪ − π cos nπ 2 sin nπ 2 cos nπ 2 ⎫⎪


=⎨ + + − 3⎬
⎪⎩ n n 2
πn 3
πn ⎪⎭

− π(−1) n 2 2
= + 0 + 3 (−1) n − 3
n πn πn

−π 2
∴ bn = ( −1) n + 3 [(−1) n − 1]
n πn

On substituting a0, an and bn in equation (1), we get,

∞ ∞
π2 ( −1)n ⎡ −π ⎤
∑ ∑ ⎢⎣ n
2
∴ f ( x) = +2 cos nx + ( −1) n + [( −1) n − 1⎥ sin nx
6 n =1
n2 n =1
πn 3 ⎦

SPECTRUM ALL-IN-ONE JOURNAL FOR ENGINEERING STUDENTS SIA GROUP


2.18 MATHEMATICS-II [JNTU-ANANTAPUR]

⎧−1
⎪ ( π + x) for − π ≤ x ≤ 0
π , π ] for the function f(x) = ⎨ 12
Q16. Find the Fourier series in [–π . Model Paper-I, Q4
⎪ (π − x) for 0 ≤ x ≤ π
⎩2

Ans: Given equation is,

⎧ –1
⎪⎪ 2 ( π + x ) for – π ≤ x ≤ 0
f(x) = ⎨
⎪ 1 ( π – x) for 0 ≤ x ≤ π
⎪⎩ 2

The Fourier series expansion of f (x) in the interval [–π, π] is given by,
∞ ∞

∑ ∑b
a0
f (x) = + an cos nx + n sin nx ... (1)
2 n =1 n =1

Where,
π
1
a0 =
π ∫ f ( x) dx
−π

1⎡
0 π ⎤ 1 ⎡ 0 −1 π
1 ⎤
=
π⎢ ∫ ∫
⎢ f ( x ) dx + f ( x ) dx ⎥ = ⎢
⎥ π ⎢−π 2
(π + x ) dx + ∫2
(π − x) dx ⎥
⎥ ∫
⎣−π 0 ⎦ ⎣ 0 ⎦

1 ⎡ −1
0
1
π ⎤ 1 ⎡ −1 ⎛ x 2 ⎞
0
1 ⎛ x 2 ⎞ ⎤
π

= ⎢
π⎢ 2 ∫
(π + x ) dx +
2

⎥∫
(π − x) dx =
π
⎢ ⎜ πx + ⎟ + ⎜ πx − ⎟ ⎥
⎢ 2 ⎜⎝ 2 ⎟⎠ −π 2 ⎜⎝ 2 ⎟⎠ 0 ⎥
⎣ −π 0 ⎦ ⎣ ⎦

1 ⎡ −1 ⎡ ⎛ ( −π ) 2 ⎞ ⎤ 1 ⎡⎛ 2 π 2 ⎞ ⎤⎤
⎢ ⎢0 − ⎜ ( −π ) + ⎥+ ⎢ ⎜ π − ⎟ − 0⎥ ⎥
2
=
π ⎢⎣ 2 ⎣ ⎝ 2 ⎟⎠ ⎦ 2 ⎣ ⎝ 2 ⎠ ⎦ ⎥⎦

1 ⎡ − 1 ⎛⎜ 2 π 2 ⎞⎟ 1 ⎪⎧ π 2 ⎫⎪⎤ 1 ⎡ − 1 ⎡ 2π 2 − π 2 ⎤ π 2 ⎤
= ⎢ ⎜π − + ⎨ ⎬⎥ = ⎢ ⎢ ⎥+ ⎥
π ⎢⎣ 2 ⎝ 2 ⎟⎠ 2 ⎪⎩ 2 ⎪⎭⎥⎦ π ⎢⎣ 2 ⎢⎣ 2 ⎥⎦ 4 ⎥⎦

1 ⎡ − π 2 ⎛⎜ π 2 ⎞⎟ ⎤
⎢ +
=
π ⎣⎢ 4 ⎜ 4 ⎟ ⎥⎥
⎝ ⎠⎦

∴ a0 = 0

Similarly,
π
1
an =
π ∫ f ( x) cos nx dx
−π

1⎡ ⎤ 1⎡0 1 ⎤
0 π π
1
= ⎢
π⎢
f (∫x ) ⋅ cos nx dx + f ( x ) ∫
⋅ cos nx dx ⎥ = ⎢ −
⎥ π ⎢− π 2
( π + x ) ∫
cos nx dx +
2 ∫
(π − x) cos nx dx ⎥

⎣− π 0 ⎦ ⎣ 0 ⎦

1 ⎡ −1 ⎤
π ⎡
0
⎡d ⎤ ⎤
= ⎢
π⎢ 2 ∫
(π + x) cos nx dx +
1
2 ∫
(π − x ) cos nx dx ⎥
⎥ ⎣
∫ ∫ ∫⎣ dx ∫
⎢Q uvdx = u vdx − ⎢ u vdx ⎥ dx ⎥
⎦ ⎦
⎣ −π 0 ⎦

Look for the SIA GROU P LOGO on the TITLE COVER before you buy
UNIT-2 (Fourier Series) 2.19

=
1 ⎧ −1

π ⎩2
[ ∫ ∫ ∫
0

−π
1
2
] [ ∫
π⎫
( π + x ) cos nx − (1 cos nx dx ) dx + ( π − x) cos nx − (−1 cos nx dx ) dx ⎬
0⎭ ∫ ∫ ]
π
1 ⎧⎪ − 1 ⎡ sin nx sin nx ⎤
0
1⎡ sin nx sin nx ⎤ ⎫⎪
=
π ⎨ ⎢
⎪⎩ 2 ⎣
( π + x ).
n
− ∫ +
n ⎥⎦ −π 2 ⎢⎣
( π − x )
n
+
n
dx ⎥ ⎬ ∫
⎦ 0 ⎪⎭

⎧⎪ − 1 ⎡ π
sin nx ( − cos nx ) ⎤ ⎫⎪
0
1 sin nx ( − cos nx ) ⎤ 1⎡
= ⎨ ⎢( π + x). − ⎥ + ⎢ ( π − x ). + ⎥ ⎬
π ⎪⎩ 2 ⎣ n n2 ⎦ −π 2 ⎣ n n2 ⎦ 0 ⎪⎭

π
1 ⎧⎪ − 1 ⎡ ( π + x ) sin nx cos nx ⎤ 0 1 ⎡ ( π − x) sin nx cos nx ⎤ ⎫⎪
= ⎨ ⎢ + + − ⎬
π ⎪⎩ 2 ⎣ n n 2 ⎥⎦ − π 2 ⎢⎣ n n 2 ⎥⎦ 0 ⎪⎭

π
1 ⎪⎧ − 1 ⎡ n( π + x) sin nx + cos nx ⎤ 0 1 ⎡ n( π − x ) sin nx − cos nx ⎤ ⎪⎫
= ⎨ ⎢ ⎥ + ⎢ ⎥ ⎬
π ⎪2 ⎣ n2 ⎦ −π 2 ⎣ n2 ⎦ 0 ⎪⎭

1 ⎧ −1 1 π⎫
⎨ 2 [n( π + x ).sin nx + cos nx ]−π + 2 [n(π − x ) sin nx − cos nx ]0 ⎬
0
=
π ⎩ 2n 2n ⎭

1 ⎧ −1
= ⎨
π ⎩ 2n2
{[n(π − 0)sin n(0) + cos n(0) ] − [( n(π − π) sin n( −π) + cos n( −π)]}

+
1
2n 2 {[n(π − π) sin nπ – cos nπ] − [(n(π − 0)sin n(0) – cos n(0)]}⎫⎬⎭

1 1
= 2 {–[1 – cos(– nπ)] + [– cos nπ – (–1)]} = {–1 + cos nπ – cosnπ + 1} = 0
2n π 2n 2 π

∴ an = 0

Similarly,
π
1
bn =
π ∫ f ( x).sin nx.dx
−π

π
1 ⎡0 ⎤ 1 ⎡ 0 −1 π
1 ⎤
=
π ⎢
⎣− π
∫ 0

⎢ f ( x ).sin nx.dx + f ( x ) sin nx.dx ⎥ = ⎢∫
⎥⎦ π ⎣ −π 2
(π + x ).sin nx.dx + ∫
0
2
(π − x) sin nx.dx ⎥

1 ⎧ −1 ⎡ π + x sinnx dx − 1 sinnx dx dx⎤ 0 + 1 ⎡ π − x sinnx dx − −1 sinnx dx dx⎤ π ⎫


= ⎨
π ⎩ 2 ⎣⎢
( ). ∫ ( ∫ ∫
) ⎥
⎦ −π 2 ⎣⎢
( ). ( ∫
) ⎥ ⎬
⎦0 ⎭ ∫ ∫
⎧⎪ − 1 ⎡ π
( − cos nx) ⎤ ⎫⎪
0
1 ( − cos nx) ( − cos nx) ⎤ 1 ⎡ − ( π − x) cos nx
=
π
⎨ ⎢( π + x )
⎪⎩ 2 ⎣ n
− ∫ n ⎥ +
⎦ −π 2 ⎣
⎢ n
+
n ∫
dx ⎥ ⎬
⎦ 0 ⎪⎭

⎧⎪ − 1 ⎡ − (π + x) cos nx sin nx ⎤ 0 π
1 1 ⎡ − ( π − x ) cos nx sin nx ⎤ ⎫⎪
= ⎨ ⎢ + ⎥ + ⎢ − ⎥ ⎬
π ⎪⎩ 2 ⎣ n n 2 ⎦ −π 2 ⎣ n n 2 ⎦ 0 ⎪⎭

SPECTRUM ALL-IN-ONE JOURNAL FOR ENGINEERING STUDENTS SIA GROUP


2.20 MATHEMATICS-II [JNTU-ANANTAPUR]

⎧⎪ − 1 ⎡ − n( π + x) cos nx + sin nx ⎤ 0 π
1 1 ⎡ − n ( π − x ) cos nx − sin nx ⎤ ⎫⎪
= ⎨ ⎢ ⎥ + ⎢ ⎥ ⎬
π ⎪⎩ 2 ⎣ n2 ⎦ −π 2 ⎣ n2 ⎦ 0 ⎪⎭

1 ⎧ −1 1 ⎫
= ⎨ [ − n( π + x) cos nx + sin nx ]0−π + 2 [ − n ( π − x) cos nx − sin nx ]0π ⎬
π ⎩ 2n 2 2n ⎭
1 ⎧ –1
= ⎨
π ⎩ 2n 2
{[ –n(π + 0) cos n(0) + sin n(0)] − [ – n(π − π)cos n(–π) + sin n(–π)]}

2 {[
– n(π − π) cos nπ – sin nπ ] − [ – n(π − 0) cos n(0) – sin n(0)]}⎬
1
+
2n ⎭

1 ⎧ −1 1 ⎫ 1 1
= ⎨ 2 [ − nπ + sin nπ] + 2 [ − sin nπ + nπ]⎬ = [nπ – sin nπ – sin nπ + nπ] = (2nπ – 2sin nπ)
π ⎩ 2n ⎭ 2n π 2n 2 π
2
2n

2 nπ 1
= 2
= [Q sin nπ = 0]
2n π n

1
∴ bn =
n
On substituting the values of a0, an and bn in equation (1), we get,

f (x) = 0 + ∑[(0)cos nx + b sin nx]
n =1
n

∞ ∞
⎛1⎞
f(x) = ∑
n =1
bn sin nx = ∑ ⎜⎝ n ⎟⎠ sin nx
n =1

⎡ 1 1 ⎤
∴ f ( x ) = ⎢sin x + sin2 x + sin3 x + ...⎥
⎣ 2 3 ⎦

2.2 FOURIER SERIES EVEN, AND ODD FUNCTIONS


Q17. Give the Fourier series expansion for even periodic functions.
Ans: Generally, a periodic function f(x) defined in the interval (–π, π) is represented by,
∞ ∞
a0
f(x) =
2
+ ∑ n =1
an cos nx + ∑b
n =1
n sin nx ... (1)

Where,
π
1
a0 =
π ∫ f ( x )dx
−π
... (2)

π
1
an =
π
−π
∫ f ( x) cos nx dx ... (3)

π
1
bn =
π
−π
∫ f ( x) sin nx dx ... (4)

When f(x) is an Even Function: From equation (2), we get,


π
1
a0 =
π
−π
∫ f ( x) dx
Look for the SIA GROU P LOGO on the TITLE COVER before you buy
UNIT-2 (Fourier Series) 2.21
π When f (x) is an Odd Function: From equation (2), we get,
2
⇒ a0 =
π ∫ f ( x) dx 1
π
0

As ‘cos nx’ is an even function, f(x) cos nx is also an


a0 =
π ∫ f ( x)dx = 0
−π
(Q f(x) is odd)

even function.
As ‘cos nx’ is an even function, f(x) cos nx becomes an
∴ From equation (3), we get, odd function
π
1 ∴ From equation (3), we get,
an =
π ∫ f ( x) cos nx dx
−π π
1
2
π
an =
π ∫ f ( x) cos nx dx = 0
−π
⇒ an =
π ∫ f ( x) cos nx dx
0
Sin nx is an odd function, f (x) sin nx becomes an even
function.
Since, sin nx is an odd function and f(x) sin nx also
becomes an odd function ∴ From equation (4), we get,
∴ From equation (4), we get, π
1
1
π bn =
π ∫ f ( x) sin nx dx
bn = ∫ f (x)sin nx dx = 0 −π
π −π
π
Therefore, it can be concluded that if a function f(x) is 2
even in the interval (–π, π), its Fourier series expansion consists ⇒ bn =
π ∫ f ( x) sin nx dx
0
of only cosine terms.
Therefore, it can be concluded that if a function f(x) is

a odd in the interval (–π, π), its Fourier series expansion consists
∴ f ( x) = 0 +
2 ∑a
n =1
n cos nx of only sine terms.

Where,
π
∴ f ( x) = ∑b
n =1
n sin nx
2
a0 =
π ∫ f ( x)dx π
0 2
π
Where, bn =
π ∫ f ( x) sin nx dx
0
2
an =
π ∫
0
f ( x ) cos nx . dx
Q19. Obtain the Fourier series for the function
Q18. Give the Fourier series expansion for odd π , π ).
f(x) = |cos x| in (–π Model Paper-II, Q4

periodic functions.
Ans: Given equation is,
Ans: Generally, a periodic function f(x) defined in the interval
(–π, π) is represented by, f(x)= | cos x |
∞ ∞

∑a ∑ b sin nx
a0 Since, f(x) = | cos x | is an even function, its Fourier
f(x) = + n cos nx + n
2 n=1 n =1
series consists of cosine terms only
... (1)

Where, a0
π
∴ f(x) =
2
+ ∑a
n =1
n cos nx ... (1)
1
a0 =
π ∫ f ( x )dx
−π
... (2)
Where,
π
1
an = ∫ f ( x) cos nx dx ... (3) 1
π
2
π
π
−π a0 =
π ∫ f ( x ) dx =
π ∫ f ( x)dx
−π 0
π π π
1 ⎡ ⎤
bn =
π ∫ f ( x) sin nx dx
−π
... (4)

⎣ −π

0

⎢Q f ( x)is even function i.e., f ( x ) dx = 2 f ( x) dx ⎥

SPECTRUM ALL-IN-ONE JOURNAL FOR ENGINEERING STUDENTS SIA GROUP


2.22 MATHEMATICS-II [JNTU-ANANTAPUR]

2⎡
π ⎤ 2 ⎡π 2 π ⎤
= ⎢
π⎢
| ∫
cos x | dx⎥

= ⎢
π⎢ ∫
cos x dx – cos x .∫
dx ⎥

⎣0 ⎦ ⎣0 π
2 ⎦

=
2
π
[ π
]
2
(sin x ) 02 – (sin x ) ππ = [1 + 1] =
2
π
4
π

4
∴ a0 =
π
π
1
an =
π ∫ f ( x) cos nx dx
−π

π ⎛ π π ⎞
2 ⎜Q f ( x )dx = 2 f ( x)dx ⎟
=
π ∫
| cos x | cos nx dx
⎜ ∫ ∫ ⎟
0 ⎝ −π 0 ⎠

⎡π 2 π ⎤
2⎢
=
π⎢
cos x∫cos nx dx + ( − cos x ∫
) cos nx dx ⎥

⎣0 π2 ⎦

⎡ ⎛ π⎞ ⎛ π ⎞⎤
⎢ Q cos x is positive in the interval ⎜ 0, ⎟ and negative in the interval ⎜⎝ 2 , π⎟⎠ ⎥
⎣ ⎝ 2⎠ ⎦

⎡π 2 π ⎤
1⎢
= π⎢ 2 cos∫nx cos x dx − 2 cos nx cos∫x dx ⎥

⎣0 π2 ⎦

⎡π 2 π ⎤
1⎢
= {cos( nx + x ) + cos( nx − x)}dx − {cos( nx + x ) + cos( nx − x)}dx⎥
∫ ∫
π⎢ ⎥
⎣0 π2 ⎦
[Q 2 cos A cos B = cos (A + B) + cos (A – B)]

⎡π 2 π ⎤
1⎢
= {cos( n + 1) x + cos( n − 1) x}dx − {cos( n + 1) x + cos( n − 1) x}dx⎥
∫ ∫
π⎢ ⎥
⎣0 π2 ⎦

⎡ π2 ⎫ ⎧π ⎫⎤
1 ⎢⎧⎪
π2 π
⎪ ⎪ ⎪
= ⎢⎨ cos( n + 1) x dx + cos( n − 1) x dx ⎬ − ⎨ cos( n + 1) x dx + cos( n − 1) xdx ⎬⎥⎥
∫ ∫ ∫ ∫
π ⎪
⎢⎣⎩ 0 0 ⎭⎪ ⎪⎩π 2 π2 ⎪⎭⎥⎦

1 ⎡⎧ sin(n + 1) x sin(n − 1) x ⎫ ⎧ sin(n + 1) x sin(n − 1) x ⎫ ⎤


π2 π
= ⎢⎨ + ⎬ −⎨ + ⎬ ⎥
π ⎢⎩ n + 1 n − 1 ⎭0 ⎩ n +1 n − 1 ⎭ π 2 ⎥⎦

⎡ ⎧⎛ π π⎞ ⎫
sin(n + 1) sin( n − 1)
1 ⎢ ⎪⎪⎜ 2 + 2⎟ −⎛ sin( n + 1).(0) sin( n − 1)(0) ⎪
⎞⎪
= ⎢ ⎨⎜ ⎜ + ⎟⎬
π ⎢⎪ n +1 n −1 ⎟ ⎝ n +1 n −1 ⎠ ⎪
⎜ ⎟
⎢⎣ ⎩⎪⎝ ⎠ ⎭⎪

⎧ ⎛ π π ⎞ ⎫⎤
⎪⎪⎛ sin(n + 1)π sin(n − 1)π ⎞ ⎜ sin(n + 1) 2 sin(n − 1) 2 ⎟⎪⎪⎥
− ⎨⎜ + ⎟ −⎜ + ⎟ ⎬⎥
⎪⎝ n + 1 n −1 ⎠ ⎜

n +1 n − 1 ⎟ ⎪⎥
⎟ ⎪⎥
⎪⎩ ⎝ ⎠ ⎭⎦

Look for the SIA GROU P LOGO on the TITLE COVER before you buy
UNIT-2 (Fourier Series) 2.23

⎡ ⎧⎛ π π⎞ ⎫ ⎧ ⎛ π π ⎞ ⎫⎤
sin(n + 1). sin( n − 1) sin(n + 1) sin(n − 1)
1 ⎢ ⎪⎪⎜ 2 2 ⎟ ⎪⎪ ⎪⎪ ⎜ 2

2 ⎟ ⎪⎪ ⎥
⎢ ⎨⎜ + ⎟ − (0 + 0)⎬ − ⎨(0 + 0) − ⎜ + ⎟ ⎬
= π ⎢⎪ n +1 n −1 n +1 n −1
⎟ ⎪⎥
⎪ ⎪ [Q sinnπ = 0]
⎜ ⎟ ⎜
⎢⎣ ⎪⎩⎝ ⎠ ⎪⎭ ⎪⎩ ⎝ ⎠ ⎪⎭ ⎦⎥

⎡ π π π π⎤
sin( n + 1) sin( n − 1) sin( n + 1) sin( n − 1) ⎥
1⎢ 2 + 2 + 2 + 2
= ⎢ ⎥ where n ≠ 1.
π⎢ n +1 n −1 n +1 n −1 ⎥
⎣⎢ ⎦⎥

1⎡ 1 ⎧ π π⎫ 1 ⎧ π π ⎫⎤
= ⎢ ⎨sin(n + 1) + sin(n + 1) ⎬ + ⎨sin( n − 1) + sin(n − 1) ⎬ ⎥
π ⎣n +1 ⎩ 2 2 ⎭ n –1 ⎩ 2 2 ⎭⎦

1⎡ 1 ⎧ π⎫ 1 ⎧ π ⎫⎤
= ⎢ ⎨2sin(n + 1) ⎬ + ⎨2sin(n − 1) ⎬⎥
π ⎣n +1 ⎩ 2 ⎭ n −1 ⎩ 2 ⎭⎦

2 ⎡ 1 ⎧ ⎛ nπ π ⎞ ⎫ 1 ⎧ ⎛ nπ π ⎞ ⎫⎤
= ⎢ ⎨sin ⎜ + ⎟ ⎬ + ⎨sin ⎜ − ⎟ ⎬⎥ [Q sin(A + B) = sinA cosB + cosA sinB]
π ⎣ n + 1 ⎩ ⎝ 2 2 ⎠ ⎭ n − 1 ⎩ ⎝ 2 2 ⎠ ⎭⎦

2 ⎡ 1 ⎧ nπ π nπ π ⎫ 1 ⎧ nπ π nπ π ⎫⎤
= ⎢ ⎨sin cos + cos sin ⎬ + ⎨sin cos − cos sin ⎬⎥
π ⎣ n +1 ⎩ 2 2 2 2 ⎭ n −1 ⎩ 2 2 2 2 ⎭⎦

2 ⎡ 1 ⎧sin nπ ⎛ nπ ⎞ ⎫ 1 ⎧ ⎛ nπ ⎞ ⎛ nπ ⎞ ⎫⎤
⎢ ⎨ (0) + cos ⎜ ⎟ (1)⎬ + ⎨sin (0) − cos ⎜ ⎟ (1)⎬⎥
⎝ 2 ⎠ ⎭ n −1 ⎩ ⎜⎝ 2 ⎟⎠
=
π ⎣ n +1 ⎩ 2 ⎝ 2 ⎠ ⎭⎦

2⎡ 1 ⎛ nπ ⎞ 1 ⎧ ⎛ nπ ⎞ ⎫⎤
cos ⎜ ⎟ + ⎨− cos ⎜⎝ ⎟⎠ ⎬⎥
π ⎣⎢ n +1 ⎝ 2 ⎠ n −1 ⎩
=
2 ⎭⎦

⎡ nπ nπ ⎤
cos cos ⎥
2⎢ 2 − 2
= ⎢ ⎥
π ⎢ n +1 n −1 ⎥
⎢⎣ ⎥⎦

2⎡ 1 1 ⎤ nπ 2 ⎡ n − 1 − (n + 1) ⎤ nπ
= ⎢ − ⎥ cos = ⎢ ⎥ cos
π ⎣ n + 1 n −1⎦ 2 π ⎣ (n + 1)(n − 1) ⎦ 2

2 ⎡ n − 1 − n − 1⎤ nπ 2 ⎡ −2 ⎤ nπ
= ⎢ ⎥ cos = ⎢ 2 ⎥ cos
π ⎣ n −1 ⎦
2
2 π ⎣ n − 1⎦ 2

−4 nπ
∴ an = cos if n ≠ 1
π (n − 1)
2
2

⎡π 2 π ⎤
2⎢
∫ ⎥

2 2
a1 = π ⎢ cos x dx – cos dx ⎥ = 0
⎣0 π
2 ⎦

∴ a1 = 0
On substituting the values of a0, an in equation (1), we get,

2 4 1 nπ
∴ f ( x) = −
π π ∑n 2
−1
cos
2
cos nx
n=2

SPECTRUM ALL-IN-ONE JOURNAL FOR ENGINEERING STUDENTS SIA GROUP


2.24 MATHEMATICS-II [JNTU-ANANTAPUR]

2sinaπ ⎡ sinx 2sin2x 3sin3x ⎤


Q20. Show that for – π < x < π , sinax = ⎢ 2 – + – ....⎥ (a is not an integer).
π ⎣1 – a 2 22 – a 2 32 – a 2 ⎦
Ans: Given equation is,
2 sin aπ ⎡ sin x 2 sin 2 x 3 sin 3 x ⎤
sinax = ⎢ 2 2
– 2 2 + 2 2 –....⎥ , for –π < x < π
π ⎣1 –a 2 –a 3 –a ⎦
since, sinax is an odd function, its Fourier series consists of sine terms only.

∴ sinax = ∑b
n =1
n sin nx ... (1)

Where,
π π
2 2
bn =
π ∫
0
f ( x) sin nx dx =
π ∫
sin ax sin nx dx
0

π
1
=
π ∫
[cos( a – n ) x – cos( a + n) x ] dx
0
[Q 2sinA sinB = cos(A – B) – cos(A + B)]

1⎡ ⎤
π π

=
π⎢ ∫ ∫
⎢ [cos( a – n) x – [cos( a + n) x ⎥ dx

⎣0 0 ⎦
π
1 ⎡ sin(a – n) x sin(a + n) x ⎤ 1 ⎡ sin( a – n) π sin( a + n ) π ⎤
= ⎢ – – – 0⎥
a + n ⎥⎦ 0 π ⎢⎣ a – n
=
π⎣ a–n a+n ⎦

1 ⎡ sin aπ cos nπ – cos aπ sin nπ sin aπ cos nπ + cos aπ sin nπ ⎤


= –
π ⎢⎣ a–n a+n ⎥

[Q sin(A + B) = sinA cosB + cosA sinB ; sin(A – B) = sinA cosB – cosA sinB]

1 ⎡ sin aπ cos nπ sin aπ cos nπ ⎤



π ⎢⎣ ⎥
= [Q sinnπ = 0]
a–n a+n ⎦

1 ⎛ 1 1 ⎞ 1 n⎛a+n–a+n⎞
= sin aπ cos nπ ⎜ – ⎟ = sin aπ (–1) ⎜ ⎟ [Q cosnπ = (–1)n]
π ⎝a–n a+n⎠ π ⎝ a2 – n2 ⎠

(–1) n 2n
= sin aπ
π(a 2 – n 2 )

(–1) n 2n
∴ bn = sin aπ
π(a 2 – n 2 )
On substituting the value of ‘bn’ in equation (1), we get,

2 sin aπ n(–1) n
sinax =
π
∑ 2
n =1 a – n
2
sin nπ

2 sin aπ ⎡ (–1) n +1 .n ⎤ 2 sin aπ ⎛ sin x





2 sin 2 x 3 sin 3 x
= ⎢ sin nπ⎥ = ⎜ 2 – 2 + 2 – ... ⎟
π ⎣⎢ n =1 n – a
2 2
⎦⎥ π ⎝1 – a 2
2 –a 2
3 –a 2

2 sin aπ ⎛ sin x 2 sin 2 x 3 sin 3x ⎞


∴ sin ax = ⎜ 2 – 2 + 2 – ...⎟
π ⎝1 – a 2
2 –a 2
3 –a 2

Hence proved.

Look for the SIA GROU P LOGO on the TITLE COVER before you buy
UNIT-2 (Fourier Series) 2.25
Q21. Find the Fourier series to represent the function
−1 ⎡ ⎧ 1 1 ⎫ ⎧ 1 1 ⎫⎤
π < x < π.
f(x) = | sin x |, –π (−1) n+1 ⎨ + ⎬−⎨ + ⎬
π ⎢⎣ n ⎭⎥⎦
=
⎩ n + 1 1 − n ⎭ ⎩ n + 1 1 −
Ans: Since, f(x) = | sin x| is an even function, its Fourier series
consists of cosine terms only. −1 ⎡ 2 2 ⎤
⎢ ( −1) n +1 −
1 − n 2 ⎥⎦
=
∞ π ⎣ 1− n 2
a0
∴ f (x) =
2
+ ∑a
n =1
n cos nx ... (1)
−2
= 2
[( −1) n ( −1)1 − 1]
− ( n − 1) π
π
1
Where, a0 =
π −π ∫
f ( x) dx
=−
2[1 + (−1) n ]
,n>2
π(n 2 − 1)
π an = 0, when n is odd,
2
=
π ∫ f ( x ) dx [Q f (x) is even function]
4
0 And an = − , when n is even, n > 2
π( n 2 − 1)
⎡ π π ⎤
⎢ −π ∫ ∫
⎢Q f ( x ) dx = 2 f ( x) dx ⎥
⎥ 1
π
⎣ ⎦
0
If n = 1, a1 =
π −π ∫
| sin x | cos x dx

π
2
=
π0 ∫
sin x dx
2
π

=
π0 ∫
sin x cos x dx

=
2
[− cos x] π0 = 2 [−(−1) + 1] = 4 π π
π π π 1 1 cos 2 x
=
π0∫sin 2 x dx = −
π 2 0
π
1
an =
π −π∫f ( x ) cos nx dx
1
= (1 − cos 2 π)= 0

π Hence the Fourier series can be written as,
2
=
π0 ∫
f ( x ) cos nx dx
2 4 ∞
cos 2 nx
⇒ f (x) = −
π π ∑ (2n)
n =1
2
−1
π
2
=
π0 ∫
sin x cos nx dx
2 4 ∞
cos 2n x
∴ f (x) = −
π π ∑ (2n − 1)(2n + 1)
n =1
π
1
=
π ∫ [sin (n + 1) x − sin( n − 1) x]dx
0
Q22. Obtain a Fourier expansion for
the interval − π < x < π .
1− cosx in

π
1 ⎡ − cos(n + 1) x cos(n − 1) x ⎤ Ans: f (x) = 1− cos x in (–π, π)
+
π ⎢⎣ n − 1 ⎥⎦ 0
=
n +1
Since f (x) = 1 − cos( − x ) = 1− cos x

1 ⎡ − cos (n + 1)π cos (n − 1)π 1 1 ⎤ ∴ f (x) is even in (–π, π)


= ⎢ + + −
π⎣ n +1 n −1 n + 1 n − 1 ⎥⎦ ∴ bn = 0 for all n.
Fourier series of f (x) is given by,
− 1 ⎡ ( −1) − 1 (−1) − 1⎤
n +1 n +1
a0 a
+
= ⎢
π ⎣ n +1 1− n ⎦
⎥ f (x) =
2
+ ∑a
n =1
n cos nx

SPECTRUM ALL-IN-ONE JOURNAL FOR ENGINEERING STUDENTS SIA GROUP


2.26 MATHEMATICS-II [JNTU-ANANTAPUR]
Where,

π
2 ⎡ x⎤
π π π
⎡ x⎤
a0 =
1
π ∫ f (x) dx =
1
π ∫ 1− cos x dx =
2
π
∫ 1− cos x dx = ⎢
π ⎢ 0 2⎥ ∫
2 sin ⎥ ⎢Q 1 − cos x = 2 sin 2 ⎥
⎣ ⎦
−π −π 0 ⎣ ⎦
π
−4 2 ⎡ x⎤
cos ⎥
π ⎢⎣
=
2 ⎦0

4 2
a0 =
π

π π
1 2
an =
π ∫ f (x)cos nx dx =
π ∫ 1− cos x cos nx dx
−π 0

π π π
⎡ ⎤
∫ ⎢⎣sin ⎛⎜⎝ n + 2 ⎞⎟⎠ x − sin ⎛⎜⎝ n − 2 ⎞⎟⎠ x⎥⎦ dx
2 x x 2 1 1
an =
π ∫ 2 sin cos nx dx =
2
2 2
π

0
sin cos nx dx =
2 π 0
0

π
2 ⎡ − cos(n + 1 / 2) x + cos(n − 1 / 2) x ⎤
= ⎢ ⎥
π ⎣ ( n + 1 / 2) ( n − 1 / 2) ⎦ 0

2 ⎡ sin nπ + sin nπ + 1 − 1 ⎤ 2 ⎡ n − 1/ 2 − n − 1/ 2 ⎤
= ⎢ n + 1/ 2 n − 1/ 2 n + 1/ 2 n − 1/ 2 ⎥ = ⎢0 + 0 + n 2 − 1/ 4 ⎥
π ⎣ ⎦ π ⎣ ⎦

− 2
an =
π( n 2 − 1 / 4 )
∴ Fourier series is given by,


− 2
f (x) =
2 2
π
+ ∑
n =1 π( n 2 − 1 / 4)
cos nx


1
=
2 2
π

π
2

n =1 ( n − 1 / 4)
2 cos nx

2.3 FOURIER SERIES IN AN ARBITRARY INTERVAL


⎧ πx, 0 ≤ x ≤1
Q23. Obtain the Fourier series for the function f(x) = ⎨ .
⎩π(2 − x), 1 ≤x≤2

Ans: Given that,


f(x) = πx when 0 ≤ x ≤ 1 and
f(x) = π(2 – x) when 1 ≤ x ≤ 2
Fourier series is,
∞ ∞
a0 nπx nπx
f(x) = +
2 n =1 ∑
an cos
l
+ ∑b
n =1
n sin
l
... (1)

∴2l = 2 ⇒ l = 1

Look for the SIA GROU P LOGO on the TITLE COVER before you buy
UNIT-2 (Fourier Series) 2.27
Then equation (1) becomes,
∞ ∞
a0
f(x) =
2
+ ∑a n cos nπx + ∑b
n =1
n sin nπx ... (2)
n =1

Here,
2l 2
1
a0 =
l ∫0
f ( x ) dx = ∫ f ( x) dx
0

1 2

= ∫
0

πx dx + π( 2 − x) dx
1

1 2
⎡ x2 ⎤ ⎡ ( 2 − x) 2 ⎤
= π ⎢ ⎥ + π⎢ ⎥
⎢⎣ 2 ⎥⎦ 0 ⎢⎣ − 2 ⎥⎦1

⎛ 1 0 ⎞ ⎡⎛ (2 − 2) 2 ⎞ ⎛ (2 − 1) 2 ⎞⎤
= π ⎜ − ⎟ + π⎢⎜⎜ ⎟−⎜
⎟ ⎜ − 2 ⎟⎥

⎝ 2 2 ⎠ ⎣⎢⎝ − 2 ⎠ ⎝ ⎠⎦⎥

π ⎛ 1⎞ π π
= + π ⎜0 + ⎟ = + =π
2 ⎝ 2⎠ 2 2
And,
2l
1 n πx
an =
l 0 ∫
f ( x) cos
l
dx

2
nπx
= ∫ f ( x) cos
0
1
dx [Q l = 1]

2
=
∫ f (x) cos nπx dx
0

1 2
= ∫
0

f (x) cos nπx dx + f (x ) cos nπx dx
1
1 2

= ∫ πx cos nπx dx + ∫ π (2 − x). cos nπx dx


0 1

1 2
⎡ ⎛ sin nπx ⎞ ⎛ cos nπx ⎞ ⎤ ⎡ ⎛ sin nπx ⎛ cos nπx ⎞ ⎞⎤
= π ⎢ x⎜ ⎟ − 1⎜ − 2 2 ⎟ ⎥ + π ⎢(2 − x)⎜⎜ − ( −1) ⎜ − 2 2 ⎟ ⎟⎟⎥
⎣ ⎝ nπ ⎠ ⎝ n π ⎠ ⎦ 0 ⎣ ⎝ nπ ⎝ n π ⎠ ⎠ ⎦1
1 2
⎡ x sin nπx cos nπx ⎤ ⎡ (2 − x ) sin nπx cos nπx ⎤
=π⎢ + 2 2 ⎥ +π⎢ − 2 2 ⎥
⎣ nπ n π ⎦0 ⎣ nπ n π ⎦1

⎡⎛ (1) sin nπ(1) cos nπ(1) ⎞ ⎛ (0) sin nπ(0) cos nπ(0) ⎞⎤
= π ⎢⎜ + ⎟−⎜ + ⎟⎥
⎣⎝ nπ n2π2 ⎠ ⎝ nπ n 2 π 2 ⎠⎦

⎡⎛ (2 − 2) sin nπ(2) cos nπ(2) ⎞ ⎛ (2 − 1) sin nπ(1) cos nπ(1) ⎞⎤


+ π ⎢⎜ − ⎟−⎜ − ⎟⎥
⎣⎝ nπ n2 π2 ⎠ ⎝ nπ n 2 π 2 ⎠⎦

⎡⎛ cos nπ ⎞ ⎛ 1 ⎞⎤ ⎡⎛ cos 2nπ ⎞ ⎛ cos nπ ⎞⎤


= π ⎢⎜ 0 + 2 2 ⎟ − ⎜ 0 + 2 2 ⎟⎥ + π ⎢⎜ 0 − 2 2 ⎟ − ⎜ 0 − 2 2 ⎟⎥ [Q cos 2nπ = (–1)2n = 1]
⎣⎝ n π ⎠ ⎝ n π ⎠⎦ ⎣⎝ n π ⎠ ⎝ n π ⎠⎦

SPECTRUM ALL-IN-ONE JOURNAL FOR ENGINEERING STUDENTS SIA GROUP


2.28 MATHEMATICS-II [JNTU-ANANTAPUR]

⎡ cos nπ 1 cos 2nπ cos nπ ⎤ 2× π


= π ⎢ 2 2 − 2 2 − 2 2 + 2 2 ⎥ = 2 2 [cos nπ – 1] [Q cos nπ = (–1)n]
⎣n π n π n π n π ⎦ n π


= [(–1)n – 1]
n 2 π2

⎧0, n is even

∴ a n = ⎨ − 4π
⎪ 2 2 , n is odd
⎩n π

2l 2
1 nπx 1 nπx
bn =
l ∫ f ( x ) sin
l
dx =
1 ∫ f ( x) sin
0
1
dx [Q l = 1]
0

= ∫ f ( x) sin nπx dx
0
1 2
= ∫
0

πx. sin nπx dx + π( 2 − x) sin nπx dx
1

1 2
⎡ ⎛ − cos nπx ⎞ ⎛ − sin nπx ⎞⎤ ⎡ ⎛ − cos nπx ⎞ ⎛ − sin nπx ⎞⎤
= π ⎢ x⎜ ⎟ − 1⎜ ⎟⎥ + π ⎢(2 − x )⎜ ⎟ − (−1)⎜ ⎟⎥
⎣ ⎝ nπ ⎠ ⎝ n π ⎠ ⎦ 0
2 2
⎣ ⎝ nπ ⎠ ⎝ n 2 π 2 ⎠⎦1

1 2
⎡ x cos nπx sin nπx ⎤ ⎡ cos nπx sin nπx ⎤ ⎡ cos nπx cos nπx ⎤
= π ⎢− + 2 2 ⎥ + π⎢(2 − x ) − 2 2 ⎥ = π ⎢− +
⎣ nπ n π ⎦0 ⎣ nπ n π ⎦1 ⎣ nπ nπ ⎥⎦

∴ bn = 0
On substituting the above values in equation (2), we get,

π cos nπx
f(x) =
2

– 2
π

n =1,3,5,7.... n2

π 4π ⎡ 1 1 1 ⎤
= − 2 ⎢ 2 cos πx + 2 cos 3πx + 2 cos 5πx + ...⎥
2 π ⎣1 3 5 ⎦
Q24. Find the Fourier series corresponding to the following f(x) defined in (–2, 2) as follows,

⎧2, − 2 ≤ x ≤ 0
f(x) = ⎨ of f(x) in (–2, 2).
⎩ x, 0 <x<2
Ans: Given function is,
⎧2, − 2 ≤ x ≤ 0
f(x) = ⎨
⎩ x, 0 < x < 2
∞ ∞
a0 nπx n πx
Let, f(x)=
2
+ ∑
n =1
a n cos
l
+ ∑b
n =1
n sin
l
in [–l, l] ... (A)

Here,
l =2
l
1
∴ a0 =
l ∫ f ( x) dx
−l

Look for the SIA GROU P LOGO on the TITLE COVER before you buy
UNIT-2 (Fourier Series) 2.29

1
2
1⎡
0 2 ⎤ 1⎡0 2 ⎤ 1⎡ ⎡ x2 ⎤ ⎤
2

∫ ∫ ∫ ∫ ∫ ⎢[ 2 x] + ⎢ ⎥ ⎥
0
= f ( x ) dx = ⎢ f ( x ) dx + ⎥
f ( x ) dx = ⎢ 2 dx + x dx ⎥ =
2 2⎢ ⎥ 2⎢ ⎥ 2⎢ −2
⎢⎣ 2 ⎥⎦ 0 ⎥
−2 ⎣−2 0 ⎦ ⎣− 2 0 ⎦ ⎣ ⎦
1⎡ 4⎤ 6
= ⎢ 2( 2 ) + ⎥ = = 3 ... (1)
2⎣ 2⎦ 2

∴ a0 = 3
2
1 nπx
∴ an =
2 ∫ f ( x) cos
−2
2
dx

1⎡ nπx ⎤ 1 ⎡ nπx ⎤
0 2 0 2
nπx nπx
=
2⎢ ∫
⎢ f ( x) cos
2
dx + f ( x) cos
2 ∫
dx ⎥ = ⎢2 cos
⎥ 2⎢ 2
dx + x cos
2
dx ⎥
⎥ ∫ ∫
⎣− 2 0 ⎦ ⎣ − 2 0 ⎦
⎡ nπx
0
nπx
2
nπx ⎤ ⎡ nπx
2⎤

1 ⎢⎢ ⎥ 1⎢ 4
2 sin x sin 2 sin 0 − cos ⎥
n πx 2 2
2 2
∫ 2 dx ⎥ = ⎢ sin ⎥
+ ( 2 sin nπ − 0) − 2
= + − nπ
2 ⎢ nπ nπ nπ ⎥ 2 ⎢ nπ 2 −2 nπ nπ ⎥
⎢ 0
⎥ ⎢ 2 ⎥
2 2 2 ⎣ 0⎦
⎣ −2 0 ⎦

⎡ 4
[sin 0 − sin(nπ)]+ 2 (0 − 0)⎤⎥ 1 4
1 ⎢ nπ nπ ⎡ ⎤
= ⎢ ⎥ = ⎢ 2 2 (cos nπ − 1)⎥
2⎢
+
4
(cos nπ − cos 0) ⎥ 2 ⎣n π ⎦
⎢⎣ n 2 π 2 ⎥⎦

2 2
∴ an = (cos nπ − 1) = 2 2 ((−1) n − 1)
n π
2 2
n π
⎧ 0 when n is even

an = ⎨ − 4 ... (2)
when n is odd
⎪⎩ n 2 π 2

2
1 nπx
∴ bn =
2 ∫ f ( x) sin
−2
2
dx

1⎡ nπx ⎤
0 2
nπx
=
2⎢
⎣− 2

⎢ f ( x ) sin
2
dx +
∫ f ( x) sin
0
2
dx ⎥
⎥⎦

1⎡ nπx ⎤
0 2
nπx
= ⎢ 2 sin
2 ⎢−2

2 ∫
dx + x sin
0
2
dx ⎥
⎥⎦ ∫
⎡ ⎛ n πx ⎞
0
⎛ nπx ⎞
2 ⎤
⎢ 2⎜ − cos ⎟ x⎜ − cos ⎟ 2 − cos
nπx ⎥
1⎢ ⎝ 2 ⎠ 2 dx ⎥
+ ⎝
2 ⎠
= ⎢
2 nπ nπ

nπ ∫

⎢ 0 ⎥
⎢⎣ 2 −2
2 0
2 ⎥⎦

⎡ nπx
2⎤
⎢ sin ⎥ 1 −4
1 ⎢− 4
[cos 0 − cos nπ] −
2
2 cos nπ +
2 2 ⎥ = ⎡ [1 − cos nπ] −
4 ⎤
cos nπ + 0⎥
= ⎢
2 ⎢ nπ nπ nπ nπ ⎥ 2 ⎣ nπ nπ ⎦
⎢ ⎥
⎣ 2 0⎦

SPECTRUM ALL-IN-ONE JOURNAL FOR ENGINEERING STUDENTS SIA GROUP


2.30 MATHEMATICS-II [JNTU-ANANTAPUR]

1 ⎡− 4 4 4 ⎤
+ cos nπ − cos nπ ⎥
2 ⎢⎣ nπ nπ
=
nπ ⎦

−2
∴ bn = ... (3)

On substituting equations (1), (2) and (3) in equation (A), we get,
( 2 n −1) ∞ ( 2 n −1) ∞
nπx −2 nπx nπx nπx
∑ ∑ nπ sin
4 2
f (x) =
3
– ∑ cos
4
+

sin ∑ =
3

n =1,3,5 n π
2 2
cos
2

2
n =1,3,5 n π
2 2 2 2 2 2
n =1 n =1

⎧0, if − 2 < x < −1



Q25. Find the Fourier series for the functions f(x) = ⎨ K, if − 1 < x < 1 .
⎪0, if1 < x < 2

Ans: The Fourier series is,

∞ ∞
a0 ⎛ nπx ⎞ ⎛ n πx ⎞
f(x) =
l
+ ∑
n =1
a n cos⎜
⎝ l ⎠
⎟+ ∑b
n =1
n sin ⎜
⎝ l ⎠
⎟ ... (1)

l
1
Where, a0 =
l −l ∫
f ( x) dx

Here period 2l = 4
⇒ l =2

1⎡ ⎤ 1 ⎡ −1
2 1 2

a0 =
2 ⎣⎢− 2 ∫
⎢ f ( x) dx ⎥ = ⎢ ∫ f ( x )dx + ∫ f ( x)dx + ∫ f ( x )dx⎥
⎦⎥ 2 ⎢⎣− 2 −1 1 ⎦⎥

1⎡
−1 1 2

= ∫ ∫
⎢ 0.dx + Kdx + 0 dx ⎥
2 ⎣⎢ − 2 ∫
−1 1 ⎦⎥

1 1
= [ Kx]
2 −1

K
= [1 − (−1)]
2

∴ a0 = K

l
1 ⎛ nπx ⎞
an = ∫
l −l
f ( x) cos⎜
⎝ l ⎠
⎟ dx

2
1 ⎛ nπx ⎞
=
2 −2∫f ( x ) cos⎜
⎝ 2 ⎠
⎟dx

1⎡
−1
⎛ nπx ⎞ ⎤
1 2
⎛ nπx ⎞ ⎛ nπx ⎞
2 ⎢−2


= ⎢ f ( x) cos⎜
⎝ 2 ⎠
⎟dx + f ( x) cos⎜
−1
⎝ 2 ⎠ ∫
⎟dx + f ( x) cos⎜
+1
⎟dx ⎥
⎝ 2 ⎠ ⎥⎦ ∫

Look for the SIA GROU P LOGO on the TITLE COVER before you buy
UNIT-2 (Fourier Series) 2.31

1⎡
−1
⎛ nπx ⎞ ⎤
1 2
⎛ nπx ⎞ ⎛ nπx ⎞
= ∫
⎢ 0. cos⎜
2 ⎣⎢ − 2 ⎝ 2 ⎠ −1

⎟ dx + K . cos⎜
⎝ 2 ⎠
⎟dx + 0. cos⎜
1
∫ ⎟ dx ⎥
⎝ 2 ⎠ ⎦⎥

⎡ ⎛ ⎛ nπx ⎞ ⎞
1

⎢ ⎜ sin ⎜ ⎟⎟ ⎥
=
1⎢
0+ K ⎜ ⎝ 2 ⎠⎟
+ 0⎥
2⎢ ⎜ nπ ⎟ ⎥
⎢ ⎜ ⎟ ⎥
⎝ 2 ⎠ −1
⎣⎢ ⎦⎥

1 2K ⎡ ⎛ nπ ⎞⎤ 2K ⎛ nπ ⎞
= .
2 nπ ⎢ 2 sin ⎜ 2 ⎟⎥ = sin⎜ ⎟
⎣ ⎝ ⎠⎦ nπ ⎝ 2 ⎠

l
1 ⎛ nπx ⎞
bn = ∫
l −l
f ( x ) sin ⎜
⎝ l ⎠
⎟dx

1
1 ⎛ nπx ⎞
= ∫
2 −1
K . sin⎜
⎝ 2 ⎠
⎟ dx

1
⎡ ⎛ nπx ⎞ ⎤
cos⎜ ⎟
K ⎢ ⎝ 2 ⎠⎥
=− ⎢ ⎥
2 ⎢ ⎛ nπ ⎞ ⎥
⎢ ⎜⎝ 2 ⎟⎠ ⎥
⎣ ⎦ −1

2K
=− [ 0] = 0
2 nπ
On substituting the corresponding values in equation (1), we get,


K 2K ⎛ nπ ⎞ ⎛ nπx ⎞
f(x) =
2
+ ∑ nπ sin ⎜⎝ 2 ⎟⎠ cos⎜⎝
n =1 2 ⎠

This is the required fourier series.

⎧ 2 π π
⎪⎪ x if − <x<
2 2
Q26. Find the Fourier series of the function f(x) = ⎨ 2 .
⎪π π 3π
if < x <
⎪⎩ 4 2 2

Ans: Consider the Fourier series,

a0 ∞ nπx ∞ nπx
f(x) = + Σ an cos + Σ bn sin
2 n =1 l n =1 l
But the length of the interval is 2π,
⇒ l = π/2
∴ The Fourier series reduces to,

a0 ∞ ∞
f(x) = + Σ an cos 2nx + Σ bn sin 2 nx
2 n =1 n=1

SPECTRUM ALL-IN-ONE JOURNAL FOR ENGINEERING STUDENTS SIA GROUP


2.32 MATHEMATICS-II [JNTU-ANANTAPUR]
Where,

3π / 2
1
a0 = ∫
l −π / 2
f ( x ) dx

⎡ π2 3π
2 ⎤ ⎛ π⎞
2⎢ π2 ⎥

x dx + ∫ ⎜⎝Q l =
2
a0 = dx ⎟
π⎢ π 4 ⎥ 2⎠
⎣− 2 π
2 ⎦

2 ⎡ x3 ⎤ 2 ⎡ π π ⎤ π ⎡ 3π π ⎤
3 3
2
2 π2 3π
= ⎢ ⎥ + . [x ]π 2 = ⎢ + ⎥+ ⎢ − ⎥
π ⎣⎢ 3 ⎦⎥ π 4 2
3π ⎣ 8 8 ⎦ 2 ⎣ 2 2⎦
−π 2

2 2π 3 π 2 π π 2 π 2
= + . = +
3π 8 2 2 6 2

4π 2 2π2 a0 π 2
a0 = ⇒ a0 = ⇒ =
6 3 2 3


1 2 nπx
an =
l −π∫f ( x ) cos
l
dx
2

2 ⎧⎪ 2 2 ⎫⎪
π 3π
2
π
2
⎛ π⎞
π ⎪− π ∫
= ⎨ x cos 2nx dx +
4
cos 2nx dx ⎬ ∫
⎪⎭
⎜Q l = ⎟
⎝ 2⎠
⎩ 2 −π 2

π 3π
2 ⎡ 2 ⎛ sin 2nx ⎞ ⎛ − cos 2nx ⎞ ⎛ − sin 2nx ⎞⎤
2
π ⎡ sin 2nx ⎤ 2
= ⎢x ⎜ ⎟ − 2 x⎜ ⎟ + 2⎜ ⎟ ⎥ + ⎢ ⎥
π ⎣ ⎝ 2n ⎠ ⎝ 4n
2
⎠ ⎝ 8n
2
⎠ ⎦ − π 2 ⎣ 2n ⎦ π 2 2

2 ⎡ π2 π π2 ⎛ − π ⎞⎤ 1 ⎡π π π ⎛ − π ⎞⎤
= ⎢ sin 2 n − sin 2n⎜ ⎟⎥ + 2 ⎢ 2 cos 2n 2 + 2 cos 2n⎜ 2 ⎟⎥
π ⎣ 8n 2 8n ⎝ 2 ⎠⎦ πn ⎣ ⎝ ⎠⎦

⎡ 1 π ⎛ − π ⎞⎤ π ⎡ 3π π⎤
– ⎢sin 2 n 2 − sin 2n⎜ 2 ⎟ ⎥ + 4n ⎢sin 2n 2 − sin 2n 2 ⎥
2
2πn ⎣ ⎝ ⎠⎦ ⎣ ⎦

2 ⎡ 2π2 ⎤ 1 ⎡ 2π ⎤ 1 π
= ⎢ sin nπ⎥ + 2 ⎢ cos nπ⎥ − 2 [2 sin nπ] + [sin 3nπ − sin nπ]
π ⎢⎣ 8n ⎥⎦ πn ⎣ 2 ⎦ 2n π 4n

π 1 1 π π
= sin nπ + 2 cos nπ − 2 sin nπ − sin nπ + sin 3nπ
2n n πn 4n 4n

⎛ π 1 ⎞ 1 π
=⎜ − 2 ⎟ sin nπ + 2 cos nπ + sin 3nπ
⎝ 4n πn ⎠ n 4n

∴ sin n π = 0
1 sin 3n π = 0
= cos nπ
n2 cos n π = ( −1) n

Look for the SIA GROU P LOGO on the TITLE COVER before you buy
UNIT-2 (Fourier Series) 2.33

1
an = 2
( −1) n
n

nπx
2
1
Finally, bn =
l −π ∫
f ( x) sin
l
dx
2


2
2
=
π −π ∫
f ( x) sin 2nx dx (Q l = π / 2)
2

π 3π
2 π2
2 2
2
=
π −π ∫
x 2 sin 2nx dx +
π +π 4 ∫
sin 2nx dx
2 2

π 3π
2 ⎡ 2 ⎛ − cos 2 nx ⎞ ⎛ − sin 2nx ⎞ ⎛ cos 2nx ⎞ ⎤ π ⎡ − cos 2nx ⎤ 2
2

= ⎢x ⎜ ⎟ − 2 x⎜ ⎟ + 2⎜ ⎟ ⎥ + ⎢ ⎥
π⎣ ⎝ 2n ⎠ ⎝ 4n
2 3
⎠ ⎝ 8n ⎠ ⎦ −π 2 ⎣ 2 n ⎦ π 2 2

2 ⎡ − π2 π 1 π π2 ⎛−π⎞ 1 ⎛ − π ⎞⎤
= ⎢ cos 2n + 0 + 3 cos 2n + cos 2n⎜ ⎟ + 0 − 3 cos 2n⎜ ⎟⎥
π ⎣⎢ 8n 2 4n 2 8n ⎝ 2 ⎠ 4n ⎝ 2 ⎠⎦⎥

π ⎡ 3π π⎤
− ⎢cos 2n − cos 2n ⎥
4n ⎣ 2 2⎦

2 ⎡ − π2 1 π2 1 ⎤ π
= ⎢ cos nπ + 3 cos nπ + cos nπ − 3 cos nπ⎥ − [cos 3nπ − cos nπ]
π ⎢⎣ 8n 4n 8n 4n ⎥⎦ 4n

2 π
= [0 + 0] − [cos 3nπ − cos nπ]
π 4n
⎡Q cos 3n π = ( −1) 3n ⎤
bn =
−π
4n
[
(−1) 3n − ( −1) n ] ⎢
⎢⎣ cos nπ = ( − 1) n ⎥⎦

bn = 0, if n is even
On substituting a0, an and bn in equation (1), we get,

π 2 ∞ ( −1) n ∞
f(x) = +Σ 2
cos 2nx + Σ 0. sin 2nx
3 n =1 n n =1

π 2 ∞ (−1) n
f(x) = +Σ cos 2nx
3 n =1 n 2

π 2 cos 2 x cos 4 x cos 6 x cos 8 x


∴ f(x) = − 2 + − 2 + + ...
3 1 22 3 42
Q27. Find Fourier series of period 3 to represent f(x) = 2x – x2 in (0, 3). Also find Fourier series of period 2 to
represent f(x) = 2x – x2 in (0, 2).
Ans: The first step in finding the Fourier series of period 2l is to represent f(x) = 2x – x2 in (0, 2l) and deduce the required results.

a0 ∞ nπx ∞ nπx
f(x) = + Σ an cos + Σ bn sin ... (1)
2 n =1 l n =1 l

SPECTRUM ALL-IN-ONE JOURNAL FOR ENGINEERING STUDENTS SIA GROUP


2.34 MATHEMATICS-II [JNTU-ANANTAPUR]
2l
1
Here, a0 =
l0 ∫
f ( x) dx

2l
1
=
l0 ∫
(2 x − x 2 )dx

2l
1 ⎡ 2 x 2 x3 ⎤
= ⎢ − ⎥
l⎣ 2 3 ⎦0

1 ⎡ 2 8l 3 ⎤
a0 = ⎢4l − ⎥
l⎣ 3 ⎦

a0 ⎛ 2l 2 ⎞
= 2l ⎜⎜ l − ⎟ ... (2)
2 ⎝ 3 ⎟⎠

2l
1 nπx
And an =
l0 ∫
f ( x) cos
l
dx

2l
1 nπx
=
l0 ∫
( 2 x − x 2 ) cos
l
dx

1⎡ ⎤
2l
nπx ⎡ nπx ⎤
∫ ∫ ∫
2
= ⎢ ( 2 x − x ) cos dx − ⎢ ( 2 − 2 x ) cos dx ⎥ dx ⎥
l⎢

l ⎣ l ⎦ 0 ⎥⎦

2l
⎡ nπx ⎡ nπx ⎤ ⎤
sin sin
1⎢ ⎢ l ⎥dx ⎥⎥
= ⎢(2 x − x ) n ∫
l − (2 − 2 x )
2
π ⎢ nπ ⎥ ⎥
l⎢ ⎢ ⎥
⎣⎢ l ⎣ l ⎦ ⎦⎥ 0

2l
⎡ ⎡ ⎛ ⎞ ⎛ ⎞ ⎤⎤
⎢ nπx ⎢ ⎜ − cos nπx ⎟ ⎜ cos nπx ⎟ ⎥ ⎥
sin ⎜
1⎢ l − ⎢(2 − 2 x) l ⎟− ⎜ ⎟ ⎥⎥
= ⎢(2 x − x ) ∫ l
2
nπ ⎢ ⎜ 2 ⎟ ⎜2 dx
2 ⎟ ⎥⎥
l
⎢ ⎢ ⎜ ⎛⎜ nπ ⎞⎟ ⎟ ⎜ ⎛⎜ nπ ⎞⎟ ⎟ ⎥ ⎥
⎢ ⎜ ⎟ ⎜ ⎟
⎝ ⎝ l ⎠ ⎠ ⎥⎦ ⎥⎦ 0
l ⎢⎣
⎣ ⎝ ⎝ l ⎠ ⎠

2l
⎡ ⎡ ⎛ ⎞ ⎤⎤
⎢ nπx ⎢ ⎜ − cos nπx ⎟ nπx ⎥ ⎥
sin sin
1⎢
= ⎢(2 x − x )
2 l − ⎢(2 − 2 x )⎜ l ⎟−2 l ⎥⎥
nπ ⎢ ⎜ 2 ⎟ 3 ⎥⎥
l
⎢ ⎢ ⎜ ⎛⎜ nπ ⎞⎟ ⎟ ⎛ nπ ⎞ ⎥ ⎥
⎜ ⎟
⎢ ⎜ ⎟
⎝ l ⎠ ⎥⎦ ⎥⎦ 0
l ⎢⎣
⎣ ⎝ ⎝ l ⎠ ⎠

2l
⎡ ⎡ ⎛ ⎞ ⎛ ⎞⎤ ⎤
⎢ nπx ⎢ ⎜ − cos nπx ⎟ ⎜ sin nπx ⎟⎥ ⎥
sin
1⎢ l − ⎢(2 − 2 x)⎜ l ⎟ + (−2)⎜ l ⎟⎥ ⎥
= ⎢(2 x − x )
2
nπ ⎢ ⎜ 2 ⎟ ⎜ 3 ⎟⎥ ⎥
l
⎢ ⎢ ⎜ ⎛⎜ nπ ⎞⎟ ⎟ ⎜ ⎛⎜ nπ ⎞⎟ ⎟⎥ ⎥
⎢ ⎜ ⎟ ⎜ l ⎟
⎝ ⎝ ⎠ ⎠⎥⎦ ⎥⎦ 0
l ⎢⎣
⎣ ⎝ ⎝ l ⎠ ⎠

Look for the SIA GROU P LOGO on the TITLE COVER before you buy
UNIT-2 (Fourier Series) 2.35

⎡ ⎛ ⎞⎤
⎢ ⎜ ⎟⎥
⎜ − cos 2nπ 2nπ ⎟ ⎥ ⎡ l2 ⎤ sin 2 n π = 0 sin 0 = 0
1 2 sin 2nπ ⎢
= ( 4l − 4l ) nπ − ⎢(2 − 4l )⎜ − 2 sin ⎟
3 ⎥
− ⎢ 0 − ( 0 − 2 ) − 0⎥
cos 2 n π = 1 cos 0 = 1
l ⎜ ⎛ nπ ⎞
2
⎛ nπ ⎞ ⎟⎥ ⎣⎢ n π
2 2
⎥⎦

l ⎢⎣ ⎜ ⎜ l ⎟ ⎜ ⎟ ⎟

⎝ ⎝ ⎠ ⎝ ⎠ ⎠⎦
l

1⎡ l2 2l 2 ⎤
= ⎢0 + 2(1 − 2l ) 2 2 − 0 − 2 2 ⎥
l ⎣⎢ n π n π ⎦⎥

2l
= [1 − 2l − 1]
n 2π2

4l 2
an = – ... (3)
n2 π2

2l 2l
1 nπx 1 nπx
Finally, bn = ∫
l0
f ( x) sin
l
dx =
l0 ∫
( 2 x − x 2 ) sin
l
dx

2l
⎡ ⎛ nπx ⎞ ⎡ ⎛ nπx ⎞⎤ ⎤
1⎢ ⎜ − cos ⎟ ⎢ ⎜ − cos ⎟ ⎥
l ⎟⎥⎥ dx ⎥
l ⎟−

2 ⎜
= ⎢(2 x − x ) ⎢ (2 − 2 x ) ⎜
l⎢ ⎜ nπ ⎟ ⎢ ⎜ nπ ⎟⎥ ⎥
⎢⎣ ⎜ ⎟ ⎜ ⎟ ⎥
⎝ l ⎠ ⎣⎢ ⎝ l ⎠⎦⎥ ⎦ 0

2l
⎡ ⎡ ⎛ ⎞ ⎡ ⎛ ⎞⎤ ⎤ ⎤
⎢ ⎛ nπx ⎞ ⎢ ⎜ − sin nπx ⎟ ⎢ ⎜ sin nπx ⎟⎥ ⎥ ⎥
⎜ − cos ⎟
1⎢ l ⎟ − ⎢(2 − 2 x )⎜ l ⎟− ⎢2⎜ ⎟⎥ ⎥ ⎥
= ⎢(2 x − x )⎜ ∫ l
2
⎜ nπ ⎟ ⎢
⎜ 2 ⎟ ⎢ ⎜ π 2 ⎟⎥ ⎥ dx ⎥
l⎢
⎜ ⎟ ⎢ ⎜ ⎛⎜ nπ ⎞⎟ ⎟ ⎢ ⎜ ⎛⎜ n ⎞⎟ ⎟⎥ ⎥ ⎥
⎢ ⎝ l ⎠ ⎢ ⎜ ⎟ ⎢⎣ ⎜⎝ ⎝ l ⎠ ⎟⎠⎥⎦ ⎥⎦ ⎥
⎣ ⎣ ⎝ ⎝ l ⎠ ⎠ ⎦0

2l
⎡ ⎡ ⎛ ⎞ ⎛ ⎞⎤ ⎤
⎢ ⎛ nπ x ⎞ ⎢ ⎜ − sin nπx ⎟ ⎜ cos nπx ⎟⎥ ⎥
⎜ − cos ⎟
1⎢ l ⎟ − ⎢(2 − 2 x)⎜ l ⎟ + (−2)⎜ l ⎟⎥ ⎥
= ⎢(2 x − x )⎜
2
nπ ⎢ ⎜ 2 ⎟ ⎜ 3 ⎟⎥ ⎥
l
⎢ ⎜ ⎟ ⎜ ⎛ nπ ⎞ ⎟ ⎜ ⎛ nπ ⎞ ⎟⎥ ⎥
⎜ ⎟ ⎢
⎢ ⎝ l ⎠ ⎢ ⎜ ⎜ l ⎟ ⎟ ⎜ ⎜ l ⎟ ⎟⎥ ⎥
⎣ ⎣ ⎝ ⎝ ⎠ ⎠ ⎝ ⎝ ⎠ ⎠⎦ ⎦ 0

2l
⎡ ⎛ ⎞ ⎛ ⎤
⎢ ⎛ − cos nπx ⎞ ⎜ − sin nπx ⎟ ⎜ − cos nπx ⎞⎟⎥
1⎢ ⎜ ⎟ ⎜ ⎟ ⎜ ⎟⎥
2 ⎜
= ⎢(2 x − x ) l ⎟ − ( 2 − 2 x )⎜ l + (−2)⎜ l
nπ 2 ⎟ 3 ⎟⎥
l ⎜ ⎟ ⎜ ⎛ nπ ⎞ ⎟ ⎜ ⎛ nπ ⎞ ⎟ ⎥
⎢ ⎜ ⎟ ⎜ ⎟ ⎟ ⎜ ⎟ ⎟
⎢⎣ ⎝ l ⎠ ⎜ ⎜
⎝ ⎝ l ⎠ ⎠ ⎝ ⎝ l ⎠ ⎠⎥⎦ 0

1⎡ l 2l 2 ⎤ 1⎡− l ⎤
⎢− (4l − 4l ) cos 2nπ − 0 − 3 3 (cos 2nπ − cos( 0))⎥ = ⎢ 4l (1 − l )⎥
2
=
l ⎢⎣ nπ nπ ⎥⎦ l ⎣ nπ ⎦

− 4l(1− l)
bn = ... (4)

SPECTRUM ALL-IN-ONE JOURNAL FOR ENGINEERING STUDENTS SIA GROUP


2.36 MATHEMATICS-II [JNTU-ANANTAPUR]
On substituting equations (2), (3) and (4) in equation (1), we get,
∞ ∞
⎛ 2l ⎞ − 4l 2 nπx − 4l (1 − l ) nπx
f(x) = 2l ⎜1 − ⎟ +
⎝ 3⎠ ∑
n =1 n π
2 2
cos
l
+ ∑
n =1

sin
l

2 ∞ ∞
⎛ 2l ⎞ 4l nπx 4l (1 − l ) nπx
∑ ∑
1 1
2x – x2 = 2l ⎜1 − ⎟ − 2 cos − sin ... (5)
⎝ 3⎠ π n =1 n
2
l π n=1 n l

To represent the Fourier series with period 3 in (0, 3), substitute 2l = 3 or l = 3/2 value in equation (5), we get,

⎛ 3⎞
∞6⎜1 − ⎟ ∞
⎛ ⎞ π 2nπx
− ⎝
3 9 1 2 n x 2⎠ 1
2x – x2 = 3⎜1 − ⎟ − 2
⎝ 3 ⎠ π n =1 n
2
cos
3 ∑ π ∑ n sin
n =1
3


9 1 2nπx 3 ∞ 1 2nπx
⇒ 2x – x2 =
π2

n =1 n
2
cos
3
+
π n =1 n
sin ∑3
... (6)

Equation (6), is the required Fourier series of period 3.


To represent the Fourier series with period 2 in (0, 2)
Let us substitute 2l = 2
⇒ l = 1 in equation (5), we get,

⎛ 2⎞ 4 1 4(1 − 1) ∞ 1
⇒ 2x – x2 = 2⎜1 −

⎟−
3 ⎠ π2 ∑
n =1 n
2
cos nπx −
π n =1 n ∑sin nπx


2 4 1
⇒ 2x – x2 = − 2
3 π ∑n
n =1
2
cos nπx

This is the required result.


2.4 EVEN AND ODD PERIODIC CONTINUATION HALF-RANGE FOURIER SINE AND COSINE
EXPANSIONS
Q28. Find the half-range sine series of f(x) = (x – 1)2 in the interval (0, 1). Model Paper-I, Q5

Ans: Given that,


f (x) = (x – 1)2 interval (0, 1)
The half-range fourier sine series is given by,
∞ ⎛ nπx ⎞
f (x) = (x –1)2 = ∑ bn sin ⎜ ⎟
n =1 ⎝ l ⎠


= ∑ bn sin (nπx) [Q l = 1]
n =1

l
2 ⎛ nπx ⎞
Where, bn =
l ∫ f ( x).sin⎜⎝
0
l ⎠
⎟ dx

1
2
=
1 ∫ f ( x).sin(nπx)dx
0
[Q l = 1]

1
2
= ( x − 1) 2 . sin (nπx )dx

1
0

Look for the SIA GROU P LOGO on the TITLE COVER before you buy
UNIT-2 (Fourier Series) 2.37
1

= 2 ( x − 2 x + 1). sin (nπx )dx



2

⎡1 1 1 ⎤

∫ π − ∫ π + ∫ sin(nπx)dx ⎥
2
= 2 x sin( n x ) dx 2 x sin( n x ) dx
⎢ ⎥
⎣0 0 0 ⎦
Let, bn = 2 [I1 + I2 +I3]
1

∫x
2
Consider, I1 = sin( nπx) dx
0

1
⎡ ⎡ − cos( nπx) ⎤ ⎡ − cos( nπx ) ⎤ ⎤
= ⎢x2 ⎢
⎣ ⎣ nπ ⎥ − ( 2 x) ⎢
⎦ ⎣ ∫ nπ ⎥ dx ⎥
⎦ ⎦0

1
⎡ − x2 2 ⎤
=⎢
⎣⎢ nπ
cos( nπx ) +
nπ ∫
x cos( nπx )dx ⎥
⎦⎥ 0

1
⎡− x2 2 ⎡ ⎡ sin( nπx) ⎤ sin( nπx) ⎤ ⎤
=⎢
⎣⎢ nπ
cos( nπx ) + ⎢x⎢
nπ ⎣ ⎣ nπ ⎦ ⎥ − 1× ∫nπ
dx ⎥ ⎥
⎦ ⎥⎦ 0

1
⎡ − x2 2 ⎡ x 1 ⎤⎤
=⎢
⎣⎢ nπ
cos( nπx) + ⎢
nπ ⎣ nπ
sin(nπx ) −
nπ ∫
sin(nπx)dx ⎥ ⎥
⎦ ⎦⎥ 0

1
⎡ − x2 2 ⎡ x 1 ⎡ − cos( nπx) ⎤ ⎤ ⎤
=⎢ cos( nπx) + ⎢ sin(nπx) − ⎥ ⎥⎥
⎣⎢ nπ nπ ⎣ nπ nπ ⎢⎣ nπ ⎦ ⎦ ⎦⎥ 0

1
⎡ − x2 2 ⎡ x 1 ⎤⎤
=⎢ cos( nπx ) + ⎢ sin(nπx ) + 2 2 cos( nπx )⎥ ⎥
⎢⎣ nπ nπ ⎣ nπ n π ⎦ ⎥⎦ 0

1
⎡ − x2 2x 2 ⎤
=⎢ cos( nπx) + 2 2 sin(nπx) + 3 3 cos( nπx )⎥
⎢⎣ nπ n π n π ⎥⎦ 0

⎡ −1 2 2 ⎤ ⎡ 2 ⎤
= ⎢ cos nπ + 2 2 sin( nπ) + 3 3 cos( nπ) ⎥ − ⎢0 + 0 + 3 3 cos 0 ⎥
⎣ nπ n π n π ⎦ ⎣ n π ⎦

−1 2 2
= ( −1) n + 0 + 3 3 ( −1) n − 3 3 [Q cos (nπ) = (–1)n, sin (nπ) = 0]
nπ n π n π

(−1) n ⎡ 2 ⎤ 2
= ⎢ − 1⎥ − 3 3
nπ ⎣ n π2 2
⎦ n π


Consider, I2 = − 2 x sin( nπx ) dx
0

1
⎡ ⎡ − cos( nπx ) ⎤ − cos( nπx) ⎤
= − 2⎢ x ⎢
⎣ ⎣ nπ ⎥ − 1×
⎦ nπ∫ dx ⎥
⎦0

SPECTRUM ALL-IN-ONE JOURNAL FOR ENGINEERING STUDENTS SIA GROUP


2.38 MATHEMATICS-II [JNTU-ANANTAPUR]
1
⎡− x 1 ⎤
= − 2⎢
⎣ nπ
cos( nπx) +

cos( nπx )dx ⎥∫
⎦0
1
⎡− x 1 ⎡ sin(nπx ) ⎤ ⎤
= − 2⎢ cos( nπx ) + ⎥
⎣ nπ nπ ⎢⎣ nπ ⎥⎦ ⎦ 0

1
⎡− x 1 ⎤
= − 2⎢ cos( nπx ) + 2 2 sin(nπx)⎥
⎣ nπ n π ⎦0

⎡⎧ − 1 1 ⎫ ⎧ 1 ⎫⎤
= − 2⎢⎨ cos nπ + 2 2 sin nπ⎬ − ⎨0 + 2 2 sin 0⎬⎥
⎣ ⎩ nπ n π ⎭ ⎩ n π ⎭⎦

⎡ −1 ⎤
= − 2 ⎢ ( −1) + 0 − 0 − 0 ⎥
n
⎣ nπ ⎦
2
= ( −1) n

1


I3 = sin(nπx)dx
0

1
⎡ − cos nπx ⎤
=⎢ ⎥
⎣ nπ ⎦ 0

−1
= [cos nπ − cos 0]

−1
= [(−1) n − 1]

1
= [1 − ( −1) n ]

⎡ ⎧⎪ ( −1) n ⎡ 2
Hence, bn = 2 ⎢⎨ ⎢
⎤ 2 ⎪⎫ ⎧ 2 ⎫ ⎧1
− 1⎥ − 3 3 ⎬ + ⎨ (−1) n ⎬ + ⎨ {
1 − ( −1) n }⎫⎬⎤⎥
⎢⎣ ⎪⎩ nπ ⎣ n π ⎦ n π ⎪⎭ ⎩ nπ ⎭ ⎩ nπ
2 2
⎭⎥⎦

⎡ (−1) n ⎡ 2 ⎤ 2 1 ⎤
= 2⎢ ⎢ − 1 + 2 − 1⎥ − 3 3 + ⎥
⎢⎣ nπ ⎣ n π ⎦ n π nπ ⎥⎦
2 2

⎡ (−1) n ⎡ 2 ⎤ 2 1 ⎤
= 2⎢ ⎢ 2 2⎥
− 3 3+ ⎥
⎢⎣ nπ ⎣ n π ⎦ n π nπ ⎥⎦

4 4 2
= (−1) n − +
n π 3 3 3 3
n π nπ

=
n π
4
3 3
[(−1) − 1]+ n2π
n


∴ f (x)= ∑ bn sin( nπx )
n =1

∞ ⎡ 4 2 ⎤
= ∑ ⎢ 3 3 [( −1) − 1] + ⎥ sin( nπx )
n
n =1 ⎣ n π nπ ⎦

Look for the SIA GROU P LOGO on the TITLE COVER before you buy
UNIT-2 (Fourier Series) 2.39

⎧1 1
⎪4 − x , 0 < x < 2
Q29. Find the half-range sine series for f(x) = ⎨ 3 1 . Model Paper-III, Q5
⎪x − , <x <1
⎩ 4 2
Ans: Given that,

⎧1 1
⎪4 − x , 0 < x < 2
f(x) = ⎨
3 1
⎪x − , < x <1
⎩ 4 2
The half-range Fourier sine series is given by,

⎛ nπx ⎞
f(x) = ∑b
n =1
n sin⎜
⎝ l ⎠

= ∑b
n =1
n sin(nπx ) [Q l = 1]

Where,

l
2 ⎛ nπx ⎞
bn =
l ∫ f ( x).sin⎜⎝
0
l ⎠
⎟ dx

l
2
=
1 ∫ f ( x).sin( nπx) dx
0

⎡1/ 2 l ⎤
∫ ∫
= 2⎢ f1 ( x ) sin(nπx ) dx + f 2 ( x) sin(nπx ) dx ⎥
⎢0 ⎥
⎣ 1/ 2 ⎦

⎡1/ 2 ⎛ 1 ⎞
l
⎛ 3⎞ ⎤
∫⎠ ⎝ 4⎠ ∫
= 2⎢ ⎜ − x ⎟ sin nπx dx + ⎜ x − ⎟ sin(nπx ) dx ⎥
⎢ 0 ⎝4 ⎥
⎣ 1/ 2 ⎦
= 2[I1 + I2]
Consider,

1/ 2
⎛1 ⎞
I1 = ∫ ⎜⎝ 4 − x ⎟⎠ sin( nπx) dx
0

1/ 2
⎡⎡ 1 ⎤ ⎡ − cos( nπx ) ⎤ ⎡ − cos( nπx) ⎤ ⎤
= ⎢ ⎢ − x⎥ ⎢
⎣⎣ 4 ⎦ ⎣ nπ ⎥

− (−1) ⎢
⎣ ∫nπ ⎥ dx ⎥
⎦ ⎦0

1/ 2
⎡ ⎛ 1− 4x ⎞ 1 ⎤
= ⎢− ⎜
⎣ ⎝ 4nπ ⎠
⎟ cos( nπx) −

cos( nπx )dx ⎥
⎦0

1/ 2
⎡⎛ 4 x − 1 ⎞ 1 ⎤
= ⎢⎜ ⎟ cos( nπx ) − 2 2 sin(nπx)⎥
⎣⎝ 4nπ ⎠ n π ⎦0

SPECTRUM ALL-IN-ONE JOURNAL FOR ENGINEERING STUDENTS SIA GROUP


2.40 MATHEMATICS-II [JNTU-ANANTAPUR]

⎡⎧⎛ ⎛ 1 ⎞ ⎞ ⎫ ⎤
⎢⎪⎜ 4⎜ ⎟ − 1 ⎟ ⎪ ⎥
⎢⎪⎜ ⎝ 2 ⎠ ⎟ cos⎛⎜ nπ ⎞⎟ − 1 sin⎛⎜ nπ ⎞⎟⎪ − ⎧⎛⎜ 0 − 1 ⎞⎟ cos 0 − 1 sin 0⎫⎥
= ⎢⎨⎜ 4nπ ⎟ ⎬ ⎨ ⎬⎥
⎝ 2 ⎠ n π ⎝ 2 ⎠⎪ ⎩⎝ 4nπ ⎠ n 2π2
2 2

⎢⎪⎪⎜ ⎟
⎪⎭ ⎥
⎢⎣⎩⎝ ⎠ ⎥⎦

1 ⎛ nπ ⎞ 1 ⎛ nπ ⎞ 1
= cos⎜ ⎟ − 2 2 sin ⎜ ⎟ +
4nπ ⎝ ⎠2 n π ⎝ ⎠2 4 n π

1
⎛ 3⎞
I2 = ∫ ⎜⎝ x − 4 ⎟⎠.sin( nπx) dx
1/ 2

1
⎡⎛ 3 ⎞ ⎡ − cos nπx ⎤ ⎡ − cos nπx ⎤ ⎤
= ⎢⎜ x − ⎟.⎢
⎣⎝ 4 ⎠ ⎣ nπ ⎦ ⎥ − 1.⎢ ∫ ⎥ dx ⎥
⎣ nπ ⎦ ⎦1 / 2

1
⎡ ⎛ 4x − 3 ⎞ 1 ⎤
= ⎢− ⎜
⎣ ⎝ 4 nπ
⎟ cos( nπx) +
⎠ nπ ∫
cos( nπx )dx ⎥
⎦ 1/ 2

1
⎡⎛ 3 − 4 x ⎞ 1 ⎤
= ⎢⎜ ⎟ cos( nπx ) + 2 2 sin(nπx)⎥
⎣⎝ 4nπ ⎠ n π ⎦ 1/ 2

⎡⎧ 1 1 ⎫ ⎧ 1 ⎛ nπ ⎞ 1 ⎛ nπ ⎞⎫⎤
= ⎢ ⎨− cos nπ + 2 2 sin nπ⎬ − ⎨ cos ⎜ ⎟ + 2 2 sin⎜ ⎟⎬⎥
⎣⎢⎩ 4nπ n π ⎭ ⎩ 4nπ ⎝ 2 ⎠ n π ⎝ 2 ⎠⎭⎦⎥

⎡⎧ − 1 ⎫ ⎧ 1 ⎛ nπ ⎞ 1 ⎛ nπ ⎞ ⎫ ⎤
= ⎢⎨ (−1) n + 0⎬ − ⎨ cos ⎜ ⎟ + 2 2 sin⎜ ⎟⎬⎥
⎣⎢⎩ 4nπ ⎭ ⎩ 4nπ ⎝ 2 ⎠ n π ⎝ 2 ⎠⎭⎦⎥

− (−1) n 1 ⎛ nπ ⎞ 1 ⎛ nπ ⎞
= − cos⎜ ⎟ − 2 2 sin⎜ ⎟
4 nπ 4 nπ ⎝ 2 ⎠ n π ⎝ 2 ⎠
∴ bn = 2[I1 + I2]

⎡ 1 ⎛ nπ ⎞ 1 ⎛ nπ ⎞ 1 (−1) n 1 ⎛ nπ ⎞ 1 ⎛ nπ ⎞ ⎤
= 2⎢ cos⎜ ⎟ − 2 2 sin⎜ ⎟ + − − cos⎜ ⎟ − 2 2 sin⎜ ⎟⎥
⎣⎢ 4nπ ⎝ 2 ⎠ n π ⎝ 2 ⎠ 4nπ 4nπ 4nπ ⎝ 2 ⎠ n π ⎝ 2 ⎠⎦⎥

⎡ −2 ⎛ nπ ⎞ 1 (−1) n ⎤
= 2⎢ 2 2 sin⎜ ⎟ + − ⎥
⎢⎣ n π ⎝ 2 ⎠ 4nπ 4nπ ⎥⎦

⎡ −4 ⎛ nπ ⎞ 1 (−1) n ⎤
= ⎢ 2 2 sin⎜ ⎟ + − ⎥
⎣⎢ n π ⎝ 2 ⎠ 2nπ 2nπ ⎦⎥

−4 ⎛ nπ ⎞ 1
= sin ⎜ ⎟ + [1 − ( −1) n ]
n π 2 2
⎝ 2 ⎠ 2 nπ


nπ ⎤
∑ ⎡⎢⎣ 2nπ [1 − (−1)
1 n 4
∴ f ( x) = ]− sin sin( nπx )
n =1 n π
2 2 2 ⎥⎦

Look for the SIA GROU P LOGO on the TITLE COVER before you buy
UNIT-2 (Fourier Series) 2.41
Q30. Obtain half-range sine series for ex in 0 < x < 1.
Ans: Given that,
f(x) = ex for 0 < x < 1
The half-range Fourier sine series is given by,


⎡ nπx ⎤
f(x) = ∑b n sin ⎢
⎣ l ⎦

n =1

= ∑b n sin(nπx )
n =1

Where,
l
2 ⎡ nπx ⎤
bn =
l ∫ f ( x) sin ⎢⎣ l ⎥⎦
dx
0


= 2 f ( x) sin( nπx)dx
0
[Q l = 1]


x
= 2 e sin( nπx)dx
0

1
⎡ ex ⎤ ⎡ e ax ⎤
= 2⎢ 2
⎣⎢1 + ( nπ)
2
(sin(nπx ) − nπ cos( nπx) ⎥
⎦⎥ 0 ⎣⎢

⎢Q e sin bxdx = 2
ax
a + b2
[ a sin bx − b cos bx ]⎥
⎦⎥

2
= [ e x [sin( nπx ) − nπ cos( nπx)]]10
n 2π 2 + 1

⎡e{(sin( nπ) − nπ cos( nπ )}⎤


2
= ⎢ ⎥
n π + 1 ⎣ − e 0 .{sin 0 − nπ cos 0} ⎦
2 2

2
= 2 2
[e{0 − nπ( −1) n } − 1.{0 − nπ}] [Q cos(nπ) = (– 1)n, sin(nπ) = 0]
n π +1

2
= 2 2
[ −2.718nπ( −1) n + nπ]
n π +1

2
= [ nπ(1 − 2.718( −1) n )]
n π +1
2 2

2nπ
= [1 − 2.718( −1) n ]
n π +1
2 2


∴ f(x) = ∑b
n =1
n sin(nπx )


⎡ 2n π ⎤
= ∑ ⎢⎣ n π
n =1
2 2
+ 1
(1 − 2.718(−1) n ⎥ sin(nπx)

SPECTRUM ALL-IN-ONE JOURNAL FOR ENGINEERING STUDENTS SIA GROUP


2.42 MATHEMATICS-II [JNTU-ANANTAPUR]
2
Q31. Find the half-range sine series of f(t) = t – t in the interval 0 < t < 1.
Ans: Given that,
f (t) = t – t2 0<t<1
The Fourier half-range sine series of f (t) in the interval (0, l) is given by,


⎛ nπt ⎞
f (t) = ∑b
n =1
n sin⎜
⎝ l ⎠
⎟ ... (1)

Where,

l
2 ⎛ nπt ⎞
bn =
l ∫ f (t ) ⋅ sin⎜⎝
0
l ⎠
⎟dt

Here, l = 1


∴ f (t) = ∑b
n =1
n ⋅ sin(nπt )

1
2 ⎛ nπt ⎞
bn =
1 ∫
(t − t 2 ) sin⎜
0
⎝ 1 ⎠
⎟dt

⎡1 1 ⎤
∫ ∫
= 2 t sin(nπt )dt − t sin(nπt )dt ⎥
⎢ 2
⎢0 ⎥
⎣ 0 ⎦

1
⎡ ⎡ − cos( nπt ) ⎤ ⎡ − cos( nπt ) ⎤ ⎤ ⎡ 2 ⎡ − cos( nπt ) ⎤ ⎡ − cos( nπt ) ⎤ ⎤
= 2⎢t ⎢
⎣⎣ nπ ⎥− ⎢
⎦ ⎣ nπ ∫ ⎥dt ⎥ − ⎢t ⎢
⎦ ⎦ ⎣ ⎣ nπ ⎦ ∫
⎥ − (2t ) ⋅ ⎢
⎣ nπ
dt ⎥ ⎥
⎦⎦ 0

1
⎡⎡ − 1 1 ⎤ ⎡ −1 2 ⎤⎤
= 2⎢⎢ t cos( nπt ) +
⎣⎣ nπ n π ∫
cos( nπt )dt ⎥ − ⎢ t 2 cos( nπt ) +
⎦ ⎣ n π n π ∫
t cos( nπt )dt ⎥ ⎥
⎦⎦ 0

1
⎡⎧ − 1 1 ⎡ sin(nπt ) ⎤ ⎫ ⎧⎪ − 1 2 2 ⎡ ⎡ sin(nπt ) ⎤ ⎡ sin(nπt ) ⎤ ⎤ ⎫⎪⎤
= 2⎢⎨ t cos( nπt ) +
⎣⎢⎩ nπ
⎢ ⎥ ⎬ − ⎨ t cos( nπt ) +
nπ ⎣ nπ ⎦ ⎭ ⎪⎩ nπ ⎢t⎢
nπ ⎣ ⎣ nπ ⎦ ⎥ − ⎢
∫ ⎥ dt ⎥ ⎬⎥
⎣ nπ ⎦ ⎦ ⎪⎭⎦⎥ 0

1
⎡⎧ − t 1 ⎫ ⎧⎪ − t 2 2 ⎡ t ⎛ − cos( nπt ) ⎞⎤ ⎫⎪⎤
= 2⎢⎨ cos( nπt ) + 2 2 sin(nπt )⎬ − ⎨ cos( nπt ) + ⎢ sin(nπt ) − ⎜ ⎟ ⎥ ⎬⎥
⎣⎢⎩ nπ n π ⎭ ⎪⎩ nπ nπ ⎣ nπ ⎝ n 2 π 2 ⎠⎦ ⎪⎭⎦⎥ 0

1
⎡−t 1 t2 2t 2 ⎤
= 2⎢ cos( nπt ) + 2 2 sin(nπt ) + cos( nπt ) − 2 2 sin(nπt ) − 3 3 cos( nπt )⎥
⎢⎣ nπ n π nπ n π n π ⎥⎦ 0
1
⎡ ⎛ −t t2 2 ⎞ ⎛ 1 2t ⎞⎤
= 2 ⎢cos( nπt )⎜⎜ + − 3 3 ⎟ + sin( nπt )⎜ 2 2 − 2 2
⎟ ⎟⎥
⎣⎢ ⎝ n π nπ n π ⎠ ⎝ n π n π ⎠ ⎦⎥ 0

⎡⎧ ⎛ −1 1 2 ⎞ ⎛ 1 2 ⎞⎫ ⎧ ⎛ 2 ⎞ ⎛ 1 ⎞ ⎫⎤
= 2⎢⎨cos( nπ)⎜ + − 3 3 ⎟ + sin(nπ)⎜ 2 2 − 2 2 ⎟⎬ − ⎨cos(0)⎜ 0 + 0 − 3 3 ⎟ + sin(0)⎜ 2 2 − 0 ⎟⎬⎥
⎢⎣⎩ ⎝ nπ nπ n π ⎠ ⎝n π n π ⎠⎭ ⎩ ⎝ n π ⎠ ⎝n π ⎠⎭⎥⎦

Look for the SIA GROU P LOGO on the TITLE COVER before you buy
UNIT-2 (Fourier Series) 2.43

⎡⎧ n⎛ −2 ⎞ ⎫ ⎧ −2 ⎫⎤ π
= 2⎢⎨(−1) ⎜ 3 3 ⎟ + 0⎬ − ⎨ 3 3 + 0⎬⎥ 2 ⎡ x2 ⎤
⎢⎣⎩ ⎝ n π ⎠ ⎭ ⎩n π ⎭⎥⎦ = ⎢ ⎥
π ⎢⎣ 2 ⎥⎦
0
[Q cos (nπ) = (– 1)n, sin(nπ) = sin0 = 0]

⎡ −2 2 ⎤ 1 2 π
= 2 ⎢ 3 3 ( −1) + 3 3 ⎥
n = [ x ]0
⎣n π n π ⎦ π

1 2
⎡ 2 n ⎤ = [ π − 0] = π
= 2 ⎢ 3 3 [1 − ( −1) ]⎥ π
⎣n π ⎦
∴ a0 = π
⎡ 4 n ⎤
= ⎢ 3 3 [1 − ( −1) ]⎥
⎣n π ⎦ π
2
an =
π ∫ f ( x).cos nx dx
⎧ 0 ; if n is even 0

∴ bn = ⎨ 8 ; if n is odd
⎪⎩ n 3 π3 π
2
Now substituting the value of ‘bn’ in equation (1), we get,
=
π ∫
x cos nx dx
0

∞ π
∑n π
8 2 ⎡ ⎡ sin nx ⎤ sin nx ⎤
sin(nπt )
f (t) =
n =1
3 3 [Q l = 1] = ⎢x⎢
π⎣ ⎣ n ⎦ ⎥ − 1×
n ∫
dx ⎥
⎦0

sin(nπt ) π

8 2 ⎡x 1 ⎤
t – t2 =
π 3
n =1 n 3 = ⎢
π ⎣n
sin nx −
n ∫
sin nx dx ⎥
⎦0

8 ⎡ 1 1 ⎤ π
= ⎢sin( πt ) + 3 sin( 2πt ) + 3 sin( 3πt ) + ...⎥ 2 ⎡x 1 ⎡ − cos nx ⎤⎤
π3 ⎣ 2 3 ⎦ = ⎢ sin nx − ⎢ ⎥
π ⎣n n ⎣ n ⎥⎦⎦ 0
Q32. Obtain half-range cosine series f(x) = x in the interval
π
1 1 1 π2 2 ⎡x 1 ⎤
0 ≤ x ≤ π. Hence show that + + + ... = . = ⎢ sin nx + 2 cos nx ⎥
12 32 52 8 π ⎣n n ⎦0
Ans: Given that,
f(x) = x 2 ⎡⎧ π 1 ⎫ ⎧ 1 ⎫⎤
= ⎢⎨ sin nπ + 2 cos nπ⎬ − ⎨0 + 2 cos 0⎬⎥
0≤x≤π π ⎣⎩ n n ⎭ ⎩ n ⎭⎦
The half range Fourier cosine series is given by,

2 ⎡ ⎧⎪ ( −1) n ⎫⎪ ⎧ 1 ⎫ ⎤
= ⎢ ⎨0 + 2 ⎬ − ⎨ 2 ⎬ ⎥

a0
f(x) = + an cos nx ... (1) π ⎢⎣ ⎪⎩ n ⎪⎭ ⎩ n ⎭ ⎥⎦
2 n =1
[Q cos nπ = (– 1)n, sin nπ = 0]
Where,
π
2 ⎡ (−1) n − 1 ⎤
2 = ⎢ ⎥
π ⎢⎣ n 2 ⎥⎦
a0 =
π ∫ f ( x)dx
0
2
π ∴ an = [( −1) n − 1]
2 n π
2
and an =
π ∫ f ( x).cos nx dx
0
⎧0, when n = even

Now, an = ⎨ − 4
, when n = odd
⎪⎩ n 2 π
π
2
a0 =
π
x dx ∫ On substituting the values of a0 and an in equation (1),
0 we get,

SPECTRUM ALL-IN-ONE JOURNAL FOR ENGINEERING STUDENTS SIA GROUP


2.44 MATHEMATICS-II [JNTU-ANANTAPUR]

π

2
f(x) = + [(−1) n − 1] cos nx
2 n =1 n 2 π

π ⎡ −4 ⎤
x= + ∑
2 n =1,3,5 ⎢⎣ n 2 π ⎥⎦
cos nx


π 4

1
⇒ x= − cos nx
2 π n =1,3,5 n 2

π 4⎡1 1 1 1 ⎤
⇒ x= − cos x + 2 cos 3 x + 2 cos 5 x + 2 cos 7 x + ......⎥ ... (2)
2 π ⎢⎣12 3 5 7 ⎦

π 4⎡ 1 1 1 ⎤
⇒ x= − ⎢cos x + cos 3x + cos 5 x + cos 7 x + ......⎥
2 π⎣ 9 25 49 ⎦
On substituting x = 0 in equation (2), we get,
π 4⎡1 1 1 1 ⎤
⇒ 0= − ⎢ 2 cos 0 + 2 cos 0 + 2 cos 0 + 2 cos 0 + ......⎥
2 π ⎣1 3 5 7 ⎦

−4 ⎡ 1 1 1 1 ⎤ −π
⇒ + + + + ......⎥ =
π ⎢⎣12 32 52 7 2 ⎦ 2

1 1 1 1 π2
⇒ + + + + ...... =
12 32 52 72 8
Hence proved.
Q33. Find the half-range cosine series of f(x) = x(π π – x) in 0 ≤ x ≤ π .
Ans: Given that,
f(x) = x(π – x) 0 ≤ x ≤ π
i.e., f(x) = x(π – x) x∈[0, π]
The half-range cosine series expansion of f(x) in [0, π] is given by,

a0 ∞
f(x) = +
2 n =1 ∑
an cos nx ... (1)

π
2
a0 =
π ∫ f ( x)dx
0

π
2
=
π ∫
x ( π − x )dx
0

2⎡ ⎤
π π

=
π⎢ ∫
⎢ πx dx − x 2 dx ⎥
⎥ ∫
⎣0 0 ⎦

2⎡ ⎤
π π
=
π⎢ ∫
⎢π x dx − x 2 dx⎥
⎥ ∫
⎣ 0 0 ⎦

⎡ π π⎤
2 ⎢ ⎡ x 2 ⎤ ⎡ x3 ⎤ ⎥
= π⎢ ⎥ − ⎢ ⎥
π ⎢ ⎢⎣ 2 ⎥⎦ ⎢⎣ 3 ⎥⎦ ⎥
⎣ 0 0⎦

2 ⎡π 2 1 ⎤
[ π − 0] − [ π 3 − 0]⎥
π ⎢⎣ 2
=
3 ⎦

Look for the SIA GROU P LOGO on the TITLE COVER before you buy
UNIT-2 (Fourier Series) 2.45

2 ⎡ π 3 π3 ⎤
= ⎢ − ⎥
π ⎢⎣ 2 3 ⎥⎦

2 π3
= ×
π 6

π2
=
3

∴ a0 = π 2 / 3

π
2
an =
π ∫ f ( x) cos nx dx
0

π
2
=
π ∫
x( π − x ). cos nx dx
0

2⎡ ⎤
π π

= ⎢
π⎢
πx cos∫nx dx − ∫
x 2 cos nx dx ⎥

⎣0 0 ⎦

2⎡ ⎤
π π

= ⎢
π⎢
π x cos nx∫dx − ∫
x 2 cos nx dx ⎥

⎣ 0 0 ⎦
π
2 ⎡ ⎧ ⎡ sin nx ⎤ sin nx dx ⎫ ⎧ 2 ⎡ sin nx ⎤ ⎡ sin nx ⎤ ⎫⎤
= ⎢ π⎨ x ⎢
π ⎢⎣ ⎩ ⎣ n ⎦ ⎥ −
n ∫
⎬ − ⎨x ⎢
⎭ ⎩ ⎣ n ⎦
⎥ − 2( x ) ⎢
⎣ n∫ ⎥ dx ⎬⎥
⎦ ⎭⎥⎦ 0

π
2 ⎡ ⎧ x sin nx 1 ⎫ ⎧⎪ x sin nx 2
2 ⎫⎪⎤
= ⎢π⎨
π ⎣⎢ ⎩ n

n ∫
sin nx dx ⎬ − ⎨
⎭ ⎪⎩ n

n ∫
x sin nx dx ⎬⎥
⎪⎭⎦⎥
0

π
2 ⎡ ⎧ x sin nx cos nx ⎫ ⎧⎪ 2 sin nx 2 ⎧ ⎡ cos nx ⎤ ⎛ cos nx ⎞ ⎫⎫⎪⎤
= ⎢π⎨
π ⎣⎢ ⎩ n
+ ⎬ − ⎨x
n ⎭ ⎪⎩
2 n
− ⎨ x ⎢−
n⎩ ⎣ n ⎦ ⎥ − ⎜−
⎝ n ⎠ ⎭⎪⎭⎦⎥∫
⎟dx ⎬⎬⎥
0

π
2 ⎡ ⎧ x sin nx cos nx ⎫ ⎧⎪ x 2 sin nx 2 ⎧ x cos nx sin nx ⎫⎫⎪⎤
= ⎢ π⎨ + ⎬−⎨ − ⎨− + ⎬⎬⎥
π ⎢⎣ ⎩ n n 2 ⎭ ⎪⎩ n n⎩ n n 2 ⎭⎪⎭⎥⎦
0

π
2 ⎡ ⎧ x sin nx cos nx ⎫ x 2 sin nx 2 2 ⎤
= ⎢π⎨ + 2 ⎬
− − 2 x cos nx + 3 sin nx ⎥
π ⎢⎣ ⎩ n n ⎭ n n n ⎥⎦ 0

π
2 ⎡ πx sin nx π cos nx x 2 sin nx 2 2 ⎤
= ⎢ + − − 2 x cos nx + 3 sin nx ⎥
π ⎣⎢ n n 2
n n n ⎦⎥ 0

2 ⎡⎧⎪ π 2 sin nπ π cos nπ π 2 sin nπ 2 2 ⎫⎪ ⎧ π cos 0 2 ⎫⎤


= ⎢⎨ + − − π cos nπ + sin n π ⎬ − ⎨0 + − 0 − 0 + sin 0⎬⎥
π ⎢⎣⎪⎩ n n2 n n2 n3 ⎪⎭ ⎩ n2 n3 ⎭⎥⎦

2 ⎡ ⎧⎪ ( −1) n π 2π( −1) n ⎪⎫ ⎧ π ⎫⎤


= ⎢ ⎨0 + − 0 − + 0 ⎬ − ⎨ 2 ⎬⎥ [Q cos (nπ) = (– 1)n, sin (nπ) = 0]
π ⎢⎣ ⎪⎩ n2 n2 ⎭⎪ ⎩ n ⎭⎥⎦

SPECTRUM ALL-IN-ONE JOURNAL FOR ENGINEERING STUDENTS SIA GROUP


2.46 MATHEMATICS-II [JNTU-ANANTAPUR]

2 ⎡ − π(−1) n π ⎤
= ⎢ − 2⎥
π ⎢⎣ n 2 n ⎥⎦
2
= 2 [ − π{( −1) + 1}]
n
n π
−2
= [(−1) n + 1]
n2
⎧0 ; if n is odd

∴ an = ⎨ − 4
⎪ 2 ; if n is even
⎩n
On substituting the values of a0 and an in equation (1), we get,
π2 / 3 π2
f(x) = x(π – n) = + when ‘n’ is odd
2 6

π2 / 3
∑− n
4
⇒ f(x) = x(π – x) = + 2
cos nx when ‘n’ is even
2 n =1


π2

cos nx
= −4 2
6 n =1 n

π2 ⎧ 1 1 ⎫
∴ f (x) = − 4⎨cos x + cos2x + cos3x +.....⎬
6 ⎩ 4 9 ⎭
Q34. Find the half-range cosine series of f(x) = x(2 – x) in 0 ≤ x ≤ 2.
Ans: Given that,
f(x) = x(2 – x) 0 ≤ x ≤ 2
The half-range cosine series is given by,

nπx

a0
f (x) = + an cos ... (1)
2 n =1 L
Where,
L
2
a0 =
L ∫ f ( x)dx
0

And
L
2 ⎛ nπx ⎞
an =
L ∫ f ( x) cos⎜⎝ L ⎠
⎟ dx
0

Here, L =2
2 2 2
2
a0 =
2 ∫
x( 2 − x ) dx = ∫ ∫
2 xdx − x 2 dx
0 0 0

2 2
⎡ x 2 ⎤ ⎡ x3 ⎤ 1 3
= 2⎢ ⎥ − ⎢ ⎥ = [2 − 0] − [2 − 0]
2

⎣⎢ ⎦⎥ 0 ⎣⎢ ⎦⎥ 0
2 3 3

8 4
= 4− =
3 3

4
∴ a0 =
3

Look for the SIA GROU P LOGO on the TITLE COVER before you buy
UNIT-2 (Fourier Series) 2.47
2 2
2 ⎛ nπx ⎞ ⎛ nπx ⎞
∫ ∫ (2 x − x
2
an = x ( 2 − x ) cos ⎜ ⎟ dx = ) cos ⎜ ⎟ dx
2 ⎝ 2 ⎠ ⎝ 2 ⎠
0 0

2 2
⎛ nπx ⎞ ⎛ nπx ⎞
∫ ∫
2
= 2 x cos ⎜ ⎟ dx − x cos ⎜ ⎟ dx
⎝ 2 ⎠ ⎝ 2 ⎠
0 0

2 2
⎡ ⎡ ⎛ nπx ⎞ ⎤ ⎛ nπx ⎞ ⎤ ⎡ ⎡ ⎛ nπx ⎞ ⎤ ⎛ nπx ⎞ ⎤
⎢ ⎢ sin ⎜ ⎟⎥ sin ⎜ ⎟ ⎥ ⎢ ⎢ sin ⎜ ⎟⎥ sin ⎜ ⎟ ⎥
⎝ ⎠
2 ⎥ ⎝ 2 ⎠ ⎝ ⎠
2 ⎥ ⎝ 2 ⎠ ⎥
= 2⎢ x ⎢ − ∫ dx ⎥ − ⎢ x 2 ⎢ − 2( x ) dx∫
⎢ ⎢ nπ / 2 ⎥ nπ / 2 ⎥ ⎢ ⎢ nπ / 2 ⎥ nπ / 2 ⎥
⎢ ⎢ ⎥ ⎥ ⎢ ⎢ ⎥ ⎥
⎢⎣ ⎣ ⎦ ⎥⎦ 0 ⎢⎣ ⎣ ⎦ ⎥⎦ 0

2 2
⎡ 2 ⎛ nπx ⎞ 2 ⎛ nπx ⎞ ⎤ ⎡ 2 2 ⎛ nπx ⎞ 4 ⎛ nπx ⎞ ⎤
= 2⎢ x sin ⎜
⎣ nπ
⎟−
⎝ 2 ⎠ nπ
sin⎜ ∫
⎟ dx ⎥ − ⎢ x sin⎜
⎝ 2 ⎠ ⎦ 0 ⎣ nπ
⎟−
⎝ 2 ⎠ nπ
x sin ⎜ ⎟ dx
⎝ 2 ⎠ ⎥⎦ 0 ∫
2 ⎡ 2⎤
⎡ ⎡ ⎛ nπx ⎞ ⎤ ⎤ ⎢ ⎧ ⎡ ⎛ nπx ⎞ ⎤ ⎛ nπx ⎞ ⎫ ⎥
⎢ − ⎜ ⎟ − ⎜ ⎟ − ⎜ ⎟
⎝ 2 ⎠ ⎥ ⎥⎥ ⎢ 2 2 ⎛ nπx ⎞ 4 ⎪⎪ ⎢ ⎝ 2 ⎠ ⎪⎪ ⎥
⎢ cos cos cos
2 ⎛ nπx ⎞ 2 ⎢ ⎝ 2 ⎠⎥
= 2⎢ x sin ⎜
⎢ nπ
⎟−
⎝ 2 ⎠ nπ ⎢ nπ / 2 ⎥ ⎥
⎥ −



x sin ⎜ ⎟− ⎨ x⎢
⎝ 2 ⎠ nπ ⎪ ⎢ nπ / 2 ⎥
⎥−
nπ / 2 ∫
dx⎬ ⎥
⎪ ⎥
⎢ ⎢ ⎥⎥ ⎢ ⎪ ⎢ ⎥ ⎪⎭ ⎥
⎣ ⎣ ⎦ ⎦0 ⎣ ⎩ ⎣ ⎦ 0⎦

2 2
⎡ 2 ⎛ nπx ⎞ 4 ⎛ nπ x ⎞ ⎤ ⎡ 2 2 ⎛ nπ x ⎞ 4 ⎧ − 2 x ⎛ nπ x ⎞ 2 ⎛ nπ x ⎞ ⎫ ⎤
= 2 ⎢ x sin ⎜
⎣ n π ⎝ 2
⎟ + 2 2 cos ⎜
⎠ n π ⎝ 2
⎟ ⎥ − ⎢ x sin ⎜
⎠ ⎦0 ⎣ n π ⎝ 2
⎟−
⎠ n π

⎩ n π
cos ⎜
⎝ 2
⎟+
⎠ n π
cos ⎜
⎝ 2 ⎠ ⎭⎦ ∫
⎟ dx ⎬ ⎥
0

2
⎡ ⎧ ⎡ ⎛ nπx ⎞ ⎤⎫ ⎤
2 ⎢ ⎪ sin ⎜ ⎟ ⎪⎥
⎡2 ⎛ nπx ⎞ 4 ⎛ nπx ⎞ ⎤ ⎢ 2 2 ⎛ nπx ⎞ 4 ⎪ − 2 ⎛ nπx ⎞ 2 ⎢⎢ ⎝ 2 ⎠ ⎥⎥⎪ ⎥
= 2 ⎢ x sin ⎜ ⎟ + cos ⎜ ⎟ − x sin ⎜ ⎟ − ⎨ x cos ⎜ ⎟ + ⎬
⎣ nπ ⎝ 2 ⎠ n2π 2 ⎝ 2 ⎠ ⎥⎦ 0 ⎢ nπ ⎝ 2 ⎠ nπ ⎪ nπ ⎝ 2 ⎠ n π ⎢ n π / 2 ⎥⎪ ⎥
⎢ ⎢ ⎥⎪ ⎥
⎢⎣ ⎩⎪ ⎣ ⎦⎭ ⎥⎦ 0

2
⎡ 4 ⎛ nπx ⎞ 8 ⎛ nπx ⎞ 2 2 ⎛ nπx ⎞ 8 ⎛ nπx ⎞ 16 ⎛ nπx ⎞ ⎤
=⎢ x sin⎜ ⎟ + 2 2 cos⎜ ⎟− x sin ⎜ ⎟ − 2 2 x cos ⎜ ⎟ + 3 3 sin ⎜ ⎟
⎣ nπ ⎝ 2 ⎠ n π ⎝ 2 ⎠ nπ ⎝ 2 ⎠ n π ⎝ 2 ⎠ n π ⎝ 2 ⎠ ⎥⎦ 0

2
⎡⎧ 4 2 2 16 ⎫ ⎛ nπx ⎞ ⎧ 8 8 ⎫ ⎛ nπx ⎞ ⎤
= ⎢⎨ x − x + 3 3 ⎬ sin ⎜ ⎟ +⎨ 2 2 − 2 2 x⎬ cos ⎜ ⎟⎥
⎣⎩ nπ nπ n π ⎭ ⎝ 2 ⎠ ⎩n π n π ⎭ ⎝ 2 ⎠ ⎦0

2
⎡⎧ 2 ⎛ 8 ⎞ ⎛ nπx ⎞ ⎫ ⎧⎛ 8 ⎞ ⎛ nπx ⎞ ⎫⎤
= ⎢⎨ ⎜ 2 x − x 2 + 2 2 ⎟ sin ⎜ ⎟ ⎬ + ⎨⎜ 2 2 (1 − x)⎟ cos ⎜ ⎟ ⎬⎥
⎢⎣⎩ nπ ⎝ n π ⎠ ⎝ 2 ⎠ ⎭ ⎩⎝ n π ⎠ ⎝ 2 ⎠ ⎭⎥⎦
0

⎡⎧ 2 ⎧ 2 8 ⎫ ⎛ nπ( 2) ⎞ ⎫ ⎧ ⎧ 8 ⎫ ⎛ nπ( 2) ⎞ ⎫ ⎤
= ⎢ ⎨ ⎨2(2) − 2 + 2 2 ⎬ sin ⎜ ⎟ ⎬ + ⎨ ⎨ 2 2 (1 − 2)⎬ cos ⎜ ⎟ ⎬⎥
⎣⎢ ⎩ nπ ⎩ n π ⎭ ⎝ 2 ⎠ ⎭ ⎩⎩ n π ⎭ ⎝ 2 ⎠ ⎭ ⎦⎥

⎡⎧ 2 ⎛ 8 ⎞ ⎛ nπ(0) ⎞ ⎫ ⎧⎛ 8 ⎞ ⎛ nπ(0) ⎞ ⎫⎤
− ⎢⎨ ⎜ 2(0) − 0 + 2 2 ⎟ sin ⎜ ⎟ ⎬ + ⎨⎜ 2 2 (1 − 0)⎟ cos ⎜ ⎟ ⎬⎥
⎣⎢⎩ nπ ⎝ n π ⎠ ⎝ 2 ⎠ ⎭ ⎩⎝ n π ⎠ ⎝ 2 ⎠ ⎭⎦⎥

⎡⎧ 16 8 ⎫ ⎧ 16 8 ⎫⎤
= ⎢⎨ 3 3 sin(nπ) − 2 2 cos( nπ)⎬ − ⎨ 3 3 sin(0) + 2 2 cos(0)⎬⎥
⎣⎩ n π n π ⎭ ⎩n π n π ⎭⎦

SPECTRUM ALL-IN-ONE JOURNAL FOR ENGINEERING STUDENTS SIA GROUP


2.48 MATHEMATICS-II [JNTU-ANANTAPUR]

8 8
= 0− (−1) n − 0 − ×1 [Q sin nπ = 0, cos nπ = (–1)n]
n π 2 2
n π22

−8
= 2 2
[1 + ( −1) n ]
n π

⎧0 ; if n is odd

∴ an = ⎨ − 16 ; if n is even
⎪⎩ n 2 π 2

On substituting the values of a0 and an is equation (1), we get,

4

−8 ⎛ nπx ⎞
3
x(2 – x) = +
2 ∑n π
n =1
2 2
[1 + ( −1) n ] cos ⎜
⎝ 2 ⎠


− 16 ⎛ nπx ⎞

4
= + cos ⎜ ⎟ [Considering for even values of n]
6 n =1 n π
2 2 ⎝ 2 ⎠


⎛ nπx ⎞
∑n
2 16 1
= 3− 2 cos ⎜
⎝ 2 ⎠

π n =1
2

2 16 ⎡1 ⎛ 2πx ⎞ 1 ⎛ 4πx ⎞ 1 ⎛ 6πx ⎞ ⎤


= − ⎢ 2 cos ⎜⎝ 2 ⎟⎠ + 2 cos⎜⎝ 2 ⎟⎠ + 2 cos ⎜⎝ ⎟ + .....⎥
3 π2 ⎣2 4 6 2 ⎠ ⎦

2 16 ⎡ 1 1 1 ⎤
= − 2 ⎢ cos πx + cos 2πx + cos 3πx + .....⎥
3 π ⎣4 16 36 ⎦
Q35. Find the half-range sine series of f(x) = x defined in the interval (0, 2).

Ans: Given range is, 0 < x < 2 ⎫ for the function f(x) = x

Extended rangeis, −2< x<0⎭
The new function is symmetrical about the origin and therefore, represents on odd function in (–2, 2)
y

x1 M L
x
−2 −1 0 1 2

y'
Figure
Hence, the fourier series for f(x) over the full period (–2, 2) will contain only sine terms given by,

n πx
f(x) = ∑b
n =1
n sin
2
[Ql = 2]

2
2 n πx
Where, bn = f (x )sin
∫ dx
20 2

Look for the SIA GROU P LOGO on the TITLE COVER before you buy
UNIT-2 (Fourier Series) 2.49
2
nπx ⎡Q sin nπ = 0, sin 0 = 0 ⎤
= ∫
0
x sin
2
dx ⎢ n ⎥
⎣cos nπ = ( −1) , cos 0 = 1⎦

2
⎡ ⎛ ⎞⎤
⎢ ⎛⎜ − cos nπx ⎞⎟ ⎜ − sin nπx ⎟⎥
= ⎢ x⎜ 2 ⎟ − 1⎜ 2 ⎟⎥
⎢ ⎜ ⎜ ⎟
nπ ⎟ ⎜ ⎛ nπ ⎞ 2 ⎟⎥
⎢ ⎜ ⎟ ⎥
⎢⎣ ⎝ 2 ⎠ ⎜ ⎜⎝ 2 ⎟⎠ ⎟⎥
⎝ ⎠⎦ 0

2
⎡ − 2x nπx 4 nπx ⎤
=⎢ cos + 2 2 sin
⎣ nπ 2 n π 2 ⎥⎦ 0

2
−2⎡ nπx 2 nπx ⎤
x cos − sin
nπ ⎢⎣ 2 ⎥⎦ 0
=
2 nπ

−2 ⎡ 2nπ 2 2 πn ⎤
⎢ 2 cos − sin − [0 − 0]
2 ⎥⎦
=
nπ ⎣ 2 nπ

−2
= [2 cos nπ − cos 0] + 24 2 [sin nπ − sin 0]
nπ n π

−4
= cos nπ [Q sin nπ = sin (0) = 0]

4(− 1)
n
bn = −

If n = 1, 2, 3, ........

4 −4 4 −4
b1 = , b2 = , b3 = , b4 = etc.
π 2π 3π 4π

nπx
∴ f(x) = ∑b
n =1
n sin
2

πx 2πx 3πx 4πx


= b1 sin + b2 sin + b3 sin + b4 sin +…
2 2 2 2

4 πx 4 2πx 4 3πx 4 4πx


x= sin − sin + sin − sin +…
π 2 2π 2 3π 2 4π 2

4 ⎡ πx 1 2πx 1 3πx 1 4πx ⎤


sin − sin + sin − sin + …⎥
π ⎢⎣
=
2 2 2 3 2 4 2 ⎦
Q36. Obtain sine series for f(x) = π x – x2 in 0 < x < π .
Ans: Given that,
f (x) = πx – x2

The sine series is ∑b
n =1
n sin nx in (0, π)

SPECTRUM ALL-IN-ONE JOURNAL FOR ENGINEERING STUDENTS SIA GROUP


2.50 MATHEMATICS-II [JNTU-ANANTAPUR]
Where,
π π
2 ⎡
π π ⎤
2 2
bn =
π ∫ f ( x). sin nx.dx =
π ∫ (πx − x 2 ). sin nx.dx =
π ⎢∫ ∫
⎢ πx sin nx.dx − x 2 .sin nx.dx ⎥
⎥⎦
0 0 ⎣0 0
Applying integration by parts, we get,

2 ⎧⎪ ⎡ ⎛ − cos nx ⎞ π
⎛ − cos nx ⎞ ⎤ ⎡ 2 ⎛ − cos nx ⎞ ⎛ − cos nx ⎞ ⎤ ⎫⎪
π

=
π
π
⎨ ⎢ x⎜
⎪⎩ ⎣ ⎝ n ⎠
⎟ − 1⎜ ∫ ⎟dx⎥ ⎢
⎝ n ⎠ ⎦0 ⎣ ⎝ n ⎠
− x ⎜ ⎟ − 2 x⎜ ∫ ⎟
⎝ n ⎠ ⎦ 0 ⎪⎭
dx⎥ ⎬

⎧ π π⎫
2 ⎪⎡ cos nx sin nx ⎤ ⎡ 2 cos nx 2 ⎛ ⎛ − sin nx ⎞ ⎛ − sin nx ⎞ ⎞⎤ ⎪
=
π
⎨⎢ − π x
⎪⎩⎣ n
+ π 2 ⎥ − ⎢− x
n ⎦ 0 ⎢⎣ n
− ⎜⎜ x⎜
n⎝ ⎝ n ⎠
⎟ − 1⎜
⎝ n ∫
dx ⎟ ⎟⎟⎥ ⎬
⎠ ⎠⎥⎦ 0 ⎪

⎧ π
⎡ 2 cos nx 2 ⎛
π⎫
2 ⎪⎡ cos nx sin nx ⎤ sin nx ⎛ cos nx ⎞ ⎞⎤ ⎪
= ⎨⎢ − π x + π 2 ⎥ − ⎢− x − ⎜⎜ − x − ⎜ 2 ⎟ ⎟⎟⎥ ⎬
π ⎪⎣ n n ⎦ 0 ⎢⎣ n n⎝ n ⎝ n ⎠ ⎠⎥⎦ 0 ⎪
⎩ ⎭
⎧⎪⎡ π π
2 cos nx sin nx ⎤ ⎡ 2 cos nx 2 2 ⎤ ⎫⎪
= ⎨⎢ − π x + π ⎥ − ⎢ − x + x sin nx + cos nx ⎥ ⎬
π ⎪⎩⎣ n n2 ⎦0 ⎣ n n2 n3 ⎦ 0 ⎪⎭
π
2 ⎡ cos nx sin nx 2 cos nx sin nx 2 cos nx ⎤
=
π ⎢− π x n + π 2 + x − 2x − ⎥
⎣ n n n2 n3 ⎦0
π
2 ⎡ 2 ⎛ − cos nx ⎞ ⎛ − sin nx ⎞ ⎛ cos nx ⎞⎤
= ⎢(π x − x )⎜ ⎟ − ( π − 2 x )⎜ ⎟ + (−2)⎜ 3 ⎟⎥
π ⎣ ⎝ n ⎠ ⎝ n 2
⎠ ⎝ n ⎠⎦ 0

2 ⎧⎪⎡ 2 2 ⎛ − cos nπ ⎞ ⎛ − sin nπ ⎞ ⎛ cos nπ ⎞⎤ ⎡ ⎛ − sin 0 ⎞ cos 0 ⎤ ⎫⎪


= ⎨⎢(π − π )⎜ ⎟ − (π − 2π)⎜ ⎟ − 2⎜ ⎟⎥ – ⎢0 − (π − 2(0))⎜ ⎟ − 2 3 ⎥⎬
π ⎪⎩⎣ ⎝ n ⎠ ⎝ n 2 3
⎠ ⎝ n ⎠⎦ ⎣ ⎝ n 2
⎠ n ⎦ ⎪⎭

2 ⎧⎡0 ( )(0) 2 cos nπ ⎤ ⎡0 2 ⎤ ⎫ 2 ⎧− 2 cos nπ + 2 ⎫


= ⎨⎢ − − π − ⎥ − ⎢ − ⎥⎬ = ⎨ 3 ⎬
π ⎩⎣ n 3 ⎦ ⎣ n 3 ⎦⎭ π ⎩ n n3 ⎭
2 2 4
= ×
π n3
{1 − cos nπ}= 3 [1 – (– 1)n]
πn
⎧0; if n is even

∴ bn = ⎨ 8
⎪ 3 ; if n is odd
⎩ πn
Thus, f(x) = πx – x2

∑ πn
8
f ( x) = 3
. sin nx
n =1
or
8⎛ sin 3x sin 5x ⎞
f ( x) = ⎜ sin x + 3 + 3 + ...⎟
π⎝ 3 5 ⎠
2.5 PARSEVAL’S FORMULA-COMPLEX FORM OF FOURIER SERIES
l
⎡1 ∞ ⎤
Q37. Prove that ∫ [f(x)]2 dx = l⎢ a 20 + ∑
(anna + b bn )⎥ provided the Fourier series for f(x) converges uniformly in
–l ⎣⎢ 2 n =1 ⎦⎥
(– l, l).
OR
Derive the Parseval’s formula. Model Paper-II, Q5
Ans:
Given that,
The Fourier series of f(x) converges uniformly in (– l, l)

Look for the SIA GROU P LOGO on the TITLE COVER before you buy
UNIT-2 (Fourier Series) 2.51
The Fourier expansion of f(x) in (– l, l) is given as,

a0 ⎛ nπx nπx ⎞
f(x) =
2
+ ∑ ⎝⎜ a
n =1
n cos
l
+ bn sin
l ⎠⎟
... (1)

Where,
l
1
a0 =
l
–l
∫ f ( x) dx ... (2)

l
1 ⎛ nπ x ⎞
an =
l ∫ f ( x ) cos ⎜⎝ l ⎠
⎟ dx and ... (3)
–l

l
1 ⎛ nπx ⎞
bn =
l ∫ f ( x) sin⎜⎝
−l
l ⎠
⎟dx ... (4)

On multiplying equation (1) by f(a) and integrating term by term from – l to l, we get,
l
∞ ⎡
⎛ nπx ⎞ ⎤⎥
l l l
⎛ nπx ⎞

−l
f ( x). f ( x) dx = ao
2 ∫ n =1⎢ ∫
f ( x)dx + Σ ⎢a n f ( x ) cos⎜
⎝ l ⎠ ∫
⎟dx + bn f ( x ) sin⎜

⎟dx
l ⎠ ⎥
... (5)
−l ⎣ −l −l ⎦
On substituting equations (2), (3) and (4) in equation (5), we get,
l
ao ∞
= ∫ [ f ( x )]2 dx = [ a o L] + Σ [a n ( a n l ) + bn (bn l ) ]
2 n =1
−l

l ∞ 2
2
= a0 + Σ [an l + bn2 l ]
2 n =1

⎡ a02 ∞ 2 ⎤
= ⎢ + Σ (an + bn )⎥
2
l
⎣⎢ 2 n =1 ⎦⎥

l
⎡ a2 ∞ ⎤

∴ [ f ( x )]2 dx = l ⎢ 0 + Σ ( an2 + bn2 ) ⎥
⎢⎣ 2 n =1 ⎥⎦
−l

Hence proved.
Q38. Derive the complex form of Fourier series.
Ans: The Fourier series expansion of a periodic function f(x) of period 2l is given as,

a0 ∞ ⎡ ⎛ nπx ⎞ ⎛ nπx ⎞⎤
f(x) = + Σ ⎢an cos⎜ ⎟ + bn sin⎜ ⎟⎥
2 n =1⎣ ⎝ l ⎠ ⎝ l ⎠⎦

l
1
Where, a0 =
l ∫ f ( x)dx
−l

l
1 ⎛ nπx ⎞
an =
l ∫ f ( x) cos⎜⎝
−l
l ⎠
⎟ dx and

l
1 ⎛ nπx ⎞
bn =
l ∫ f ( x) sin⎜⎝
−l
l ⎠
⎟ dx

SPECTRUM ALL-IN-ONE JOURNAL FOR ENGINEERING STUDENTS SIA GROUP


2.52 MATHEMATICS-II [JNTU-ANANTAPUR]

⎡ ⎛ jnπx −
jnπx ⎞ ⎛ jnπx jnπx ⎞⎤
a0 ∞ ⎢ ⎜ e l + e l ⎟ ⎜ e l − e− l ⎟⎥ ⎛ jθ − jθ jθ jθ ⎞
+ Σ ⎢an ⎜ ⎟ + bn ⎜ ⎟⎥ ⎜Q cos θ = ⎛⎜ e + e ⎞
⎟ sin θ = e − e ⎟
f(x) =
2 n =1⎢ ⎜ ⎜ ⎜ ⎟ ⎟

2 ⎟⎟ ⎜⎜ 2i ⎟⎟⎥ ⎝ ⎝ 2 ⎠1 2j

⎣⎢ ⎝ ⎠ ⎝ ⎠⎦⎥
jnπ x jnπ x jnπ x jnπ x
a0 ∞ ⎡ an l an − l bn l bn − l ⎤
⇒ + Σ ⎢
f(x) = 2 n=1 2 e + e + e − e ⎥
⎣⎢ ⎦⎥
2 2j 2j

a0 ∞ ⎡ l ⎡ an bn ⎤ − l ⎡ an bn ⎤ ⎤
jnπx jnπx
+ Σ
= 2 n =1⎢ e ⎢ + ⎥ + e ⎢ − ⎥⎥
⎢⎣ ⎣ 2 2j⎦ ⎣ 2 2 j ⎦ ⎥⎦

a0 ∞ ⎡⎛ an − jbn ⎞ l ⎛ an + jbn ⎞ − l ⎤
jnπx jnπx
+ Σ
f(x) = 2 n =1⎢ ⎜ ⎟e + ⎜ ⎟e ⎥
⎢⎣⎝ 2 ⎠ ⎝ 2 ⎠ ⎥⎦

a0 1 1
Let, C0 = , Cn = (a – jbn); C–n = (a + jbn)
2 2 n 2 n
On substituting the corresponding values in above equation, we get,

∞ ⎡ jnπ x jnπ x ⎤

f(x) = C0 + Σ ⎢Cne l + C−ne l ⎥ ... (1)
n=1 ⎢ ⎥⎦

The value of ‘Cn’ can be calculated as,
1
Cn = (an − jbn ) ... (2)
2
On substituting the value of ‘an’ and ‘bn’ in equation (2), we get,

1 ⎡1 ⎛ nπx ⎞ ⎤⎥
l l
⎛ nπx ⎞ 1
Cn = ⎢
2 ⎢l ∫
f ( x) cos⎜
⎝ l ⎠
⎟dx − j
l
f ( x ) sin⎜ ⎟dx
⎝ l ⎠ ⎥⎦ ∫
⎣ −l −l

1⎡ ⎛ nπ x ⎞ ⎤⎥
l
⎛ nπ x ⎞

= 2l ⎢ cos ⎜ ∫
⎝ l ⎠ ⎟ − j sin ⎜⎝ l ⎟⎠ f ( x)dx

⎣ −l ⎦
l nπx
1 −j
= 2l ∫
−l
e l .f ( x ) dx [Q e–iθ = cosθ – sinθ]

l − jnπ x
1
⇒ Cn =
2 ∫ f ( x)e l .dx
... (3)
−l

On substituting n = – n in equation (3), we get,


l jnπx
1
Cn =
2l ∫
−l
f ( x )e l .dx
... (4)

From equation (3) and (4), we get,


l jnπx
1 −
Cn = 2l ∫
−l
f ( x) e l .dx
for n = 0, ± 1, ± 2, +3,....

∴ Equation (1) can be written as,

∞ jnπx
f ( x) = Σ C n e l
n = −∞

This is the required complex from of Fourier series.

Look for the SIA GROU P LOGO on the TITLE COVER before you buy
UNIT-2 (Fourier Series) 2.53
Q39. Find the complex form of the Fourier series of f(x) = e–x in − |≤ x ≤| .

Ans: Given function is,

f(x)= e–x

Interval, −| ≤ x ≤ |

Complex form of Fourier series = ?

The complex form of fourier series is given as,

∞ jnπx
f(x) = Σ C n e l ... (1)
n = −∞

Here,

l = 1 and

l − jnπx
1
Cn =
2l
−l
∫ f ( x )e l dx

The value of Cn can be obtained as,

l
1
∫e
− x − jnπx
Cn = e dx
2l
−l

l
1

− (1+ jnπ )
= 2 e dx
−l

1
1 ⎡ e −(1+ jnπ) ⎤
= ⎢ ⎥
2 ⎢⎣ − (1 + jnπ) ⎥⎦
−1

1 ⎡ e −(1+ jnπ) + e (1+ jnπ) ⎤


= ⎢− ⎥
2 ⎣⎢ 1 + jnπ ⎦⎥

e − (1+ jnπ) − e − (1+ jnπ)


=
2(1 + jnπ)

e1.e jnπ − e −1.e − jnπ


=
2(1 + jnπ)

e[cos nπ + j sin nπ] − e −1[cos nπ − j sin nπ]


= [Q cos nπ = (–1)n, sin nπ = 0]
2(1 + jnπ)

e(−1) n − e −1 (−1) n
=
2(1 + jnπ)

⎡ e − e −1 ⎤ (−1) n ×1 − jnπ
= ⎢ 2 ⎥ (1 + jnπ)(1 − jnπ)
⎢⎣ ⎥⎦

SPECTRUM ALL-IN-ONE JOURNAL FOR ENGINEERING STUDENTS SIA GROUP


2.54 MATHEMATICS-II [JNTU-ANANTAPUR]

⎛ −x ⎞
⎜Q sinh x = e − e
x
(−1) n (1 − jnπ) sinh1 ⎟
= ⎜ 2 ⎟
1+ n π2 2
⎝ ⎠

(−1) n (1 − jnπ) sinh 1


∴ Cn =
1+ n2π2

On substituting the corresponding values in equation (1), we get,

∞ ⎡ ( −1) n (1 − jnπ) sinh1 ⎤


jnπx
e–x = n =Σ−∞ ⎢ ⎥e , which is required complex fourier series.
⎣⎢ 1+ n π2 2
⎦⎥

Look for the SIA GROU P LOGO on the TITLE COVER before you buy
UNIT-3 (Fourier Transform) 3.1

UNIT FOURIER TRANSFORM

3
PART-A
SHORT QUESTIONS WITH SOLUTIONS
Q1. State Fourier integral theorem. Model Paper-I, Q1(e)
Ans: The Fourier integral theorem of f(x) is defined as,
∞ ∞
1
f(x) =
π ∫ ∫ f (t ) cos λ(t − x)dt.dλ
0 −∞

Q2. Define Fourier sine integral and cosine integral.


Ans:
Fourier Sine Integral: The Fourier sine integral of f(x) is defined as,
∞ ∞
2
f(x) =
π
0
∫ 0

sin λx f (t ) sin λ t dt dλ

Fourier Cosine Integral: The Fourier cosine integral of f(x) is defined as,
∞ ∞
2
f(x) =
π ∫
0

cos λx f (t ) cos λ t dt dλ
0


1 − cos πλ π
Q3. Using Fourier integral show that, ∫
0
λ
λ )dλ
sin(xλ λ =
2
, 0 < x < π. Model Paper-III, Q1(e)

π
Ans: Let, f (x) = , 0 < x < π
2
Applying Fourier sine integral to f (x),
∞ ∞
2
f (x) =
π ∫ 0
sin( λ x ) dλ ∫ f (t ) sin λtdt
0
∞ π
2
=
π ∫ sin( λ x) d λ ∫ 1sin(λt ) dt
0 0

∞ π
2
sin( λ x ) d λ ⎡
− cos(λt ) ⎤
=
π ∫
0

⎣ λ ⎥
⎦0

2 1 − cos( λπ )
=
π ∫
0
λ
sin (λx)dλ ... (1)


1 − cos( λπ ) π
= ∫
0
λ
sin (λx) dλ =
2
π
∴ f ( x) = [0 ≤ x ≤ π]
2

SPECTRUM ALL-IN-ONE JOURNAL FOR ENGINEERING STUDENTS SIA GROUP


3.2 MATHEMATICS-II [JNTU-ANANTAPUR]
Q4. Define the Fourier transform of function f(x). Q7. Define Fourier sine and cosine transforms of
Ans: The Fourier transform of a function f(x) is defined as, f(x).
Ans:

Fourier Sine Transform: The Fourier sine transform of the
∫ f ( x) e
isx
F(s) = F.T{f(x)} = dx function f(x) is,
−∞

Q5. State the properties of Fourier transform.
Ans: Model Paper-II, Q1(e)
Fs(S) = FST{f(x)} = ∫ f ( x) sin sx dx
0
The properties of Fourier transforms are as follows, Fourier Cosine Transform: The Fourier cosine transform of
(a) Linear Property: If f(x) and g(x) are two functions whose the function f(x) is defined as,
Fourier transform is given by, F(s) and G(s) respectively, then, ∞
F[a f(x) + b g(x)] = a. F(s) + b G(s)
Where, a and b are arbitrary constants.
Fc(S) = FCT {f(x)} = ∫ f ( x) cos sx dx
0
(b) Scaling Property: If f(x) is a function whose complex Q8. Find Fourier cosine transform of,
Fourier transform is given by F(s),
⎧cosx, 0 < x < a
Then, f(x) = ⎨ .
⎩ 0, x≥a
1 ⎛s⎞
F{f(ax)} = F⎜ ⎟ Ans: Given that,
a ⎝a⎠
f (x)= cos x, 0 < x < a
Where, a ≠ 0. = 0, x ≥ a
(c) Shifting Property: If a function f(x) has a complex Fourier Fourier cosine transform of f (x) is,
transform F(s),

Then, 1
F{f(x–a)} = eisa.F(s) Fc [f (x)] =
2 ∫ f ( x ) cos sx dx
0
(d) Modulation Property: If a function f (x) has a complex
Fourier transform F(s), then, ⎡a ∞

1
1 = ⎢∫ f ( x) cos sx dx + ∫ f ( x ) cos sx dx ⎥
F{f(x) cos ax}= [F(s + a) + F(s – a)] 2 ⎢⎣ 0 a ⎥⎦
2
Q6. Find the Fourier transform of, 1 ⎡ ⎤
a

= ⎢∫ cos x cos sx dx + 0 ⎥
2 ⎣⎢ 0 ⎦⎥
⎧⎪e ikx , a<x <b
f(x) = ⎨
⎪⎩0, x < a and x > b
1 ⎡ ⎤
a

= ⎢∫ cos(1 + s) x + cos (1− s ) xdx⎥


Ans: By the definition, 2 ⎣⎢ 0 ⎦⎥
F{f(x)} = F(s)
1 ⎡a a ⎤
∞ = ⎢∫ cos(1 + s) x dx + ∫ cos (1 − s) x dx⎥
2
∫ f ( x) . e ⎢⎣ 0 ⎥⎦
isx
= dx 0

−∞
1 ⎡ sin(1 + s) x sin(1 − s) x ⎤
a
= +
2 ⎢⎣ 1 + s 1 − s ⎥⎦ 0
⎡ a b ∞ ⎤
= ⎢ ∫ f ( x ) .e isx dx + ∫ f ( x ) e isx dx + ∫ f ( x) . e isx dx⎥
⎢ ⎥
⎣ −∞ a b ⎦
1 ⎡ sin(1 + s)a sin(1 − s )a ⎤
+
2 ⎢⎣ 1 + s 1 − s ⎥⎦
=
⎡ b ⎤

= ⎢0 + e . e dx + 0⎥
ikx isx
⎢ a ⎥ Q9. Find Fourier sine transform series of xe–ax.
⎣ ⎦
Ans: Model Paper-I, Q1(f)
b To find the Fourier sine transform of xe–ax
b
ei (k + s) x Fourier sine transform is given by,
∫e
ikx isx
= .e dx =
i( k + s )
a a −d
Fs[x f (x)] = Fc [ f ( x)]
ds
e i ( k + s )b − e i ( k + s ) a
= ∞
i (k + s )
Fc [ f ( x )] = ∫ f ( x) cos sxdx
0

Look for the S IA GROUP LOGO on the TITLE COVER before you buy
UNIT-3 (Fourier Transform) 3.3

∫e
− ax
Fs (e–ax) = cos sxdx
0

a
Fs (e–ax) =
s + a2
2

−d ⎛ a ⎞
Fs (xe–ax) = ⎜ ⎟
ds ⎝ s 2 + a 2 ⎠

⎛ − 2s ⎞
= − a⎜ ⎟
⎜ 2
⎝ s +a
2
( )
2 ⎟

2as
(s )
= 2
2
+ a2

1
Q10. Find Fourier sine transform of . Model Paper-II, Q1(f)
x

Ans: Given that,

1
f (x) =
x
Fourier sine transform of f (x) is,


2
Fs[f(x)] =
π ∫ f ( x) sin sx dx
0


⎡1 ⎤ 2 1
Fs ⎢ ⎥ =
⎣x⎦ π ∫ x sin sx dx
0

dt
Let, sx = t ⇒ s dx = dt ⇒ dx =
s


2 s dt
=
π ∫ t sin t
0
s


2 sin t
=
π ∫
0
t
dt

2 π
=
π 2

π ⎡ ∞ sin t π⎤
=
2
⎢Q ∫
⎢⎣ 0 t
dt = ⎥
2 ⎥⎦

SPECTRUM ALL-IN-ONE JOURNAL FOR ENGINEERING STUDENTS SIA GROUP


3.4 MATHEMATICS-II [JNTU-ANANTAPUR]
–ax
Q11. Find the Fourier cosine transforms of e sin ax.
Ans: Fourier cosine transform for f (x) is,
∞ ∞

∫ f ( x) cos sx dx = ∫ e
− ax
Fc (S) = sin ax cos sx dx
0 0

∞ ∞
1 1 − ax
=
20 ∫
[e − ax sin(a + s ) x ]dx +
20 ∫
e sin( a − s ) xdx [Q 2sinA cosB = sin(A + B) + sin(A – B)]

∞ ∞
1⎡ e − ax ⎤ 1⎡ e − ax ⎤
= ⎢ 2 [ −a sin( a + s ) x − ( a + s ) cos( a + s ) x]⎥ + ⎢ 2 [−a sin( a − s) x − (a − s) cos( a − s ) x]⎥
2 ⎣ a + (a + s) 2
⎦0 2 ⎣ a + ( a − s) 2
⎦0

⎡ e − ax ⎤

−ax
⎢Q e sin bx dx = 2 2
(− a sin bx − b cos bx) ⎥
⎣⎢ a +b ⎦⎥

1⎡ 1 1 ⎤
= ⎢ 2 [a + s ] + 2 ( a − s )⎥
2 ⎣ a + (a + s ) 2
a + (a − s) 2

Q12. Find the Fourier cosine transform of 5e–2x + 2e–5x.
Ans: Given that,
f (x) = 5e–2x + 2e–5x
The Fourier cosine transform is given as,

Fc ( f (x)) = ∫ f ( x) cos px dx
0

∞ ∞ ∞


−2 x
+ 2e −5 x ) cos px dx = 5e − 2 x cos px dx + 2e −5 x cos px dx
= (5e
0

0

0

∞ ∞

∫ ∫
−2x
=5 e cos px dx + 2 e −5 x cos px dx
0 0

∞ ∞
⎡ e−2x ⎤ ⎡ e −5 x ⎤
=5⎢ 2 ( −2 cos px + p sin px ) ⎥ + 2 ⎢ ( −5 cos px + p sin px ) ⎥
⎣2 + p ⎣5 + p
2 2 2
⎦0 ⎦0

⎡ ∞ − ax e − ax ⎤
⎢⎣ 0

⎢Q e cos bx dx = 2
a +b 2
( −a cos bx + b sin bx )⎥
⎥⎦

⎡ 2 ⎤ ⎡ 5 ⎤
= 5 ⎢0 + ⎥ + 2 ⎢0 + 2 ⎥
⎣ p + 4⎦
2
⎣ p + 25 ⎦

10 10
= + 2
p + 4 p + 25
2

∴ It is the required solution.


Q13. Define the inverse Fourier transform.
Ans: The inverse Fourier transform of f(S) is defined as,

1
∫ F ( S )e
−isx
f(x) = I.F.T {F(S)} = ds

−∞

Look for the S IA GROUP LOGO on the TITLE COVER before you buy
UNIT-3 (Fourier Transform) 3.5
Q14. Define the inverse Fourier sine and cosine transform.
Ans:
Inverse Fourier Sine Transform: The inverse Fourier transform of f(s) is defined as,

2
f(x) = I.F.T {F(S)} =
π0 ∫
Fs ( S ) sin sx ds

Inverse Fourier Cosine Transform: The inverse Fourier cosine transform of Fc(S) is defined as,

2
f(x) = IFCT {Fc(S)} =
π ∫
Fc (S ) cos sx ds
0

Q15. Define finite Fourier sine and cosine transforms.


Ans:
Finite Fourier Sine Transform: The finite Fourier sine transform of f(x) which is defined in 0 < x < k as,
k
nπx
Fs(n) =
0
∫ f ( x) sin
k
dx , n = 1, 2, 3,.....

Finite Fourier Cosine Transform: The finite Fourier cosine transform of f(x) which is defined in 0 < x < k as,
k
nπx
Fc(n) = ∫ f ( x) cos
0
k
dx , n = 1, 2, 3,...

Q16. Define inverse finite Fourier sine and cosine transforms. Model Paper-III, Q1(f)
Ans:
Inverse Finite Fourier Sine Transform: The inverse finite Fourier sine transform of Fs(n) is defined as,

nπx

2
F(x) = Fs (n) sin , 0<x<k
k n =1 k
Inverse Finite Fourier Cosine Transform: The inverse finite Fourier cosine transform of Fc(n) is defined as,

nπx
∑ F (n) cos
2
F(x) = c , 0<x<k
k n =1
k

Q17. Find the finite Fourier cosine transform of f(x) = x2 in (0, l).
Ans: Given that,
f (x) = x2 in (0, 1)
Finite Fourier cosine transform of f (x) is given by,
l
⎛ nπx ⎞ 1

Fc(n) = ∫
0
f ( x ) cos ⎜
⎝ l ⎠
⎟ dx =
∫x
2
cos nπxdx (Q l =1)
0

2 ⎡ cos πx − cos nπx ⎤


1 1 1
sin nπx
− ⎢x − − ∫
dx ⎥
2
= x
nπ 0 nπ ⎢ nπ 0 nπ ⎥
⎣ 0 ⎦
1
1 2 1 2 1 2 sin nπx
= [ x sin nπx ] + 2 2 [ x cos nπx ] –
nπ 0 n π 0
n π
2 2
nπ 0

1 2 2
= (sin nπ) + 2 2 (cos nπ) – 3 3 sin nπ
nπ n π nπ

sin nπ ⎡ 2 ⎤ 2 2
⎢1 − n 2 π 2 ⎥ + n 2 π 2 cos nπ = n 2 π 2 (−1)
n
=
nπ ⎣ ⎦

SPECTRUM ALL-IN-ONE JOURNAL FOR ENGINEERING STUDENTS SIA GROUP


3.6 MATHEMATICS-II [JNTU-ANANTAPUR]

PART-B
ESSAY QUESTIONS WITH SOLUTIONS
3.1 FOURIER INTEGRAL THEOREM (ONLY STATEMENT) – FOURIER SINE AND COSINE
INTEGRALS
Q18. Obtain an expression for sine and cosine integrals.
Ans: According to Fourier integral theorem,
∞ ∞
1
f(x) =
π ∫ ∫ f (t ) cos(λt − λx)dt.dλ
0 −∞

∞ ∞
1
=
π 0 −∞ ∫∫
f (t )[cos λ t cos λ x + sin λ t.sin λx ]dt .dλ (Q cos(A – B) = cosA cosB + sinA sinB)

∞ ∞ ∞ ∞
1 1
=
π ∫∫
0 −∞
f (t ) cos λt cos λx dtdλ +
π ∫ ∫ f (t ) sin λt. sin λx dt.dλ
0 −∞

∞ ∞ ∞ ∞
1 1
=
π0 ∫
−∞

cos λ x f (t ). cos λ t dt dλ +
π0 −∞

sin λx f (t ) sin λt dt.dλ ∫ ... (1)

Case (i): If f(x) is an odd function and f(t) cosλt is also an odd function and f(t) sinλt is an even function. Then, the first term on the
right side of equation (1) becomes zero.
∞ ∞
1
f(x) =
π
sin λx ∫
0 −∞
∫ f (t ) sin λt dt.dλ
∞ ∞
2
f ( x) =
π ∫ ∫
sin λ x f (t ).sin λ t dt.d λ ... (2)
0 0

Equation (2) represents the Fourier sine integral.


Case (ii): If f(x) is an even function and f(t) sinλt is also an even function and f(t) cosλt is an odd function then the second term on
the right side of equation (1) becomes zero.
∞ ∞
1
f(x) =
π ∫
cos λx
0 −∞
∫ f (t ) cos λt dt.dλ
∞ ∞
2
∴ f(x) = π ∫
0

cos λ x f (t ) cos λt dt.dλ
0
... (3)

Equation (3) is known as Fourier cosine integral.



cos ωx
∫ 1+ ω
π −x
Q19. Using Fourier integral theorems show that 2
dω = e .
0
2


cos ωx π −x
Ans: To prove that
∫ 1+ ω
0
2
dω =
2
e using Fourier integral theorem.

The Fourier cosine integral is given as,

2
∞ ⎡∞ ⎤
f(x) =
π
cosλx
⎢0∫ ⎥∫
⎢ f (t ) cos λtdt ⎥ dλ ... (1)
0 ⎣ ⎦

Look for the S IA GROUP LOGO on the TITLE COVER before you buy
UNIT-3 (Fourier Transform) 3.7
Let, f (x) = e , f (t) = e and put λ = ω in equation (1), we get,
–x –t

2
∞ ⎡∞ ⎤
e = –x
π
cos ωx
⎢ ∫ ⎥ ∫
⎢ e −t cos ωt dt ⎥ dω
0 ⎣0 ⎦


2 ⎡ ⎛ 1 ⎞⎤
⇒ e–x
=
π ⎣ ∫
⎢cos ωx ⎜⎝ 2
1 + ω 2⎟
⎠ ⎥⎦

0

∞ ∞
⎡ e −ax ⎤ a ⎤

− ax
[Q L {cos at} = e cos px dx = ⎢ 2 (−a cos px + p sin px)⎥ = 2 2⎥
⎣⎢ a + p ⎦⎥ 0 a + p ⎦⎥
2
0


2 cos ωx
⇒ e–x = ∫
π 1 + ω2
0


π −x cos ωx

2
e = ∫ 1+ ω
0
2


cos ω x π −x
∴ ∫ 1+ ω
0
2
dω =
2
e

αx dx= e–ααx.
Q20. Solve the integral equation f(x) cosα ∫
0
Model Paper-III, Q6

Ans: Given equation is,


∫ f ( x) cos αxdx = e –αx


... (1)
0

∫ f ( x) cos αxdx = F (α)


0
c
... (2)

∴ From equations (1) and (2), it can be observed that,


Fc(α) = e–αx ... (3)
By inversion formula, we get,

2
f(x) =
π ∫
[e −αx cos(αx )]dx
0


2⎛ α ⎞ ⎡ ∞
⎡ e −as ⎤ a ⎤
f(x) = ⎜ ⎟
π ⎝ α + x2 ⎠ ⎢ ∫
⎢Q L{cos at} = e − as cos psds = ⎢ 2
⎣⎢ a + p 2
(−a cos ps + p sin ps)⎥ =
⎥⎦ 0

a + p 2 ⎦⎥
2 ... (4)
⎣ 0

On substituting equation (4), in equation (2), we get,


2⎛ α ⎞
f(x) = ⎜ ⎟ in Fc(α)
π ⎝ α2 + x2 ⎠

= ∫ f ( x) cos αxdx
0

SPECTRUM ALL-IN-ONE JOURNAL FOR ENGINEERING STUDENTS SIA GROUP


3.8 MATHEMATICS-II [JNTU-ANANTAPUR]
We get,


2⎛ α ⎞
Fc(α) = ∫ π ⎜⎝ α 2
+x 2⎟

cos αxdx
0


2 ⎛ α ⎞
⇒ Fc(α) = ⎜ ∫ ⎟ cos αxdx
π ⎝ α2 + x 2 ⎠
... (5)
0

From equation (3),


Fc(α) = e–αx
On substituting Fc(α) = e–αx in equation (5), we get,


2 α
e –αx
= ∫
π (α 2 + x 2 )
cos αxdx ... (6)
0

From equations (1) and (6), we get,

∞ ∞
2 α
∫ f ( x) cos αxdx = ∫ π (α
0 0
2
+ x2 )
cos αxdx

= e–αx
Hence proved.

3.2 FOURIER TRANSFORM – FOURIER SINE AND COSINE TRANSFORMS – PROPERTIES


Q21. State and prove the properties of Fourier transform.
Ans: The salient properties of Fourier transforms are,
(a) Linear Property: If f(x) and g(x) are two functions whose Fourier transform is given by, F(s) and G(s) respectively, then,
F[a f(x) + b g(x)] = a. F(s) + b G(s)
Where, a and b are arbitrary constants.
Proof: By definition∞of Fourier transform,


F(s) = e isx f ( x) dx
−∞
Similarly,


isx
G(s) = e g ( x ) dx
−∞


F[a f(x) + b g(x)] = e isx [ af ( x) + bg ( x)]dx
−∞

∞ ∞

∫e ∫e
isx isx
= [ af ( x)]dx + [bg ( x)]dx
−∞ −∞

∞ ∞

=a ∫
−∞

e isx f ( x ) dx + b e isx g ( x) dx
−∞

= a. F(s) + b. G(s).

∴ F [af ( x) + b.g ( x)] = a.F ( s) + b.G(s)

Look for the S IA GROUP LOGO on the TITLE COVER before you buy
UNIT-3 (Fourier Transform) 3.9
(b) Scaling Property: If f(x) is a function whose complex Fourier transform is given by F(s),
Then,
1 ⎛s⎞
F{f(ax)} = F⎜ ⎟
a ⎝a⎠
Where, a ≠ 0.
Proof : By the definition of Fourier transform we get,

∫e
F(s) = isx
. f ( x )dx
−∞

∫e
isx
F{f(ax)}= . f ( ax ) dx
−∞
Let, ax = u
u 1
⇒ x= = (u)
a a
du
⇒ dx =
a

(ua ). f (u ).⎛ du ⎞
∫e
is
∴ f{f(ax)} = ⎜ ⎟
−∞
⎝ a ⎠

1 i ( s a )u
=
a ∫e
−∞
. f (u ).du

1 ⎛s⎞
= .F ⎜ ⎟ [Q By definition of Fourier transform]
a ⎝a⎠
(c) Shifting Property: If a function f(x) has a complex Fourier transform F(s),
F{f(x – a)} = eisa.F(s).
Proof : By definition of Fourier transform,

∫e
isx
F(s) = . f ( x )dx
−∞

∫e
isx
F{f(x–a)} = . f ( x − a ) dx
−∞
Let, x – a = u
⇒ x= u+a
dx= du



F{f(x – a)}= e is ( u + a ) . f (u ).du
−∞

∫e
isu + isa
= . f (u ).du
−∞

∫e
isu isa
= e . f (u ).du
−∞

∫e
isa isu
=e . f (u ).du = eisa . F(s)
−∞

∴ F{ f ( x − a)} = e isa .F ( s)

SPECTRUM ALL-IN-ONE JOURNAL FOR ENGINEERING STUDENTS SIA GROUP


3.10 MATHEMATICS-II [JNTU-ANANTAPUR]
(d) Modulation Property: If a function f (x) has a complex Fourier transform F(s), then,
1
F{f(x) cosax} = [F(s + a) + F(s – a)]
2
Proof: By the definition of Fourier transform,

∫e
isx
F(s) = f ( x) dx
−∞

∫e
isx
F{f(x).cosax} = .[ f ( x ) cos ax]dx
−∞

∫e
isx
= . cos ax f ( x ) dx
−∞

⎡ e iax + e −iax ⎤ (e iθ + e −iθ )
=
−∞

e isx .⎢

⎣ 2
⎥ f ( x ) dx
⎦⎥
[Q cosθ =
2
]


1
∫ [e + e isx .e −iax ] f ( x) dx
isx iax
= e
2
−∞

1
∫ [e
ix ( s + a )
= + e ix ( s −a ) ] f ( x )dx
2
−∞
∞ ∞
1 1
∫ [ e ix ( s + a ) . f ( x )]dx + ∫ [e
ix ( s − a )
= . f ( x )]dx
2 2
−∞ −∞

1 1
= F ( s + a) + F ( s − a)
2 2
1
= [ F ( s + a ) + F ( s − a )]
2
1
∴ F { f ( x ) cos ax} =
[ F (s + a) + F ( s − a )
2
Q22. Show that the Fourier sine transform of,

⎧x , for 0 < x < 1



f(x)= ⎨2 − x , for 1< x < 2 is 2 sin s(1− cos s) .
⎪0 , for x > 2 s2

Ans: Given that,
f (x)= x, when 0 < x < 1 limits lies between 0 and 1.
f (x)= 2 – x, when 1 < x < 2 limits lies between 1 and 2.
f (x) = 0, when x > 2 limits lies between 2 and ∞.
By sine transform,

f (x) = ∫ f ( x) sin sxdx


0

1 2 ∞

fs(s) = ∫
0

1

f ( x ) sin sxdx + f ( x ) sin sxdx + f ( x ) sin sxdx
2

1 2

fs(s) = ∫0

x sin sxdx + ( 2 − x ) sin sxdx + 0
1

Look for the S IA GROUP LOGO on the TITLE COVER before you buy
UNIT-3 (Fourier Transform) 3.11
By integration by parts, we get, 0 ∞

fs(s) = ⎡ cos sx ⎤
⎢− x s ⎥ +
⎣ ⎦0
1 1


cos sx
s

dx + ⎢− ( 2 − x)

cos x ⎤
s ⎥⎦1
+ −
2

s
2
cos sx
∫ dx
= ∫
−∞

e x(1+ is ) dx + e x( is −1) dx
0
0 1

1 2
− cos s 1 ⎛ sin sx ⎞ ⎡ cos s ⎤ 1 ⎡ sin sx ⎤ 0 ∞
= + ⎜ ⎟ + 0+ − ⎡ e x (1+ is ) ⎤ ⎡ e x (is −1) ⎤
s s ⎝ s ⎠ 0 ⎢⎣ s ⎥⎦ s ⎢⎣ s ⎥⎦1 =⎢ ⎥ +⎢ ⎥
⎢⎣ 1 + is ⎥⎦ − ∞ ⎢⎣ is − 1 ⎥⎦ 0
cos s 1 ⎛ sin s ⎞ cos s 1
=− + ⎜ ⎟+ − 2 (sin 2 s − sin s )
s s⎝ s ⎠ s s 0 ∞
⎡ e x (1+is ) ⎤ ⎡ e − x (1+is ) ⎤
=⎢ ⎥ +⎢ ⎥
⎣⎢ 1 + is ⎦⎥ − ∞ ⎣⎢ is − 1 ⎦⎥ 0
1 1
= 2
sin s − (sin 2 s − sin s)
s s2

sin s sin 2 s sin s 2 sin s sin 2 s e0 e −∞ e −∞ e0


= − + = − = − + −
s2 s2 s2 s2 s2 1 + is 1 + is is − 1 is − 1

2 sin s − sin 2 s 0⎡ 1 1 ⎤
= =e ⎢ − ⎥
s2 ⎣ 1 + is is − 1 ⎦
2 sin s (1 − cos s )
= ⎡ is − 1 − 1 − is ⎤
s2 =1⎢ ⎥
Q23. Find the Fourier transforms of f(x) = e–|x| and ⎣ (1 + is )(is − 1) ⎦

cos xt π –|x| −2
deduce that ∫ 1+ t 2
dt =
2
e . Hence show
=
−2
(is ) − 12 =
0 i s −1
2 2

4s
that F(xe–|x|) = i . Model Paper-I, Q6 −2
(1 + s 2 )2 = (Q i2 = –1)
( −1) s 2 − 1
Ans: We know that,
∞ −2 −2
Fourier transform of f(x) = ∫
−∞
f ( x ) e isx dx =
− s2 −1
=
− ( s 2 + 1)

Given that, 2
= = f(s)
f(x) = e–|x| 1+ s2
⎧⎪e − x if x > 0 ∴ The inverse Fourier transform of f(s) is f(x).
⇒ f(x) = e–|x| = ⎨ x
⎪⎩ e if x < 0 ∞
1
∫e
− isx
⇒ f(x) = f ( s ) ds
F[f(x)] = ? 2π
−∞
The Fourier transform of f(x) is given as,
∞ 2
On substituting, f(s) = in the above equation,

isx
F[f(x)] = f ( x ) e dx 1+ s2
−∞ we get,
∞ ∞
1 ⎛ 2 ⎞
∫e
− isx
⇒ F[f(x)] = ∫
−∞
e −| x| e isx dx f(x) =

−∞
⎜ ⎟ ds
⎝1+ s2 ⎠

0 ∞ ∞
2 ⎛ 1 ⎞
∫e
− isx
= ∫
−∞

e x e isx dx + e − x e isx dx
0
=

−∞
⎜ ⎟ ds
⎝1+ s2 ⎠

0 ∞ ∞
1 − isx ⎛ 1 ⎞
= ∫e
x + isx
dx + e ∫
− x + isx
dx f(x) =
π ∫e
−∞
⎜ ⎟ ds
⎝1+ s2 ⎠
−∞ 0

SPECTRUM ALL-IN-ONE JOURNAL FOR ENGINEERING STUDENTS SIA GROUP


3.12 MATHEMATICS-II [JNTU-ANANTAPUR]
Given that, Q24. Find the Fourier transform of,
f(x) = e–|x| ⎧1, for | x | < a
f(x) = ⎨ and hence evaluate
∞ ⎩0, for | x | > a > 0
1 − isx ⎛ 1 ⎞

π ∫e ⎜ ⎟ ds = e–|x|
⎝1+ s2 ⎠

sin x ∞


sin as.cos xs
−∞

0
x
dx and
∫ s
ds.
∞ −∞
⎛ 1 ⎞
⇒ ∫
−∞
e − isx ⎜
⎝ 1 + s 2
⎟ ds = πe–|x|

Ans: The Fourier transform of the function f (x) is,
∞ a


⎛ 1 ⎞
F(s) = ∫ ∫
f (t ) e ist dt = 1 . e ist dt


−∞
⇒ (cos sx − i sin sx ) ⎜ ⎟ ds = πe–|x| −a

−∞
⎝ 1 + s 2

a
⎡ e ist ⎤ e isa − e −isa
∞ = ⎢ ⎥ =
⎛ cos sx i sin sx ⎞ ⎣ is ⎦ − a is
⇒ ∫ ⎜⎝ 1 + s
−∞
2

1+ s2 ⎠
⎟ ds = πe–|x|
2i sin sa 2 sin sa
= =
On equating real and imaginary parts, we get, is s

cos sx ⎡ e isa − e −isa ⎤
⇒ ∫ 1+ s
−∞
2
ds = πe–|x| ⎢

Q sin sa =
2i


To evaluate the integrals (a) and (b), apply the inversion
Let, s = t,
formula of F(s),
∞ ∞
cos xt 1
⇒ ∫ 1+ t
−∞
2
dt = πe–|x| f (x) =
2π − ∞ ∫
F ( s ) e −isx ds ... (1)

∞ 2 sin sa
cos xt
⇒ 2 ∫ 1+ t
0
2
dt = πe–|x| (a) On substituting, F(s) =
s
in equation (1),


1 2 sin sa −ixs

cos xt π −| x |
f (x) =
2π − ∞ s ∫ e ds
∴ ∫ 1+ t
0
2
dt =
2
e

1 sin sa
= ∫ (cos xs − i sin xs ) ds
d π s
F(xnf(x)) = – i f(s) −∞
ds
∞ ∞
1 sin sa cos xs

i sin sa sin xs
=–i
d ⎛ 2 ⎞
⎜ ⎟
=
π s
ds −
π
−∞
∫ s
ds
−∞
ds ⎝ 1 + s 2 ⎠
The integrand in the second integral is an odd function
and hence this integral vanishes. Therefore,
d ⎛ 1 ⎞
= –i2 ⎜ ⎟ ∞
ds ⎝ 1 + s 2 ⎠ 1 sin sa cos sx
f (x) =
π −∞ ∫ s
ds
(1 + s 2 ) 0 − 1( 2 s )
= –i2
(1 + s 2 ) 2 ∞
1 sin sa cos sx ds
−2 s
⇒ =
π −∞∫ s
= –i2
(1 + s 2 ) 2
= π f (x)
4s ⎧π for | x |≤ a
=i =⎨ ... (2)
⎩0 for | x |> a
2 2
(1 + s )

Look for the S IA GROUP LOGO on the TITLE COVER before you buy
UNIT-3 (Fourier Transform) 3.13
(b) If x = 0, then equation (2) yields,

sin sa
−∞
∫ s
ds = π


sin sa π
⇒ ∫ s
ds =
2
0
2
Q25. Find the Fourier cosine transform of e − x . Model Paper-II, Q6
Ans: Given that,
f (x) = e − x
2

Fourier cosine transform of f(x) is given as,



Fc (p) = f ( x) cos( px) dx
0

∫e
− x2
∴ Fc (p) = cos px dx
0

∫e
− x2
Let, Fc (p) = cos px dx = I ... (1)
0

On differentiating equation (1) on both sides with respect to ‘p’, we get,



dI d
∫e
− x2
= (cos px) dx
dp dp
0

∫e
− x2
= ( − x sin px) dx
0

On multiplying and dividing by 2 on RHS we get,



dI 1

2
= ( −2 xe − x ) sin px dx
dp 2
0


dI 1

2
⇒ = sin px ( −2 xe − x ) dx
dp 2
0

1⎡ ⎤
∞ ∞
dI ⎡d ⎤
∫ ∫ ∫
− x2 2
= ⎢sin px (−2 xe )dx − ⎢ sin px ⎥ (−2 xe − x )dx dx ⎥
dp 2 ⎢ ⎣ dx ⎦ ⎥
⎣ 0 0 ⎦

⎡Q f ( x) g ( x ) dx = f ( x ) g ( x ) dx − ⎡ f ′( x ) g ( x ) dx ⎤ dx ⎤
⎢⎣ ∫ ∫ ∫
⎢⎣ ∫ ⎥⎦ ⎥⎦

1⎡
[ ] − ∫ [( p cos px)e ]dx⎤⎥⎥

dI ∞
⎡Q − 2 xe − x 2 dx = e − x 2 ⎤

− x2 − x2
= ⎢ sin px.e 0 ⎢⎣ ⎥⎦
dp 2 ⎢
⎣ 0 ⎦

1 ⎡ ⎤

dI
∫ [ Q e– ∞ = 0, sin 0 = 0]
2
= ⎢[ 0 − 0] − p e − x cos pxdx⎥
dp 2 ⎢ ⎥
⎣ 0 ⎦

SPECTRUM ALL-IN-ONE JOURNAL FOR ENGINEERING STUDENTS SIA GROUP


3.14 MATHEMATICS-II [JNTU-ANANTAPUR]
∞ On substituting p = 0 in equation (2), we get,
dI − p − x2
dp
=
2 ∫
e cos px dx I = c e–0 = c
0
π
∴ c= [ From equation (3)]
−p 2
= I [ From equation (1)]
2
π
On substituting c = in equation (2), we get,
dI − p 2
⇒ = dp
I 2 p2
Integrating on both sides, we get, π − 4
I= e
2
1 −p
∫ I dI = ∫ 2
dp
∴ Fc{p} = I =
π −
e
p2
4 [ From equation (1)]
2
1 ⎡− p2 ⎤
⇒ log I = ⎢ ⎥ + log c p2
2 ⎢⎣ 2 ⎥⎦ − x2 π −
∴ The fourier cosine transform of e e 4
2
− p2
⇒ log I = + log c 1
4 Q26. Find the Fourier sine transform of .
x(x 2 + a 2 )
p2
− Ans: Model Paper-III, Q7
⇒ I = ce [ Q loge x = a ⇒ x = ea]
4 ... (2)
Given that,
On substituting p = 0 in equation (1), we get,
1
∞ f(x) =
x( x + a 2 )
2

∫e
2
−x
I= cos 0 dx
0 Fourier cosine transform of f(x) is given as,

I = ∫e
−x 2
.dx FS(p) = ∫ f ( x) sin px dx
0
0

Let, t = x2 ∞
sin px
1 1
⇒ FS(p) = ∫ x( x 2
+ a2)
dx ... (1)
dt = 2x dx ⇒ dx = dt = dt 0
2x 2 t On differentiating equation (1) with respect to ‘p’, we

get,
dt
∫e
−t ∞
I=
d 1 d
0
2 t
dp
{FS ( p )} =
∫ x( x
0
2
. (sin px ) dx
+ a 2 ) dp

1
⇒ I= ∫
(t ) −1 / 2 e −t dt ∞
1
2
0 = ∫ x( x
0
2
+ a2 )
.x. cos px dx

∞ 1
1 −1
⇒ I= ∫t 2 e −t dt ∞
1
2
0 = ∫ (x
0
2
+ a2 )
cos px dx ... (2)

⎡ ∞ ⎤ ∞
1 ⎛1⎞
⇒ I = Γ⎜ ⎟
2 ⎝2⎠ ⎢ ∫
⎢Q Γ ( p) = e −t t p −1dt ⎥

1
= ∫ 2 ye
−( x 2 + a 2 ) y 2
dy
⎣ 0 ⎦ Q 2
x +a 2
0

1 ⎡ 1 ⎤ On multiplying cos px on both sides of above equation,


⇒ I= π ⎢Q Γ 2 = π ⎥ we get,
2 ⎣ ⎦

cos px

2
+ a2 ) y2
π 2 2 = cos px.2 ye −( x dy
⇒ I= ... (3) x +a
2 0

Look for the S IA GROUP LOGO on the TITLE COVER before you buy
UNIT-3 (Fourier Transform) 3.15
Integrating on both sides with respect to x from 0 to ∞.
∞ ∞∞
cos px
∫x ∫ ∫ cos px.2 ye
−( x 2 + a 2 ) y 2
dx = dy dx
0
2
+a 2
0 0

∞∞

∫ ∫ cos px.2 ye
−x2y2 2 2
= .e − a y
dy dx
0 0

∞ ⎡∞ 2 2 ⎤
∫ ∫
2 2
= 2 ye − a y
⎢ e − x y cos px dx ⎥ dy ... ( 3)
⎢0 ⎥
0 ⎣ ⎦

∫e
− x2 y 2
Let, cos px dx = S ... (4)
0

On differentiating on both sides with respect to p, we get,



dS d
∫e
− x2 y 2
= (cos px ) dx
dp dp
0

∫e
− x2 y 2
= ( − x sin px ) dx
0


1
∫ 2 y .e
− x2 y 2
=
2
(− x sin px )dx
2y2 0


1
∫ (−2 xy .e
2 − x2 y2
= 2
) sin px dx
2y 0


1
∫ [sin px.(−2 xy e
2 − x2 y 2
= 2
)]dx
2y 0

1 ⎡ ⎤ ⎤⎥
∞ ∞
⎡d
∫ ∫ ∫
2 − x2 y2 2 − x2 y2
= ⎢sin px − 2 xy e − ⎢ dx sin px ( −2 xy e ) dx ⎥ dx ⎥
2 y 2 ⎢⎣ 0 0
⎣ ⎦ ⎦

1 ⎡
( ) − ∫ [( p cos px)(e ]⎤


⎡Q − 2 xe − x 2 dx = e − x 2 ⎤

2 2
−x2y2
= ⎢ sin px.e − x y ) ⎥ dx
2 y ⎢⎣
2 0
⎥ ⎢⎣ ⎥⎦
0 ⎦

1 ⎡ ∞ ⎤

2 2
= ⎢0 − 0 − p (e − x y cos px )dx ⎥ [Q e–∞ = 0, sin 0 = 0]
2y2 ⎢ ⎥
⎣ 0 ⎦
1 − pS ⎡ ∞ ⎤

2 2
= 2
[ − pS ] = 2 ⎢Q S = e − x y cos px dx ⎥
2y 2y ⎢ ⎥
⎣ 0 ⎦

dS − pS
∴ =
dp 2 y2

dS dp
⇒ =−p 2
S 2y

SPECTRUM ALL-IN-ONE JOURNAL FOR ENGINEERING STUDENTS SIA GROUP


3.16 MATHEMATICS-II [JNTU-ANANTAPUR]
Integrating on both sides with respect to p, we get, Therefore, equation (5), becomes,
1 p ∞
∫ S dS = − ∫ 2 y 2
dp
S = ∫e
− x2 y 2
cos px dx
0

p2 1 π − p2 / 4 y2
⇒ log S = − . + log C1 = e
2 2y2 2y

− p2 π − p2 / 4 y2
⇒ log S = + log C1 On substituting S = e in equation (3), we get,
4y2 2y

−p 2
/4 y2 ∞ ∞
⇒ S = C1e ... (5) cos px π − p2 / 4 y2
∫ 2 ye
−a2 y2
On substituting p = 0 in equation (4), we get, ∫x
0
2
+ a2
dx =
0
.
2y
.e dy

∫e
− x2 y 2 ∞
S= dx [Q cos 0 = 1]

2 2
y − p2 / 4 y2
0 = π e−a dy
0
Let, x2y2 = t ⇒ xy = t
⎛ 2 ⎞
∞ −a 2 ⎜ y 2 + p ⎟
⎜ ⎟

2 2
1 = π e ⎝
4 a y ⎠ dy
∴ ydx = dt
2 t 0

1 p
π − 2. 2 a .a
2
⇒ dx = dt = π. .e
2y t 2a
∞ π − pa
1

.e
∴ S= e −t dt =
2a
0
2y t
d π − Pa
∞ ⇒ [ FS ( p)] = e
1 dp 2a
⇒ S=
2y ∫
t −1/ 2 e −t dt
0 On integrating above equation with respect to ‘P’ on
both sides, we get,
∞ 1
1 −1
⇒ S=
2y ∫
t 2 e −t dt d
[ FS ( p)] =
π − Pa
e dp
0 dp 2a

1 ⎛1⎞ ⎡ ∞ ⎤
π ⎡ e − ap ⎤
⇒ S= Γ⎜ ⎟
2y ⎝ 2 ⎠ ⎢ ∫
⎢Q Γ( p ) = e − t t p −1dt ⎥
⎥ FS(P) = 2a ⎢ − a ⎥
⎣ 0 ⎦ ⎣⎢ ⎦⎥
1 ⎡ ⎛1⎞ ⎤
⇒ S= π ⎢Q Γ⎜ ⎟ = π ⎥
2y ⎣ ⎝2⎠ ⎦ π − ap
= e + C1
− 2a
π
⇒ S= π
2y ∴ FS ( P ) = e − ap + C1
π − 2a 2
On substituting S = when p = 0 in equation (5),
2y Q27. Find the Fourier cosine transform of e–ax and
we get, ∞
cospx
π
= C1 e 0
hence evaluate ∫a 2
+ p2
dx .
2y 0
Ans: Given function is,
π Fourier cosine transmission of f(x) is given as,
⇒ C1 =
2y f(x) = e–ax

Look for the S IA GROUP LOGO on the TITLE COVER before you buy
UNIT-3 (Fourier Transform) 3.17
From definition,

Fc(p) = ∫ f ( x) cos pxdx
0

a
∫e
− ax
= cos pxdx = ... (1)
0
a + p2
2

⎡ ∞ ⎤
⎢ 0

⎢Q L{cos at} = e − ax cos pxdx ⎥

⎢ ∞

⎢ ⎧ e − ax ⎫ ⎥
⎢ =⎨ 2 (− a cos px + p sin px )⎬ ⎥
⎩a + p
2
⎢ ⎭0 ⎥
⎢ ⎥
⎢ a ⎥
= 2
⎢ a + p2 ⎥
⎣⎢ ⎦⎥

By Fourier cosine transform, we get,



2
f(x) =
π ∫
Fc ( p ) cos pxdp
0


2 a
⇒ f(x) = ∫
π a + p2
0
2
cos pxdp [Q From equation (1)]


2a cos px
⇒ e–ax =
π ∫a
0
2
+ p2
dp


dp = π e − ax
cos px
∫a 2
+ p2 2a
0


cos px π − ax
∫a
a
∴ The Fourier cosine transform of e –ax
is and dp = e
a +p2 2
0
2
+p 2 2a


⎧1, | x | < a sinax
Q28. Find the Fourier transform of, f(x) = ⎨
⎩0, | x | > a
and hence find the value of ∫
0
x
dx .

Ans: Given that,

⎧1 | x | < a
f(x) = ⎨
⎩0 | x | > a
The Fourier transform of the function f(x) is,


isx
F[f(x)] = e f ( x ) dx
−∞

−a a ∞

= ∫ e isx f ( x ) dx + ∫ ∫
e isx f ( x) dx + e isx f ( x )dx
−∞ −a a

SPECTRUM ALL-IN-ONE JOURNAL FOR ENGINEERING STUDENTS SIA GROUP


3.18 MATHEMATICS-II [JNTU-ANANTAPUR]
a a
⎡ e isx ⎤ 1

isx
= e (1) dx = ⎢ ⎥ Q29. Prove that is self-reciprocal under Fourier
−a
⎢⎣ is ⎥⎦ −a x
e isa − e −isa sine transform.
=
is 1
Ans: To prove is self reciprocal under Fourier sine
2 1 x –x x
= sin h(isa ) [Q sin hx = (e – e )]
is 2 transform,
2 sin sa 1
∴ F [f(x)] = F(s) = [Q sin h(ix) = isinx] Let f(x) =
s x
We get,
⎧ 1 ⎫
∞ FS{f(x) = FS ⎨ ⎬
⎩ x⎭
∫e
isx
F(s) = f ( x) dx
−∞ ∞
1
By inversion formula,
= ∫
0
x
sin sxdx


1 ⎧ 1 ⎫
∫ F ( s )e
− isx
f(x) = ds And FC{f(x)} = FC ⎨
2π ⎬
−∞ ⎩ x⎭
∞ ∞
⎧1 , | x |< a 1

1
2π ∫
2 sin sa −isx
s
.e ds = ⎨
⎩0 , | x |> a
= ∫
0
x
cos sxdx
−∞

Taking LHS, ∞ ∞
⎧ 1 ⎫ ⎧ 1 ⎫ 1 1

FC ⎨
⎩ x⎭
⎬ + iFS ⎨
⎩ x⎭
⎬= ∫ x
cos sx dx + i ∫ x
sin sx dx
1 2 sin sa 0 0

2π ∫ s
.(cos sx − i sin sx ) ds

−∞ 1
[Q e – ix
= cos x – i sin x]
= ∫ (cos sx + i sin sx)
0
x
dx

∞ ∞ ∞
1 2 sin sa i 2 sin sa

2π ∫ s
cos sxds −
2π ∫ s
sin sxds = ∫e
isx
.
1
x
dx
−∞ −∞ 0

∞ ∞
1 2 sin sa

∫ ∫e ( x) −1 / 2 dx
isx
. cos sx ds =
2π s
−∞ 0

[Q Second integral is an odd function] ∞ 1


−1


sin sa ⎧π , | x |< a
= ∫ e isx ( x ) 2 dx


0
⇒ cos sxds = ⎨
−∞
s ⎩0 ,| x |> a Let, isx = – t

Let, x = 0 t
⇒ x=−
is

sin sa
∫ s
ds = π
⇒ dx = −
dt
is
−∞

∞ 1
∞ −1
π ⎧ 1 ⎫ ⎧ 1 ⎫ ⎛ − t ⎞ 2 ⎛ − dt ⎞

sin as −t
⎬ + iFS ⎨
⇒ ∫ s
ds =
2
∴ FC ⎨
⎩ x⎭ ⎩ x⎭
⎬=
0
e .⎜ ⎟ ⎜
⎝ is ⎠ ⎝ is ⎠

0

∞ 1
∞ −1
π ⎧ 1 ⎫ ⎧ 1 ⎫ − t ⎛ − t ⎞ 2 ⎛ − dt ⎞
or ∫
sin ax
x
dx =
2
⇒ FC ⎨
⎩ x⎭
⎬ + iFS⎨
⎩ x⎭
⎬ = −
0
e .⎜

⎟ ⎜
is ⎠ ⎝ is ⎠ ∫⎟
0

Look for the S IA GROUP LOGO on the TITLE COVER before you buy
UNIT-3 (Fourier Transform) 3.19
1
−1 ∞ 1 1
(−1).(−1) 2 −1
Q30. Find the Fourier cosine transform of, f(x) = .

−t 2
= 1
e t dt 1+ x 2
−1+1
(is ) 2 0
Ans: Given that,
1
−1+1 ∞ 1 1
( −1) 2 −1 f(x) =
=
is ∫
0
(e −t t 2 ) dt 1+ x2
To find the Fourier cosine transform of f(x).
(i 2 )1 / 2 1 ∞ ∞
= S cos px
(is) 1/ 2
2 Let, k = FC[f(x)] = ∫ f ( x) cos px dx = ∫ 1 + x 2
dx ... (1)
0 0
⎡ ∞ ⎤
⎢ ∫
⎢Q n = e − x ( x) n −1 dx ⎥


cos px
⎣ 0 ⎦ On differentiating, k = F C[f(x)] = ∫ 1+ x 2
dx with
1 0
1−
(i ) 2 1 respect to ‘p’, we get,
=
s 2 ∞
dk d x sin px
(i ) 2
1
1 dp
=
dp
{FC [ f ( x)]} = −
1+ x2
dx ∫ ... (2)
0
=
s 2

x 2 sin px
⎛ π
1
π ⎞2
=− ∫ x(1 + x 2
)
dx
⎜ cos + i sin ⎟ 0
⎝ 2 2⎠ 1
= ∞
s 2 ( x 2 + 1 − 1) sin px
π π
=− ∫ x(1 + x 2 )
dx
cos + i sin 0
4 4 π
= ∞
s (1 + x 2 ) sin px − sin px
⎡ 1 ⎤
=− ∫ x (1 + x 2 )
dx
0
⎢Q = π⎥
⎣ 2 ⎦ ∞
⎡ (1 + x 2 ) sin px sin px ⎤
1
+i
1 =− ⎢


∫ x (1 + x 2
)

x (1 + x 2 ⎥
) ⎥

dx
2 2 0
= π
s ∞ ∞
dk sin px sin px
⎛ 1
= ⎜⎜ +i
1 ⎞
⎟ π

dp
=− ∫
x
dx +
x (1 + x 2 ) ∫
dx
2 s ⎟⎠
0 0
⎝ 2s

π π dk π sin px
=
2s 2s
+i −
dp = 2
+
0
x (1 + ∫
x 2
)
dx

Equating real and imaginary parts on both sides, we get,


⎡ ∞ sin ax π ⎤
⎧ 1 ⎫ π
FC ⎨
⎩ x⎭
⎬=
2s
⎢Q

⎢ 0 x
dx = (if a > 0)⎥
2 ⎥
⎣ ⎦
⎧ 1 ⎫ π Again differentiating with respect to ‘p’, we get,
Fs ⎨ ⎬=
⎩ x⎭ 2 s ∞
d2 cos px
A function f(x) is equal to F(s) then the function f(x) is
said to be self reciprocating. dp 2
{FC [ f ( x)]} = ∫ 1+ x
0
2
dx

1 1 1
∴ As Fourier sine transform of is . Hence, is d2
x s x {FC [ f ( x)]} = FC[f(x)]
dp 2
self reciprocal under Fourier sine transform.
Hence proved. ⇒ (D2 – 1) FC[f(x)] = 0

SPECTRUM ALL-IN-ONE JOURNAL FOR ENGINEERING STUDENTS SIA GROUP


3.20 MATHEMATICS-II [JNTU-ANANTAPUR]
General solution of the above is,
FC[f(x)] = C1ep + C2e–p ... (3) d a
⇒ [ F (α )] = 2 ... (1)
Put p = 0 in equation (1), then dα α + a2
∞ On integrating equation (1) with respect to ‘α’, we get,
1 π
FC[f(x)] = ∫ 1+ x
0
2
dx = [tan −1 x]∞
0 =
2 ∞
a
d π
F (α) = Fs [ f ( x )] = ∫α 2
+ a2

And {FC [ f ( x )]} = − (when p = 0) ... (4) 0
dp 2
∴ From equation (3), we get, ⎡ ∞ a ⎤
−1 ⎛ x ⎞
π
⎢Q

⎢ 0 x2 + a2
dx = tan ⎜ ⎟
⎝ a⎠ ⎥
+ c ⎥
C1 + C2 = and ⎣ ⎦
2
π −1 ⎛ α ⎞
C1 – C 2 = − [Q From equations (3) and (4)] ∴ F (α) = Fs [ f ( x )] = tan ⎜ ⎟ + c ... (2)
2 ⎝a⎠
On solving the above two equations, we get,
For α = 0, we get,
π
C1 = 0 and C2 = F(α) = 0
2
On substituting α, f (α) values in equation (2), we get,
π −p
∴ FC[f(x)] = 0e + e
p
2 0 = tan–1(0) + c

1 p −P ⇒ 0= 0 + c
∴ The Fourier cosine transform of is, FC [ f ( x)] = e
1+ x 2 2
c=0
–ax
e
Q31. Find the Fourier sine transform of , where
x ⎛α⎞
∴ F (α ) = Fs [ f ( x )] = tan −1 ⎜ ⎟
a > 0. ⎝a⎠
Ans: Given that,
e − ax −1 ⎛ α ⎞
e − ax Hence the Fourier Sine transform of = tan ⎜ ⎟ ,
f(x) = a ⎝a⎠
x
Where a > 0 where a > 0
By definition of Fourier sine transform, 3.3 INVERSE TRANSFORMS
∞ Q32. Find the inverse Fourier cosine transform f(x)
Fs[ f(x)] = ∫ f ( x) sin αx dx ⎧⎪ a − p , w hen p < 2a
0 of FC (p) = ⎨ 2
⎪⎩ 0 , w hen p ≥ 2a
∞ − ax
e Ans: Given that,
Fs[ f(x)] = ∫
0
x
sin αx dx = F(α)
⎧⎪ a − p , when p < 2 a
FC ( p ) = ⎨ 2
Differentiating on both sides with respect to ‘α’, we get, ⎪⎩ 0 , when p ≥ 2 a
∞ By the definition of inverse Fourier cosine transform, we
e − ax ⎛ d ⎞
d

[ F (α)] = ∫ ⎜
x ⎝ dα
sin αx⎟ dx

have,

0
2

f(x) =
π ∫
FC ( p ) cos pxdp
e − ax 0

d

[ F ( α)] = ∫ x
x. cos αx dx
2⎡
2a ∞ ⎤
∫ ∫
⎢ FC ( p) cos pxdp + FC ( p) cos pxdp⎥
0
=
π⎢ ⎥
⎣0 2a ⎦
⎡ ∞
a ⎤
⎢ ∫
⎢Q e − ax cos bx dx =
2
+ 2

⎥ 2⎡ ⎛
2a
p⎞
∞ ⎤
⎣ 0 a b ⎦
π⎢ ⎝ 2⎠∫
= ⎢ ⎜ a − ⎟ cos pxdp + (0) cos pxdp ⎥
⎥ ∫
⎣0 2a ⎦

Look for the S IA GROUP LOGO on the TITLE COVER before you buy
UNIT-3 (Fourier Transform) 3.21

2⎡ ⎤
2a 2a
p
=
π⎢ ∫
⎢ a cos pxdp −
∫ 2
cos pxdp⎥

⎣0 0 ⎦

2⎡ ⎤
2a 2a
1
=
π⎢ ∫
⎢a cos pxdp −
2 ∫ p cos px.dp ⎥

⎣ 0 0 ⎦

∫{ ∫ }
2 ⎡ ⎛ sin px ⎞ ⎤
2a 2a
1⎡ ⎤
= π ⎢a ⎜⎝ x ⎟⎠ − 2 ⎢⎣ p cos pxdp −
⎢⎣ 0
∫ (1). cos pxdp dp ⎥ ⎥
⎦0 ⎥

2 ⎡ ⎡ sin 2ax sin x(0) ⎤ 1 ⎡ sin px sin px ⎤ ⎤


2a
⎢ a
= π ⎢ x
⎢⎣ ⎣
− −
x ⎥⎦ 2 ⎢⎣
p
x

x
dp ⎥ ⎥
⎦ 0 ⎥⎦ ∫
2 ⎡⎛ a
2a ⎤
⎞ 1 ⎡ p sin px ⎛ − cos px ⎞⎤
= ⎢⎜ sin 2ax ⎟ − ⎢ −⎜ ⎟⎥ ⎥
π ⎢⎝ x ⎠ 2⎣ x ⎝ x 2 ⎠⎦ 0 ⎥
⎣ ⎦

2 ⎡a
2a ⎤
1 ⎧ p sin px cos px ⎫
= ⎢ sin 2ax − ⎨ + ⎬ ⎥
π ⎢x 2⎩ x x 2 ⎭0 ⎥⎦

2 ⎡ a sin 2 ax 1 ⎡ ⎧ 2 a sin 2ax cos 2ax ⎫ ⎡ ⎧ 0 sin( 0) x cos x (0) ⎫ ⎤


= ⎢ − ⎢⎨ + ⎬⎢ − ⎨ + ⎬⎥
π⎣ x 2 ⎣⎩ x x 2 ⎭⎣ ⎩ x x 2 ⎭⎦

2 ⎡ a sin 2ax a sin 2ax cos 2ax 1 ⎤


− − + 2⎥
π ⎢⎣
=
x x 2x 2 2x ⎦
2⎡ 1 cos 2ax ⎤
= ⎢ − ⎥
π ⎣ 2x 2
2x 2 ⎦
2 ⎡1 − cos 2ax ⎤
π ⎢⎣ 2 x 2 ⎥
=

2 sin 2 ax
= ×
π x2
2 sin 2 ax
=
πx 2

p
Q33. Find the inverse Fourier sine transform f(x) of FS (p) = .
1+ p 2
Ans: Given that,

p
FS ( p) =
1+ p 2

By the definition of inverse Fourier sine transform,


2
f (x) =
π ∫
FS ( p ) sin( px) dp
0


2 ⎛ p ⎞
f (x) =
π ∫ ⎜⎜⎝ 1 + p
0
2
⎟ sin( px ) dp


... (1)

SPECTRUM ALL-IN-ONE JOURNAL FOR ENGINEERING STUDENTS SIA GROUP


3.22 MATHEMATICS-II [JNTU-ANANTAPUR]
Multiply and divide by ‘p’, we get, Again differentiating equation (3) with respect to ‘x’,
we get,

2 ⎛ p ⎞ p
f (x)= ∫⎜
⎜1+ p2
⎟ × sin px dp
⎟ p d −2 dp

d
π
0⎝ ⎠
dx
( f ′( x )) =
π 1 + p dx
0
2 ∫
. (cos px )


2 p 2 sin( px ) ∞
⇒ f (x) =
π ∫ p(1 + p 2 )
.dp
f ′′(x) =
−2 dp
∫ ( − sin px × p )
0 π 1+ p2
0

∞ ∞
2 p 2 + 1 −1
⇒ f (x) =
π ∫ p(1 + p
0
2
)
sin px.dp f ′′(x) =
2
π ∫
p sin px
1+ p2
dp ... (4)
0

(Adding and subtracting 1 in the numerator) ⇒ f ′′(x) = f(x) [From equation (1)]

2 ⎡ ⎛⎜ (1 + p 2 )

1 ⎞ ⎤ ⇒ f ′′(x ) – f(x) = 0
⇒ f (x) = ⎢
∫ − ⎟ sin( px)dp ⎥
π ⎢ ⎜⎝ p(1 + p 2 ) p(1 + p 2 ) ⎟⎠ ⎥
⎣0 ⎦ d 2 f ( x)
⇒ – f (x) = 0 ... (5)
dx 2
2 ⎡ ⎛⎜ 1 ⎤

1 ⎞ The auxillary equation of the above equation is,
⇒ f (x) = ⎢ −∫ ⎟ sin( px)dp ⎥
π ⎢ ⎜⎝ p p(1 + p 2 ) ⎟⎠ ⎥ M2 – 1 = 0
⎣0 ⎦
⇒ M2 = 1

2 ⎡ sin( px) ⎤
∞ ∞ M= 1 =±1
sin( px)
⇒ f (x) =
π⎢

p ∫ .dp −
p (1 + p 2
)
dp ⎥
⎥ ∫ ∴ M1 = +1 and M2 = –1.
⎣0 0 ⎦
The general solution of the differential equation (5) is
given by,
⎛ ∞ sin px π ⎞
2 ⎡π ⎤

⎜Q = ⎟
⇒ f (x) = ⎢ −
π ⎢2
sin( px)
.∫
dp
p(1 + p 2 ) ⎥⎦
⎥ ⎜ ∫ p 2⎟
f(x) = c1 e M1 x + c2 e M 2 x
⎣ 0 ⎝ 0 ⎠
f(x) = c1ex + c2e–x ... (6)
∞ On differentiating equation (6) with respect to x, we get,
2 π 2 sin px
∴ f (x) = × −
π 2 π ∫ p(1 + p 2
)
dp
d
0 ⇒ (f(x)) = c1ex + c2e–x(–1)
dx
∞ = c1ex – c2e–x ... (7)
2 sin px
f (x) = 1 −
π ∫ p (1 + p
0
2
)
dp ... (2) Let substitute x = 0 in equation (2), we get,

2 sin p ( 0)
∫ p (1 + p
Differentiating equation (2) with respect to ‘x’, we get, ⇒ f (x) = 1 − dp
π 2
)
0
d ⎡ 2 ⎤

d sin px
dx
( f ( x )) = ⎢1 − dp ⎥
dx ⎢ π p (1 + p ) ⎥
2 ∫ ∞
⎣ ⎦ 2
0
= 1−
π ∫
(0) dp
0

d 2 dp d
⇒ f ′(x) = dx (1) − π ∫ . (sin px )
p (1 + p 2 ) dx
=1
0 Let x = 0 is equation (3), we get,

∞ ∞
df ( x) − 2 cos p(0)

2 p cos px
⇒ f ′(x) = 0 −
π ∫ p(1 + p
0
2
)
.dp
dx
=
π
0
1+ p2
dp

∞ ∞
− 2 cos px df ( x) − 2 cos 0
⇒ f ′(x) =
π 1+ p2
dp ∫ ... (3) dx
=
π 1+ p2
0
∫ dp
0

Look for the S IA GROUP LOGO on the TITLE COVER before you buy
UNIT-3 (Fourier Transform) 3.23
∞ 0 ∞
−2 1 1 1
=
π 1+ p2
0
∫dp (Q cos 0 = 1) =
2π ∫
−∞
e p ( y −ix ) dp +
2π ∫
e − p ( y +ix ) dp
0


[ ]
∞ ⎡ ⎤
0
−2 1 1 ⎡ e p ( y −ix) ⎤ 1 ⎡ e − p ( y +ix ) ⎤
=
π
tan −1 x
0 ⎢Q
⎣ ∫
1 + x 2
dx = tan −1 x ⎥
⎦ = ⎢ ⎥ + ⎢
2π ⎣⎢ y − ix ⎦⎥ − ∞ 2π ⎣⎢ − ( y + ix ) ⎦⎥

0

=
−2
π
[
tan −1 ( ∞) − tan −1 (0) ] =
1
[e 0 − e − ∞ ] −
1
[e −∞ − e 0 ]
2π( y − ix ) 2π( y + ix )
− 2 ⎡π ⎤
− 0⎥ 1 1
π ⎢⎣ 2
= = [1 − 0] − [0 − 1]

2π( y − ix ) 2π( y + ix )
−2 π ⎡ −∞ 1 1 ⎤
= × ⎢Q e = ∞ = ∞ = 0⎥
π 2 ⎣ e ⎦
= –1 1 1
= +
df ( x) 2π( y − ix ) 2π( y + ix )
On substituting and f(x) values in equation (7),
dx y + ix + y − ix
=
we get, 2π( y − ix )( y + ix )
⇒ 1 = c1e0 + c2e–0 and –1 = c1e0 – c2e–0
2y
⇒ c1 + c2 = 1 ... (8) =
2π( y − i 2 x 2 )
2
and c1 – c2 = –1 ... (9)
Solving equations (8) and (9), we get, y
= (Q i2 = –1)
⇒ c1 = 0 and c2 = 1 π( y + x 2 )
2

On substituting c1 and c2 values in equation (6), we get, y


∴ F −1[ F ( p)] = f ( x) =
∴ f(x) = (0)ex + 1 . e–x π ( y 2 + x2 )

f ( x) = e − x 3.4 FINITE FOURIER TRANSFORMS

Q34. Find the inverse Fourier transform f(x) of Q35. Find the finite cosine transform of,
F(p) = e–|p|y.
π x2
Ans: Given that, f(x) =
−x+ , 0 < x < π. Model Paper-I, Q7
F(p) = e–|p|y 3 2π
Ans: Given function is,
By the definition of inverse Fourier transform,
π x2
∞ f (x) = –x+ , 0<x<π
1 2π
∫e
ipx
.F ( p ) dp 3
f(x) =
2π Finite cosine of f(x) is given that,
−∞
l
nπx
1
∞ Fc(n) = ∫ f ( x) cos dx
∫e
ipx −| p| y l
= e dp 0

−∞
π
⎛π x 2 ⎞⎟ nπx
1
0 ∞
0⎝

= ⎜⎜ − x +
3 2 π ⎟ cos π dx

∫e ∫
− ipx py
= e dp + e −ipx e − py dp

−∞ 0 π
⎛π x2 ⎞
(Q | p |= − p if p ≤ 0


= ⎜⎜ − x +
3 2
⎟ cos nx dx
π ⎟⎠
and = + p if p ≥ 0) 0

π
1 ⎡ ⎤
0 ∞ π
⎛π x 2 ⎞ sin nx sin nx ⎛ x⎞
=
2π ⎢ ∫ ∫
⎢ e p ( y −ix ) dp + e − p ( y +ix ) dp ⎥

= ⎜⎜ − x + ⎟⎟
⎝3 2π ⎠ n
− ∫ n ⎝
⎜ − 1 + ⎟ dx
π⎠
⎣−∞ 0 ⎦ 0 0

SPECTRUM ALL-IN-ONE JOURNAL FOR ENGINEERING STUDENTS SIA GROUP


3.24 MATHEMATICS-II [JNTU-ANANTAPUR]
π
1 ⎛x ⎞
=0–
n ∫ ⎜⎝ π − 1⎟⎠ sin nx dx
0
[Q sin nx |0π = 0]

⎧⎪⎛ π
⎛ 1 ⎞ ⎫⎪
π
−1 x ⎞ − cos nx − cos nx
=
n
⎨⎜ − 1 + ⎟
⎪⎩⎝ π ⎠ n 0
− ∫
0
n
⎜ ⎟ dx ⎬
⎝ π ⎠ ⎪⎭

π
1 ⎡⎛ π⎞ ⎤ 1
= 2 ⎢⎜ − 1 + π ⎟(−1) − (−1)⎥ − n π − cos nx dx
n ⎣⎝ ⎠
n

⎦ 0

π⎞
1 ⎛⎜ 1 − sin nx ⎟
= –
n 2 ⎜⎝ nπ n ⎟
0⎠

1
= −0
n2
= 1/n2 ∀ n > 0
When,
n = 0,
π
⎛ x2 ⎞
0⎝

Fc(n) = ⎜⎜ π 3 − x + ⎟⎟ dx
2π ⎠

π
⎡π x 2 x3 ⎤ π2 π2 π3
= ⎢ x− + ⎥ = – +
⎣3 2 6π ⎦ 0 3 2 6π

π2 π2
= –
2 2

=0

π.
Q36. Find the finite Fourier sine and cosine transform of f(x) defined by f(x) = 4x where 0 < x < 4π
Ans: Given function is,
f(x) = 4x for 0 < x < 4π
By the definition of finite Fourier sine transform,
l
⎛ nπx ⎞
FS(n) = ∫ f ( x) sin⎜⎝
0
l ⎠
⎟dx


⎛ nπx ⎞
= ∫ 4 x. sin⎜⎝ 4π ⎟⎠dx
0


⎛ nx ⎞
= ∫ 4 x. sin⎜⎝ 4 ⎟⎠dx
0


⎡ ⎛ nx ⎞ ⎧ d ⎛ nx ⎞ ⎫ ⎤
⎢⎣ ∫
= ⎢4 x sin⎜ ⎟dx − ⎨
⎝ 4 ⎠ ⎩ dx

(4 x). sin ⎜ ⎟dx ⎬dx ⎥
⎝ 4 ⎠ ⎭ ⎥⎦ 0 ∫

Look for the S IA GROUP LOGO on the TITLE COVER before you buy
UNIT-3 (Fourier Transform) 3.25

⎡ ⎛ ⎛ nx ⎞ ⎞ ⎛ ⎛ nx ⎞ ⎞ ⎤
⎢ ⎜ − cos ⎜ ⎟ ⎟ ⎜ − cos⎜ ⎟ ⎟ ⎥
⎜ ⎝ 4 ⎠ ⎟ − 4.⎜ ⎝ 4 ⎠⎟ ⎥
= ⎢ 4 x⎜ ∫⎜ ⎛ n ⎞ ⎟dx ⎥
⎢ ⎛n⎞ ⎟
⎢ ⎜ ⎜ ⎟ ⎟ ⎜ ⎜ ⎟ ⎟ ⎥
⎣ ⎝ ⎝4⎠ ⎠ ⎝ ⎝ 4 ⎠ ⎠ ⎦0


⎡ ⎛ nx ⎞ ⎛ nx ⎞ ⎤
⎢ − 4 x × 4 cos⎜ 4 ⎟ 4 × 4 cos⎜ ⎟ ⎥
⎝ ⎠+ ⎝ 4 ⎠ dx ⎥
=⎢
⎢ n ∫ n ⎥
⎢ ⎥
⎣ ⎦0


⎡ ⎛ nx ⎞ ⎛ nx ⎞ ⎤
⎢ − 16 x cos ⎜ 4 ⎟ 16 cos⎜ ⎟ ⎥
⎝ ⎠+ ⎝ 4 ⎠ dx ⎥
=⎢
⎢ n ∫ n ⎥
⎢ ⎥
⎣ ⎦0

4π 4π
⎡ ⎛ nx ⎞ ⎤ ⎡ ⎛ nx ⎞ ⎛ ⎛ nx ⎞ ⎞ ⎤
⎢ − 16 x cos⎜ ⎟ ⎥ ⎢ − 16 x cos⎜ ⎟ ⎜ sin ⎜ ⎟ ⎟ ⎥
⎝ 4 ⎠ 16 ⎛ nx ⎞ ⎝ 4 ⎠ 16 ⎜ ⎝ 4 ⎠ ⎟⎥
=⎢ + cos⎜ ⎟dx ⎥ = ⎢
∫ + ∫ ⎜ ⎛ n ⎞ ⎟⎥
⎢ n n ⎝ 4 ⎠ ⎥ ⎢ n n
⎢ ⎥ ⎢ ⎜⎜ ⎜ ⎟ ⎟⎟ ⎥
⎣ ⎦0 ⎢⎣ ⎝ ⎝ 4 ⎠ ⎠ ⎦⎥ 0

4π 4π
⎡ ⎛ nx ⎞ ⎤ ⎡ ⎛ nx ⎞ ⎤
⎢ − 16 x cos⎜ ⎟ ⎥ ⎢ − 16 x cos⎜ ⎟ ⎥
⎝ ⎠
4 16 × 4 ⎛ ⎞
nx ⎝ ⎠
4 64 ⎛ ⎞
nx
=⎢ + sin⎜ ⎟⎥ = ⎢ + 2 sin⎜ ⎟⎥
⎢ n n×n ⎝ 4 ⎠⎥ ⎢ n n ⎝ 4 ⎠⎥
⎢ ⎥ ⎢ ⎥
⎣ ⎦0 ⎣ ⎦0

⎡⎧ ⎛ n(4π) ⎞ ⎫ ⎧ ⎛ n(0) ⎞ ⎫⎤
⎢⎪ − 16(4π) cos⎜ ⎟ ⎪ ⎪ − 16(0) cos ⎜ ⎟ ⎪⎥
⎢⎪ ⎝ 4 ⎠ 64 ⎛ n(4π) ⎞⎪ ⎪ ⎝ 4 ⎠ 64 ⎛ n(0) ⎞⎪⎥
= ⎢⎨ + 2 sin⎜ ⎟⎬ − ⎨ + 2 sin⎜ ⎟⎬
⎪ n n ⎝ 4 ⎠⎪ ⎪ n n ⎝ 4 ⎠⎪⎥
⎢ ⎥
⎪ ⎪⎭ ⎪⎩ ⎪⎭⎥
⎣⎢⎩ ⎦

⎡⎧ ⎛ 4nπ ⎞ ⎫ ⎧ ⎛0⎞ ⎫⎤
⎢⎪ − 64π cos⎜ ⎟ ⎪ ⎪ 0 cos ⎜ ⎟ ⎪⎥
⎢⎪⎨ ⎝ 4 ⎠ 64 ⎛ 4nπ ⎞⎪ ⎪ ⎝ 4 ⎠ + 64 ⎪⎥
=⎢ + sin ⎜ ⎟ −
⎬ ⎨ sin (0 )⎬⎥ (Q sin(0) = 0)
n 2
⎝ 4 ⎠⎪ ⎪ n 2
⎢⎪⎪ ⎪⎥
n n
⎢⎣⎩ ⎪⎭ ⎪⎩ ⎪⎭⎥⎦

⎡⎧ − 64π cos (nπ) 64


= ⎢⎨
⎫ ⎤
+ 2 sin(nπ)⎬ − {0 + 0}⎥ (Q cos nπ = (−1) n
and sin nπ = 0 )
⎣⎩ n n ⎭ ⎦

⎡ − 64π(− 1)n 64 ⎤
=⎢ + 2 (0)⎥ (Q sin nπ = 0)
⎣⎢ n n ⎦⎥

− 64π(−1) n
=
n

− 64π(−1) n
∴ FS(n) =
n

SPECTRUM ALL-IN-ONE JOURNAL FOR ENGINEERING STUDENTS SIA GROUP


3.26 MATHEMATICS-II [JNTU-ANANTAPUR]
The finite Fourier cosine transform is given by,
l
⎛ nπx ⎞
FC(n) = ∫ f ( x) cos⎜⎝
0
l ⎠
⎟ dx

4π 4π
⎛ nπx ⎞ ⎛ nx ⎞
= ∫
0
4 x cos⎜
⎝ 4π ⎠
⎟ dx = ∫ 4 x. cos⎜⎝ 4 ⎟⎠dx
0


⎡ ⎛ ⎛ nx ⎞ ⎞ ⎛ ⎛ nx ⎞ ⎞ ⎤

4π ⎢ ⎜ sin⎜ ⎟ ⎟ ⎜ sin⎜ ⎟ ⎟ ⎥
⎛ nx ⎞ ⎧ d ⎛ nx ⎞ ⎫ ⎤ ⎝ 4 ⎠⎟
⎜ − 4⎜⎜ ⎝ ⎠ ⎟⎟dx ⎥
⎢ 4
⎢⎣ ∫
= ⎢ 4 x cos⎜ ⎟ dx − ⎨
⎝ 4 ⎠ ⎩ dx
∫ ∫
( 4 x) cos ⎜ ⎟ dx ⎬dx ⎥
⎝ 4 ⎠ ⎭ ⎥⎦ 0
= 4 x⎜
⎢ ⎛n⎞ ⎟ ∫ ⎛n⎞ ⎥
⎢ ⎜ ⎜ ⎟ ⎟ ⎜ ⎜ ⎟ ⎟ ⎥
⎣ ⎝ ⎝4⎠ ⎠ ⎝ ⎝ 4 ⎠ ⎠ ⎦0

4π 4π
⎡ ⎛ ⎛ nx ⎞ ⎞ ⎛ nx ⎞ ⎤ ⎡ ⎛ nx ⎞ ⎡ ⎛ nx ⎞ ⎤ ⎤
⎢ ⎜ 4 sin⎜ ⎟ ⎟ 4 sin⎜ ⎟ ⎥ ⎢ 16 x sin ⎜ ⎟ ⎢ − cos ⎜ ⎟ ⎥ ⎥
⎜ ⎝ 4 ⎠⎟−4 ⎝ 4 ⎠ dx ⎥ ⎢ ⎝ 4 ⎠ 16 ⎢ ⎝ 4 ⎠ ⎥⎥
= ⎢ 4 x⎜
⎢ n ⎟ n ∫

=⎢
n

n ⎢ ⎛ n ⎞ ⎥⎥
⎢ ⎜ ⎟ ⎥ ⎢ ⎢ ⎜ ⎟ ⎥⎥
⎣ ⎝ ⎠ ⎦0 ⎢⎣ ⎣ ⎝ 4 ⎠ ⎦ ⎥⎦ 0

4π 4π
⎡ ⎛ nx ⎞ ⎤ ⎡ ⎛ nx ⎞ ⎤
⎢16 x sin⎜ ⎟ ⎥ ⎢16 x sin ⎜ ⎟ ⎥
⎝ ⎠
4 16 4 ⎛ nx ⎞ ⎝ ⎠
4 64 ⎛ nx ⎞
=⎢ + × . cos⎜ ⎟⎥ = ⎢ + 2 cos ⎜ ⎟⎥
⎢ n n n ⎝ 4 ⎠⎥ ⎢ n n ⎝ 4 ⎠⎥
⎢ ⎥ ⎢ ⎥
⎣ ⎦0 ⎣ ⎦0

⎡⎧ ⎛ n(4π) ⎞ ⎫ ⎧ ⎛ n(0) ⎞ ⎫⎤
⎢⎪16(4π) sin⎜ ⎟ ⎪ ⎪16(0) sin⎜ ⎟ ⎪⎥
⎢⎪ ⎝ 4 ⎠ 64 ⎛ n(4π) ⎞⎪ ⎪ ⎝ 4 ⎠ 64 ⎛ n(0) ⎞⎪⎥
= ⎢⎨ + cos ⎜ −
⎟⎬ ⎨ + cos ⎜ ⎟⎬
n n2 ⎝ 4 ⎠⎪ ⎪ n n2 ⎝ 4 ⎠ ⎪⎥
⎢⎪ ⎥
⎪ ⎪⎭ ⎪⎩ ⎪⎭⎥
⎣⎢⎩ ⎦

⎡⎧ ⎛ 4nπ ⎞ ⎫ ⎤
⎢⎪ 64π. sin⎜ ⎟ ⎪ ⎥
⎢⎪⎨ ⎝ 4 ⎠ 64 ⎛ 4nπ ⎞⎪ ⎧ 64 ⎫⎥
=⎢ + cos ⎜ ⎟ ⎬ − ⎨0 + cos (0 )⎬
n n2 ⎝ 4 ⎠⎪ ⎩ n 2 ⎭⎥
⎢⎪⎪ ⎪ ⎥
⎣⎢⎩ ⎭ ⎦⎥

⎡⎧ 64π. sin nπ 64 ⎫ ⎧ 64 ⎫⎤
= ⎢⎨ + 2 cos nπ⎬ − ⎨ 2 (1)⎬⎥
⎣⎩ n n ⎭ ⎩n ⎭⎦

⎡ 64π × (0) 64(−1) n 64 ⎤


=⎢ + − 2⎥ [Q sin nπ = 0 and cos nπ = (–1)n]
⎣⎢ n n2 n ⎦⎥

64(−1) n 64
= − 2
n2 n

=
64
n2
((−1) − 1)
n

∴ FC (n) =
64
n2
(
(−1)n − 1 )
Look for the S IA GROUP LOGO on the TITLE COVER before you buy
UNIT-3 (Fourier Transform) 3.27
2
⎛ x⎞
Q37. Find the finite Fourier sine transform of f(x) defined by f(x) = ⎜ 1 − ⎟ , where 0 < x < π .
⎝ π⎠
Ans: Given function is,
2
⎛ x⎞
f(x) = ⎜ 1– ⎟ , where 0 < x < π
⎝ π⎠
The Finite Fourier sine transform of f(x) is given by,
l
⎛ nπx ⎞
FS(n) = ∫ f ( x) sin⎜⎝
0
l ⎠
⎟dx

π 2 π
⎛ x⎞ ⎛ nπx ⎞ ⎛ x⎞
2

0

= ⎜1 − π ⎟ sin ⎜ π ⎟ dx =
⎝ ⎠ ⎝ ⎠ ∫ ⎜1 − ⎟ sin (nx )dx
⎝ π⎠
0

π
⎡⎛ x ⎞ 2 ⎧⎪ d ⎛ x ⎞ 2 ⎫⎪ ⎤
⎢⎝ π ⎠ ∫
= ⎢⎜1 − ⎟ sin( nx ) dx − ∫ ∫
⎨ ⎜1 − ⎟ sin( nx ) dx ⎬dx ⎥
⎪⎩ dx ⎝ π ⎠ ⎪⎭ ⎥⎦
⎣ 0

π
⎡⎛ x ⎞ 2 ⎛ − cos( nx) ⎞ ⎛ x ⎞⎛ − 1 ⎞⎛ − cos( nx) ⎞ ⎤
= ⎢⎜1 − ⎟ ⎜
⎢⎣⎝ π ⎠ ⎝ n ⎠ ∫
⎟ − 2⎜1 − ⎟⎜ ⎟⎜
⎝ π ⎠⎝ π ⎠⎝ n
⎟dx ⎥
⎠ ⎥⎦ 0

π
⎡⎛ x ⎞ 2 ⎛ − cos( nx ) ⎞ 2 ⎛ x ⎞ ⎤
= ⎢⎜1 − ⎟ ⎜ ⎟− ⎜1 − ⎟(cos( nx ) )dx ⎥

⎣⎢⎝ π ⎠ ⎝ n ⎠ nπ ⎝ π ⎠ ⎦⎥ 0

π
⎡⎛ x ⎞ 2 ⎛ − cos( nx) ⎞ 2 ⎡⎛ x ⎞ ⎧ d ⎛ x⎞ ⎫ ⎤⎤
= ⎢⎜1 − ⎟ ⎜
⎢⎣⎝ π ⎠ ⎝ n
⎟− ∫ ∫ ∫
⎢⎜1 − ⎟ cos( nx)dx − ⎨ ⎜1 − ⎟ cos( nx)dx ⎬dx ⎥ ⎥
⎠ nπ ⎣⎢⎝ π ⎠ ⎩ dx ⎝ π ⎠ ⎭ ⎦⎥ ⎥⎦ 0

π
⎡⎛ x ⎞ 2 ⎛ − cos( nx) ⎞ 2 ⎡⎛ x ⎞⎛ sin(nx) ⎞ ⎛ − 1 ⎞⎛ sin(nx) ⎞ ⎤ ⎤
= ⎢⎜1 − ⎟ ⎜
⎢⎣⎝ π ⎠ ⎝ n
⎟− ⎢⎜1 − ⎟⎜
⎠ nπ ⎣⎝ π ⎠⎝ n ⎠
⎟ − ⎜ ⎟⎜ ∫
⎝ π ⎠⎝ n ⎠ ⎦ ⎥⎦ 0
⎟dx ⎥ ⎥

π
⎡⎛ x ⎞ 2 ⎛ − cos( nx) ⎞ 2 ⎡⎛ x ⎞⎛ sin(nx) ⎞ 1 ⎛ sin(nx) ⎞ ⎤ ⎤
= ⎢⎜1 − ⎟ ⎜ ⎟− ⎢⎜1 − ⎟⎜ ⎟+ ⎜ ⎟dx ⎥ ⎥
⎢⎣⎝ π ⎠ ⎝ n ⎠ nπ ⎣⎝ π ⎠⎝ n ⎠ π ⎝ n ⎠ ⎦ ⎥⎦
0

π
⎡⎛ x ⎞ 2 ⎛ − cos( nx) ⎞ 2 ⎡⎛ x ⎞⎛ sin(nx ) ⎞ 1 ⎛ − cos( nx ) ⎞⎤ ⎤
= ⎢⎜1 − ⎟ ⎜
⎢⎣⎝ π ⎠ ⎝ n
⎟− ⎢⎜1 − ⎟⎜
⎠ nπ ⎣⎝ π ⎠⎝ n ⎠ π ⎝
⎟+ ⎜
n2 ∫ ⎟⎥ ⎥
⎠⎦ ⎥⎦ 0

π
⎡⎛ x ⎞ 2 ⎛ − cos( nx) ⎞ 2 ⎛ x ⎞⎛ sin(nx) ⎞ 2 ⎛ cos( nx) ⎞⎤
= ⎢⎜1 − ⎟ ⎜ ⎟ − ⎜1 − ⎟⎜ ⎟+ 2 ⎜ ⎟⎥
⎢⎣⎝ π ⎠ ⎝ ⎠ nπ ⎝ π ⎠⎝ n ⎠ nπ ⎝ n
2
n ⎠⎥⎦ 0

⎡⎧⎪⎛ π ⎞ 2 ⎛ − cos nπ ⎞ ⎛ 0 ⎞ 2 ⎛ − cos( n × 0) ⎞⎫⎪ ⎧ 2 ⎛ π ⎞⎛ sin(n × π) ⎞


= ⎢⎢⎨⎜1 − π ⎟ ⎜ ⎟ − ⎜1 − ⎟ ⎜
π⎠ ⎝
⎟⎬ − ⎨ ⎜1 − ⎟⎜
π π
⎟−
⎣⎩⎪⎝ ⎠ ⎝ n ⎠ ⎝ n ⎠ ⎪
⎭ ⎩ n ⎝ ⎠⎝ n ⎠

2 ⎛ 0 ⎞⎛ sin(n × 0) ⎞⎫ ⎧ 2 ⎛ cos( nπ) ⎞ 2 ⎛ cos( n × 0) ⎞⎫⎤


⎜1 − ⎟⎜ ⎟⎬ + ⎨ 2 ⎜ ⎟− 2 ⎜ ⎟ ⎬⎥
nπ ⎝ π ⎠⎝ n ⎠ ⎭ ⎩ nπ ⎝ n
2
⎠ nπ ⎝ n2 ⎠⎭⎦⎥

SPECTRUM ALL-IN-ONE JOURNAL FOR ENGINEERING STUDENTS SIA GROUP


3.28 MATHEMATICS-II [JNTU-ANANTAPUR]

⎡⎧ 1 ⎫ ⎧ 2 ⎛ π ⎞⎛ 0 ⎞ 2 ⎫ ⎧⎪ 2 ⎛ (−1) n ⎞ ⎫⎤
⎟ − 2 ⎛⎜ 1 ⎞⎟⎪⎬⎥
= ⎢⎨0 + n ⎬ − ⎨ nπ ⎜1 − π ⎟⎜ n ⎟ − nπ (1 − 0)(0)⎬ + ⎨ 2

⎜ n ⎟ nπ ⎝ n ⎠⎪⎥ [Q cos nπ = (–1)n]
⎭ ⎪⎩ nπ
2 2 2
⎢⎣⎩ ⎭ ⎩ ⎝ ⎠⎝ ⎠ ⎝ ⎠ ⎭⎦

⎡⎧ 1 ⎫ ⎧ 2
[ ⎫⎤
= ⎢⎨ n ⎬ − {0}+ ⎨ 3 2 (−1) − 1 ⎬⎥
n
]
⎣⎩ ⎭ ⎩n π ⎭⎦

1 2(−1) n 2
= + 3 2 − 3 2
n n π n π

1 2( −1) n 2
∴ FS (n) = + 3 2 − 3 2
n nπ nπ

x
Q38. Find the finite Fourier sine and cosine transforms of f(x) defined by f(x) = , where 0 < x < 4.
π
x
Ans: Given function is, f(x) = , for 0 < x < 4.
π
The finite Fourier sine transform of the function f(x) is given by,
l 4
⎛ nπx ⎞ ⎛x⎞ ⎛ nπx ⎞
FS(n) = ∫
0
f ( x ) sin ⎜
⎝ l ⎠
⎟dx = ∫ ⎜⎝ π ⎟⎠ sin⎜⎝
0
4 ⎠
⎟ dx

4
⎡x ⎛ nπx ⎞ ⎧d ⎛x⎞ ⎛ nπx ⎞ ⎫ ⎤
=⎢
⎢⎣ π ∫
sin ⎜
⎝ 4 ⎠ ∫
⎟ dx − ⎨ ⎜ ⎟ sin ⎜
⎩ dx ⎝ π ⎠

⎟ dx ⎬dx ⎥
⎝ 4 ⎠ ⎭ ⎥⎦ 0

4
⎡ ⎡ ⎛ nπx ⎞ ⎤ ⎛ ⎛ nπx ⎞ ⎞ ⎤ ⎡ ⎡ ⎛ nπx ⎞ ⎤ ⎛ nπx ⎞ ⎤
4

⎢ ⎢ − cos⎜ ⎟⎥ ⎜ − cos⎜ ⎟⎟ ⎥ ⎢ ⎢ − 4 cos⎜ ⎟⎥ 4 cos⎜ ⎟ ⎥


⎢ x ⎝ 4 ⎠ 1 ⎜ ⎝ 4 ⎠ ⎟ ⎥
= ⎢
⎢ π ⎢ ⎛ nπ ⎞ ⎥
⎥ − ∫
π ⎜ ⎛ nπ ⎞ ⎟ ⎥
dx = ⎢ x
⎢π ⎢
⎢ ⎝

4 ⎠⎥ 1
+
⎥ π ∫
⎝ 4 ⎠ ⎥

dx

⎢ ⎢ ⎜ ⎟ ⎥ ⎜ ⎜ ⎟ ⎟ ⎥ ⎢ ⎥
⎣ ⎣ ⎝ 4 ⎠ ⎦ ⎝ ⎝ 4 ⎠ ⎠ ⎦ 0 ⎣⎢ ⎢⎣ ⎥
⎦ ⎦⎥ 0
4 4
⎡ ⎡ ⎛ nπx ⎞ ⎤ ⎤ ⎡ ⎡ ⎛ nπx ⎞ ⎤ ⎡ ⎛ nπx ⎞ ⎤ ⎤
⎢ ⎢ − 4 cos⎜ ⎟⎥ ⎥ ⎢ ⎢ − 4 cos⎜ ⎟⎥ ⎢ sin ⎜ ⎟ ⎥⎥
x
=⎢ ⎢ ⎝ 4 ⎠ ⎥ + 4 cos⎛ nπx ⎞dx ⎥ = ⎢ x ⎢ ⎝ 4 ⎠⎥ 4 ⎢ ⎝ 4 ⎠ ⎥⎥
⎢π ⎢ nπ ⎥ nπ 2

⎝ 4

⎠ ⎥ ∫ ⎢ π ⎢ n π ⎥
+ 2
n π ⎢ ⎛ nπ ⎞ ⎥ ⎥
⎢ ⎢ ⎥ ⎥ ⎢ ⎢ ⎥ ⎢ ⎜ ⎟ ⎥⎥
⎣ ⎣ ⎦ ⎦ 0 ⎢⎣ ⎣ ⎦ ⎣ ⎝ 4 ⎠ ⎦ ⎥⎦ 0

4 4
⎡ ⎡ ⎛ nπx ⎞ ⎤ ⎤ ⎡ ⎛ nπx ⎞ ⎛ nπx ⎞ ⎤
⎢ ⎢ − 4 cos⎜ ⎟⎥ ⎥ ⎢ − 4 x cos ⎜ 4 ⎟ 16 sin ⎜ 4 ⎟ ⎥
x ⎝ 4 ⎠ ⎥ + 4 × 4 sin⎛ nπx ⎞⎥ ⎝ ⎠ ⎝ ⎠⎥
=⎢ ⎢ ⎜ ⎟ = ⎢ +
⎢π ⎢ nπ ⎥ n 2 π3 ⎝ 4 ⎠⎥ ⎢ nπ 2
n π
2 3

⎢ ⎢ ⎥ ⎥ ⎢ ⎥
⎣ ⎣ ⎦ ⎦0 ⎣ ⎦0

⎡⎧ ⎛ nπ(4) ⎞ ⎛ nπ(4) ⎞ ⎫ ⎧ ⎛ nπ(0) ⎞ ⎛ nπ(0) ⎞ ⎫⎤


⎢⎪ − 4(4) cos⎜ ⎟ 16 sin ⎜ ⎟ ⎪ ⎪ − 4(0) cos ⎜ ⎟ 16 sin⎜ ⎟ ⎪⎥
⎢⎪ ⎝ 4 ⎠+ ⎝ 4 ⎠⎪− ⎪ ⎝ 4 ⎠+ ⎝ 4 ⎠ ⎪⎥
= ⎢⎨ ⎬ ⎨ ⎬⎥
nπ 2 n 2π3 nπ 2 n 2 π3
⎢⎪ ⎪ ⎪ ⎪⎥
⎢⎣⎪⎩ ⎪⎭ ⎪⎩ ⎪⎭⎥

⎡⎧ − 16 cos(nπ) 16 sin (nπ)⎫ ⎧ 16 sin (0)⎫⎤


= ⎢⎨ + ⎬ − ⎨0 + 2 3 ⎬⎥ [Q sin(0) = 0]
⎣⎩ nπ 2 n 2 π3 ⎭ ⎩ n π ⎭⎦

⎡⎧ − 16 cos (nπ) 16 sin (nπ)⎫ ⎤


= ⎢⎨ + ⎬ − {0 + 0}⎥
⎣⎩ nπ 2
n π
2 3
⎭ ⎦

Look for the S IA GROUP LOGO on the TITLE COVER before you buy
UNIT-3 (Fourier Transform) 3.29
16 cos (nπ ) 16 sin (nπ )
=− + [Q sin nπ = 0 and cos nπ = (–1)n]
nπ 2 n 2 π3

16(− 1)n 16(0) 16(− 1)n


=− + = −
nπ 2 n 2 π3 nπ 2

16(− 1)
n
∴ FS(n) = −
nπ 2
The finite Fourier cosine transform of f(x) is given by,
l
⎛ nπx ⎞
FC(n)= ∫ f ( x) cos⎜⎝
0
l ⎠
⎟ dx

4 4
⎛ x ⎞ ⎛ nπx ⎞ ⎡x ⎛ nπx ⎞ ⎧ d ⎛ x⎞ ⎛ nπx ⎞ ⎫ ⎤
0

= ⎜ π ⎟ cos⎜ 4 ⎟ dx =
⎝ ⎠ ⎝ ⎠
⎢ ∫
⎢⎣ π
cos⎜
⎝ 4 ⎠ ∫
⎟ dx − ⎨ ⎜ ⎟ cos ⎜
⎩ dx ⎝ π ⎠
∫ ⎟ dx ⎬dx ⎥
⎝ 4 ⎠ ⎭ ⎥⎦ 0

4 4
⎡ ⎛ ⎛ nπx ⎞ ⎞ ⎛ ⎛ nπx ⎞ ⎞ ⎤ ⎡ ⎡ ⎛ nπx ⎞ ⎤ ⎛ nπx ⎞ ⎤
⎢ ⎜ sin⎜ ⎟⎟ ⎜ − sin⎜ ⎟⎟ ⎥ ⎢ ⎢ 4 sin⎜ ⎟⎥ 4 sin⎜ ⎟ ⎥
⎢ x ⎜ ⎝ 4 ⎠⎟ 1⎜ ⎝ 4 ⎠⎟ ⎥ ⎢x ⎢ ⎝ 4 ⎠⎥ − 1 ⎝ 4 ⎠ dx ⎥
=
⎢ π ⎜ ⎛ nπ ⎞ ⎟
− ∫
π ⎜ ⎛ nπ ⎞ ⎟ ⎥
dx =
⎢π ⎢ nπ ⎥ π ∫ nπ ⎥
⎢ ⎜ ⎜ ⎟ ⎟ ⎜ ⎜ ⎟ ⎟ ⎥ ⎢ ⎢ ⎥ ⎥
⎣ ⎝ ⎝ 4 ⎠ ⎠ ⎝ ⎝ 4 ⎠ ⎠ ⎦0 ⎣ ⎣ ⎦ ⎦0

4 4
⎡ ⎛ nπx ⎞ ⎤ ⎡ ⎛ nπx ⎞ ⎡ ⎛ nπx ⎞ ⎤ ⎤
⎢ x 4 sin⎜ 4 ⎟ π ⎥ ⎢ 4 sin⎜ ⎟ ⎢ − cos ⎜ 4 ⎟ ⎥ ⎥
⎛ ⎞ ⎝ ⎠− 4 ⎛ n x ⎞ ⎛ ⎞
= ⎢⎜ ⎟
⎢⎝ π ⎠ nπ nπ 2
sin ⎜ ∫ ⎟
⎝ 4 ⎠ ⎥
dx ⎥ = x
⎢⎜ ⎟
⎢⎝ π ⎠
⎝ 4 ⎠− 4
nπ nπ 2

⎢ ⎛ ⎞ ⎥⎥
n

π
⎠ ⎥⎥



⎦0 ⎢ ⎢ ⎜ 4 ⎟ ⎥⎥
⎣ ⎣ ⎝ ⎠ ⎦⎦ 0

4 4
⎡ ⎛ nπx ⎞ ⎛ nπx ⎞ ⎤ ⎡ ⎛ nπx ⎞ ⎛ nπx ⎞ ⎤
⎢ 4 x sin⎜ 4 ⎟ 4 × 4 × cos⎜ 4 ⎟ ⎥ ⎢ 4 x sin⎜ 4 ⎟ 16 cos⎜ 4 ⎟ ⎥
⎝ ⎠+ ⎝ ⎠⎥ ⎝ ⎠+ ⎝ ⎠⎥
=⎢ = ⎢
⎢ nπ 2
n π
2 3
⎥ ⎢ nπ 2
n π
2 3

⎢ ⎥ ⎢ ⎥
⎣ ⎦0 ⎣ ⎦0

⎡⎧ ⎛ nπ( 4) ⎞ ⎛ nπ( 4) ⎞ ⎫ ⎧ ⎛ n π (0 ) ⎞ ⎛ nπ( 0) ⎞ ⎫⎤


⎢ ⎪ 4( 4) sin ⎜ ⎟ 16 cos⎜ ⎟ ⎪ ⎪ 4(0) sin ⎜ ⎟ 16 cos⎜ ⎟ ⎪⎥
⎢ ⎪ ⎝ 4 ⎠+ ⎝ 4 ⎠⎪− ⎪ ⎝ 4 ⎠+ ⎝ 4 ⎠ ⎪⎥
= ⎨ ⎬ ⎨ ⎬
⎢⎪ nπ 2 n 2 π3 ⎪ ⎪ nπ 2 n 2 π3 ⎪⎥
⎢⎪ ⎪ ⎪ ⎪⎭⎥⎦
⎣⎩ ⎭ ⎩

⎡⎧16 sin nπ 16 cos nπ ⎫ ⎧ 16 cos 0 ⎫⎤


= ⎢⎨ + ⎬ − ⎨0 + 2 3 ⎬⎥
⎣⎩ nπ n2π3 ⎭ ⎩ n π ⎭⎦
2

⎡ 16 sin nπ 16 cos nπ 16 ⎤
=⎢ + − 2 3⎥ [Q cos 0 = 1]
⎣ nπ 2 n 2 π3 n π ⎦

⎡16(0) 16(− 1)n 16 ⎤


= ⎢ 2 + 2 3 − 2 3⎥ [Q sin nπ = 0 and cos nπ = (–1)n]
⎣⎢ nπ n π n π ⎦⎥

16(− 1)n
=
n π
2 3
16
n π
16
n π
[
− 2 3 = 2 3 ( −1) n − 1 ]
16 ⎡
∴ FC (n) = (−1)n − 1⎤⎦
n 2π 3 ⎣

SPECTRUM ALL-IN-ONE JOURNAL FOR ENGINEERING STUDENTS SIA GROUP


3.30 MATHEMATICS-II [JNTU-ANANTAPUR]
π – x) in 0 < x < π .
Q39. Find the finite Fourier sine and cosine transform of f(x) = x (π Model Paper-I, Q7

Ans: Given function is, f(x) = x (π – x) in 0 < x < π.


The Finite Fourier sine transform of f(x) is given by,

l
⎛ nπx ⎞
FS(n) = ∫ f ( x) sin⎜⎝
0
l ⎠
⎟dx

π π
⎛ nπx ⎞
= ∫0
f ( x ) sin ⎜
⎝ π ⎠
⎟dx = ∫ f ( x) sin nx dx
0

π π

∫ ∫ ( πx − x
2
= x( π − x) sin nx dx = ) sin nx dx
0 0

[ ]
π π
⎡ ⎧d ⎫ ⎤
∫ 0
⎣ ⎩ dx ∫
= (πx − x 2 ) sin( nx ) dx − ⎢ ⎨ ( πx − x 2 ) sin nxdx ⎬dx ⎥
⎭ ⎦0 ∫
π π
⎡ 2 ⎡ − cos nx ⎤ ⎤ ⎡ ⎛ − cos nx ⎞ ⎤
= ⎢(πx − x )⎢
⎣ ⎣ n ⎥ ⎥ − ⎢ (π − 2 x )⎜
⎦⎦ 0 ⎣ ⎝ ∫n ⎠ ⎦0
⎟dx ⎥

π π
⎡ ⎡ − cos nx ⎤ ⎤ ⎡⎡ ⎛ − cos nx ⎞ ⎛ − cos nx ⎞ ⎤ ⎤
= ⎢( πx − x 2 ) ⎢

⎥ ⎥ − ⎢ ⎢( π − 2 x ⎜
⎣ n ⎦ ⎦ 0 ⎢⎣ ⎣ ⎝ n ⎠∫
⎟ − ( −2) ⎜
⎝ n ∫ ⎠ ⎦ ⎥⎦ 0∫
⎟dx ⎥ dx ⎥

π π
⎡ 2 ⎡ − cos nx ⎤ ⎤
⎡⎡ ⎛ − cos nx ⎞ ⎛ − sin nx ⎞ ⎤ ⎤
= ⎢(πx − x )⎢ n ⎥ ⎥ − ⎢⎢(π − 2 x ⎜
⎣ ⎣ ⎦ ⎦ 0 ⎢⎣⎣ ⎝ n
⎟ + (2) ⎜
⎠ ⎝ n ∫
2
⎟dx ⎥ ⎥
⎠ ⎦ ⎥⎦ 0 ∫ ∫
π π
⎡ 2 ⎛ − cos nx ⎞ ⎤ ⎡⎡ ⎛ − sin nx ⎞ ⎛ sin nx ⎞ ⎤ ⎤
= ⎢ ( πx − x )⎜
⎣ ⎝ n
⎟ ⎥ − ⎢ ⎢( π − 2 x )⎜
⎠ ⎦ 0 ⎢⎣ ⎣ ⎝ n 2
⎟ − 2 ⎜ 2 ⎟ dx ⎥ ⎥
⎠ ⎝ n ⎠ ⎦ ⎦⎥ 0∫
π π
⎡ 2 ⎛ − cos nx ⎞⎤ ⎡ ⎛ − sin nx ⎞ ⎛ − cos nx ⎞⎤
= ⎢(πx − x )⎜
⎣ ⎝
⎟⎥ − ⎢(π − 2 x )⎜
n ⎠⎦ 0 ⎣ ⎝ n 2
⎟−2 ⎜
⎠ ⎝ n3 ⎠⎦ 0
⎟⎥ ∫
π π
⎡ 2 ⎛ − cos nx ⎞⎤ ⎡ sin nx cos nx ⎤
= ⎢(πx − x )⎜ ⎟⎥ + ⎢(π − 2 x ) 2 − 2 3 ⎥
⎣ ⎝ n ⎠⎦ 0 ⎣ n n ⎦0

⎡⎧ 2 ⎛ − cos nπ ⎞ 2 ⎛ − cos n(0) ⎞ ⎫ ⎧ sin nπ sin n(0) ⎫ ⎧ cos nπ cos n (0) ⎫⎤


= ⎢ ⎨(π × π − π )⎜ ⎟ − (π (0 ) − (0 ) ⎜ ⎟ ⎬ + ⎨(π − 2 π) 2 − ( π − 2(0)) ⎬ − 2⎨ − ⎬⎥
⎣⎢ ⎩ ⎝ n ⎠ ⎝ n ⎠⎭ ⎩ n n 2 ⎭ ⎩ n3 n 3 ⎭⎦

⎡ ⎧⎪ ( −1) n 1 ⎫⎪⎤
= ⎢ − 2⎨ 3 − 3 ⎬ ⎥
⎣⎢ ⎪⎩ n n ⎪⎭⎦⎥

=
−2
n3
[(−1) − 1]
n

2
∴ Fs (n ) = [1 − (−1) n ]
n3

Look for the S IA GROUP LOGO on the TITLE COVER before you buy
UNIT-3 (Fourier Transform) 3.31
The finite Fourier cosine transform of f(x) is given by,
l
⎛ nπx ⎞
FC(n) = ∫ f ( x) cos⎜⎝
0
l ⎠
⎟dx

π
⎛ nπx ⎞
= ∫ x(π − x) cos⎜⎝
0
π ⎠
⎟ dx

= ∫ x(π − x) cos(nx)dx
0

∫ (πx − x
2
= ) cos( nx) dx
0

[ ]
π π
⎡ ⎧d ⎫ ⎤
∫ ∫
= (πx − x ) cos( nx)dx − ⎢ ⎨ (πx − x ) cos( nx )dx ⎬dx ⎥ ∫
2 2
0 ⎣ ⎩ dx ⎭ ⎦0

π π
⎡ 2 ⎛ sin nx ⎞ ⎤ ⎡ ⎧ ⎛ sin nx ⎞ ⎫ ⎤
= ⎢ ( πx − x )⎜

⎟ ⎥ − ⎢ ⎨( π − 2 x )⎜
⎝ n ⎠ ⎦ 0 ⎢⎣ ⎩ ∫ ⎟ ⎬dx ⎥
⎝ n ⎠ ⎭ ⎦⎥ 0

π π
⎡ 2 ⎛ sin nx ⎞ ⎤ ⎡⎡ ⎛ sin nx ⎞ d ⎛ sin nx ⎞ ⎤ ⎤
= ⎢ ( πx − x )⎜

⎟ ⎥ − ⎢ ⎢ ( π − 2 x) ⎜
⎝ n ⎠ ⎦ 0 ⎢⎣ ⎣ ⎝ n ⎠
⎟ dx −∫dx ∫
( π − 2 x) ⎜ ⎟dx ⎥ ⎥
⎝ n ⎠ ⎦ ⎥⎦ 0 ∫
π π
⎡ 2 ⎛ sin nx ⎞ ⎤ ⎡ ⎛ − cos nx ⎞ ⎛ − cos nx ⎞ ⎤
= ⎢(πx − x )⎜
⎣ ⎝ n
⎟ ⎥ − ⎢ ( π − 2 x )⎜
⎠⎦ 0 ⎣ ⎝ n 2
⎟ − (−2)⎜
⎠ ⎝ n2 ⎠ ⎦0 ∫
⎟dx ⎥

π π
⎡ 2 ⎛ sin nx ⎞ ⎤ ⎡ ⎛ − cos nx ⎞ ⎛ sin nx ⎞⎤
= ⎢(πx − x )⎜ ⎟⎥ − ⎢(π − 2 x)⎜ ⎟ − 2⎜ ⎟⎥
⎣ ⎝ n ⎠⎦ 0 ⎣ ⎝ n 2 ⎠ ⎝ n 3 ⎠⎦ 0

π π
⎡ 2 ⎛ sin nx ⎞ ⎤ ⎡ ⎛ cos nx ⎞ ⎛ sin nx ⎞⎤
= ⎢(πx − x )⎜ ⎟⎥ + ⎢(π − 2 x)⎜ 2 ⎟ + 2⎜ 3 ⎟⎥
⎣ ⎝ n ⎠⎦ 0 ⎣ ⎝ n ⎠ ⎝ n ⎠⎦ 0

⎡⎧ 2 ⎛ sin nπ ⎞ 2 ⎛ sin n(0) ⎞ ⎫ ⎧ ⎛ cos nπ ⎞ ⎛ cos n(0) ⎞⎫ ⎧ sin nπ sin n(0) ⎫⎤


= ⎢⎨(π × π − π )⎜ n ⎟ − ( π(0) − (0) ⎜ n ⎟ ⎬ + ⎨( π − 2π)⎜ 2 ⎟ − ( π − 2(0))⎜ ⎟⎬ + 2 ⎨ − ⎬⎥
⎢⎣⎩ ⎝ ⎠ ⎝ ⎠⎭ ⎩ ⎝ n ⎠ ⎝ n 2 ⎠⎭ ⎩ n 3 n3 ⎭⎥⎦

⎡ ⎛ ( −1) n ⎞ ⎤
⎜ ⎟ − ( π)⎛⎜ (1) ⎞⎟ + 2( 0) ⎥
= ⎢ 0 + ( − π )⎜ 2 ⎟
⎢⎣ ⎝ n ⎠ ⎝ n2 ⎠ ⎥⎦

⎡ π(−1) n − π ⎤
= ⎢− − 2⎥
⎢⎣ n2 n ⎥⎦

=
−π
n 2
[1 + (−1) ] n

∴ FC(n) =
−π
n2
[1 + (−1) ] n

SPECTRUM ALL-IN-ONE JOURNAL FOR ENGINEERING STUDENTS SIA GROUP


3.32 MATHEMATICS-II [JNTU-ANANTAPUR]

x2 π
Q40. Find the finite cosine transform of f(x) defined by f(x) = − , where 0 < x < π .
2π 6
Ans: Given function is,

x2 π
f(x) = − , where 0 < x < π.
2π 6
The Finite Fourier cosine transform of f(x) is given by,
l
⎛ nπx ⎞
FC(n) = ∫ f ( x) cos⎜⎝
0
l ⎠
⎟ dx

π π
⎛ x 2 π ⎞ ⎛ nπx ⎞ ⎛ x2 π ⎞

0⎝
∫ ⎟
= ⎜ 2π − 6 ⎟ cos ⎜ π ⎟ dx =
⎠ ⎝ ⎠ ∫ ⎜

− ⎟
⎜ 2π 6 ⎟ cos(nx) dx

0

π π
⎡⎛ x 2 π ⎞ ⎤ ⎡ ⎧⎪ d ⎛ x 2 π ⎞ ⎫ ⎤
⎟ cos nxdx ⎪⎬dx ⎥
= ⎢⎜

⎢⎣⎝ 2π 6 ⎟⎠

− ⎟ cos nxdx ⎥ − ⎢ ⎨ ⎜ −∫
⎥⎦ 0 ⎢⎣ ⎪⎩ dx ⎜⎝ 2π 6 ⎟⎠

⎪⎭ ⎥⎦
0

π π π
⎡⎛ x 2 π ⎞⎛ sin nx ⎞⎤ ⎡ ⎧⎛ 2 x ⎞⎛ sin nx ⎞⎫ ⎤ ⎡⎛ x 2 π ⎞⎛ sin nx ⎞⎤ ⎡ ⎛ x ⎞⎛ sin nx ⎞ ⎤ π

⎢⎣⎝

⎠⎝ ⎠⎥⎦ 0 ⎢⎣ ⎩⎝ ⎠⎝ ∫
= ⎢⎜ 2π − 6 ⎟⎜ n ⎟⎥ − ⎢ ⎨⎜ 2π ⎟⎜ n ⎟⎬dx ⎥ =
⎠⎭ ⎦⎥ 0
⎢⎜⎜ − ⎟⎜ ∫
⎟⎥ − ⎢ ⎜ ⎟⎜
⎢⎣⎝ 2π 6 ⎟⎠⎝ n ⎠⎥⎦ 0 ⎣ ⎝ π ⎠⎝ n ⎠ ⎦ 0
⎟dx ⎥

π
⎡⎛ x 2 π ⎞⎛ sin nx ⎞ ⎤ ⎡ x sin nx ⎧ d ⎛ x ⎞ sin nx ⎫ ⎤
= ⎢⎜

− ⎟⎜
⎟ ⎟⎥ − ⎢
⎣⎢⎝ 2π 6 ⎠⎝ n ⎠ ⎦⎥ 0 ⎣⎢ π n ∫
dx − ⎨ ⎜ ⎟
⎩ dx ⎝ π ⎠
∫ n
dx ⎬dx ⎥
⎭ ⎦⎥

π π
⎡⎛ x 2 π ⎞⎛ sin nx ⎞ ⎤ ⎡ x ⎡ − cos nx ⎤ ⎛ 1 ⎞⎛ − cos nx ⎞ ⎤
= ⎢⎜ − ⎟⎜ ⎟ ⎥ −⎢ ⎢
⎢⎣⎜⎝ 2π 6 ⎟⎠⎝ n ⎠ ⎥⎦ 0 ⎣ π ⎣ n
2 ⎥

− ⎜ ⎟ ⎜∫
⎝ π ⎠⎝ n 2 ⎠ ⎦ 0
⎟ dx ⎥

π π
⎡⎛ x 2 π ⎞⎛ sin nx ⎞⎤ ⎡ − x ⎛ cos nx ⎞ 1 ⎤
= ⎢⎜

− ⎜ ⎟ ⎟⎥ − ⎢ ⎜ 2 ⎟ + 2 cos nxdx ⎥
⎢⎣⎝ 2π 6 ⎟⎠⎝ n ⎠⎥⎦ 0 ⎣ π ⎝ n ⎠ πn ⎦0

π π
⎡⎛ x 2 π ⎞⎛ sin nx ⎞ ⎤ ⎡ − x ⎛ cos nx ⎞ 1 ⎛ sin nx ⎞ ⎤
= ⎢⎜⎜ − ⎟⎜
⎟ ⎟ ⎥ −⎢ ⎜ ⎟ + .⎜ ⎟ ⎥
⎣⎢⎝ 2π 6 ⎠⎝ n ⎠ ⎦⎥ 0 ⎣ π ⎝ n ⎠ πn ⎝ n ⎠ ⎦ 0
2 2

π π
⎡⎛ x 2 π ⎞⎛ sin nx ⎞⎤ ⎡ x ⎛ cos nx ⎞ sin nx ⎤
= ⎢⎜⎜ − ⎟⎜ ⎟ ⎥ + ⎢ ⎜ 2 ⎟− 3 ⎥
⎢⎣⎝ 2π 6 ⎟⎠⎝ n ⎠⎥⎦ 0 ⎣ π ⎝ n ⎠ πn ⎦ 0

⎡⎧⎪⎛ π 2 π ⎞⎛ sin nπ ⎞ π ⎛ cos nπ ⎞ ⎛ sin nπ ⎞⎫⎪ ⎧⎪⎛ 0 2 π ⎞⎛ sin n(0) ⎞ 0 ⎛ cos n(0) ⎞ sin n(0) ⎫⎪⎤
= ⎢⎨⎜⎜ − ⎟⎜ ⎟+ ⎜ ⎟−⎜ ⎟⎬ − ⎨⎜⎜ − ⎟⎜ ⎟+ ⎜ ⎟− ⎬⎥
⎢⎣⎪⎩⎝ 2π 6 ⎟⎠⎝ n ⎠ π ⎝ n 2 ⎠ ⎝ πn3 ⎠⎪⎭ ⎪⎩⎝ 2π 6 ⎟⎠⎝ n ⎠ π ⎝ n
2
⎠ πn 3 ⎪⎭⎥⎦
=
⎡⎧⎪⎛ π π ⎞ 1(−1) n ⎫⎪ ⎧⎛ π⎞ ⎫⎤
⎢⎨⎜ − ⎟(0) + − 0⎬ − ⎨⎜ 0 − ⎟(0) + 0 − 0⎬⎥
⎢⎣⎪⎩⎝ 2 6 ⎠ ⎪⎭ ⎩⎝
2
n 6⎠ ⎭⎥⎦

(−1) n
=
n2

(−1)n
∴ FC (n) =
n2

Look for the S IA GROUP LOGO on the TITLE COVER before you buy
UNIT-4 (Partial Differential Equations) 4.1

UNIT PARTIAL DIFFERENTIAL


EQUATIONS
4
PART-A
SHORT QUESTIONS WITH SOLUTIONS
Q1. Define order and degree with reference to partial differential equation. Model Paper-I, Q1(g)
Ans:
Order: The order of a partial differential equation is equal of the order to highest partial derivative for a particular equation.
Degree: The degree of a partial differential equation is equal to the power of highest partial derivative.
Examples
∂z ∂z
(i) x +y =z
∂x ∂y
IOrder 1 and degree 1.

∂2N ∂2N
(ii) + 2 = 3t 2 + x
∂t 2
∂x
Order 2 and degree 1.
2
⎛ ∂2N ⎞
(iii) ⎜ ⎟ + ∂N = t − x
⎜ ∂t 2 ⎟ ∂x
⎝ ⎠
Order 2 and degree 2.
Q2. Form partial differential equation by eliminating the arbitrary constants z = ax + by + a2 + b2.
Ans: Model Paper-II, Q1(h)
Given that,
z = ax + by + a2 + b2 ... (1)
On differentiating equation (1) partially with respect to ‘x’ and ‘y’, we get,
∂z
=a
∂x
⇒ zx = a
∂z
⇒ a =p [Q = zx = p]
∂x

∂z
= zy = b
∂y

∂z
⇒ b= q [Q
∂y = zy = q]
On substituting the values of ‘a’ and ‘b’ in equation (1), we get,
⇒ z = px + qy + p2 + q2.

SPECTRUM ALL-IN-ONE JOURNAL FOR ENGINEERING STUDENTS SIA GROUP


4.2 MATHEMATICS-II [JNTU-ANANTAPUR]
Q3. Form partial differential equation by eliminating On differentiating equation (4) partially with respect to
the arbitrary constants z = (x2 + a2) (y2 + b2). ‘y’, we get,
Ans: Given that,
z = (x2 + a2) (y2 + b2) ... (1) ∂z 1
= zy = axey + a2.e2y.2
Differentiating equation (1) partially with respect to ‘x’, ∂y 2
we get,
∂z
∂z ⇒ q = (aey)x + (aey)2 [Q = zy = q]
= zx = 2x (y2 + b2) ∂y
∂x
⇒ q = p.x + p2 [From equation (2)]
∂z
⇒ 2
p = 2x (y + b )2
[Q zx = = p] Q5. Form the partial differential equation by
∂x
eliminating the arbitrary function from z = f(x2– y2).
p Ans: Model Paper-III, Q1(g)
⇒ = y 2 + b2
2x Given that,
p z = f(x2 – y2) ... (1)
∴ y2 + b 2 =
2x On differentiating equation (1) partially with respect to
On differentiating equation (1) partially with respect to ‘x’, we get,
‘y’, we get,
∂z
∂z = zx = p = f ′ (x2 – y2).2x
∂x
= zy = 2y(x2 + a2)
∂y or
∂z p
⇒ q = 2y(x2 + a2) [Q zy = =q] f ′ (x2 – y2) = ... (2)
∂y 2x

q On differentiating equation (1) partially with respect to


⇒ = x 2 + a2 ‘y’, we get,
2y
∂z
q = z = q = f ′ (x2 – y2) . (–2y)
∴ x +a =
2 2 ∂y y
2y
On substituting the values of x 2 + a2 , y 2 + b2 in −q
equation (1), we get, f ′ (x2 – y2) = ... (3)
2y
q p From equation (2) and equation (3), we get,
z= .
2 y 2x
p −q
pq =
2x 2y
z=
4 xy
p −q
⇒ =
∴4 xyz = pq x y
Q4. Form partial differential equation by ⇒ py = – qx
eliminating the arbitrary constants z = axey +
∴ py = qx = 0
1 2 2y
a e + b. Model Paper-I, Q1(h)
Q6. Form the partial differential equation by
2
Ans: Given that, eliminating the arbitrary function from xyz =
f(x2 + y2 + z2).
1 2 2y
z = ax ey + a e +b ... (1) Ans: Given that,
2
xyz = f(x2 + y2 + z2) ... (1)
On differentiating equation (4) partially with respect to
On differentiating equation (1), partially with respect to
‘x’, we get,
x, we get,
∂z
= zx = aey ⎛ ∂z ⎞ ⎛ ∂z ⎞
∂x y⎜ z + x ⎟ = f ′( x 2 + y 2 + z 2 ) ⎜ 2 x + 2 z ⎟ ... (2)
⎝ ∂ x ⎠ ⎝ ∂x⎠
∂z
⇒ p = aey [Q = zx = p] ... (2)
∂x ⇒ y(z + xp) = f ′( x 2 + y 2 + z 2 ) (2 x + 2 zp ) ... (3)

Look for the SIA GROU P LOGO on the TITLE COVER before you buy
UNIT-4 (Partial Differential Equations) 4.3
On differentiating equation (1), partially with respect to Integrating equation (1) wtih respect to a and b partially,
y, we get, we get,

⎛ ∂z ⎞ ⎛ ∂z ⎞ ∂z
x⎜⎜ z + y ⎟⎟ = f ′( x 2 + y 2 + z 2 ) ⎜⎜ 2 y + 2 z ⎟⎟ Consider,
⎝ ∂ y⎠ ⎝ ∂y ⎠ ∂a
⇒ 0 = y3 + 0
⇒ x(z + yq) = f ′( x + y + z ) (2 y + 2 zq )
2 2 2
... (4) ⇒ y3 = 0 ⇒ y = 0
Dividing equations (3) and (4), we get, ∂z
Consider,
y ( z + xp ) (2 x + 2 zp) ∂b
=
x( z + yq) (2 y + 2 zq) ⇒ 0 = 0 + x2
∴ (y + zq) y (z + xp) = (x + zp) x (z + yq) ⇒ x2 = 0 ⇒ x = 0
On substituting the values of x and y in equation (1), we
Q7. Form the partial differential equation by get,
eliminating the arbitrary function from, z = f
z =0+0
(sinx + cosy).
z = 0 (constant)
Ans: Given that,
∴z =k
z = f(sinx + cosy) ... (1)
On differentiating equation (1) with respect to ‘x’, we get, ∂ u ∂ 2u
Q9. Write the three possible solutions of = .
∂ t ∂x 2
∂z
= zx = f ′ (sinx + cosy).cosx Ans: Model Paper-II, Q1(g)
∂x
Given partial differentiation equation is,
∂z
p = cosx. f ′ (sinx + cosy) [Q = zx = p] ∂u ∂ 2u
∂x = ... (1)
∂t ∂x 2
p The three possible solutions of equation(1) are,
⇒ f ′ (sinx + cosy) = ... (2)
cos x (i) u (x, t) = (A1epx + B1e–px) e p
2
t
On differentiating equation (1) with respect to ‘y’, we get, (ii) u (x, t) = A2 + B2x
∂z
2
(iii) u (x, t) = (A3cospx + B3sinpx) e – p t
= z = f ′ (sinx + cosy) (–siny)
∂y y
Q10. Write the boundary conditions for the following
problem. A rectangular plate is bounded by the
∂z line x = 0, y = 0 and y = b. Its surfaces are
q= – siny . f ′ (sinx + cosy) [Q ∂y = zy = q]
insulated. The temperature along x = 0 and y =
0 are kept at 0ºC and the others are kept at 100ºC.
−q Ans: Model Paper-III, Q1(h)
⇒ f ′ (sinx + cosy) = ... (3) Let, u(x, y) be a function.
sin y
According to the given data, the rectangular plate is
Equating equations (3) and (2), we get, shown in figure.

p −q
=
cos x sin y
y = b, u = 100
∴ p siny = –q cosx
x=0 x =a
or u = 100
u=0
p siny + q cosx = 0

Q8. Solve by the method of separation of variable y = 0, u = 0


py3 + qx2 = 0. Figure
Ans: Given that, The boundary conditions are,
py3 + qx2 = 0 (i) u(0, y) = 0
Let p = a and q = b (ii) u(x, 0) = 0
(iii) u(a, y) = 0
∴ z = ay3 + bx2 ... (1)
(iv) u(x, b) = 0

SPECTRUM ALL-IN-ONE JOURNAL FOR ENGINEERING STUDENTS SIA GROUP


4.4 MATHEMATICS-II [JNTU-ANANTAPUR]

PART-B
ESSAY QUESTIONS WITH SOLUTIONS
4.1 FORMATION OF PARTIAL DIFFERENTIAL EQUATIONS BY ELIMINATION OF ARBITRARY
CONSTANTS AND ARBITRARY FUNCTIONS
Q11. Obtain partial differential equation by eliminating the arbitrary constants for the following equations,
(a) z = ax2 + by2

x2 y2
(b) 2z = + .
a2 b2
Ans:
(a) z = ax2 + by2
Given that,
z = ax2 + by2 ... (1)
On differentiating equation (1) partially with respect to ‘x’ and ‘y’, we get,

∂z
⇒ zx = = 2ax
∂x

zx
a=
2x
Let, p = zx

∂z
zy = = 2by
∂y

zy
b =
2y
Let, q = zy
On substituting the values of ‘a’ and ‘b’ in equation (1), we get,

⎛ zx ⎞ 2 ⎛ z y ⎞ 2
⇒ z = ⎜ ⎟ x + ⎜⎜ ⎟⎟ y
⎝ 2x ⎠ ⎝ 2y ⎠

x. z x y. z y
⇒ z= +
2 2

x.z x + y.z y
⇒ z=
2
⇒ 2z = x.zx + y.zy

∴ 2 z = x. p = yq

x2 y 2
(b) 2z = +
a2 b2
Given that,

x2 y 2
2z = + ... (1)
a 2 b2

Look for the SIA GROU P LOGO on the TITLE COVER before you buy
UNIT-4 (Partial Differential Equations) 4.5
On differentiating equation (1) partially with respect to On differentiating equation (1) partially with respect to
‘x’, we get, ‘y’, we get,
∂z 2 x ∂z
⇒ 2. = = zy = q = 2(y – b)
∂x a 2 ∂y
2x q
⇒ 2.zx = ⇒ y–b= ... (3)
a2 2
x Substituting equations (2) and (3) in equation (1), we get,
⇒ zx = 2
a 2 2
⎛ p⎞ ⎛q⎞
x x ∂z ⇒ z = ⎜ ⎟ + ⎜ ⎟ +1
⇒ p= ⇒ a2 = [Q p = zx = ] ⎝ 2 ⎠ ⎝ 2⎠
a2 p ∂x
p2 q2
On differentiating equation (1) partially with respect to ⇒ z = + +1
‘y’, we get, 4 4

∂z 2 y ∴ 4z = p 2 = q 2 + 4
2. =
∂y b 2
z = ae − b
2
t
(b) cos bx
y
⇒ zy = Given that,
b2
−b 2
t
y ∂z z = ae cos bx ... (1)
⇒ q= [Q q = zy = ]
b 2
∂y On differentiating equation (1) partially with respect to
‘x’, we get,
y
⇒ b2 = ∂z
= – ae −b t sin bx (b)
2
q
∂x
On substituting the values of a2 and b2 in equation (1),
we get, ∂z 2
⇒ = – abe −b t sin bx ... (2)
x y 2 2 ∂x
⇒ 2z = +
⎛ x ⎞ ⎛ y⎞ On differentiating equation (2) with respect to ‘x’,
⎜⎜ ⎟⎟ ⎜⎜ ⎟⎟ we get
⎝ p⎠ ⎝q⎠

2z =
px 2 qy 2
+
∂ ⎡ ∂z ⎤ ∂
⎢ ⎥ =
∂x ⎣ ∂x ⎦ ∂z
2
[
− abe −b t . sin(bx ) ]
x y
∂2z −b 2 t
∴2 z = px = qy ⇒ = − abe cos bx (b )
∂x 2
Q12. Obtain the partial differential equation by
eliminating the arbitrary constants ∂2z
= − ab 2e −b t cos bx
2
(a) z = (x – a)2 + (y – b)2 + 1 ... (3)
∂x 2
2
(b) z = ae −b t
cos bx. Model Paper-III, Q8 On differentiating equation (1) partially with respect to
Ans: ‘t’, we get,
(a) z = (x – a)2 + (y – b)2 + 1
Given that, ∂z − b 2t
= ae cos bx.( −b 2 )
z = (x – a)2 + (y – b)2 + 1 ... (1) ∂t
On differentiating equation (1) partially with respect to
‘x’, we get, ∂z
= − ab 2 e −b t cos bx
2
⇒ ... (4)
∂t
∂z
= zx = p = 2(x – a) From equations (3) and equation (4), we get,
∂x

p ∂2z ∂z
⇒ x–a = ... (2) =
2 ∂x 2 ∂t

SPECTRUM ALL-IN-ONE JOURNAL FOR ENGINEERING STUDENTS SIA GROUP


4.6 MATHEMATICS-II [JNTU-ANANTAPUR]
Q13. Form the partial differential equation by Q14. Form the differential equations of all planes
eliminating the arbitrary constants a, b from which are at a constant distance ‘a’ from the
1 1 origin.
2z = (x + a) 2 + (y − a) 2 + b .
Ans: It is required to find the differential equation of all planes
Ans: Given that, which are at a constant distance ‘a’ from origin.
[
2z = x + a + y − a + b ] The equation of the plane in normal form as,
On differentiating with respect to x, we get, lx + my + nz = a ... (1)
2∂ z 1 Where l, m, n are the direction cosines of the normal
= +0 from the origin to the plane.
∂x 2 x+a
Then we have l2 + m2 + n2 = 1
4∂z 1
∂x
= ⇒ n = (1 − l 2 − m 2 )
x+a
Therefore, equation (1), becomes,
1
⇒ 4p =
x+a lx + my + (1 − l 2 − m 2 ) .z = a ... (2)

1 On differentiating equation (2) partially with respect to x


⇒ x+a = on both sides, we get,
4p
∂z
1 l+ (1 − l 2 − m 2 ) =0
⇒ x+a = ∂x
16 p 2
1 ∂z
a= –x ... (1) ⇒ l+ (1 − l 2 − m 2 ) . p = 0 [Q = p] ... (3)
∂y
16 p 2
On differentiating given equation with respect to y, On differentiating equation (2) partially with respect to
we get, y, we get,
2∂z 1 ∂z
= +0 m+ (1 − l 2 − m 2 ) . =0
∂y 2 y−a ∂y

4∂z 1 ∂z
⇒ = m+ (1 − l 2 − m 2 ) .q = 0 [Q = q] ... (4)
∂y y−a ∂y
Eliminating l, m from equations (2), (3) and (4).
1
4q = From equations (3) and (4), we get,
y−a
l = − (1 − l 2 − m 2 ) . p and ... (5)
1
⇒ y−a =
4q m = − (1 − l 2 − m 2 ) .q ... (6)
1 Squaring on both sides and then adding the above two
⇒ y–a =
16 q 2 equations, we get,
1 l2 + m2 = (1 – l2 – m2)p2 + (1 – l2 – m2) q2
⇒ y = +a ... (2) ⇒ l2 + m2 = (1 – l2 – m2) (p2 + q2)
16q 2
⇒ l2 + m2 = p2 + q2 – l2p2 – l2q2 – m2p2 – m2q2
On substituting the value of ‘a’ from equation (1) in
⇒ l2 (1 + p2 + q2) + m2 (1 + p2 + q2) = p2 + q2
equation (2), we get,
⇒ (l2 + m2) (1 + p2 + q2) = p2 + q2
1 1
y = + −x
16q 2
16 p 2 p2 + q 2
⇒ l2 + m2 =
(1 + p 2 + q 2 )
1 1
⇒ y + x= + ∴ (1 – l2 – m2) = 1 – (l2 + m2)
16 p 2 16q 2
1 1 p2 + q2 ⎡ 1 ⎤
∴16( x + y ) = + =1– 2 2
=⎢ 2⎥
p2 q2 1+ p + q ⎢⎣1 + p + q ⎥⎦
2

Look for the SIA GROU P LOGO on the TITLE COVER before you buy
UNIT-4 (Partial Differential Equations) 4.7

1
On substituting (1 – l2 – m2) = in equations (5) and (6), we get,
1+ p2 + q2

1
l = − . p and
(1 + p 2 + q 2 )

1
m =− .q
(1 + p 2 + q 2 )
On substituting the values of l, m, (1 – l2 – m2) in equation (2), we get,
1 1 1
⇒ − . px − .qy + .z = a
(1 + p 2 + q 2 ) (1 + p 2 + q 2 ) (1 + p 2 + q 2 )

−1
⇒ [ px + qy − z ] = a
(1 + p 2 + q 2 )

⇒ px + qy – z = – a (1 + p 2 + q 2 )

2 2
px + qy + a (1 + p + q ) = z
∴ The required differential equation of all planes which are at a constant distance ‘a’ from origin is given by,

z = px + qy + a (1 + p 2 + q 2 )

x 2 y2 z 2
Q15. Form the partial differential equation by eliminating constants a, b and c from + + =1
a2 b2 c2
Ans: Given equation is,

x2 y2 z2
+ + =1 ... (1)
a2 b2 c2
In equation (1), there are 3 arbitrary constants and 2 independent variables. Thus, differential equation of order greater than
1 is otained. Treating ‘z’ as a function of ‘x’ and ‘y’.
Differentiating equation (1) partially with respect to ‘x’ and ‘y’, we get,
2x 2z ∂z
+ × =0
a2 c2 ∂x

x z ∂z
+ × =0 ... (2)
a 2
c 2
∂x
Similarly,
2y 2z ∂z
+ × =0
b 2
c 2
∂y

y z ∂z
+ ×
b 2
c 2 ∂y = 0 ... (3)

Again differentiating equation (2) partially with respect to ‘x’, we get,

1 1 ⎛ ∂z ⎞
2
z ⎛ ∂2 z ⎞
+ ⎜ ⎟ + 2 ⎜ ⎟
a2 c ⎝ ∂x ⎠
2
c ⎜ ∂x 2 ⎟ = 0
⎝ ⎠

c 2 ⎛ ∂z ⎞
2
⎛ ∂2 z ⎞
+ ⎜ ⎟ + ⎜ ⎟
a 2 ⎝ ∂x ⎠
z ⎜ ∂x 2 ⎟ = 0 ... (4)
⎝ ⎠

SPECTRUM ALL-IN-ONE JOURNAL FOR ENGINEERING STUDENTS SIA GROUP


4.8 MATHEMATICS-II [JNTU-ANANTAPUR]
From equation (2), On substituting the value of f ′ (x2 + y2 + z2) from equation
(2) in equation (3), we get,
x z ∂z
+ × =0
a 2
c 2 ∂x 1+ p
⇒ 1 + q= (2y + 2zq) .
2 x + 2 zp
Multiplying with c2 on both sides, we get,
( y + zq)(1 + p)
2 z ⎛ ∂z ⎞ ⇒ 1 + q= ( x + zp)
c
⇒ =– ⎜ ⎟
a 2 x ⎝ ∂x ⎠ ⇒ (1 + q) (x + zp) = (y + zq) (1 + p)
⇒ x + zp + xq + zpq = y + yp + zq + zpq
c2 ⇒ x – y = yp + zq – zp – xq
On substituting value in equation (4), we get,
a2 ∴ x – y = (y – z) p + (z – x)q
Q17. Form the partial differential equation by
2
z ⎛ ∂z ⎞ ⎛ ∂z ⎞ ⎛ ∂2 z ⎞ eliminating the arbitrary function from z = yf(x)
− ⎜ ⎟ + ⎜ ⎟ +z⎜ ⎟
x ⎝ ∂x ⎠ ⎝ ∂x ⎠ ⎜ ∂x 2 ⎟ = 0 + x g(y).
⎝ ⎠
Ans: Given that,
∴ The required partial differential equation is, z = y f (x) + x g(y) ... (1)
By differentiating equation (1) partially with respect to
2 x, we get,
z ∂z ⎛ ∂z ⎞ ∂2 z
= ⎜ ⎟ +z 2
x ∂x ⎝ ∂x ⎠ ∂x ∂z
= y f '(x) + g(y)
∂x
Q16. Obtain the partial differential equation by ∴ yf '(x) + g(y) = p ... (2)
eliminating the arbitrary function from x+y+z =
By differentiating equation (1), partially with respect to
f(x2+y2+z2).
y, we get,
Ans: Given that,
∂z
= f (x) + xg'(y)
x + y + z = f(x2 + y2 + z2) ... (1) ∂y

Differentiating equation (1) partially with respect to ‘x’, ∴ f (x) + xg'(y) = q ... (3)
we get, Since, equations (1), (2) and (3) are not sufficient to
eliminate f, g, f ' and g', the second order partial derivatives are
∂z ∂z determined as,
= 2 xf ′( x + y + z ) + 2 z. f ′( x 2 + y 2 + z 2 )
2 2 2
1+
∂x ∂x ∂2 z
= r = y f ''(x) ... (4)
∂x 2
⇒ 1 + zx = f ′ (x2 + y2+ z2) [2x + 2z zx]
∂2z
∂z = s = f '(x) + g'(y) ... (5)
∂x∂y
⇒ 1 + p = (2x + 2z.p) f ′ (x2 + y2 + z2) [Q = zx = p]
∂x
∂2z
= t = x g''(y) ... (6)
1+ p ∂y 2
f ′ (x2 + y2 + z2) = ... (2)
2 x + 2 zp
From equation (2),
On differentiating equation (1) with respect to ‘y’, y f '(x) = p – g(y)
we get,
1
f ′( x) = [ p − g ( y )] ... (7)
y
1 + zy = 2y f ′ (x2 + y2 + z2) + 2z zy f ′ (x2 + y2 + z2)
From equation (3),
1 + q = (2y + 2zq) f ′ (x2 + y2 + z2) ... (3) x g'(y) = q – f (x)

⎡ ∂z ⎤ 1
⎢Q = zy = q ⎥ g ′( y ) = [q − f ( x )] ... (8)
⎣ ∂y ⎦ x

Look for the SIA GROU P LOGO on the TITLE COVER before you buy
UNIT-4 (Partial Differential Equations) 4.9
From equation (5), we get, Q19. Form the partial differential equation by
s = f '(x) + g'(y) eliminating the arbitrary function from
z = xf 1 (x+t) + f 2 (x+t).
1 1
s= [p – g(y)] + [q – f (x)] Ans: Given equation is,
y x
z = x f1 (x + t) + f2 (x + t) ... (1)
x[ p − g ( y )] + y[q − f ( x)] To form the partial differential equation by eliminating
⇒ s=
xy arbitrary functions.
⇒ sxy = x[p – g(y)] + y[q – f (x)] ∴ Differentiate equation (1), with respect to ‘x’ and ‘t’.
⇒ xys = xp – x g(y) + yq – y f (x) First, differentiating with respect to ‘x’, we get,
xys = px + qy – [y f (x) + x g(y)]
(Q From equation (1)) ∂z
= f (x + t) + xf1′ + f 2′ ... (2)
∂x 1
xys = px + qy − z
On differentiating with respect to ‘t’, we get,
This is the required partial differential equation.
Q18. Form the partial differential equation by ∂z
eliminating the arbitrary function from z = f(y) = xf1′ + f 2′ ... (3)
∂t
+ φ (x + y).
Ans: The given function is, Again differentiating equations (2) and (3) partially with
respect to ‘x’ and ‘t’, we have,
z = f(y) + φ(x + y) ... (1)
By differentiating equation (1), partially with respect to
∂2 z
x, we get, = f1′ + xf1′′+ f1′ + f 2′′
∂x 2
∂z
⇒ = 0 + φ'(x + y) . 1 = 2 f1′ + xf1′′+ f 2′′
∂x ... (4)

∂z ∂2 z
⇒ = p = φ'(x + y) ... (2) = xf1′′ + f 2′′ ... (5)
∂x ∂t 2
By differentiating equation (1), partially with respect to
Again differentiating equation (3) partially with respect
y, we get,
to ‘x’, we get,
∂z
⇒ = f '(y)+ φ'(x + y) . 1
∂y ∂2 z
= xf1′′ + f1′ + f 2′′ ... (6)
∂x∂t
∂z
⇒ = q = f '(y) + φ'(x + y) ... (3) On adding equations (4) and (5), we get,
∂y
On differentiating equation (2) partially with respect to ∂2 z ∂ 2 z
‘x’, we get,
+ = 2 f1′ + xf1′′ + f 2′′ + xf1′′ + f 2′′
∂x 2 ∂t 2
∂2z = 2 f1′ + 2 xf1′′ + 2 f 2′′
⇒ = φ'' (x + y) .1
∂x 2
= 2 ( f1′ + xf1′′ + f 2′′)
∂2z
⇒ = r = φ'' (x + y) ... (4)
∂x 2 ⎛ ∂2z ⎞
On differentiating equation (3), partially with respect to = 2 ⎜⎜ ⎟
⎟ [Q From equation (6)]
⎝ ∂x∂ t ⎠
‘x’,
∂2z ⎛ ∂2z ⎞
⇒ = φ''(x + y).1 ∂2 z ∂ 2 z ⎜ ⎟
∂x∂y + = 2⎜ ∂x∂t ⎟
∂x 2 ∂t 2 ⎝ ⎠
∂2 z
⇒ = s = φ''(x + y) ... (5)
∂x ∂y ∂2z ⎛ ∂2 z ⎞ ∂ 2 z
− ⎜ ⎟
∴ From equations (4) and (5), we get,

∂x 2
2⎜ ∂x∂t ⎟ + ∂t 2 = 0
⎝ ⎠
r = s or r – s = 0
This is the required partial differential equation. This is the required partial differential equation.

SPECTRUM ALL-IN-ONE JOURNAL FOR ENGINEERING STUDENTS SIA GROUP


4.10 MATHEMATICS-II [JNTU-ANANTAPUR]
Q20. Form the partial differential equations by Q21. Form the partial differential equation by
eliminating the arbitrary functions z = f(2x + y) + eliminating the arbitrary function from
g(3x – y).
⎛ y⎞
Ans: Given equation is, z = yf ⎜ ⎟ .
⎝x⎠
z = f(2x + y) + g(3x – y) ... (1)
On differentiating equation (1) partially, with respect to Ans: Given that,
‘x’, we get,
⎛ y⎞
z = y.f ⎜ ⎟ ... (1)
∂z ⎝ x⎠
⇒ = f'(2x + y).2 + g'(3x – y).3
∂x On differentiating equation (1) with respect to ‘x’, we get,
∂z ⎛ y ⎞⎛ − 1 ⎞
⇒ = p = 2f '(2x + y) + 3g'(3x – y) ... (2) zx = y. f ′⎜ ⎟⎜ ⎟
∂x ⎝ x ⎠⎝ x 2 ⎠
On differentiating equation (1) partially, with respect to ⎛ − y2 ⎞ ⎛ y ⎞
⎜ ⎟
‘y’, we get, p = ⎜ 2 ⎟ f ′⎜ x ⎟
⎝ x ⎠ ⎝ ⎠
∂z
⇒ = f'(2x + y).1 + g'(3x – y).(–1) ⎛y⎞ px 2
∂y f ′⎜ ⎟ −
⇒ = ... (2)
⎝x⎠ y2
∂z
⇒ = q = f '(2x + y) – g'(3x – y) ... (3) On differentiating equation (1) with respect to ‘y’, we get,
∂y
⎛ y ⎞⎛ 1 ⎞ ⎛ y⎞
On differentiating equation (2) partially, with respect to zy = y. f ′⎜ ⎟⎜ ⎟ + f ⎜ ⎟.1
‘x’, we get, ⎝ x ⎠⎝ x ⎠ ⎝ x⎠

∂2z ⎛ y⎞ ⎛ y⎞ ⎛ y⎞
⇒ = 2f"(2x + y).2 + 3g"(3x – y).3 ⇒ q = ⎜ ⎟ f ′⎜ ⎟ + f ⎜ ⎟
∂x 2 ⎝ x⎠ ⎝x⎠ ⎝x⎠

∂2z ⎛ y⎞ y ⎛y⎞
⇒ p2 = = 4f"(2x + y) + 9g"(3x – y) ... (4) ⇒ f ⎜ ⎟ = q – . f ′⎜ ⎟ ... (3)
∂x 2 ⎝ ⎠
x x ⎝x⎠
On differentiating equation (3) partially with respect to On substituting equation (3) in equation (1), we get,
‘y’, we get,
y ′⎛ y ⎞
⇒ f ⎜ ⎟]
∂2 z z = y [q –
x ⎝x⎠
= f"(2x + y).1 – g"(3x – y)(–1)
∂y 2
y ⎛ y⎞
2 ⇒ z = [qx – y f ′⎜ ⎟ ]
∂ z x ⎝x⎠
⇒ q2 = = f "(2x + y) + g"(3x – y) ... (5)
∂y 2
⎛ y⎞
On differentiating equation (2) partially, with respect to On substituting f ′⎜ ⎟ value from equation (2), we get,
⎝x⎠
‘y’, we get,

∂2z y ⎛ px 2 ⎞
⇒ ⎜− ⎟
= 2f"(2x + y).1 + 3g"(3x – y)(–1) ⇒ z= [qx – y ⎜ y2 ⎟ ]
∂x∂y x ⎝ ⎠

∂2z y px 2
⇒ pq = = 2f"(2x + y) – 3g"(3x – y) ... (6) ⇒ z= (qx + )
∂xδy x y
Adding equations (4) and (6), we get,
y ⎛ qxy + px 2 ⎞
p2 + pq= 4f"(2x + y) + 9g"(3x – y) + 2f"(2x + y) – 3g"(3x – y) ⎜ ⎟
⇒ z= ⎜ y ⎟
= 6f"(2x + y) + 6g"(3x – y) x ⎝ ⎠
= 6[f "(2x + y) + g"(3x – y)]
1
= 6(q2) [ Q From equation (5)] ⇒ z= .x(qy + px)
2 2 x
p + pq = 6q
∴ p2 – 6q2 + pq = 0 is the required differential equation. ⇒ z = px + qy

Look for the SIA GROU P LOGO on the TITLE COVER before you buy
UNIT-4 (Partial Differential Equations) 4.11
Q22. Form the partial differential equation by eliminating the arbitrary function from, z = eax + by f (ax – by).
Ans: Given that,
z = eax + by f (ax – by) ... (1)
On differentiating equation (1) with respect to ‘x’, we get,
∂z
= zx
∂x
= eax+by. f ′ (ax – by) . a + f(ax – by) . eax + by . a
p = a [ eax + by f ′ (ax – by) + eax + by . f(ax – by)]
p = a [eax + by f ′ (ax – by) + z] [Q From equation (1)]

p
= eax + by . f ′ (ax – by) + z ... (2)
a
On differentiating equation (1) partially with respect to ‘y’, we get,
∂z
=z
∂y y

= eax+by. f ′ (ax – by) . (–b) + f(ax – by) . eax + by . (b)


⇒ q = b [–eax + by . f ′ (ax – by) + f(ax – by) .eax + by]
⇒ q = b [–eax + by. f ′ (ax – by) + z] [Q From equation (1)]

q
= –eax + by f ′ (ax – by) + z ... (3)
b
Adding equation (2) and equation (3), we get,
p q
+ = eax+by f ′ (ax – by) + z – eax + by f ′ (ax – by) + z
a b

pb + qa
⇒ = 2z
ab
⇒ pb + qa = 2abz.
Q23. Form the partial differential equation by eliminating arbitrary function from f(x2 + y2, z – xy) = 0.
Ans: Given equation is,
f (x2 + y2, z – xy) = 0 ... (1)
Let, x2 + y2 = u and z – xy = v ... (2)
∴ From equations (1) and (2), we get,
f (u, v) = 0 ... (3)
On differentiating equation (3) with respect to x, by using chain rule, we get,
∂f ∂u ∂f ∂u ∂z ∂f ∂v ∂f ∂v ∂z
+ + + =0
∂u ∂x ∂u ∂z ∂x ∂v ∂x ∂v ∂z ∂x

∂f ∂f ∂f ∂f
⇒ (2x) + (0) p + (– y) + 1. p = 0
∂u ∂u ∂v ∂v

∂z
Where, p =
∂x

∂f ∂f ∂f
⇒ (2x) + (– y) + (p) = 0
∂u ∂v ∂v

∂f ∂f
⇒ (2x) + (p – y) =0 ... (4)
∂u ∂v

SPECTRUM ALL-IN-ONE JOURNAL FOR ENGINEERING STUDENTS SIA GROUP


4.12 MATHEMATICS-II [JNTU-ANANTAPUR]
On differentiating equation (3) with respect to ‘y’ by On differentiating equation (1) partially with respect to
using chain rule, we get, y, we get,

∂f ∂u ∂f ∂u ∂z ∂f ∂v ∂f ∂v ∂z ∂f ⎡ ∂u ∂u ∂z ⎤ ∂f ⎡ ∂v ∂v ∂z ⎤
+ + +
∂v ∂y ∂v ∂z ∂y
=0 ⎢ + . ⎥+ ⎢ + . ⎥
∂u ∂y ∂u ∂z ∂y ∂u ⎣ ∂y ∂z ∂y ⎦ ∂v ⎣ ∂y ∂z ∂y ⎦ = 0

∂f ∂f ∂f ∂f
⇒ (2 y) + (0) (q) + (– x) + 1.q = 0 ∂f ⎡ ∂z ⎤ ∂f ⎡ ∂z ⎤
∂u ∂u ∂v ∂v ⇒ ⎢1 + 1. ⎥ + ⎢2 y + 2 z. ⎥ = 0
∂u ⎣ ∂y ⎦ ∂v ⎣ ∂y ⎦
∂z
Where, =q ∂z
∂y Let =q
∂y
∂f ∂f ∂f
⇒ (2 y) – x + q=0 ∂f ∂f
∂u ∂v ∂v ⇒ (1 + q ) + ( 2 y + 2 zq ) = 0 ... (3)
∂u ∂v
∂f ∂f
⇒ (2 y) + (q – x) =0 ... (5) ∂f ∂f
∂u ∂v Eliminating and between equations (2) and (3),
∂u ∂v
∂f ∂f we get,
Eliminating and from equations (4) and (5), we get,
∂u ∂v
(1 + p) 2( x + zp )
=0
2 x ( p − y) (1 + q) 2( y + zq)
=0
2 y (q − x ) ⇒ (1 + p). 2(y + zq) – 2(x + zp).(1 + q) = 0
⇒ 2x (q – x) – (p – y) 2y = 0 ⇒ (1 + p) (y + zq) – (x + zp).(1 + q) = 0
⇒ 2 xq – 2x2 – 2yp + 2y2 = 0 ⇒ y + zq + yp + zpq – x – xq – zp – zpq = 0
⇒ 2
xq – x – yp + y = 0 2 ⇒ (y – z)p + (z – x)q = x – y
⇒ xq – yp = x – y 2 2 ∴ The partial differential equation by eliminating ‘f ’
from f(x + y + z, x2 + y2 + z2) = 0 is,
∴ The required partial differential equation by eliminating
arbitrary function from f (x2 + y2, z – xy) = 0 is xq – yp = x2 – y2. (y – z)p + (z – x)q = x – y
Q24. Form the PDE by eliminating ‘f’ from f(x + y + z, Q25. Form the partial differential equation by eliminating
x2 + y2 + z2) = 0. Model Paper-I, Q8 arbitrary function φ from φ(x + y + z, x2 + y2 – z2) =
Ans: Given that, 0.
f(x + y + z, x2 + y2 + z2) = 0 Ans: Given that,
φ(x + y + z, x2 + y2 – z2) = 0
Let, x + y + z = u
Let, x + y + z = u and
And x2 + y2 + z2 = v x2 + y2 – z2 = v
∴ f(u, v) = 0 ... (1) ⇒ φ(u, v) = 0 ... (1)
On differentiating equation (1) partially with respect to Differentiating equation (1) partially with respect to x by
‘x’ by chain rule, we get, chain rule, we get,

∂f ⎡ ∂u ∂u ∂z ⎤ ∂f ⎡ ∂v ∂v ∂z ⎤ ∂φ ⎡ ∂u ∂u ∂z ⎤ ∂φ ⎡ ∂v ∂v ∂z ⎤
+ . + + .
+ . + + . ∂u ⎢⎣ ∂x ∂z ∂x ⎥⎦ ∂v ⎢⎣ ∂x ∂z ∂x ⎥⎦
=0
∂u ⎢⎣ ∂x ∂z ∂x ⎥⎦ ∂v ⎢⎣ ∂x ∂z ∂x ⎥⎦
=0

∂φ ⎡ ∂z ⎤ ∂φ ⎡ ∂z ⎤
∂f ⎡ ∂z ⎤ ∂f ⎡ ∂z ⎤ ⇒ 1 + (1) ⎥ + ⎢2 x + (−2 z ) ⎥ = 0
⇒ ⎢1 + 1. ∂x ⎥ + ∂v ⎢ 2 x + 2 z. ∂x ⎥ = 0 ∂u ⎢⎣ ∂x ⎦ ∂v ⎣ ∂x ⎦
∂u ⎣ ⎦ ⎣ ⎦
∂φ
∂z ⇒ [1 + p ]+ ∂φ [2 x − 2 zp ] = 0 ... (2)
∂u ∂v
Let =p
∂x Where,

∂f ∂f ∂z
⇒ (1 + p ) + ( 2 x + 2 zp ) = 0 ... (2) p=
∂u ∂v ∂x

Look for the SIA GROU P LOGO on the TITLE COVER before you buy
UNIT-4 (Partial Differential Equations) 4.13
Similarly differentiating equation (1) partially with respect Similarly differentiating equation (1), partially with
to y, respect to y, we get,

∂φ ⎡ ∂u ∂u ∂z ⎤ ∂φ ⎡ ∂v ∂v ∂z ⎤ ∂φ ⎡ ∂u ∂u ∂z ⎤ ∂φ ⎡ ∂v ∂v ∂z ⎤
⎢ + . ⎥+ ⎢ + . ⎥ =0 ⇒ ⎢ + ⋅ ⎥+ ⎢ + ⋅ ⎥ =0
∂u ⎣ ∂y ∂z ∂y ⎦ ∂v ⎣ ∂y ∂z ∂y ⎦ ∂u ⎣ ∂y ∂z ∂y ⎦ ∂v ⎣ ∂y ∂z ∂y ⎦
∂φ ⎡ 1 ∂z ⎤ ∂φ ⎡ ∂z ⎤
∂φ ⎡ ∂z ⎤ ∂φ ⎡ ∂z ⎤ ⇒ ⎢ + 0 ⋅ ⎥ + ⎢2 y + 2 z ⋅ ⎥ = 0
⇒ ⎢1 + (1) ⎥ + ⎢2 y + (−2 z ) ⎥ = 0 ∂u ⎣ x ∂x ⎦ ∂v ⎣ ∂y ⎦
∂u ⎣ ∂y ⎦ ∂v ⎣ ∂y ⎦
∂φ ⎡ 1 ⎤ ∂φ
∂φ + [2 y + 2 zq ] = 0
⇒ [1 + q] + ∂φ [2 y − 2 zq] = 0 ... (3)

∂u ⎢⎣ x ⎥⎦ ∂v
... (3)
∂u ∂v
Where, ∂z
Where, q =
∂z ∂y
q =
∂y
∂φ ∂φ
Eliminating and from equations (2) and (3), we get,
∂φ ∂φ ∂u ∂v
Eliminating and between equations (2) and (3),
∂u ∂v
−y
we get, 2( x + zp )
x2 =0
(1 + p) 2( x − zp) 1
2( y + zq)
=0 x
(1 + q) 2( y − zq)
−2 y 2
⇒ 2(1 + p) (y – zq) – 2(x – zp) (1 + q) = 0 ⇒ 2
( y + zq ) − ( x + zp ) = 0
x x
⇒ (1 + p) (y – zq) – (1 + q) (x – zp) = 0
⇒ y – zq + py – zpq – [x – zp + xq – zpq] = 0 −y 1
⇒ ( y + zq ) −
( x + zp ) = 0
⇒ y – zq + py – zpq – x + zp – xq + zpq = 0 x 2
x
⇒ y – x + p(y + z) – q(x + z) = 0 ⇒ – y(y + zq) – x(x + zp) = 0
⇒ p(y + z) – q(x + z) = x – y ⇒ – y2 – yzq – x2 – xzp = 0
This is the required partial differential equation. ⇒ x2 + y2 + xzp + yzq = 0
Q26. Form the partial differential equation by eliminating This is the required partial differential equation.
⎛y 2 2 2⎞ 4.2 METHOD OF SEPARATION OF VARIABLES
φ from φ ⎜ , x + y + z ⎟ = 0.
⎝x ⎠
Q27. Explain the stepwise procedure to solve second
Ans: Given that, order equations using method of separation of
variables.
⎛y ⎞
φ ⎜ , x2 + y2 + z 2 ⎟ = 0 Ans:
⎝ x ⎠ Method of Separation of Variables
y Step 1: For the given second order equations, Assume
Let, = u and z = X(x), Y(y) ... (1)
x
Where,
x2 + y2 + z2 = v X(x) denotes a function of x
⇒ φ(u, v) = 0 ... (1) Y(y) denotes a function of y.
On differentiating equation (1), partially with respect to Step 2: Partially differentiate equation (1) with respect to x and
‘x’ by chain rule, we get, y, substitute in the given equation and write it in the form of
∂φ ⎡ ∂u ∂u ∂z ⎤ ∂φ ⎡ ∂v ∂v ∂z ⎤ f(X, X', X'' ....) = g(Y, Y', Y''...) ... (2)
+ + +
∂u ⎢⎣ ∂x ∂z ∂x ⎥⎦ ∂v ⎢⎣ ∂x ∂z ∂x ⎥⎦
=0 Which is separable in X and Y.
Step 3: The LHS of equation (2) is a function of only x and RHS
is a function of only y and both are equal for all values of x and
∂φ ⎡ − y ∂z ⎤ ∂φ ⎡ ∂z ⎤
⇒ + 0⋅ ⎥ + 2x + 2 z ⎥ = 0 y. Therefore equating LHS and RHS to same constant (say λ),
∂u ⎢⎣ x 2 ∂x ⎦ ∂v ⎢⎣ ∂x ⎦ which is called separation constant.
∴ f(X, X', X''....) = K ... (3)
∂φ ⎡ − y ⎤ ∂φ
⇒ + [2 x + 2 zp ] = 0 ... (2) g(Y, Y', Y''...) = K ... (4)
∂u ⎢⎣ x 2 ⎥⎦ ∂v Step 4: Solve for X using equation (3)
Step 5: Similarly solve for Y using equation (4).
∂z
Where, p = Step 6: Substitute the corresponding values of X and Y in
∂x equation (1), therefore the required solution is obtained.

SPECTRUM ALL-IN-ONE JOURNAL FOR ENGINEERING STUDENTS SIA GROUP


4.14 MATHEMATICS-II [JNTU-ANANTAPUR]
Q28. Solve by the method of separation of variables. Q29. Solve by separation of variables 3ux + 2uy = 0
ux = 2ut + u where u(x, 0) = 6e–3x with u(x, 0) = 4e– x. Model Paper-II, Q8
Ans: Given that,
Ans: Given that, 3ux + 2uy = 0 with u(x, 0) = 4e– x
∂u ∂u ∂u ∂u
= 2 +u ... (1) ⇒ 3 +2 =0 ... (1)
∂x ∂t ∂x ∂y
Let, u(x, y) = X(x) Y(y) = XY ... (2)
Let, u(x, t) = X(x)T(t) ... (2)
= XT be a solution of equation (1), then, ∂u ∂u
= X 'Y and = XY '
∂x ∂y
∂u ∂u
= X'T and = XT' ... (3) On substituting above values in equation (1), we get,
∂x ∂t 3X 'Y + 2 XY ' = 0
On substituting equations (2) and (3) in equation (1), we Dividing by XY, we get,
get,
⎛ X '⎞ ⎛ Y'⎞
3 ⎜ ⎟ + 2⎜ ⎟ = 0
X ′ 2T ′ ⎝ X ⎠ ⎝Y ⎠
X'T = 2XT' + XT ⇒ = +1
X T On equating each side to an arbitrary constant K.
Equating each side to some arbitrary constant k. ⎛ X'⎞
3 ⎜ ⎟ = − 2⎛⎜ ⎞⎟ = K
Y'
X ′ 2T ′ ⎝X ⎠ ⎝Y ⎠
⇒ = +1 = k
X T ⎛ X'⎞ ⎛Y'⎞
i.e., 3⎜ ⎟ = K − 2⎜ ⎟ = K
⎝X ⎠ ⎝Y ⎠
X′
⇒ = k ⇒ X' − kX = 0 ⇒ 3 X ' = KX ⇒ − 2Y ' = KY
X
KX − KY
⇒ X' = ⇒ Y' =
2 ⎛ KY ⎞
2T ′ ⎛ k −1⎞ 3
+1 = k ⇒ T ′ = ⎜
⎛ KX ⎞
⇒ X = c1e kx and ⎟T = 0 ⎜ ⎟
Y = C2 e
−⎜ ⎟
⎝ 2 ⎠
T ⎝ 2 ⎠ X = C1 e ⎝ 3 ⎠

⎛ k −1 ⎞
Substituting these X and Y values in equation (2), we get,
⎜ ⎟t
⇒ T = c2 e⎝ 2 ⎠ ⎛ ⎛ KX ⎞ ⎞ ⎛
⎜ ⎟
⎛ KY ⎞ ⎞
−⎜ ⎟
⎜ ⎝ 3 ⎠⎟ ⎜ ⎝ 2 ⎠⎟
u(x, y) = ⎜ C1 e ⎟⎜ C 2 e ⎟
On substituting the values of X and T in equation (2), we ⎜ ⎟⎜ ⎟
⎝ ⎠⎝ ⎠
get,
⎛ KX ⎞ ⎛ KY ⎞
⎛ k −1 ⎞ ⎜ ⎟ −⎜ ⎟
⎜ ⎟t e⎝ 3 ⎠ e ⎝ 2 ⎠
u(x, t) = (c1 ekx).c2 e ⎝ 2 ⎠ = C1 C 2
⎛ KX ⎞ ⎛ KY ⎞
⎜ ⎟ −⎜ ⎟
⎛ k −1 ⎞
⎜ ⎟t = B e⎝ 3 ⎠
e ⎝ 2 ⎠ [ Q B = C1 C2 ]
⎝ 2 ⎠
= c1c2ekx e
⎛ KX ⎞ ⎛ KY ⎞
⎜ ⎟ −⎜ ⎟
⎛ k −1 ⎞
⎜ ⎟t ∴ u(x, y) = B e⎝ 3 ⎠ e ⎝ 2 ⎠ ... (3)
⎝ 2 ⎠
= A ekx e Taking, y = 0 in equation (3), we get,
Where, A = c1c2 ... (4) KX

Taking, t = 0 in equation (4), ⇒ u(x, 0) = Be 3 ... (4)


But given,
u(x, 0) = Aekx
u(x, 0) = 4e–X ... (5)
6e–3x = Aekx From equations (4) and (5), we get,
A = 6 and k = –3. KX

Since u(x, 0) = 6 e–3x 4e–X = Be 3


Comparing the terms on both sides, we get, B = 4 and
On comparing the terms both sides, we get, K = –3.
u(x, t) = 6e–3x e–2t or 3Y
u(x, t) = 6e– (3x+2t) ∴ u(x, y) = 4 e − X e 2

Which is the particular solution of equation (1). Which is the required particular solution.

Look for the SIA GROU P LOGO on the TITLE COVER before you buy
UNIT-4 (Partial Differential Equations) 4.15
Integrating equation (9) with respect to ‘t’, we get,
∂ 2u
Q30. Solve = e − t cosx , given that u = 0 when
∂x∂t 1 −t
T = (e ) (−1) + a2
C
∂u
t = 0 and = 0 when x = 0. Show also that as
∂t
− e −t
t tends to ∞, u tends to sinx. T= + a2 ... (11)
C
∂ 2u
Ans: To solve = e–t cosx ... (1) Here, a1 and a2 are constants.
∂x.∂t
Substituting equations (10) and (11) in equation (5),
Assume that, u(x, t) = e–t cosx ... (2)
we get,
Given conditions are,
(i) u = 0 when t = 0 ⎛ − e −t ⎞
⇒ u(x, 0) = 0 ... (3) u = (C sin x + a1) ⎜⎜ + a2 ⎟

⎝ C ⎠
∂u
(ii) = 0 when x = 0 From equation (3),
∂t

∂ ⎛ − e0 ⎞
∂t
u ( 0, t ) = 0 ... (4) u(x, 0) = (C sin x + a1) ⎜⎜ + a2 ⎟

⎝ C ⎠
Let the solution of equation (2) be,
u(x, t) = X(x).T(t) ... (5) ⎛ −1 ⎞
Here, ⇒ 0 = (C sin x + a1) ⎜ + a2 ⎟
⎝C ⎠
X is a function of only ‘x’ and
T is a function of only ‘t’. −1
C sin x + a1 ≠ 0, + a2 = 0
∂X ∂T C
∴ X'= and T ' =
∂x ∂t
1
2 ∴ a2 = ... (12)
∂X ∂T ∂ ( XT ) C
⇒ X 'T '= =
∂x. ∂t ∂x.∂t
Since,
But, XT = u from equation (5),
∂u
∂2 u = XT' + X'T
∴ X 'T ' = ... (6) ∂t
∂x. ∂t
⎛ e −t ⎞ ⎛ −t ⎞
⎟ + (C cos x )⎜ − e + a ⎟
On substituting equation (6) in equation (1), we get,
X ' T ' = e–t cosx = (C sin x + a1 )⎜⎜ ⎟ ⎜ C 2⎟
⎝ C ⎠ ⎝ ⎠
On separating the variables ‘x’ and ‘t’, we get,
[Q From equations (8), (9), (10) and (11)]
−t
X' e From equation (4),
= ... (7)
cos x T'
∂ ⎛ e −t ⎞ ⎛ −t ⎞
⎟ + (C cos 0)⎜ − e + a ⎟
Since x and t are independent variables, equation (7) is
u ( 0, t ) = (C sin 0 + a1) ⎜
true only if both L.H.S and R.H.S are equal to the same arbitrary ∂t ⎜ C ⎟ ⎜ C 2⎟
constant C. ⎝ ⎠ ⎝ ⎠

X' e −t ⎛ e −t ⎞ ⎛ −t ⎞
∴ = C and =C ⇒ 0 = (0 + a1) ⎜⎜ ⎟ + C (1)⎜ − e + a ⎟
cos x T' ⎟ ⎜ C 2⎟
⎝ C ⎠ ⎝ ⎠
X '= C cos x ... (8)
e −t ⎛ e −t ⎞
−t ⇒ 0 = a1 + C⎜ − + a2 ⎟
e C ⎜ C ⎟
T '= ... (9) ⎝ ⎠
C
Integrating equation (8) with respect to ‘x’, we get, e −t ⎛ − e −t ⎞
⇒ a1 = − C⎜⎜ + a2 ⎟

X = C sin x + a1 ... (10) C ⎝ C ⎠

SPECTRUM ALL-IN-ONE JOURNAL FOR ENGINEERING STUDENTS SIA GROUP


4.16 MATHEMATICS-II [JNTU-ANANTAPUR]
On substituting these values in given equation, we get,
e −t
⇒ a1 = e–t – Ca2 X' Y = 4XY'
C
X ′ 4Y ′
⇒ a1 e–t = C(e–t – Ca2) ⇒ =
X Y
⎛ ⎛ 1 ⎞⎞ Since x and y are independent variables.
⇒ a1 e–t = C ⎜⎜ e −t − C ⎜ ⎟ ⎟⎟
⎝ ⎝ C ⎠⎠ ∴ Each side is equal to the constant, say K

[Q From equation (12)] X′


∴ =K
⇒ a1 e–t = C(e–t – 1) X
4Y ′
1 ⎛1 ⎞ And =K
a1 = C ⎜ t −1⎟ Y
et ⎝e ⎠
log X = Kx + log C1
a1 ⎛1− e t ⎞ X
t= C ⎜⎜ t ⎟
⎟ ⇒ = eKx
e ⎝ e ⎠ C1
∴ X = C1eKx
t
∴ a1 = C (1 − e ) ... (13) 4 log Y = Ky + logC2

∴ The required solution is, Y4


⇒ = eKy
u = X.T C2
⇒ Y 4 = C2eKy
⎛ − e −t ⎞
= (C sinx + a1) ⎜⎜ + a2 ⎟
⎟ ∴ Y = 4 C 2 eKy/4
⎝ C ⎠
[ Q From equations (10) and (11)] ∴ The unique solution of the given equation is,
u = C1eKx · 4 C 2 eKy/4
⎛ − e −t 1 ⎞
= (C sin x + C (1 − e ))⎜⎜ + ⎟
t

⎝ C C ⎟⎠ ⇒ u(x, y) = C1 4 C 2 eKx + Ky/4


[Q From equations (12) and (13)] ⇒ u(0, y) = Ce0 + Ky/4 [Q C1 4 C 2 = C]
⎛ − e −t + 1 ⎞ ⇒ 8e–3y = Ce Ky/4
= C[sin x + (1 − e )] ⎜⎜ ⎟
t
C ⎟ K
⎝ ⎠ ∴ C = 8, = –3 ⇒ K = –12
4
u( x, t ) = [sin x + (1 − e t )] (1 − e −t ) i.e., u = 8 e– 3y
Q32. Solve by the method of separation of variables
u(x,t) = [sinx(1 – e–t)] + [(1 – et) (1 – e–t)]
= sinx(1 – e–t) [Q (1 – et) (1 – e–t) = 0] ∂2z ∂z ∂z
−2 + = 0.
∂x 2 ∂x ∂y
When, t → ∞
u(x, ∞) = sinx(1 – 0) Ans: Here z is a function of X and Y
u(x, ∞) = sinx Let, z = XY
Where, X is a function of x and that of y
Hence proved.
∂2z ∂z ∂z
∂u ∂u Then 2 =X" Y, = X' Y, = XY'
Q31. Solve =4 where u(0, y) = 8e–3y by the ∂x ∂x ∂y
∂x ∂y
On substituting this value of z in the given equation,
method of separation of variables.
we get,
Ans: Here u is a function of x and y X" Y – 2X' Y + XY' = 0
Let, u = XY ⇒ (X" – 2X') Y = – XY'
Where X is a function of x and that of y
X ′′ − 2 X ′ Y′
⇒ =– ... (1)
∂u ∂u X Y
then = X' Y and = XY'
∂x ∂y Since x and y are independent variables.

Look for the SIA GROU P LOGO on the TITLE COVER before you buy
UNIT-4 (Partial Differential Equations) 4.17
∴ Equation (1) can be true if each side is equal to the same Since X and T are independent variable we can equate to
constant (say a), it by constant, we get,

X ′′ − 2 X ′ −Y ′ X ′′ 1 T ′′
∴ = =a = 2 =K
X Y X a T

X ′′ − 2 X ′ X ′′ ∂X
⇒ =a = K i.e., X' =
X X ∂x
X" – 2X' – aX = 0
⇒ (D2 – K)X = 0
On solving the ordinary linear equation, we get,
Case (i): If K is positive constant, say p2
⎡ d⎤
(D2 – 2D – a) X = 0 ⎢Q D =
⎣ dx ⎥⎦ (D2 – p2)X = 0
Auxiliary roots are 0, 1 ± 1 + a A. E D2 – p2 = 0 ⇒ D = ± p
∴ C.I X = C1 e–px + C2epx
(1− (1+a ) x )
(1+ (1+ a ) x )
∴ The solution is, X = C1 e + C2 e Case (ii): If K is negative say ‘–p2’, we get,
−Y ′
And the solution of =a (D2 + p2)X = 0
Y
⇒ – log Y = ay + log C A. E D2 + p2 = 0
D = ± ip
⎛Y ⎞
⇒ log ⎜ ⎟ = – ay ∴ C.I X = C3 cos px + C4 sin px
⎝C ⎠
Case (iii): If K is zero, then,
Y
⇒ = e–ay (D2 – 0)X = 0
C
A. E D2 = 0
∴ Y = Ce–ay
D = 0, 0
∴ The required solution,
z = XY T ′′
∴ C.I X = C5 + C6x and =K
[
z = C1e (1+ (1+ a ) x )
+ C2 e (1− (1+ a ) x )
]Ce − ay
a 2T

2 dT
∂ 2u ∂ 2u ( D 1 – a2K)T = 0 i.e., T' =
Q33. Solve 2
= a 2 2 by separation of variable dt
∂t ∂x
method. Case (i): If K is positive say ‘–p2’, we get,

Ans: The given partial differentiation,


2
( D 1 + a2p2)T = 0
∂ u
2
2
∂ u 2

2 =a ... (1)
∂t ∂x 2 2
A.E D 1 + a2 p2 = 0
The solution of the form u = XT where X is a function D' = ± ap
of x and that of T.
∴ C.I T = C7 e–apt + C8 eapt
∂ u
2
∂ u 2
Case (ii): If K is negative say ‘–p2’, we get,
Then 2 = XT" and = X"T
∂t ∂x 2
2
( D 1 + a2p2)T = 0
Equation (1) becomes,
XT" = a2X"T 2
A. E D 1 + a2 p2 = 0
T ′′ X ′′ D1 = ± ap
⇒ 2 =
a T X ∴ C.I T = C9 cos (apt) + C10 sin (apt)

SPECTRUM ALL-IN-ONE JOURNAL FOR ENGINEERING STUDENTS SIA GROUP


4.18 MATHEMATICS-II [JNTU-ANANTAPUR]
Case (iii): If K is zero,

2
( D 1 – 0)T = 0

A. E D1 = 0, 0

∴ C.I T = C11 + C12 t

The possible solution of a differential equation is u = XT

u = (C1e–px + C2epx)(C7e–apt + C8eapt)

⇒ u = (C3 cos px + C4 sin px)(C9 cos (apt) + C10 sin (apt))

And u = (C5 + C6 x)(C11 + C12t)

4.3 SOLUTIONS OF ONE-DIMENSIONAL WAVE EQUATION, HEAT EQUATION


Q34. Write complete solutions of wave equation.
Ans: One of the engineering applications of partial differential equations is in wave equations.
The one-dimensional wave equation is,

∂2 y 2 ∂ y
2
= c
∂t 2 ∂x 2

Vibration of a Stretched String (Derivation)


Consider a string (elastic) of length ‘l’, fixed lightly between points O and A and subjected to constant tension T. Neglecting
the effect of gravity. The string at rest is allowed to vibrate with no external force acting on it, assume that each point on the string
OA makes small vibrations at right angles to equilibrium position of string OA, entirely in one plane.
Let end ‘O’ be origin,
OA is along X-axis
OY ⊥lar to X-axis is Y-axis
So that the motion is entirely one-dimensional in XY-plane.
Figure (a) shows the string is at rest.
y
y
T
Q δ + δθ
δy P
θ y
x x
x T
O A O A
δx
Figure (a): String is at Rest Figure (b): String in Vibration at Time ‘t’
Figure (b) shows the string making vibrations perpendicular to X-axis.
At t the string is at position OPA. Consider the motion of two points say P and Q on the string.
P(x, y) and Q(x + δx, y + δy), where the tangents make angle θ and (θ + δθ) with X-axis.

∂2 y
Acceleration of element moving upward =
∂t 2

Look for the SIA GROU P LOGO on the TITLE COVER before you buy
UNIT-4 (Partial Differential Equations) 4.19
Vertical component of force action on this element (PQ)
= T sin(θ + δθ) – Tsinθ
= T [sin(θ + δθ) – Tsinθ] [Qθ is small]

⎡ dy ⎤
= T [tan(θ + δθ) – tanθ] ⎢Q tan θ = dx ⎥
⎣ ⎦

⎡⎧ ∂y ⎫ ⎧ ∂y ⎫ ⎤
= T ⎢⎨ ⎬ −⎨ ⎬ ⎥
⎣⎩ ∂x ⎭ x +δx ⎩ ∂x ⎭ x ⎦
If m is the mass per unit length of string, then by Newton’s second law of motion.
Force = Mass × Acceleration

∂2 y ⎡⎧ ∂y ⎫ ⎧ ∂y ⎫ ⎤
m.δx. =T ⎢⎨ ⎬ −⎨ ⎬ ⎥
∂t 2 ⎣⎩ ∂x ⎭ x +δx ⎩ ∂x ⎭ x ⎦

⎡ ⎧ ∂y ⎫ ⎧ ∂y ⎫ ⎤
⎢ ⎨ ⎬ −⎨ ⎬ ⎥
∂ y T ⎢ ⎩ ∂x ⎭ x + δx ⎩ ∂x ⎭ x ⎥
2
=
∂t 2 m ⎢ δx ⎥
⎢ ⎥
⎣ ⎦
Taking limit as Q → P
δx → O
Applying limits we get,

∂2 y T ∂2 y
=
∂t 2 m ∂x 2

⎡ f ( x + h ) − f ( x) d ⎤
⎢Q hlt = f ( x) ⎥
⎣ → 0 h dx ⎦
T
Let, c2 =
m
∂2 y 2 ∂ y
2
= c
∂t 2 ∂x 2
The above partial differential equation gives the transverse vibration of string, which is known as one-dimensional wave
equation.
Q35. If a string of length ‘l ’ is initially at rest in equilibrium position and each of its points is given the
πx
velocity v0 sin3 , find the displacement y(x, t).
l
Ans: Wave equation is,

2 ∂ y
2
∂2 y
= a ... (1)
∂t 2 ∂x 2
Boundary conditions are,
(i) y(0, t) = 0
(ii) y(l, t) = 0
∂y
(iii) (x, 0) = 0
∂t
∂y ⎛ πx ⎞
(iv) (x, 0) = v0 sin3 ⎜ ⎟
∂t ⎝ l ⎠
y(x, t) = (c1 cos px + c2 sin px) (c3 cos pat + c4 sin pat) which satisfies the boundary conditions . ... (2)

SPECTRUM ALL-IN-ONE JOURNAL FOR ENGINEERING STUDENTS SIA GROUP


4.20 MATHEMATICS-II [JNTU-ANANTAPUR]
By applying condition (i) in equation (2), we get,
y(0, t) = c1(c3 cos pat + c4 sin pat)
Where, c3 cos pat + c4 sin pat ≠ 0

∴c1 = 0

On substituting in equation (2), we get,


y(x, t) = c2 sin px(c3 cos pat + c4 sin pat) ... (3)
By applying condition (ii) in equation (3), we get,
y(l, t) = c2 sin pl(c3 cos pat + c4 sin pat)
0 = c2 sin pl(c3 cos pat + c4 sin pat)
When c2 ≠ 0 and c3 cos pat + c4 sin pat ≠ 0
∴ sin pl = nπ
⇒ pl = nπ


p=
l
On substituting in equation (3), we get,

nπx ⎛ nπat nπat ⎞


y(x, t) = c2 sin ⎜ c3 cos + c4 sin ⎟ ... (4)
l ⎝ l l ⎠
On differentiating equation (4) with respect to ‘t’, we get,

∂y nπx ⎛ nπa nπat nπa nπat ⎞


( x, t ) = c2 sin ⎜ − c3 sin + c4 cos ⎟
∂t l ⎝ l l l l ⎠
By applying condition (iii), we get,

∂y nπx ⎛⎜ c nπa ⎞⎟
(x,0) = c2 sin 4
∂t l ⎝ l ⎠

nπx
0 = c2 sin (c4 nπa)
l

⎡ nπx ⎤
∴c4 = 0 ⎢Q c2 ≠ 0 and sin l ≠ 0⎥
⎣ ⎦
On substituting above value in equation (4), we get,

nπx nπ at
y(x, t) = c2 sin c3 cos
l l

nπx nπ at
y(x, t) = cn sin cos [Put c2 c3 = cn]
l l
∴ General solution is,


nπx nπat
y(x, t) = ∑c
n =1
n sin
l
cos
l
... (5)

Look for the SIA GROU P LOGO on the TITLE COVER before you buy
UNIT-4 (Partial Differential Equations) 4.21
Given,

πx 1
y(x, 0) = v0 sin3 and sin3 x = (3 sin x − sin 3x )
l 4

sin 3 πx 1
= (3 sin πx – sin 3πx) ... (6)
l 4l
From equations (5) and (6),

nπx v0
∑c
n =1
n sin
l
=
4l
(3 sin πx – sin 3πx)

sin πx sin 2πx sin 3πx v0


c1 + c2 + c3 + .... = (3 sin πx − sin 3πx )
l l l 4l
On equating like coefficients, we get,

3v0 −v0
c1 = , c = 0, c3= , c = 0, c5 = 0
4l 2 4l 4
On substituting these values in equation (6), we get,

3v0 πx πat v0 3πx 3πat


y(x, t) = sin cos − sin cos
4 l l 4 l l
Q35. A tightly stretched string with fixed end points x = 0 and x = 1 is initially at rest in its equilibrium
position. If it is set on vibrating by giving each of its points a velocity λ x(1–x), find the displacement of
the string at any distance x from end at any time ‘t’. Model Paper-II, Q9

Ans: Given boundary conditions are,


y(0, t) = y(l, t) = 0
Since,

nπct nπct ⎞ nπx
y(x, t) = ∑ ⎛⎜⎝ a
n =1
n cos
l
+ bn sin
l ⎠
⎟ sin
l
… (1)

As the string is at rest initially,


y(x, 0) = 0
∴ From equation (1),

nπx
0= ∑a
n =1
n sin
l

⇒ an = 0


∴y(x, t) = ∑b n sin ⎛⎜ nπct ⎞⎟ sin ⎛⎜ nπx ⎞⎟ … (2)
n =1 ⎝ l ⎠ ⎝ l ⎠


∂y nπc
And =
∂t n =1 l
∑bn cos nπct sin nπx
l l


πc
=
l
∑ nb
n =1
n sin
nπx
l

SPECTRUM ALL-IN-ONE JOURNAL FOR ENGINEERING STUDENTS SIA GROUP


4.22 MATHEMATICS-II [JNTU-ANANTAPUR]

δy
But, = λx (l – x) when t = 0
δt


πc
∴ λx (l – x) =
l
∑ nb
n =1
n sin
nπx
l

1
πc
∫ n(1 − n)
2
⇒ nbn =
l l 0

1
πc 2λ ⎡ ⎛ −l nπx ⎞ ⎛ − l2 nπx ⎞ ⎛ l3 nπx ⎞⎤
⎢ x (l − x)⎜ cos ⎟ − (l − 2 x )⎜ sin ⎟ + ( −2)⎜ 3 3 cos ⎟⎥
.nbn =
⎝ nπ ⎜ ⎟ ⎜ l ⎟⎠⎦⎥
⎝n π ⎝n π
2 2
l l ⎣⎢ l ⎠ l ⎠
0

4λ l 2 4λ l 2
= (1 – cos nπ) = [1 – (– 1)n]
n 3π 3 n 3π 3

= 0, When n is even

8λ l 2
= 3 3 , When n is odd
n π

8λl 2
i.e., , taking n = 2m – 1
π3 (2m − 1) 3

8λl 3
⇒ bn =
cπ ( 2m − 1) 4
4

From equation (2), the required solution is,


8λl 3 ⎛ (2m − 1)πct ⎞ (2m − 1)πx
∑ (2m − 1)
1
y(x, t) = sin ⎜ ⎟ sin
cπ 4 4 ⎝ l ⎠ l
m =1

Q37. Find the deflection u(x, y, t) of the square membrane with a = b = 1 and c = 1, if the initial velocity is
zero and the initial deflection is f(x, y) = A sin π x sin 2π y. Model Paper-I, Q9

Ans: Given that,


a = b = 1 and f (x, y) = A sin πx sin 2πy
On substituting in generalised Euler’s formula,
1 1

Amn = 4 ∫ ∫ A sin πx sin 2πy sin mπx sin nπy dy dx


0 0

1
⎡1 ⎤

= 4A sin πx sin mπx dx ⎢ ∫ sin 2πy sin nπy dy ⎥ = 0 for m ≠ 1
0 ⎢⎣0 ⎥⎦

⎛1⎞
1 ⎡ 1 1⎤
= 4A ⎜ ⎟
⎝ 2⎠
∫ sin 2πy sin nπy dy, for m = 1
⎣⎢ 0

⎢Q sin πx sin πx dx = ⎥
2 ⎦⎥
0

Look for the SIA GROU P LOGO on the TITLE COVER before you buy
UNIT-4 (Partial Differential Equations) 4.23
1

= 2A ∫
0
sin 2πy sin nπy dy = 0, for n ≠ 2

⎛1⎞
= 2A × ⎜ ⎟ (for n = 2)
⎝ 2⎠
∴ A12 = A ... (1)

2 2
⎛ m⎞ ⎛ n⎞
Also, pmn = πc ⎜ ⎟ +⎜ ⎟
⎝ a ⎠ ⎝b⎠

= π × 1 (1) 2 + ( 2) 2 = 5 π ... (2)

ab
On substituting equations (1) and (2) in u(x, y, t) = A (A cos pt + Bmnsin pt)
4 mn mn
∴ Required solution is u(x, y, t) = A sin πx sin 2πy cos ( 5 πt)
Q38. The points of trisection of a string are pulled aside through the same distance on opposite sides of the
position of equilibrium and the string is released from rest. Derive an expression for the displacement
of the string at subsequent time and show that the midpoint of the string always remains at rest.
Ans: Let B and C be the points of trisection of the string OA of length l, say initially the string is hold in the form OB' C 'A.
Where,
BB' = CC' = a (say)
3a
The equation of OB' is, y = x
l

− 2a ⎛ l⎞
The equation of B' C' is, y – a = ⎜x − ⎟
l ⎝ 3⎠
3
y ⎛l ⎞
B′⎜ , a ⎟
⎝3 ⎠

C x
0
B A'(1, 0)
⎛ 2l ⎞
C′⎜ , − a ⎟
⎝3 ⎠
Figure
− 6a ⎛ l⎞
y–a = ⎜x− ⎟
l ⎝ 3⎠

⎛ 6⎛ l ⎞⎞
⇒ y = a⎜⎜1 − ⎜ x − ⎟ ⎟⎟
⎝ l⎝ 3 ⎠⎠

⎛ 6x ⎞ ⎛ 6x ⎞
= a⎜1 − + 2 ⎟ = a⎜ 3 − ⎟
⎝ l ⎠ ⎝ l ⎠

3a
⇒ y= (l – 2x)
l
3a
The equation of C'A is, y = (x – l)
l
Here the boundary conditions are,
y(0, t) = y (l, t) = 0

SPECTRUM ALL-IN-ONE JOURNAL FOR ENGINEERING STUDENTS SIA GROUP


4.24 MATHEMATICS-II [JNTU-ANANTAPUR]
And the initial conditions are

⎧ 3a l
⎪ l x 0≤ x≤
3
⎪⎪
3 a
y(x, 0) = ⎨ (l − 2 x ),
l 2l
≤x≤
⎪ l 3 3
⎪ 3a 2l
⎪⎩ l ( x − l ), 3
≤ x≤l

∂y
and = 0 when t = 0
∂t
Therefore, wave equation is,

⎛ nπct ⎞ ⎛ nπx ⎞
y(x, t) = ∑a n cos⎜ ⎟ sin⎜
⎝ l ⎠ ⎝ l ⎠

n =1

Where,

2 ⎡ 3a nπx ⎤
l/3 2l / 3 l
2 l
⎛ n πx ⎞ ⎛ nπx ⎞ 3a nπx 3a
an =
l ∫ y(x,0) sin⎜
⎝ l ⎠ l ⎢⎣ 0 ⎝ ∫
⎟ dx= ⎢ l x sin⎜ l ⎟dx +
⎠ ∫ l
(l − 2 x ) sin
l
dx + ∫ l
( x − l ) sin
l
dx ⎥

0 l /3 l /3 ⎦

⎡ ⎛ ⎞
l
⎡ ⎛ ⎞
2l
3
⎛ − π ⎞ π
3 2
⎛ − l
6a ⎢ x⎜ cos nπ x ⎞ − l 2
n π x ⎢ l n x l n x
⎟ −1⎜ sin ⎟ (l − 2 x)⎜ cos ⎜
⎟ − (−2) 2 2 sin ⎟
= 2 ⎢ ⎝ nπ l ⎠ ⎝ n2π 2 l ⎠ +⎢ ⎝ nπ l ⎠
l ⎣ 0 ⎣ ⎝n π l ⎠l
3

l
⎛−l nπx ⎞ ⎛ − l 2 nπx ⎞
+ ( x − l )⎜ cos ⎟ − 1⎜ 2 2 sin ⎟
⎝ nπ l ⎠ ⎝n π l ⎠ 2l
3

6a ⎡⎛ − l 2 nπ l2 nπ ⎞ ⎛ l 2 2 nπ 2l 2 2 nπ l 2 nπ 2l 2 nπ ⎞
= ⎢⎜⎜ cos + 2 2 sin ⎟+⎜
⎟ ⎜ cos − sin + cos + sin ⎟
l2 ⎣⎢⎝ 3nπ 3 n π 3 ⎠ ⎝ 3nπ 3 n π
2 2
3 3nπ 3 n π
2 2
3 ⎟⎠

⎛ l2 2 nπ l2 2 nπ ⎞ ⎤ 6a 3l 2 ⎛ nπ 2nπ ⎞ 18a nπ
−⎜ cos + 2 2 sin ⎟⎥ = ⎜sin − sin ⎟ = 2 2 sin (1 + (−1) n )
⎝ 3n π 3 n π 3 ⎠ ⎦⎥ l n π ⎝
2 2 2
3 3 ⎠ n π 3

⎛ 2nπ ⎛ 3nπ − nπ ⎞ nπ nπ nπ nπ ⎞
⎜since, sin = sin ⎜ ⎟ = sin nπ cos – cos nπ sin = 0 – (–1)n sin = – (–1)3 sin ⎟
⎝ 3 ⎝ 3 ⎠ 3 3 3 3 ⎠
an = 0, when n is odd
36a nπ 9a 2mπ
= sin , when n is even i.e., sin Taking, n = 2m
n 2π 2 3 m 2π 2 3

2mπ 2mπct 2mπx
∑m
9a 1
Hence, y(x,t) = 2 sin cos sin
π m =1
2 3 l l


⎛ l ⎞ 9a 2mπ 2mπct
∑m
1
Also, y⎜ ,t⎟ = 2 sin cos sin mπ
⎝2 ⎠ π m =1
2 3 l

= 0 since sin m π = 0
The displacement of the midpoint of the string is zero for all the values of t.
Thus the midpoint of the string is always at rest.

Look for the SIA GROU P LOGO on the TITLE COVER before you buy
UNIT-4 (Partial Differential Equations) 4.25
Q39. A tightly stretched unit square membrane starts vibrating from rest and its initial displacement is
k sin 2π x sin π y. Show that the deflection at any instant is k sin 2π x sin π y cos( 5 π ct).
Ans: Given that,
Taking a = b = 1, f(x, y) = k sin 2πx sinδπ y.
1 1

∴ Amn = 4 ∫ ∫ k sin 2πx sin πy sin mπx sin nπy dy dx


0 0

1
⎡1 ⎤
=4k ∫
0
sin 2πx sin mπx dx ⎢

⎢⎣ 0
sin πy sin nπy dy ⎥ = 0; for n ≠ 1
⎥⎦
1 1
⎛1⎞ 1
= 4k ⎜ ⎟
⎝ 2⎠

0
sin 2πx sin mπx dx, for n = 1 ∫
Q sin πx sin πx dx =
0
2


= 2k sin 2πx sin mπy dx = 0, for m ≠ 2
0

⎛1⎞
= 2k × ⎜ ⎟ ; for m = 2
⎝2⎠

∴ A12 = k and pmn = πc m2 + n2

= πc 2 2 + 12 = πc 5
∴ The required solution is obtained by, substituting the values of A12 and P12 in,
∞ ∞
⎛ mπx ⎞ ⎛ nπy ⎞
u= ∑ ∑ sin ⎝⎜
m =1 n =1
a ⎠
⎟ sin ⎜
⎝ b ⎠
⎟ [Amn cos pt]

∞ ∞
⎛ mπx ⎞ ⎛ nπy ⎞
∴ f (x, y) ∑∑ A
m =1 n =1
mn sin ⎜ ⎟ sin ⎜
⎝ a ⎠ ⎝ b ⎠

u(x, y, t) = Amn f (x, y)cos pt


[Q f (x, y) = Initial deflection]
u(x, y, t) = k sin 2πx sin πy cos πc 5 t
Q40. The ends A and B of a rod 20 cm long have the temperature at 30°C and 80°C until study states prevail.
The temperatures of the ends are changed at 40°C and 60°C respectively. Find the temperature
distribution in the rod at time t.

∂u ∂ 2u
Ans: Let = α2 2 ... (1)
∂t ∂x
The boundary conditions are,
u(0, t) = 30°C ... (2)
and u(l, t) = 80°C ... (3)
∂u
Where l is the length of the rod. Under steady state conditions =0
∂t
From equation (1),

∂ 2u
= 0 ⇒ u = ax+b ... (4)
∂x 2
and from equation (2), we get,
b = 30
SPECTRUM ALL-IN-ONE JOURNAL FOR ENGINEERING STUDENTS SIA GROUP
4.26 MATHEMATICS-II [JNTU-ANANTAPUR]
On substituting b value in equation (4), we get, ut(x, 0) = u(x, 0) – us(x)
⇒ u = ax + 30
50 x ⎛ 20 x ⎞
80 = la + 30 [Q u(1, t) = 80°] = + 30 − ⎜ + 40 ⎟
l ⎝ l ⎠
50
⇒ a= From equations (8) and (10), we get,
l
Hence, equation (4) becomes,
30 x
⇒ ut(x, 0) = − 10
50 x l
⇒ u= + 30 ... (5)
l
Therefore, the transient solution is given by,
The subsequent temperature distribution is determined
from the boundary value problem (1) with boundary conditions,
−α 2 p 2t
ut(x, t) = (A cos px + B sin px) e ... (11)
u(0, t) = 40 ... (6)
and u(l, t) = 60 ... (7) With boundary conditions
and the initial temperature is,
ut(0, t) = 0 ... (12)
50 x
u(x, 0) = + 30 ... (8) and ut(l, t) = 0 ... (13)
l
Let, u(x, t) = us(x, t) + ut(x) ... (9) and the initial condition is,
[Q u(x, t) is non zero]
Where us (x) is a solution of (1) satisfying conditions 30 x
ut(x, 0) = − 10 ... (14)
equations (5) and (6) and is called steady state solution whereas l
ut(x, t) is called the transient solution which decreases with the
increase of time. From equation (11), A = 0

Let, us(x) = px+q −α 2 p 2t


⇒ ut(x, t) = B sin px e ... (15)
Putting x = 0 and using equation (5), we get q = 40
⇒ 40 = l Also, from equation (12), sin px = 0 for all n so that

Thus, us(x) = kx+40.



Putting x = l and using equation (6), we get, p = , n = 1, 2, 3,........
l
60 = kl + 40
nπx
20 ⇒ ut(x, t) = B sin − α 2 n 2 π2 t l2 , n = 1, 2, 3 ... (16)
⇒ k=
l l e

Hence, the general solution is,


20 x
⇒ us(x) = + 40 ... (10)
l ∞
nπx −α 2 n 2 π2 t
ut(x, t) = u(x, t) – us(x) ut(x, t) = ∑b
n =1
n sin
l
e l2
, n = 1, 2, 3 ... (17)
So that,
ut(0, t) = u(0, t) – us(0) On substituting t = 0 in equation (16) and then comparing
equation (13), we get,
= 40 – 40

= 0 and nπx
ut(l, t) = u(l, t) – us(l)
30 x
− 10 = ∑b n sin
l
... (18)
l n =1

⎛ 20 x ⎞
= 60 – ⎜ + 40 ⎟ l
⎝ l ⎠ 2 ⎛ 30 x ⎞ nπx
bn = ⎜
l 0⎝ l ∫
− 10 ⎟ sin
⎠ l
dx
=0

Look for the SIA GROU P LOGO on the TITLE COVER before you buy
UNIT-4 (Partial Differential Equations) 4.27
l
2 ⎡⎛ 30 x ⎞⎛ nπx ⎞⎛ l ⎞ 30 ⎛ nπx ⎞ l 2 ⎤
= ⎢⎜ − 10 ⎟⎜ − cos ⎟⎜ ⎟ − ⎜ − sin ⎟ ⎥
l ⎣⎝ l ⎠⎝ l ⎠⎝ nπ ⎠ l ⎝ l ⎠ n 2 π2 ⎦ 0

2 ⎡⎛ 30l ⎞ ⎤
=− ⎢⎜ − 10 ⎟ cos nπ − (− 10 )cos 0⎥
nπ ⎣⎝ l ⎠ ⎦

bn = –
2

[
20 x(− 1)n + 10 =
20

]2(− 1) + 1 ,
n
[ ]
n = 1, 2,...
On substituting value of bn in equation (16), we get,


− 20 nπx −α 2 n 2 π2 t
ut(x, t) = ∑
n =1

[( −1) n 2 + 1] sin
l
e l2
... (19)


20 x − 20 nπx −α 2 n 2 π 2 t
⇒ u(x, t) = l
+ 40 + ∑n =1

[( −1) n 2 + 1] sin
l
×e l2

20 ∞
(− 1)n 2 + 1 sin nπx × e − α n π t 20

2 2 2 2
Writing l = 20 then u(x, t) = x + 40 –
π n =1
n 20
Q41. A bar 100 cm long, with insulated sides, has its ends kept at 0°C and 100°C until steady state conditions
prevail. The two ends are then suddenly insulated and kept 80°C. Find the temperature distribution.
Ans: The temperature U(x, t) along the bar satisfies the equation.

∂u ∂ 2u
= c2 ... (1)
∂t ∂x 2
By law of heat conduction, the rate of heat flow is proportional to the gradient of the temperature.
Thus, if the ends x = 0 and x = l = 100 cm of the bar are insulated so that no heat can flow through the ends. The boundary
conditions are,

∂u
(0, t ) = 0
∂x
∂u
(l , t ) = 0 for all ‘t’ ... (2)
∂x

∂ 2u
Initially under steady state conditions = 0. Its solution is u = ax + b.
∂x 2
Since u = 0 for x = 0 and u = 100 for x = l.
b = 0 and a = 1.
Thus, the initial condition is
u(x, 0) = x 0<x<1 ... (3)
The solution of equation (1) is of the form
− c 2 p 2t
u(x, t) = (c1 cos px + c2 sin px) e ... (4)
On differentiating partially with respect to ‘x’ we get,

∂u
= (− c1 p sin px + c2 p cos px ) e −c pt
2
... (5)
∂x
SPECTRUM ALL-IN-ONE JOURNAL FOR ENGINEERING STUDENTS SIA GROUP
4.28 MATHEMATICS-II [JNTU-ANANTAPUR]
Putting x = 0.

⎛ ∂u ⎞ 2 2
⎜ ⎟ = c2 pe − c p t = 0 for all t (QBy equation (2))
⎝ ∂x ⎠ x =0
c2 = 0
Putting x = l in equation (5)
⎛ ∂u ⎞
⎜ ⎟ = c1p sin ple
− c 2 p 2t
for all t (QBy equation (2))
⎝ ∂x ⎠ x = l
c1p sin pl = 0 (Q p ≠ 0 either c1 = 0 or sin pl = 0)
or pl = n π

p= , n = 0, 1, 2, ......
l
Hence equation (4) becomes,
n 2 π2
nπx −c 2 l2
t
u(x, t) = c1cos e
l
The most general solution of equation (1) satisfying the boundary conditions equation (2) is,

nπx −c 2 n 2 π2 t
u(x, t) = ∑ A cos
n =0
n
l
e l2


nπx −c 2 n 2 π2 t
= A0 + ∑ A cos
n =0
n
l
e l2
(Where, An = c1) ... (6)

Putting t = 0.

nπx
u(x, 0) = A0+ ∑ A cos
n =1
n
l
=x (Q by equation (3))
By expanding ‘x’ into a half range cosine series in (0, 1)

a0 nπx
Thus, x=
2
+ ∑a
n =1
n cos
l
l
2
Where, a0 = ∫
l 0
x dx = l

l
2 nπx 2l
and an = ∫
l 0
x cos
l
dx = 2 2 (cos nπ − 1)
n π

⎧ 0 when n is even

= ⎨ − 4l
for n odd
⎪⎩ n 2 π 2

a0 l
A0 = = and
2 2
An = an = 0 for n even
−4l
= for n odd
n 2 π2
Hence equation (6) takes the form,

l − 4l nπx −c 2 n 2 π 2 t
u(x, t) = +
2 m =1 n π∑
2 2
cos
l
e l2

l 4l ∞
1 (2n − 1)πx e −c
∑ (2n − 1)
−c 2 ( 2 n−1)2 π2 t l 2
= − cos
2 π2 n =1
2
l

Look for the SIA GROU P LOGO on the TITLE COVER before you buy
UNIT-4 (Partial Differential Equations) 4.29
Q41. The temperature at one end of a bar is 50 cm long with insulated sides is kept at 0oC
and that the other
end is kept at 100oC until steady state condition prevails. The two ends are then suddenly insulated so
that the temperature gradient is zero at each and thereafter. Find the temperature distribution.
Ans: Let the length of the rod be ‘l’ cm
Where, l = 50 cm
The steady state solution is, u = ax + b
and the boundary conditions are,
u(0) = 0 and u(50)= 100
⇒ b = 0 and a =2
∴ u = ax + b
∴ u = 2x.
One-dimensional heat equation is,

∂u ∂ 2u
= α2 . 2 ... (1)
∂t ∂x
Given that the ends are insulated and hence no heat flows through the ends. The boundary conditions are,

⎛ ∂u ⎞
⎜ ⎟ =0 ... (A)
⎝ ∂t ⎠ x = 0

⎛ ∂u ⎞
⎜ ⎟ =0 ... (B)
⎝ ∂t ⎠ x =l
The initial condition is,
u(x, 0) = 2x , 0 < x < l
The solution of equation (1) is,

u(x, t) = (A cos px + B sin px) e − p α 2t


2
... (2)
On differentiating equation (2) partially with respect to ‘x’, we get,

∂u
= (–Ap sin px + Bp cos px) e − p α2 t
2

∂x
Using equation (A) we get,

Bp e − p α 2t
2
=0 (Q cos 0 = 1)
⇒ B =0

∂u
= – Ap sin px e − p α t
2 2

∂x
Using equation (B),

–Ap sin pl e −α
2
p 2t
=0 (Q B = 0)
⇒ sin pl = 0
⇒ pl = nπ

⇒ p =
l

⎛ nπx ⎞ − α2 n2 π 2t /l 2
∴ u(x, t) = A cos ⎜ ⎟.e
⎝ l ⎠

SPECTRUM ALL-IN-ONE JOURNAL FOR ENGINEERING STUDENTS SIA GROUP


4.30 MATHEMATICS-II [JNTU-ANANTAPUR]
∴ The most general solution is,

∴ u(x, t) = ∑
n =1
An cos
nπx − α2 n2 π 2t /l 2
l
e

∑ A cos
nπx − α 2 n 2 π 2t / l 2
∴ u(x, t) = A0+ n e
n =1 l
Let t = 0,

⇒ u(x, 0) = A0 + ∑
n =1
An cos
nπx
l
But u(x, 0) = 2x, 0 < x < l. (initial condition)
Expanding u(x, 0) in half-range cosine series,

⎛ nπx ⎞
u(x, 0) =
a0
2
+ ∑a
n =1
n cos ⎜
⎝ l ⎠

l l
2 ⎡ 2x2 ⎤

2 2
a0 = 2x dx = ⎢ ⎥ = . l2
l 0 l ⎣ 2 ⎦ l
0

a0 = 2l
a0
∴ A0 = =l
2
l
4 ⎡ ⎛ nπx ⎞ l 2 ⎤
l
2 nπx nπx ⎞ l ⎛
an =
l ∫
0
2x cos
l
dx = ⎢ x
l ⎣ ⎝
⎜ sin ⎟ +
l ⎠ nπ ⎝
1 ⎜ cos ⎟. ⎥
l ⎠ n2 π2 ⎦ 0

4 ⎡ l2 ⎤
× ⎢ 2 2 [cos nπ − cos 0]⎥ = 2 2 [(–1)n – 1]
4l
=
l ⎣n π ⎦ n π
4l
∴ An = an = [(–1)n – 1].
n2 π2

4l
∴ u(x, t) = l + ∑n π
n =1
2 2 [(–1)n – 1]cos
nπx
l e
− α 2 n 2 π 2t / l 2

On substituting, l= 50 in the above equation, we get,


− α 2 n 2 π 2t

[(−1) n − 1] nπx
200
∴ u(x, t) = 50 – 2 .
π n =1 n 2 ∑
cos
50
e ( 50 ) 2
.

Q43. A homogeneous rod of conducting material of length 100 cm has its ends kept at zero temperature and
the temperature initially is
u(x, 0) = x; 0 < x < 50
= 100 – x; 50 < x < 100
Find the temperature u(x, 0) at any t.
Ans: The general solution of the heat equation is,
∂u 2 ∂ u
2
=C ... (1)
∂t ∂x 2
Which describes the temperature distribution u(x, t) subject to the boundary conditions u(0, t) = 0 is given by,
∞ 2
⎛ nπx ⎞ ⎛ cnπ ⎞
u(x, t) = Σ bn sin⎜ ⎟ .C − ⎜ ⎟ t ... (2)
n =1 ⎝ 100 ⎠ ⎝ 100 ⎠
Where 0 < x < 100.

Look for the SIA GROU P LOGO on the TITLE COVER before you buy
UNIT-4 (Partial Differential Equations) 4.31
On substituting the initial condition the half-range sine series for u(x, 0) in 0 < x < 100 is obtained as,


⎛ nπx ⎞
u(x, 0)= ∑b
n =1
n sin ⎜ ⎟
⎝ 100 ⎠

100
2 ⎛ nπx ⎞
⇒ bn =
100 ∫ u ( x, 0) sin⎜⎝ 100 ⎟⎠ dx
0

1 ⎡ ⎤
50 100
⎛ nπx ⎞ ⎛ nπx ⎞
=
50 ⎢ 0


⎢ x sin ⎜ ⎟ dx +
⎝ 100 ⎠ ∫ (100 − x)sin ⎜ ⎟ dx⎥
50
⎝ 100 ⎠ ⎥⎦

1 ⎡ ⎛ nπx ⎞ ⎤ − x sin ⎛⎜ nπx ⎞⎟ dx


50 100 100
⎛ nπx ⎞
=
50 ⎢ ∫
⎢ x sin ⎜

⎟ dx + 100 sin ⎜
100 ⎠ ⎝ ∫
⎟ dx ⎥
100 ⎠ ⎥ 50 ⎝ 100 ⎠ ∫ ... (3)
⎣0 50 ⎦
Considering,

50
⎡ ⎛ nπx ⎞ ⎛ nπx ⎞ ⎤
50 ⎢ cos⎜ ⎟ sin ⎜ ⎟⎥
⎛ n πx ⎞ ⎝ 100 ⎠ + 1. ⎝ 100 ⎠ ⎥
0

x sin ⎜
⎝ 100 ⎠
⎟ dx = ⎢− x .
⎢ nπ n2π2 ⎥
⎢ 100 ⎥
⎣ 100 ⎦ x =0

2
100 ⎡ ⎛ nπ ⎞⎤ ⎛ 100 ⎞ ⎛ nπ ⎞
=− ⎢50 cos⎜ 2 ⎟⎥ + ⎜ ⎟ sin ⎜ ⎟
nπ ⎣ ⎝ ⎠⎦ ⎝ nπ ⎠ ⎝ 2 ⎠

50
100
⎛ nπx ⎞ ⎛ n πx ⎞
Similarly, − ∫ x sin ⎜ ⎟ dx = x sin ⎜
⎝ 100 ⎠ 100

⎝ 100 ⎠
⎟ dx
50

50
⎡ ⎛ nπx ⎞ ⎛ nπx ⎞ ⎤
⎢ cos⎜ ⎟ sin ⎜ ⎟⎥
⎝ 100 ⎠ + 1. ⎝ 100 ⎠ ⎥
= ⎢(− x ).
⎢ nπ ⎛ nπ ⎞ ⎥
⎢ ⎜ ⎟
⎣ 100 ⎝ 100 ⎠ ⎥⎦ x =100

2
⎛ nπ ⎞ ⎛ 100 ⎞ ⎛ nπ ⎞ 100
2
100
=− .50 cos⎜ ⎟ + ⎜ ⎟ sin ⎜ ⎟+ cos nπ − 0
nπ ⎝ 2 ⎠ ⎝ nπ ⎠ ⎝ 2 ⎠ nπ

100
⎡ ⎛ nπx ⎞ ⎤
⎢ − cos⎜ ⎟
100 ⎠ ⎥⎥
100
⎛ nπx ⎞ ⎝
Considering, 100 sin ⎜ ∫ ⎟ dx = 100
⎝ 100 ⎠

⎢ nπ ⎥
⎢ ⎥
50
⎣ 100 ⎦ x =50

100 100 ⎛ nπ ⎞
=− .100 cos( nπ) + 100 cos⎜ ⎟
nπ nπ ⎝ 2 ⎠

SPECTRUM ALL-IN-ONE JOURNAL FOR ENGINEERING STUDENTS SIA GROUP


4.32 MATHEMATICS-II [JNTU-ANANTAPUR]
Adding these equations and substituting in equation (3), we get,

⎧ 0 if n is even
1 ⎡ ⎛ 100 ⎞ ⎛ nπ ⎞ ⎤ ⎪
2

bn = ⎢2 .⎜ ⎟ sin ⎜ ⎟⎥ = ⎨ 400 sin ⎛ n π ⎞ if n is odd


50 ⎣⎢ ⎝ nπ ⎠ ⎝ 2 ⎠⎦⎥ ⎪ 2 2 ⎜ ⎟
⎩n π ⎝ 2 ⎠
n = 2m – 1

n 1
=m−
2 2
nπ π
∴ = mπ −
2 2
⎛ nπ ⎞ ⎛ π⎞
sin ⎜ ⎟ = sin ⎜ mπ − ⎟ = –cos mπ
⎝ 2 ⎠ ⎝ 2⎠
= (–1)m–1 0 if n is even

400
= (− 1)m −1 if n = 2m – 1
n π
2 2

400 ∞ (− 1) ⎧ (2 m − 1)πx ⎫ (c (2 m −1)π 100 )2 t


m −1
∴ u(x, t) = 2 ∑
π m =1 (2 m − 1)2
sin ⎨
⎩ 100 ⎭
⎬ e 0 < x < 100 .

Q44. Find the temperature in a laterally insulated rod of length L whose both ends are insulated and the
L L
initial temperature u(x, 0) = x if 0 < x < and L – x if < x < L.
2 2
Ans: Given that,
Length of the insultated rod = L

L
Initial temperature (t = 0) u(x, 0) = x, 0 < x <
2

L
= L − x, <x<L ... (1)
2
The equation for conduction of heat is,

∂u 2 ∂ u
2
= C ... (2)
∂t ∂x 2
The boundary conditions are,
u(0, t)= 0 ∀ t ... (3)
u(L, t) = 0 ∀ t [Q Both ends are insulated ⇒ No heat can flow through end] ... (4)
The solution of equation (2) is,

u(x, t) = (C1cos px + C2 sin px) e −C


2
p 2t
... (5)
Put x = 0 in equation (5), we get,
2
u(0, t) = C1e −C p 2t

2
⇒ 0 = C1e −C p 2t [Q From equation (3)]

⇒ C1 = 0
2
∴ u(x, t)= C 2 sin px.e −C p2t ... (6)

Look for the SIA GROU P LOGO on the TITLE COVER before you buy
UNIT-4 (Partial Differential Equations) 4.33
Put, x = L in equation (6), we get,

2
u(L, t) = C 2 sin pL.e −C p 2t

2
−C p 2t
⇒ 0 = C 2 sin pL.e [Q From equation (4)]

As C2 ≠ 0 ⇒ sin Lp = 0

⇒ Lp = nπ


⇒P=
L

∴ Equation (6) reduces to,

C 2 n 2 π 2t
nπx − L2
u(x, t) = C 2 sin e ... (7)
L

∴ The most general solution of equation (2) is of the form

∞ C 2 x 2 π 2t
nπx −
u(x, t) =
n =1

Cn sin
L
.e L2
... (8)

Put t = 0 in equation (8), we get,


nπx
u(x, 0) = ∑C
n =1
n sin
L
... (9)

In order to satisfy the boundary conditions of equation (1),

∑C
n =1
n sin nx = u(x, 0) = x in 0 ≤ x ≤ L/2

= L – x in L/2 ≤ x ≤ L ... (10)

The expnasion of u(x, 0) in half-range sine series in the interval (0, L) is obtained as,


nπx
Since u(x, 0) = ∑ A sin
n =1
n
L ... (11)

100
2 nπx
Where, An = L ∫ u ( x, 0) sin
0
L
dx

SPECTRUM ALL-IN-ONE JOURNAL FOR ENGINEERING STUDENTS SIA GROUP


4.34 MATHEMATICS-II [JNTU-ANANTAPUR]
On comparing equations (9) and (11), we get,
Cn = An

2⎡ nπx ⎤
L/2 L
nπx


Cn = L ⎢ ∫ x sin
L ∫
dx ( L − x ) sin
L
dx ⎥

⎣ 0 L/2 ⎦

2⎡ nπx ⎞ ⎤
L/2 L/2 L L
nπx ⎛ nπx ⎞ nπx ⎛
= ⎢x
L⎢ ∫ sin
L
dx − ∫ ∫ ⎜1. sin
⎝ L
dx ⎟dx + ( L − x ) sin
⎠ L ∫
dx − ⎜ (−1) sin
⎝ ∫L ∫
dx ⎟dx⎥
⎠ ⎥⎦
⎣ 0 0 L/2 L/2

⎧⎡ ⎛ nπx ⎞ ⎤
L/2
⎛ nπx ⎞ ⎡ ⎛ nπx ⎞ ⎤
L
⎛ nπx ⎞ ⎫
⎪⎢ x⎜ − cos ⎟⎥ L / 2 − cos ⎜ ⎟ ⎜ − cos ⎟ L ⎜ − cos ⎟
2 ⎪⎢ ⎝ L ⎠⎥ ⎝ L ⎠

⎝ L ⎠

⎝ L ⎠ ⎪⎪
= ⎨
L ⎪⎢ nπ ⎥

nπ ∫ ⎢
dx + ( L − x)
⎢ nπ


+
nπ ∫
dx ⎬

⎪⎢⎣ ⎥ ⎢ ⎥
0 L/2
L ⎦ L ⎣ L ⎦ L ⎪⎭
⎩ 0 L / 2

⎧ ⎡ nπx ⎤
L/2
⎡ nπx ⎤ ⎫
L
⎪ nπ ⎞ L ⎢ sin sin ⎪
2⎪ ⎛L L L ⎥ + 0 + L × L cos nπ − L ⎢ L ⎥ ⎪
= ⎨− ⎜ × cos − 0⎟ + ⎢ ⎥ ⎢ ⎥ ⎬
L ⎪ ⎝ 2 nπ 2 ⎠ nπ ⎢ nπ ⎥ 2 nπ 2 nπ ⎢ nπ ⎥ ⎪
⎪⎩ ⎣ L ⎦0 ⎣ L ⎦ L / 2 ⎪⎭

2 ⎧⎪ L2 nπ L L ⎡ ⎡ nπ ⎤ L
2
nπ L L ⎡ nπ ⎤ ⎤ ⎫⎪
= ⎨− cos + × ⎢ ⎢sin − 0⎥ + cos − × ⎢ sin nπ − sin ⎥ ⎥ ⎬
L ⎪⎩ 2nπ 2 nπ nπ ⎣⎢ ⎣ 2 ⎦ 2 nπ 2 nπ nπ ⎣ 2 ⎦ ⎥⎦ ⎪⎭

2 L2 ⎧ 1 nπ 1 nπ 1 nπ 1 1 nπ ⎫
= × ⎨− cos + sin + cos − sin nπ + sin ⎬
L nπ ⎩ 2 2 nπ 2 2 2 nπ nπ 2 ⎭

2L ⎡ 1 nπ ⎤ 4L nπ
= ⎢ 2. sin ⎥ = 2
sin [Q sin nπ = 0]
nπ ⎣ nπ 2 ⎦ ( nπ) 2

4L nπ
∴ Cn = 2 2
sin
n π 2

When n is even, Cn = 0

4L
When n is odd, Cn = 2 2
( −1) n
n π

On substituting in equation (8), we get,

∞ C 2 n 2 π 2t
(−1) n nπx −

4L L2
u(x, t) = sin e
π2 n =1,3,5 n2 L

∞ C 2 ( 2 n −1) 2 π 2t
( −1) n ( 2n − 1) πx −

4L L2
sin e
u(x, t) = π 2 n =1 ( 2 n − 1) 2
L
(odd)

Look for the SIA GROU P LOGO on the TITLE COVER before you buy
UNIT-4 (Partial Differential Equations) 4.35
Q45. A rod of length 100 cm length has its ends A After reaching the steady state condition, the
and B kept at 0° and 100° centigrade until steady temperature at the ends A and B are now changed to 20°C and
state conditions prevail. The temperatures at 60°C respectively. The boundary conditions are,
the ends are changed to 20°C and 60°C u (0, t) = 20°C ... (7)
respectively. Find the temperature in the rod.
u (L, t) = 60°C ... (8)
Ans: Given that,
The temperature in the intermediate period is given by,
Length of rod, L = 100 cm
U(x, t) = Us (x) + Ut (x, t) ... (9)
Initial temperature of the end A, θA1 = 0°C Where, Us (x) represents the steady state solution as
Initial temperature of the end B, θB1 = 100°C given by equation (6) and Ut (x, t) represents the transient solu-
Final temperature of the end A, θA2 = 20°C tion.
Final temperature of the end B, θB2 = 60°C Us (x) is evaluated by solving the equation,
Temperature in the rod, at a later time t, u(x, t) = ? ∂ 2u
=0
The one-dimensional heat conduction equation is given by, ∂x 2
∂u ∂ 2u Thus, its solution is,
= C2 ... (1) Us (x)= ax + b ... (10)
∂t ∂x 2
From equation (7),
Under steady state conditions,
x = 0 , u = 20
∂u Substituting the values in equation (10), we get,
=0
∂t 20 = a. 0 + b
∴ Equation (1) becomes, ⇒ b = 20
∂ 2u And also, from equation (8), we have,
=0 ... (2)
∂x 2 x = L, u = 60
The solution of equation (2) is given by, Substituting the values in equation (10), we get,
u = ax + b ... (3) ⇒ 60 = a (L) + 20 [Q b = 20]
⇒ 60 – 20 = aL
The boundary conditions are,
⇒ 40 = aL
u (0, t) = 0°C ... (4)
⇒ a = 40/L
u (L , t) = 100°C ... (5)
From equation (4), ∴ a = 40 / L
When, x = 0, u = 0 Thus, equation (10) becomes,
On substituting the above values in equation (3), we get,
⎛ 40 ⎞
0 = a (0) + b ∴ Us(x)= ⎜ ⎟ x + 20 ... (11)
⎝ L ⎠
∴b = 0 From equation (9),
Ut (x, t) = u(x, t) – Us (x)
From equation (5),
∴ Ut (0, t) = U (0, t) – Us (0)
When, x = L, u = 100
= 20 – 20
On substituting the values in equation (3), we get,
[From equations (7) and (11)]
∴ 100 = a (L) + 0 =0
Also,
⇒ a = 100 / L
Ut (L, t) = U (L, t) – Us (L)
Hence, equation (3) is given as,
⎡⎛ 40 ⎞ ⎤
x
= 60 – ⎢⎜ ⎟ × L + 20⎥
⎛ 100 ⎞ ⎣⎝ ⎠
L ⎦
∴ u (x, 0) = ⎜ ⎟ ... (6)
⎝ L ⎠ [ From equations (8) and (11)]
Equation (6) represents the initial temperature = 60 – [40 + 20]
distribution in the rod. = 60 – 60 = 0

SPECTRUM ALL-IN-ONE JOURNAL FOR ENGINEERING STUDENTS SIA GROUP


4.36 MATHEMATICS-II [JNTU-ANANTAPUR]
Ut(x, 0) = U(x, 0) – Us (x)

⎡⎛ 100 ⎞ ⎤ ⎡⎛ 40 ⎞ ⎤
= ⎢⎜ ⎟ x ⎥ − ⎢⎜ ⎟ x + 20⎥ [From equations (6), (11)]
⎣⎝ L ⎠ ⎦ ⎣⎝ L ⎠ ⎦

60
= x − 20
L

⎛ 60 ⎞
∴ Ut (x, 0) = ⎜ ⎟ x − 20 ... (12)
⎝ L⎠
Thus,
The boundary conditions of the transient solution Ut (x, t) are,
Ut(0, t) = 0 ... (13)
Ut(L, t) = 0 ... (14)

⎛ 60 ⎞
Ut (x, 0) = ⎜ ⎟ x − 20 ... (15)
⎝ L⎠
The solution of Ut (x, t) is given by,

Ut (x, t) = (C1 cos px + C2 sin px) e −c


2
p 2t
... (16)
Put x = 0 in equation (16), we get,

Ut (0, t) = (C1 cos 0 + C2 sin 0) e −c


2
⇒ p 2t

⇒ 0 = (C1 + 0) – C2p2t [From equation (3) Ut (0, t) = 0]

⇒ C1 = 0

Put x = L in equation (16)


−c 2
p 2t
Ut (L, t) = (C1 cos PL + C2 sin PL) e

0 = (0 + C2 sin PL) e −c
2
p 2t
⇒ [Q C1 = 0 and Ut (L, t) = 0 from equation (14)]

⇒ [C2 sin pL] e −c


2
p 2t =0

⇒ C2 sin pL = 0
⇒ sin pL =0 [Q C2 ≠ 0]
⇒ pL = nπ [Q sin nπ = 0]


p=
L
Thus,
Substituting the values of C1 and p in equation (16), we get,
2
2 ⎡ nπ ⎤
⎡⎛ nπ ⎞ ⎤ −c ⎢ L ⎥ t
⎣ ⎦
Ut (x, t)= C2 sin ⎢⎜ ⎟ x ⎥ e
⎣⎝ L ⎠ ⎦

⎡ c 2n2π 2 ⎤
⎡⎛ nπ ⎞⎤ − ⎢⎣⎢ L2 ⎦⎥
⎥t
⇒ Ut (x, t) = C2 sin ⎢⎜ x ⎟⎥ e ... (17)
⎣⎝ L ⎠⎦

Look for the SIA GROU P LOGO on the TITLE COVER before you buy
UNIT-4 (Partial Differential Equations) 4.37
The most general solution is given by,
⎡ c 2 n 2 π2 ⎤
∞ ⎛ nπx ⎞ ⎢⎣⎢ L2 ⎦⎥
⎥t
Ut(x, t)= ∑ Bn sin⎜ ⎟e ... (18)
n=1 ⎝ L ⎠
Put t = 0 in equation (18), we get,

nπx ⎞
Ut (x, 0) = ∑ Bn sin ⎛⎜ ⎟ ... (19)
n =1 ⎝ L ⎠
In order to satisfy the boundary condition given by equation (15),
We have,
∞ ⎛ nπx ⎞ ⎡ 60 ⎤
∑ Bn SM ⎜ ⎟ = x – 20
n =1 ⎝ L ⎠ ⎢⎣ L ⎥⎦
The value of co-efficient Bn is given as,
L
2 ⎡⎛ 60 ⎞ ⎤ ⎛ nπ ⎞
Bn = ⎢ ∫
⎜ ⎟ x − 20⎥ sin⎜ ⎟ x dx
L ⎣⎝ L ⎠
0 ⎦ ⎝ L⎠

2 ⎡⎛ 60 ⎞ ⎤
L L
⎛ nπ ⎞ ⎛ nπ ⎞
= ⎢
L ⎢⎝ L ⎠ ∫ ⎝ L ⎠ ⎝ L⎠ ∫
⎜ ⎟ x sin⎜ ⎟ x − 20 sin⎜ ⎟ x dx ⎥

⎣ 0 0 ⎦

⎧ ⎡ ⎡ ⎤ ⎡ ⎤ ⎤
L

L⎫
⎛ n π ⎞
⎪ ⎢ ⎢ − cos⎜ ⎟ x ⎥ ⎛ n π ⎞ ⎛ nπ ⎞ ⎤ ⎪
⎢ − cos ⎜ ⎟ x ⎥ ⎥ ⎢ − cos ⎜ ⎟ x ⎥
2 ⎪ 60 ⎝ L ⎠ ⎥ − 1× ⎢ ⎝ L ⎠ ⎥ dx ⎥ − 20⎢ ⎝ L⎠ ⎥ ⎪
= ⎨ ⎢ x⎢
L ⎪ L ⎢ ⎢ nπ / L ⎥ ∫
⎢ nπ / L ⎥ ⎥ ⎢ nπ / L ⎥ ⎪

⎪ ⎢⎣ ⎢⎣ ⎥



⎥ ⎥
⎦ ⎦0


⎥ ⎪
⎦0 ⎭

⎧ ⎡ ⎡ ⎛ nπ ⎞ ⎤ ⎤
L ⎫
⎪ ⎢ sin ⎜ ⎟ x ⎥ ⎪
2 ⎪ 60 ⎢ − xL ⎡ ⎛ nπ ⎞ ⎤ L ⎢⎢ ⎝ L ⎠ ⎥⎥ ⎥
L
⎡ 20 L ⎛ nπ ⎞ ⎤ ⎪
= ⎨ ⎢cos⎜ ⎟ x ⎥ + +⎢ cos⎜ ⎟ x ⎥ ⎬
L ⎪ L ⎢ nπ ⎣ ⎝ L ⎠ ⎦ nπ ⎢ nπ / L ⎥ ⎥ ⎣ nπ ⎝ L ⎠ ⎦0 ⎪
⎪ ⎢⎣ ⎢

⎥⎥
⎦⎦ 0 ⎪
⎩ ⎭

⎧ L L⎫
2 ⎪ 60 ⎡ − xL ⎡ ⎛ nπ ⎞ ⎤ L2 ⎡ ⎛ nπ ⎞ ⎤ ⎤ ⎡ 20 L ⎛ nπ ⎞ ⎤ ⎪
= ⎨ ⎢ ⎢cos ⎜ ⎟ x ⎥ + ⎢sin ⎜ ⎟ x ⎥ ⎥ + ⎢ cos ⎜ ⎟ x ⎥ ⎬
L ⎪ L ⎢⎣ nπ ⎣ ⎝ L ⎠ ⎦ n 2 π 2 ⎣ ⎝ L ⎠ ⎦ ⎥⎦ 0 ⎣ nπ ⎝ L ⎠ ⎦0 ⎪
⎩ ⎭

L
2 ⎡ − 60 x ⎛ nπ ⎞ 60 L ⎛ nπ ⎞ 20 L ⎛ nπ ⎞ ⎤
= ⎢ π cos ⎜ ⎟ x + 2 2 sin⎜ ⎟ x + cos⎜ ⎟ x ⎥
L⎣ n ⎝ L ⎠ n π ⎝ L⎠ nπ ⎝ L ⎠ ⎦0

L
2 ⎡ − 20 x ⎛ nπx ⎞ 60 L ⎛ nπ ⎞ ⎤
= ⎢ cos ⎜ ⎟[3x − L ]+ 2 2 sin⎜ ⎟ x ⎥
L ⎣ nπ ⎝ L ⎠ n π ⎝ L ⎠ ⎦0

2 ⎧⎪⎡ − 20 ⎛ nπ ⎞ 60L ⎡ nπ ⎤ ⎤ ⎡ − 20 60 L ⎤ ⎫⎪
= L ⎨⎢ nπ cos⎜ L L ⎟[(3 × L) − L] + 2 2 sin ⎢ L × L ⎥ ⎥ − ⎢ nπ cos 0[(3 × 0) − L] + 2 2 sin 0⎥ ⎬
⎪⎩⎣ ⎝ ⎠ n π ⎣ ⎦⎦ ⎣ n π ⎦ ⎪⎭

2 ⎧⎡ − 40 L ⎤ ⎡ 20 L ⎤ ⎫
= ⎨⎢ (−1) n + 0⎥ − ⎢ + 0⎥ ⎬ [Q sin nπ = 0, sin (0) = 0, cos nπ = (–1)]
L ⎩ ⎣ nπ ⎦ ⎣ nπ ⎦⎭

SPECTRUM ALL-IN-ONE JOURNAL FOR ENGINEERING STUDENTS SIA GROUP


4.38 MATHEMATICS-II [JNTU-ANANTAPUR]
2 ⎡ − 40 L 20 L ⎤
(−1) n −
L ⎢⎣ nπ nπ ⎥⎦
=

2 ⎡ − 20 L ⎤
= [1 + 2(−1) n ]⎥
L ⎢⎣ nπ ⎦

=
−40

[
1 + 2( −1) n ]

∴ bn =
−40

[
1 + 2( −1) n ]
On substituting the value of bn in equation (18), we get,

⎡ c 2n2π 2 ⎤
∞ − 40 −
⎛ nπx ⎞ ⎢⎢⎣ L ⎥⎦
⎥t
Ut (0, t) = ∑ [1 + 2(−1) n ] sin⎜ ⎟e
n=1 nπ ⎝ L ⎠

⎡ c 2 n2π 2 ⎤
− 40 ∞ [1 + 2(−1) n ] ⎛ nπx ⎞ − ⎢⎣⎢ L ⎦⎥
⎥t
⇒ Ut (0, t) = ∑ sin ⎜ ⎟e ... (20)
π n =1 n ⎝ L ⎠

Hence, substituting equation (11) and equation (17) in equation (9), we get,

⎡ c 2n2π 2 ⎤
⎛ 40 ⎞ 40 ∞ [1 + 2(−1) n ] ⎛ nπx ⎞ − ⎢⎢⎣ L ⎥⎦
⎥t
U(x, t) = ⎜ ⎟ x + 20 − ∑ sin⎜ ⎟e
⎝ L⎠ π n=1 n ⎝ L ⎠

Put L = 100,

⎡ c 2 n 2 π2 ⎤

⎡ 40 ⎤ 40 ∞ [1 + 2(−1) n ] ⎛ nπx ⎞ ⎢⎣⎢ 100 ⎦⎥
⎥t
∴ U(x, t) = ⎢ ⎥ x + 20 − ∑ sin ⎜ ⎟e
⎣100 ⎦ π n=1 n ⎝ 100 ⎠

⎡ c 2n 2π 2 ⎤
40 ∞ ⎡ 1 + 2( −1) n ⎤ ⎛ nπx ⎞ − ⎢⎣⎢ 100 ⎦⎥
⎥t
U ( x, t ) = 0.4 x + 20 − ∑ ⎢ ⎥ sin ⎜ ⎟e
π n =1 ⎢⎣ n ⎥⎦ ⎝ 100 ⎠

Q46. A bar of length L is laterally insulated with its ends A and B kept at 0° and 100° respectively until steady
state condition is reached. The temperature at A is raised to 30° and that at B is reduced to 80° simul-
taneously. Find the temperature in the rod at a later time. Model Paper-III, Q9

Ans: Given that,


Length of the rod = L

Initial temperature at the end A, θ A1 = 0°C

Initial temperature at the end B, θ B1 = 100°C

Final temperature at the end A, θ A2 = 30°C

Final temperature at the end B, θ B2 = 80°C

Temperature in the rod, at a later time t, u(x, t) = ?

Look for the SIA GROU P LOGO on the TITLE COVER before you buy
UNIT-4 (Partial Differential Equations) 4.39
One-dimensional heat conduction equation is given by, On substituting these values in equation (10), we get,
∂u 30 = a(0) + b
2 ∂ u
2
=c ... (1)
∂t ∂x 2
∴ b = 30
Under steady state conditions,
Using equation (8),
∂u
=0 If x = L, u = 80
∂t
∴ Equation (1) becomes, ⇒ 80 = a[L] + 30 [Q b = 30]

∂ 2u ⇒ aL= 50
=0 ... (2)
∂x 2 ∴ a = 50 / L

Solution of equation (2) is given by,
u = ax + b ... (3) ⎡ 50 ⎤
The boundary conditions are, ∴ us(x) = ⎢ ⎥ x + 30 ... (11)
⎣L⎦
u(0, t) = 0 ... (4)
From equation (9),
u(L, t) = 100 ... (5)
ut(x, t) = u(x, t) – us(x)
Using equation (4),
∴ ut(0, t)= u(0, t) – us(0)
i.e., if x = 0, u = 0
= 30 – 30 [Q From equations (7) and (11)]
On substituting these values in equation (3), we get,
=0
0 = a(0) + b
Also,
∴b = 0
ut(L, t) = u(L, t) – us(L)
Using equation (5),
⎡⎛ 50 ⎞ ⎤
i.e., if x = L, u = 100 = 80 – ⎢⎜⎝ ⎟⎠ L + 30⎥
∴ 100 = a[L] + 0 ⎣ L ⎦
[Q From equations (8) and (11)]
⇒ ∴ a = 100 / L = 80 – 80
=0
⎡100 ⎤
∴ u(x, 0) = ⎢ ⎥x ... (6) ut(x, 0) = u(x, 0) – us(x)
⎣ L ⎦
Equation (6) represents the initial temperature ⎡100 ⎤ ⎡⎛ 50 ⎞ ⎤
distribution in the rod. =⎢ x ⎥ − ⎢⎜ ⎟ x + 30⎥
⎣ L ⎦ ⎣ L ⎝ ⎠ ⎦
After reaching steady state condition, the temperature [Q From equations (6) and (11)]
at the ends A and B are now changed to 30 and 80 respectively.
50
The boundary conditions are, = x − 30
L
u(0, t) = 30 ... (7)
⎛ 50 ⎞
u(L, t) = 80 ... (8) ∴ ut(x, 0) = ⎜ ⎟ x − 30 ... (12)
The temperature in the intermediate period is given by, ⎝ L⎠

u(x, t) = us(x) + ut(x, t) ... (9) Thus, the boundary conditions of the transient solution
Where us(x) represents the steady state solution as ut(x, t) are,
given by equation (6) and ut(x, t) represents the transient ut(0, t)= 0 ... (i)
solution.
ut(L, t) = 0 ... (ii)
∂ u
2
us(x) is evaluated by solving the equation = 0. ⎛ 50 ⎞
∂x 2 ut(x, 0)= ⎜ ⎟ x − 30 ... (iii)
⎝ L⎠
Hence its solution is,
us(x) = ax + b ... (10) The solution of ut(x, t) is given by,
Using equation (7),
ut(x, t) = [c1 cos px + c2 sin px] e − c
2
p 2t
... (13)
If x = 0, x = 30

SPECTRUM ALL-IN-ONE JOURNAL FOR ENGINEERING STUDENTS SIA GROUP


4.40 MATHEMATICS-II [JNTU-ANANTAPUR]
Put x = 0 in equation (13), we get,

ut(0, t) = [c1 cos 0 + c2 sin 0] . e − c


2
p 2t

2
. ⇒ 0 = [ c1 + 0]e − c p 2t

∴ c1 = 0
Put x = L in equation (13), we get,
− c 2 p 2t
ut(L, t) = [ c1 cos( pL ) + c 2 sin( pL )]e
2
⇒ 0 = [0 + c2 sin( pL)]e −c p 2t [Q c1 = 0 and ut(L, t) = 0 from equation (ii)]
⇒ c2 sin pL = 0
⇒ sin pL = 0 [Q c2 ≠ 0]
⇒ pL = sin–1 (0)
⇒ pL = nπ


∴p=
L
Thus, substituting the values of c1, c2 and p in equation (13), we get,
2
2 ⎡ nπ ⎤
⎡ nπ ⎤ −c ⎢ L ⎥ .t
⎣ ⎦
ut(x, t) = c2 . sin ⎢ ⎥.x.e
⎣L⎦

⎡ c 2n 2π 2 ⎤
−⎢ ⎥.t
⎡ nπ ⎤ ⎢⎣ L ⎥⎦
⇒ ut(x, t) = c 2 . sin ⎢ ⎥.xe ... (14)
⎣L⎦
The most general solution is given by,

⎡ c 2 n 2 π2 ⎤
∞ −⎢ ⎥t
⎛ nπx ⎞
ut(x, t) = ∑
n =1
Bn sin⎜
⎝ L ⎠
⎟.e ⎣⎢ L ⎦⎥
... (15)

Put t = 0 in equation (15), we get,



⎛ nπx ⎞
⇒ ut(x, 0)= ∑B
n =1
n sin⎜
⎝ L ⎠
⎟ .e–0


⎛ nπx ⎞
⇒ ut(x, 0)= ∑B
n =1
n sin⎜
⎝ L ⎠
⎟ ... (16)

In order to satisfy the boundary condition given by equation (iii), equating equation (16) and equation (iii), we get,

⎛ nπx ⎞ ⎡ 50 ⎤
∑B
n =1
n sin⎜ ⎟ = ⎢ ⎥ x − 30
⎝ L ⎠ ⎣L⎦
The value of Bn can be obtained as,
L
2 ⎡⎛ 50 ⎞ ⎤ ⎛ nπx ⎞
Bn = ⎢ ∫
⎜ ⎟ x − 30⎥. sin⎜
L ⎣⎝ L ⎠
0 ⎦ ⎝
⎟dx
L ⎠

2 ⎡ 50 ⎛ nπx ⎞ ⎤
L L
⎛ nπx ⎞
= ⎢
L⎢ L
x sin ⎜
⎝∫ L ⎠
⎟ dx − 30 sin⎜

⎟dx ⎥
L ⎠ ⎥ ∫
⎣ 0 0 ⎦
Look for the SIA GROU P LOGO on the TITLE COVER before you buy
UNIT-4 (Partial Differential Equations) 4.41

L
⎡ ⎧ ⎡ ⎛ nπx ⎞ ⎤ ⎛ nπx ⎞ ⎫ ⎡ ⎛ nπx ⎞ ⎤ ⎤
⎢ ⎪ ⎢ − cos⎜ ⎟⎥ − cos⎜ ⎟ ⎪ ⎢ − cos ⎜ ⎟ ⎥⎥
2 ⎢ 50 ⎪ ⎢ ⎝ L ⎠⎥ − ⎝ L ⎠ dx ⎪ − 30⎢ ⎝ L ⎠ ⎥⎥
= ⎢ ⎨
L L ⎪ ⎢
x.
nπ ⎥ nπ ⎬
⎪ ⎢ ∫ nπ ⎥⎥
⎢ ⎢ ⎥ ⎢ ⎥⎥

⎢⎣ ⎩ ⎣ L ⎦ L ⎪
⎭ ⎣ L ⎦ ⎥⎦ 0

L
2 ⎡ 50 ⎧ − xL ⎛ nπx ⎞ L ⎛ nπx ⎞ ⎫ 30 L ⎛ nπx ⎞ ⎤
= ⎢ ⎨ . cos⎜ ⎟ + 2 2 sin ⎜ ⎟⎬ + cos⎜ ⎟⎥
L ⎣⎢ L ⎩ nπ ⎝ L ⎠ n π ⎝ L ⎠⎭ n π ⎝ L ⎠ ⎦⎥ 0

L
2 ⎡ 50 x ⎛ nπx ⎞ 50 ⎛ nπx ⎞ 30L ⎛ nπx ⎞⎤
= ⎢− cos⎜ ⎟ + 2 2 sin⎜ ⎟+ cos⎜ ⎟⎥
L ⎣ nπ ⎝ L ⎠ n π ⎝ L ⎠ nπ ⎝ L ⎠⎦ 0

L
2 ⎡ 10 ⎛ nπx ⎞ 50 ⎛ nπx ⎞⎤
= ⎢− cos ⎜ ⎟.[−5 x + 3L ] + 2 2 sin⎜ ⎟⎥
L ⎣ nπ ⎝ L ⎠ n π ⎝ L ⎠⎦ 0

2⎧ − 10 ⎛ nπL ⎞ 50 ⎛ nπL ⎞⎫ ⎧ − 10 50 ⎫
= ⎨(−5L + 3L) × cos ⎜ ⎟. + 2 2 sin⎜ ⎟⎬ − ⎨[(−5 × 0) + 3L ] × cos 0 + 2 2 sin 0⎬
L⎩ nπ ⎝ L ⎠ n π ⎝ L ⎠⎭ ⎩ nπ n π ⎭

2 ⎧ 20 L 30L ⎫
= ⎨ cos nπ + 0 + ⎬
L ⎩ nπ nπ ⎭

2 ⎧ 20L 30L ⎫
= ⎨ (−1) n + ⎬
L ⎩ nπ nπ ⎭

2 ⎧10L ⎫
= ⎨ [2(−1) n + 3]⎬
L ⎩ nπ ⎭

20
= [ 2( −1) n + 3]

20
∴ Bn = [3 + 2.( −1) n ]

On substituting the value of Bn in equation (15), we get,

⎡ c 2n 2π2 ⎤
∞ −⎢ ⎥t
⎡ nπ ⎤

20
[3 + 2(−1) n ]. sin ⎢ ⎥ x.e ⎢⎣
L ⎥⎦
ut(x, t) =
n =1
nπ ⎣L⎦

⎡ c2n 2π 2 ⎤
∞ −⎢ ⎥t
⎡ nπ ⎤

20 ⎣⎢ L ⎦⎥
⇒ [3 + 2(−1) ]. sin ⎢ ⎥ x.e
n
ut(x, t) = ... (17)
nπ n =1 ⎣L⎦

Hence, substituting equations (11) and (17) in equation (9), we get,

⎡ c 2 n2π 2 ⎤
∞ −⎢ ⎥t
⎛ 50 ⎞ ⎡ nπ ⎤

20 ⎣⎢ L ⎦⎥
u ( x, t ) = ⎜ ⎟ x + 30 + [3 + 2( −1) n ]. sin ⎢ ⎥ x.e
⎝ L⎠ nπ n =1 ⎣L⎦

SPECTRUM ALL-IN-ONE JOURNAL FOR ENGINEERING STUDENTS SIA GROUP


4.42 MATHEMATICS-II [JNTU-ANANTAPUR]
Q47. A homogeneous rod of length L with insulated On substituting the values in equation (3), we get,
sides has its ends A and B are maintained at 100 = a(L) + b
temperatures 50°C and 100°C until steady state
conditions prevail. The temperature at end A is ⇒ a(L) + 50 = 100 [Q b = 50]
suddenly raised to 90°C and at the same time ⇒ aL = 100 – 50
the temperature at B is lowered to 60°C. Show ⇒ aL = 50
that the temperature at the midpoint of the rod
remains unaltered for all times regardless of the 50
⇒ a=
material of the rod. L
Ans: Given that, Hence, the initial condition can be expressed as,
Length of the homogeneous rod = L units U(x, 0) = ax + b
Initial temperature at the end A, θ A1 = 50°C ⇒ aL = 100 – b = 100 – 50 = 50

Initial temperature at the end B, θB1 = 100°C 50


∴ a =
L
Final temperature at the end A, θ A2 = 90°C
50
⇒ u(x, 0) = x + 50 ... (4)
Final temperature at the end B, θ B2 = 60°C L
To prove that, When the temperature at the ends A and B are changed
Temperature at the midpoint of the rod remains unaltered suddenly to 90°C and 60°C respectively.
for all times regardless of the material of the rod. Under this state as t approaches ∞, the temperature again
We know that, attains steady state.
One dimensional heat conduction equation is given by, Hence,
u(x, t) ≈ u(x)
∂u ∂ 2u
= α2 2 ... (1) As such that,
∂t ∂x
Consider, ∂ 2u
=0
The temperature at any point P of the rod = u(x, t) i.e., ∂x 2
AP = x
The solution remains same as given by equation (3),
The boundary conditions are,
∴ u(0, t) = u(0) = 90°C
u(0, t) = 50°C
u(L, t) = 100°C and u(L, t) = u(L) = 60°C
Prior to the temperature change at the ends A and B, ∴ ax + b = u
when t = 0, the heat flow was independent of time. ⇒ a(0) + b = 90
Hence,
Under steady state condition,
∴ b = 90°

∂u And,
=0 a(L) + b = 60
∂t
∴ Equation (1) reduces to, ⇒ aL + 90 = 60
⇒ aL = 60 – 90
∂ 2u
=0 ... (2)
∂x 2 −30
∴a =
The solution of equation (2) is given by, L
u = ax + b ... (3) Thus, the solution is,
Using the boundary limits,
If x = 0, u = 50 ⎛ − 30 ⎞
u(x, t) = u(x) = ⎜ ⎟ x + 90° ... (5)
Substituting the values in equation (3), we get, ⎝ L ⎠
50 = a(0) + b Assuming that,
∴ b = 50 ⎡ 30 ⎤
u(x, t) = – ⎢ ⎥ x + 90 + U(x, t) ... (6)
If x = L, u = 100 ⎣L⎦

Look for the SIA GROU P LOGO on the TITLE COVER before you buy
UNIT-4 (Partial Differential Equations) 4.43
Using u(0, t) = 90°, in equation (6), we get, The solution of equation (13) is of the form,
2
p 2t
⎡ 30 ⎤ U(x, t) = (C1 cospx + C2 sinpx)⋅ e − c ... (15)
90 = – ⎢ ⎥ × 0 + 90 + U(0, t)
⎣L⎦ Put x = 0 in equation (15),

U(0, t) = (C1 cos0 + C2 sin0) ⋅ e − c


2
p 2t
⇒ 90 – 90 = U(0, t) ⇒
⇒ U(0, t) = 0 ... (7) ⇒ 0 = C1 e −c
2
p 2t

∴ U(0, t)= 0 for all t [Q U(0, t) = 0 ∀ t from equation (13)]


Using u(L, t) = 60, in equation (6), we get,
⇒ ∴ C1 = 0
⎡ 30 ⎤ Put x = L in equation (15), we get,
60 = – ⎢ ⎥ × L + 90 + U(L, t)
⎣L⎦ 2
p 2t
⇒ U(L, t) = (C1 cos(pL) + C2 sin(pL))⋅ e − c
⇒ 60 = – 30 + 90 + U(L, t) 2
p 2t
⇒ 0 = 0 + C 2 sin( pL ) e −c [Q U(L, t) = 0
⇒ U(L, t) = 0 ... (8) ∀ t and C1 = 0]
∴ U(L, t) = 0 for all t ⇒ c2 sin(pL) = 0
Further at t = 0, i.e., the instant, the temperature is ⇒ C2 = 0 or sin(pL) = 0
changed at both the ends, the initial temperature distribution is If C2 = 0, then the solution of U(x, t) given by equation
that which is given by equation (4). (15) vanishes,
Hence, C2 ≠ 0
Put t = 0 in equation (6),
∴ sin(pL) = 0
⎡ 30 ⎤ pL = sin–1 (0)
u(x, 0) = – ⎢ ⎥ x + 90 + U(x, 0) ... (9) ⇒ pL = nπ
⎣L⎦

Equating equation (4) and (9), we get, ∴p=
L
⎡ 50 ⎤ ⎡ 30 ⎤ Thus, on substituting the values of C1, C2 and p in
⇒ ⎢ L ⎥ x + 50 = – ⎢ L ⎥ x + 90 + U(x, 0)
⎣ ⎦ ⎣ ⎦ equation (15), we get,
2
2 ⎡ nπ ⎤
⎡ 50 ⎤ ⎡ 30 ⎤ ⎡ nπx ⎤ −c ⎢ L⎥ t
⇒ U(x, 0) = ⎢ ⎥ x + ⎢ ⎥ x + 50 − 90 U(x, t) = C 2 sin ⎢ ⎥ ⋅e
⎣ ⎦
⎣ ⎦
L ⎣L⎦ ⎣ L ⎦

⎡ c 2 n 2 π 2t ⎤
⎡ 80 ⎤ −⎢ ⎥
⇒ U(x, 0) = ⎢ ⎥ x − 40 ... (10) ⎡ nπx ⎤ ⎣⎢ L
2
⎦⎥
⎣L⎦ U(x, t) = C 2 sin ⎢ ⎥ ⋅e ... (16)
⎣ L ⎦
Further using equation (1), we get, The most general solution of equation (12) satisfying
the boundary conditions, equation (12) and equation (13) is
2 ∂ U
2
∂U given by,

∂t ∂x 2
⎡ c 2 n 2 π 2t ⎤
∞ −⎢ ⎥
⎛ nπx ⎞

2
Thus, ⎢⎣ L ⎥⎦
⇒ U(x, t) = Bn ⋅ sin ⎜ ⎟e
n =1 ⎝ L ⎠
The conditions can be written as,
... (17)
∂ U
2
1 ∂U Put t = 0 in equation (17),
⇒ = 2 ... (11)
∂x 2 α ∂t ⎡ c 2 n 2 π 2 ×0 ⎤
∞ −⎢ ⎥
⎡ nπx ⎤
U(0, t) = 0 for all t ... (12) ⇒ U(x, 0) = ∑
n =1
Bn ⋅ sin ⎢
⎣ L ⎦
⎥ ⋅e
⎣⎢ L2 ⎦⎥

U(L, t) = 0 for all t ... (13)



⎡ nπx ⎤
⎡ 80 ⎤
U(x, 0) = ⎢ ⎥ x − 40
⎣L⎦
... (14) ⇒ U(x, 0) = ∑B
n =1
n ⋅ sin ⎢
⎣ L ⎦
⎥ ... (18)

SPECTRUM ALL-IN-ONE JOURNAL FOR ENGINEERING STUDENTS SIA GROUP


4.44 MATHEMATICS-II [JNTU-ANANTAPUR]
Equating equations (18) and (14), we get,


⎡ nπx ⎤ ⎡ 80 ⎤ x − 40
∑B
n =1
n sin ⎢ ⎥ =⎢ ⎥
⎣ L ⎦ ⎣L⎦

The value of Bn can be found as,

L
2 ⎡ 80 ⎤ ⎡ nπx ⎤
Bn =
L ⎢⎣ L
0
∫x − 40⎥ sin ⎢
⎦ ⎣ L ⎦
⎥ dx

2 ⎡ ⎡ 80 ⎤ ⎡ nπx ⎤ ⎤⎥
L L
⎡ nπx ⎤
= ⎢

L ⎢ ⎢⎣ L ⎥⎦
x ⋅ sin ⎢ L ⎥
⎣ ⎦
dx − 40 ∫ sin ⎢ L ⎥ dx
⎣ ⎦ ⎥⎦
⎣0 0

2 ⎡ 80 ⎡ nπx ⎤ ⎤⎥
L L
⎡ nπx ⎤
= ⎢
L⎢ L ∫ x sin ⎢
⎣ L ⎦⎥ dx − 40∫ sin ⎢

dx
L ⎥⎦ ⎥
⎣ 0 0 ⎦

L
⎡ ⎧ ⎡ ⎡ nπx ⎤ ⎤ ⎡ nπx ⎤ ⎫ ⎧ ⎡ nπx ⎤ ⎫⎤
⎢ ⎪ ⎢ − cos ⎢ ⎥ ⎥ − cos ⎢ ⎥ ⎪ ⎪ − cos ⎢ ⎥ ⎪⎥
2 80 ⎪ ⎣ L ⎦⎥ − ⎣ L ⎦ ⎪ − 40⎪ ⎣ L ⎦ ⎪⎥
= ⎢ ⎨x⎢ ∫ ⎬ ⎨ ⎬⎥
L⎢ L ⎪ ⎢ nπ ⎥ nπ ⎪ ⎪ nπ ⎪⎥
⎢ ⎪ ⎢ L ⎥ L ⎪⎭ ⎪⎩ L ⎪⎭⎥
⎢⎣ ⎩ ⎣ ⎦ ⎦0

L
⎡ ⎧ ⎡ ⎡ nπx ⎤ ⎤ ⎫ ⎤
⎢ ⎪ ⎢ sin ⎢ ⎥ ⎪ ⎥
2 80 ⎪ − L ⎡ nπx ⎤ L ⎢ ⎣ L ⎥⎦ ⎥ ⎪ ⎡L ⎡ nπx ⎤ ⎤ ⎥
= ⎢⎢ ⎨ + +
nπ ⎥ ⎬⎪
x cos ⎢ ⎥ 40⎢ cos ⎢ L ⎥⎥ ⎥
L L ⎪ nπ ⎣ L ⎦ nπ ⎢ ⎣ nπ ⎣ ⎦⎦
⎢ ⎢ ⎥ ⎥

⎢⎣ ⎩ ⎣ L ⎦⎭⎪ ⎥⎦ 0

L
2 ⎡ − 80 ⎛ nπx ⎞ 80 L ⎡ nπx ⎤ 40 L ⎡ nπx ⎤ ⎤
= ⎢ x cos⎜ ⎟ + 2 2 sin ⎢ ⎥ + nπ cos ⎢ L ⎥ ⎥
L ⎣ nπ ⎝ L ⎠ n π ⎣ L ⎦ ⎣ ⎦⎦ 0

L
2 ⎡ 40 ⎡ nπx ⎤ 80 L ⎡ nπx ⎤ ⎤
= ⎢ cos ⎢ ⎥ (−2 x + L) + 2 2 sin ⎢ ⎥⎥
L ⎣ nπ ⎣ L ⎦ n π ⎣ L ⎦⎦ 0

⎡⎧ 40 ⎡ nπL ⎤ 80 ⎡ nπL ⎤ ⎫ ⎡⎧ 40 80 ⎫⎤
= ⎢⎨ cos ⎢ ⎥ ⋅ (−2 L + L) + 2 2 L sin ⎢ ⎥ ⎬ – ⎢⎨ cos 0 ⋅ (−2(0) + L) + 2 2 L sin 0⎬⎥
⎢⎣⎩ nπ ⎣ L ⎦ n π ⎣ L ⎦ ⎭ ⎣ ⎩ nπ n π ⎭⎦

2 ⎡⎧ − 40 L ⎫ ⎧ 40L ⎫⎤
⎨ cos nπ + 0⎬ − ⎨ + 0⎬ ⎥
L ⎢⎣⎩ nπ
=
⎭ ⎩ nπ ⎭⎦

2 ⎡ − 40 L ⎤
= ⎢ (cos nπ + 1) ⎥
L ⎣ nπ ⎦

Look for the SIA GROU P LOGO on the TITLE COVER before you buy
UNIT-4 (Partial Differential Equations) 4.45

−80
= [cos nπ + 1]

−80
= [(−1) n + 1]

If n is odd, Bn = 0

−80
If n is even, let Bn = B2m = [( −1) 2 m + 1]
2m × π

−160
⇒ Bn = B2m =
2 mπ

−80
⇒ Bn = B2m =

⎡ ( 2 m ) 2 ⋅ π 2 c 2t ⎤
∞ −⎢ ⎥
− 80 ⎡ 2mπx ⎤
∴ U(x, t) = ∑ mπ sin ⎢⎣
m =1
L ⎥⎦
⋅e ⎣⎢ L2 ⎦⎥
[ From equation (17) ]

∞ −4 m 2 π 2 c 2 t
− 80 ⎡ 2mπx ⎤

1 L2
⇒ sin ⋅e
π m=1 m ⎢⎣ L ⎥⎦
U(x, t) = ... (19)

Substituting equation (19) in equation (6), we get,

∞ −4 m 2 π 2 c 2t
⎡ − 30 ⎤ ⎡ 2mπx ⎤

80 1 L2
u(x, t) = ⎢ ⎥ x + 90 − π sin ⎢ ⎥ ⋅e ... (20)
⎣ L ⎦ m =1
m ⎣ L ⎦

Hence the required temperature distribution is,

∞ − 4 m 2 π 2 c 2t
⎡ − 30 ⎤ ⎡ 2mπx ⎤

80 1 L2
u ( x, t ) = ⎢ ⎥ x + 90 − π sin ⎢ ⎥e
⎣ L ⎦ m =1
m ⎣ L ⎦

Temperature at Midpoint

L
Put x = in equation (20), we get,
2

⎛ L ⎞ ⎡ − 30 ⎤ ⋅ ⎡ L ⎤ + 90 − 80 [ 0]
u⎜ , t⎟ = ⎢ ⎥ ⎢ ⎥ [Q sin mπ = 0]
⎝ 2 ⎠ ⎣ L ⎦ ⎣2⎦ π

⎛L ⎞
⇒ u ⎜ , t ⎟ = – 15 + 90
⎝2 ⎠

⎛L ⎞
∴u ⎜ , t⎟ = 75 ... (21)
⎝2 ⎠

⎛L ⎞
From equation (21), it can be observed that, the temperature distribution at the midpoint of the rod i.e., U ⎜ , t ⎟ is
⎝2 ⎠
independent of t and also independent of the material of the rod as it does not depend on C.
SPECTRUM ALL-IN-ONE JOURNAL FOR ENGINEERING STUDENTS SIA GROUP
4.46 MATHEMATICS-II [JNTU-ANANTAPUR]
4.4 TWO DIMENSIONAL LAPLACE EQUATION ∴ Equation (6) becomes,
UNDER INITIAL AND BOUNDARY CONDITIONS
nπx
u(x, y) = C2sin (C3 e nπy/l – C3e –nπy/l)
2 2
∂ u ∂ u l
Q48. Solve + = 0 which satisfies the
∂x 2 ∂y 2
nπx nπy / l
conditions u(0, y) = u(l, y) = u(x, 0) = 0 and u(x, y) = C 2C 3 sin (e − e − nπy / l )
l
πx

u(x, a) = sin . Replacing C2, C3 by bn, we have,
l
nπx nπy/l
∂ 2u ∂ 2u u(x, y) = bn sin (e – e –nπy/l)
Ans: The given equation is + =0 ... (1) l
∂x 2 ∂y 2
nπx nπy
The three possible solutions of equation (1) are, = 2bn sin . sinh ... (7)
l l
u = (C1epx + C2e–px)(C3 cos py + C4 sin py) ... (2)
⇒ u = (C1 cos px + C2 sin px)(C3epy + C4e–py) ... (3) Putting, y = a, we get,
u = (C1x + C2)(C3 y + C4) ... (4) nπx nπx nπa
u(x, a) = sin =2bn sin . sinh
Keeping in view the boundary conditions, l l l
The only possible solution is (3),
nπa 1
u(x, y) = (C1 cos px + C2 sin px)(C3epy + C4e–py) ⇒ 2bn sinh = 1 ⇒ bn =
l nπa
... (5) 2 sinh
l
Since u(0, y) = 0
Hence, equation (7) reduces to,
0 = C1 (C3epy + C4e–py)
C1 = 0 nπy ⎤

On substituting C1 = 0 in equation (5), we get, nπx ⎢ sinh l ⎥
u(x, y) = sin ⎢ ⎥
u(x, y)= C2 sin pl (C3epy + C4e–py) ... (6) l ⎢ sinh nπa ⎥
⎢⎣ l ⎥⎦
u(l, y) = 0
∴ 0 = C2 sin pl (C3epy + C4e–py) Which is the required solution.
⇒ sin pl = 0 Q49. A rectangular plate with insulated surface is 10
⇒ pl = nπ cm wide and so long compared to its width that
it may be considered infinite in length without
nπ introducing an appreciable error. If the tempera-
∴ p= , n being integer
l ture of the short edge y = 0 is given by,
y u = 20x for 0 ≤ x ≤ 5 and u = 20 (10 – x) for 5 ≤ x
≤ 10 and the two long edges x = 0, x = 10 as
nπ x
u = sin
l
well as the other short edge are kept at 0°C.
Find the temperature u at any point.

P(Q, Y) Ans: The temperature u(x, y) at any point p(x, y) satisfies the
equation.

∂ 2u ∂ 2u
0 u=0 x + =0 ... (1)
∂x 2 ∂y 2
Figure
The boundary conditions are,
Also u(x, 0) = 0
u(0, y) = 0 for all values of y ≥ 0 ... (2)
nπx u(10, y) = 0 ∀ y ≥ 0 ... (3)
0 = C2 sin (C3 + C4)
l
u(x, ∞) = 0 in 0 ≤ x ≤ 10 ... (4)
C3 + C4 = 0
∴ C3 = – C 4 ⎧20 x 0≤ x ≤5
u(x, 0) = ⎨ ... (5)
⎩20(10 − x ), 5 ≤ x ≤ 10

Look for the SIA GROU P LOGO on the TITLE COVER before you buy
UNIT-4 (Partial Differential Equations) 4.47
Thus the three possible solutions of equation (1) are,
u = (C1epx + C2e–px)(C3 cos py + C4 sin py) ... (6)
= (C1 cos px + C2 sin px)(C3epy + C4e–py) ... (7)
= (C1 x + C2)(C3y + C4) ... (8)
The solution (6) cannot satisfies the condition (2), we get u ≠ 0 for x = 0 ∀ y
The solution (8) cannot satisfies the condition (4). Thus the only possible solution is (7),
∴ u(x, y) = (C1 cos px + C2 sin px)(C3epy + C4e–py) ... (9)
Since u(0, y) = 0
∴ 0 = C1 (C3epy + C4e–py)
C1 = 0
∴ Equation (9) reduces to,
u(x, y)= C2 sin px (C3epy + C4e–py) ... (10)
∴ u(10, y) = 0
∴ 0 = C2 sin 10p (C3epy + C4e–py)
⇒ sin 10p = 0
i.e., 10p = nπ

p= , Where n being an integer.
10
Also, u(x, ∞) = 0
∴ C3 = 0
Hence from equation (10) a solution satisfying equations (2), (3) and (4) is,
nπx –nπy/10
u(x, y) = C2C4 sin e
10
Replacing C2, C4 by bn. The most general solution is,
∞ − nπy
nπx
u(x, y) = ∑
n =1
bn sin
10
⋅e 10
... (11)
Putting y = 0, we get,

nπx
u(x, 0) = ∑b
n =1
n sin
10

1⎡ nπx ⎤
10 5 10
2 nπx nπx
bn =
10 0 ∫
u ( x, 0) sin
10
dx = ⎢
5 ⎢0

20 x sin
10∫ dx +
5

20(10 − x) sin
10
dx ⎥
⎥⎦

5 10
⎡ ⎛ ⎞⎤ ⎡ ⎛ ⎞⎤
⎢ ⎛⎜ − cos nπx ⎞⎟ ⎜ − sin nπx ⎟ ⎥ ⎢ ⎛

nπx ⎞

⎜ − sin nπx ⎟ ⎥
⎢ ⎜ 10 ⎜ 10 ⎟⎥ ⎢ 10 ⎟ − (−1)⎜ 10 ⎟ ⎥
= 4 ⎢x ⎟ − (1)⎜ ⎟ ⎥ + 4⎢(10 − x )⎜ − cos ⎜ 2 ⎟⎥
nπ ⎟ 2 nπ ⎟
⎢ ⎜⎝ ⎠ ⎜⎜ ⎛⎜ nπ ⎞⎟ ⎟⎟ ⎥ ⎢ ⎜
⎝ ⎜ ⎛ nπ ⎞ ⎟ ⎥

⎜ ⎝ ⎠ ⎟ ⎟
⎢⎣ 10 ⎝ ⎝ 10 ⎠ ⎠ ⎥⎦ 0 ⎢⎣ 10 ⎠ ⎝ 10 ⎠ ⎥⎦ 5

⎡ − 50 nπ ⎛ 10 ⎞
2
nπ ⎤ ⎡ 50 nπ ⎛ 10 ⎞
2
nπ ⎤ 800 nπ
=4 ⎢ cos + ⎜ ⎟ sin ⎥ + 4 ⎢ cos + ⎜ ⎟ sin ⎥ = 2 2 sin
⎢⎣ n π 2 nπ
⎝ ⎠ 2 ⎥⎦ ⎢⎣ nπ 2 ⎝ nπ ⎠ 2 ⎥⎦ n π 2

⎧ 800
⎪ ( −1) n −1/2 , when n is odd
= ⎨ n 2π 2
⎪ 0, when n is even

Hence from equation (11), the required solution is,

( −1) n −1 / 2 nπx −nπy /10
800
∴ u(x, y)= 2
π

n =1,3,5 n 2
sin
10
e

SPECTRUM ALL-IN-ONE JOURNAL FOR ENGINEERING STUDENTS SIA GROUP


4.48 MATHEMATICS-II [JNTU-ANANTAPUR]
Q50. Infinitely long plane uniform plate is bounded From condition (iii), u = 0 when y → ∞
by two parallel edges and an end at right angles
to them. The breadth is π , this end is maintained On substituting C1,p value and condition (iii) in equation
at a temperature µ 0 at all points and other edges (1), we get,
are at zero temperature. Determine the 0 = C2 sin nx C3
temperature at any point of the plate in the
steady state. C3 = 0
Ans: Steady state temperature u(x, y) at a point p(x, y) is, ∴ Equation (3) becomes,

∂ 2u ∂ 2u u(x, y) = C2 sin nx (C4e–ny) ... (4)


+ = 0.
∂x 2 ∂y 2 ∴ u(x, y) = Cn sin nxe–ny [Put C2C4 = Cn]

y General solution is,

0o p(x, y) 0o
u(x, y) = ∑C
n =1
n sin nx e − ny ... (5)

∑C
µo
u(x, 0) = n sin nx [Put y = 0] ... (6)
n =1
The boundary conditions are,
From condition (iv), we get,
(i) u(0, y) = 0 (ii) u(π, y) = 0
u(x, 0) = µo Where 0 < x < π
(iii) u(x, ∞) = 0 (iv) u(x, 0) = µo By half-range sine series, we get,

u = 0 when y → ∞, we have, ∞

u(x, y) = (C1 cos px + C2 sin px) (C3 epy + C4e–py)


u(x, 0) = ∑b
n =1
n sin nx ... (7)
... (1)
Equating equations (6) and (7), we get,
From above condition (i), u = 0 when x = 0.
Cn = bn
On substituting in equation (1), we get,
π π
2 2µ o ⎡ − cos nx ⎤
0 = (C1 (1) + 0) (C3 epy + C4e–py) bn =
π0 ∫
µ o sin nx dx = π ⎢ n ⎥
⎣ ⎦0
0 = C1 (C3 epy + C4 e–py) ⇒
−2µ o
∴ C1 = 0 = [(–1)n –1]

∴ Equation (1), becomes, Cn = 0 When n is even

4µ o
u(x, y) = C2 sin px(C3epy + C4e–py) ... (2) Cn = When n is odd

From condition (ii), u = 0 when x = π.
4µ o
∴ C2n – 1 =
On substituting C1 value and condition (ii) in equation ( 2n − 1) π
(1), we get,
On substituting the above value in equation (5), we get,
0 = C2 sin pπ (C3 epy + C4 e–py)

sin pπ = 0 u(x, y) = ∑C
n =1
2 n −1 sin( 2 n − 1) xe
− ( 2 n −1) y

pπ = nπ ⇒ p=n ∞
4µ o
= ∑ (2n − 1)π sin( 2n − 1) xe
n =1
− ( 2 n −1) y

∴ u(x, y) = C2 sin nx (C3eny + C4e–ny) ... (3)

Look for the SIA GROU P LOGO on the TITLE COVER before you buy
UNIT-4 (Partial Differential Equations) 4.49
Q51. Consider a semi-infinite insulated plate in the first quadrant of the xy plane bounded by y = 0, x = 0,
x = l. Determine the temperature u(x, y) at any point of the plate in the steady state given that, u(x, 0)
= f(x), 0 ≤ x ≤ l and u(0, y) = u(l, y) = 0 for y ≥ 0 and u(x, y) = as y → ∞
Ans: The temperature u(x, y) at any point (x, y) satisfies the equation,

∂ 2u ∂ 2u
+ =0 ... (1)
∂x 2 ∂y 2

The boundary conditions are,


u(0, y) = 0 for all values of y ... (2)
u(l, y) = 0 for all values of y ... (3)
u(x, ∞) = 0 for all 0 ≤ x ≤ l ... (4)
u(x, 0) = f (x) for all 0 ≤ x ≤ l ... (5)
The three possible solutions of equation (1) are,
u(x, y) = (A1epx + B1e–px) (C1 cos py + D1 sin py) ... (6)
u(x, y) = (A2x + B2) (C2y + D2) ... (7)
u(x, y) = (A1 cos px + B1 sin px) (C1 epy + D1 e–py) ... (8)
The solution of equations (6) and (7) doesn’t satisfies condition (4),
Thus, the only possible solution is equation (8).
For the condition (4) assume C1 = 0, otherwise u will not be infinite as y → ∞.
∴ u(x, y) = (A1 cos px + B1 sin px) D1 e–py
Let A1 D1= A and B1 D1 = B
⇒ u(x, y) = (A cos px + B sin px)e–py ... (9)
Using condition (2), i.e.,
For x = 0, i.e.,
u(x, y) = 0
(A cos 0 + B sin 0) e–py = 0
⇒ Ae–py = 0 (Q From condition (2))
⇒ A=0 (Q e–py ≠ 0)
∴ Equation (5) becomes,
u(x, y) = B sin px e–py ... (10)
Using condition (3) in equation (10),
For x = l, we get,
B sin pl e–py = 0
⇒ sin pl = 0 (Q B ≠ 0 and e–py ≠ 0)
⇒ pl = nπ for n = 1, 2, 3,......


⇒ p= for n = 1, 2, 3......
l
⇒ Equation (10) becomes,

nπx − nπy / l
u(x, y) = Bn sin e , for n = 1, 2, 3.....
l

SPECTRUM ALL-IN-ONE JOURNAL FOR ENGINEERING STUDENTS SIA GROUP


4.50 MATHEMATICS-II [JNTU-ANANTAPUR]
The most general solution of homogeneous equation (1) satisfying conditions (2) (3) and (4) is,


nπx − nπy / l
u(x, y) = ∑B
n =1
n sin
l
e ... (11)

To find Bn, put y = 0 in above equation,


nπx
u(x, 0) = ∑B
n =1
n sin
l
= f (x), 0 ≤ x ≤ l (given)


nπx
= ∑B
n =1
n sin
l
= f (x)

By half range sine series of f (x) in [0, l].

l
2 nπx
Bn =
l ∫ f ( x) sin
0
l
dx , for n = 1, 2, 3..... ... (12)

∴ Equations (11) and (12) are the required solution.

∂ 2u ∂ 2 y
Q52. Derive the solution of the Laplace equation + = 0 . Given that, u(0, y) = u(a, y) = u(x, b) = 0 and
∂x 2 ∂y 2
u(x, 0) = u(x, 0).
Ans: Assuming,
Given that,

∂ 2u ∂ 2u
+ =0 ... (1)
∂x 2 ∂y 2

u(x, 0) = x(a – x) = x(a – x)


The boundary conditions are,
u(0, y) = 0
u(a, y)= 0
u(x, b)= 0
and u(x, 0) = x(a – x)
The given equation i.e., equation (1) is of the form,
u(x, y) = (c1 cos px + c2 sin px)(c3 epy + c4 e–py ) ... (2)
Now, using the boundary condition u(0, y) = 0 in equation (2), we get,
u(0, y) = [c1 cos p(0) + c2 sin p(0)] (c3 epy + c4 e–py) = 0
⇒ [c1 cos 0 + c2 sin 0] (c3 epy + c4 e–py) = 0
⇒ (c1 + 0) (c3 epy + c4 e–py ) = 0
⇒ c1(c3 epy + c4e–py ) = 0
∴ c1 = 0 ... (3)
On substituting value of c1 in equation (2), we get,
u(x, y) = (c2 sin px) (c3 epy + c4 e–py) ... (4)

Look for the SIA GROU P LOGO on the TITLE COVER before you buy
UNIT-4 (Partial Differential Equations) 4.51
Using the boundary condition u(a, y) = 0 in equation (4), we get,

u(a, y) = (c2 sin pa) (c3 epy + c4 e–py) = 0

⇒ c2 sin pa = 0

⇒ sin pa = 0

⇒ pa = nπ


⇒ p=
a

On substituting the value of p in equation (4), we get,

nπy nπ
⎡ ⎛ nπ ⎞⎤ − y
u(x, y) = ⎢c2 sin⎜ x ⎟⎥(c3 e a + c4e a )
⎣ ⎝ a ⎠⎦

nπy nπy
⎛ nπx ⎞ −
= sin ⎜ ⎟( c2 c3 e a + c2 c4 e a )
⎝ a ⎠

Let, A = c2 c3 and B = c2 c4

nπy nπy
⎛ nπx ⎞ −
∴ u(x, y) = sin ⎜ ⎟( Ae a + Be a ) ... (5)
⎝ a ⎠

Using the boundary condition u(x, b) = 0 in equation (5).

⎛ nπx ⎞ ⎛⎜
nπb nπb ⎞

∴ u(x, b) = sin⎜ ⎟ A ⋅ e a + Be a ⎟ = 0
⎝ a ⎠ ⎜⎝ ⎟

nπb nπb

∴ Ae a + Be a =0 ... (6)

Let bn be a constant whose value is given by,

nπb nπb

bn = − 2( Ae a − Be a )

nπb nπb
− 1
⇒ Ae a − Be a =– bn ... (7)
2
Using equations (6) and (7), equation (5) can be written as,

⎛ nπx ⎞ 1
u(x, y) = sin ⎜ ⎟ ⋅ bn e
⎝ a ⎠ 2
[
n π (b − y ) / a
− e − nπ( b − y ) / a ]
nπx nπ(b − y )
u(x, y) = bn sin sin h ... (8)
a a

But, the most general solution of equation (1) is,


⎛ nπx ⎞ ⎛ nπ(b − y ) ⎞
u(x, y) = ∑b
n =1
n sin⎜
⎝ a
⎟ sinh⎜
⎠ ⎝ a


... (9)

SPECTRUM ALL-IN-ONE JOURNAL FOR ENGINEERING STUDENTS SIA GROUP


4.52 MATHEMATICS-II [JNTU-ANANTAPUR]
Using the boundary condition,
u(x, 0) = x(a – x) in equation (9).

nπx nπb
∴ u(x, 0) = ∑b
n =1
n sin
a
. sinh
a
= x( a − x)


nπb nπx
⇒ ∑b
n =1
n sin h
a
sin
a
= x ( a − x)

a
nπb 2 nπx
Where, bn sin h
a
=
a ∫
x( a − x ) sin
0
a
.dx

a
2⎡ 2 ⎛ − cos nπx / a ⎞ ⎛ − sin nπx / a ⎞ ⎛ π ⎞⎤
⎟ + (−2)⎜ cos n x / a ⎟⎥
= ⎢(ax − x )⎜ ⎟ − (a − 2 x)⎜⎜ ⎟ ⎜ ⎟
a ⎣⎢ ⎝ nπ / a ⎠ ⎝ (nπ / a) ⎠
2
⎝ (nπ / a) ⎠⎦⎥
3
0

⎡ ⎛ ⎞ ⎛ ⎞⎤
⎢ ⎛ nπa ⎞ ⎜ − sin nπa ⎟ ⎜ cos n πa ⎟⎥
2⎢ ⎜ − cos ⎟ ⎜ ⎟ ⎜ ⎟⎥
a a a
= ⎢( a.a − a )⎜ ⎟ − ( a − 2.a )⎜ ⎟ + ( −2)⎜
2
π ⎟⎥
a ⎜ ⎟ ⎜ ⎛n ⎞ π
2
⎟ ⎜ ⎛n ⎞π
3
⎟⎥
⎢ ⎜ n ⎟ ⎜ ⎟ ⎜ ⎟
⎢⎣ ⎝ a ⎠ ⎜ ⎟ ⎜ ⎟⎥
⎝ ⎝ a⎠ ⎠ ⎝ ⎝ a⎠ ⎠⎦

2⎡ 2 ⎛ − cos 0 ⎞ ⎛ − sin 0 ⎞ ⎛
⎟ + ( −2)⎜ cos 0 ⎟ ⎥
⎞⎤
− ⎢ ( a.0 − ( 0) )⎜⎜ ⎟⎟ − ( a − 2.0)⎜
a ⎢⎣ n π / a ⎜ ( nπ / a ) 2 ⎟ ⎜ (n π / a) 3 ⎟⎥
⎝ ⎠ ⎝ ⎠ ⎝ ⎠⎦

2⎡ cos nπ ⎤ 2 ⎡ 1 ⎤
= ⎢0 − 0 − 2 3⎥
− ⎢0 − 0 − 2. 3⎥
a ⎢⎣ (n π / a ) ⎥⎦ a ⎢⎣ ( n π / a ) ⎥⎦

4a 2 4a 2
=− cos nπ +
n 3 π3 n 3 π3

4a 2
= [1 − cos nπ]
n 3 π3

nπb 8a 2
∴ bn sin h = 3 3 when n is odd.
a n π
(or)

nπb
bn sin h = 0 when n is even.
a
Therefore, when ‘n’ is odd, equation (9) can be written as,

8a 2 sin h nπ(b − y ) / a nπx
u(x, y) =
π3

n =1,3,5... n sin h nπb / a
3
. sin
a


8a 2 sin h(2 n + 1)π(b − y ) / a (2n + 1)πx
⇒ u(x, y) =
π 3 ∑ (2n + 1)
n =0
3
sin h (2n + 1) πb/a
. sin
a

Which is the required solution.

Look for the SIA GROU P LOGO on the TITLE COVER before you buy
UNIT-5 (Z-Transform) 5.1

UNIT Z-TRANSFORM

5
PART-A
SHORT QUESTIONS WITH SOLUTIONS

Q1. Find the Z-transform of the following sequences,


1
(i)
n!

1
(ii)
(n + 1)!

1
(iii)
(n + 2)!
(iv) cos h nθθ
n
(v) a + b n


(vi) πn – 4 sin – 7a.
4
Ans:
1
(i)
n!
Given that,

1
un =
n!

Z(un) = Σ un . z–n
n =0

⎛1⎞ ∞ 1
Z⎜ ⎟= Σ . z–n
⎝ n! ⎠ n =0 n!

1 –1 1 –2 1 –3
=1+ z + z + z + ..
1! 2! 3!
1 1 1
=1+ + 2 + + ...
z 2! z 3! z 3

⎛ 1 ⎞ 1/z
∴ Z⎜ ⎟=e
⎝ n! ⎠
SPECTRUM ALL-IN-ONE JOURNAL FOR ENGINEERING STUDENTS SIA GROUP
5.2 MATHEMATICS-II [JNTU-ANANTAPUR]

−2
(ii)
1 2⎛ z z −3 ⎞
z
= ⎜ ⎜ + + ...⎟⎟
(n + 1)!
⎝ 2! 3! ⎠
Given that,
1 ⎛ ⎛ 1 ⎞ ⎛ 1 ⎞ 2 ⎛ 1 ⎞3 ⎞
un = ⎜ ⎜ ⎟ ⎜ ⎟ ⎜ ⎟ ⎟
( n + 1)!
2⎜ ⎝ z⎠ ⎝z⎠ ⎝ z⎠ 1⎟

= z ⎜1 + + + + ... − 1 − ⎟
1! 2! 3! z
Z(un) = Σ un . z–n ⎜ ⎟
n =0 ⎜ ⎟
⎝ ⎠
⎡ 1 ⎤ ∞ 1
Z⎢ Σ
⎥ n =0 ( n + 1)! z
= –n
+ 2 ⎛ 1/ z 1⎞
⎣ ( n 1)! ⎦ = z ⎜e −1− ⎟
⎝ z⎠
∞ 1 1
= Σ z–n × × z
n =0 ( n + 1)! z = z 2 (e1 / z − 1 − z −1 )

∞ (iv) θ
cosh nθ
1
= Σ z–n × z–1 × z Given that,
n =0 ( n + 1)!
un = cosh nθ
∞z − ( n +1)
=z Σ ∞
n = 0 ( n + 1)! Z(un) = Σ un . z–n
n =0

⎛1 1 1 1 1 1 ⎞
=z⎜ . + 2
. + 3 . + ...⎟ ⎛ e nθ + e − nθ ⎞
⎝ z 1! z 2! z 3! ⎠ Z(cosh nθ) = Z ⎜⎜ ⎟

⎝ 2 ⎠
⎡ 1 ⎛ 1 ⎞ 2 ⎛ 1 ⎞3 ⎤
⎢ ⎜ ⎟ ⎜ ⎟ ⎥ 1 nθ –nθ
= z ⎢1 + z +
⎝ z ⎠ + ⎝ z ⎠ + ..... − 1⎥ = Z(e + e )
⎢ 1! 2! 3! ⎥ 2
⎢ ⎥
⎣⎢ ⎦⎥ 1 nθ 1
= Z(e ) + Z(e–nθ)
2 2
= z[e1/ z − 1]
1 z 1 z
⎛ 1 ⎞ = × θ + ×
2 z − e 2 z − e −θ
∴ Z ⎜⎜ ⎟⎟ = z(e1/z –1)
⎝ (n + 1)! ⎠
⎡ z − an z ⎤
⎢Q Z (e ) = z − e a and Z (e ) = z − e − a ⎥
an
1 ⎣ ⎦
(iii) (n+ 2)!
1 ⎡ z z ⎤
Given that,
⎢ +
z − e −θ ⎥⎦
= θ
2 ⎣z −e
1
un =
(n + 2)!
z ⎡ z − e −θ + z − e θ ⎤
= ⎢ θ −θ ⎥
∞ 2 ⎣ ( z − e )( z − e ) ⎦
Z(un) = Σ un . z–n
n =0

z ⎡ 2 z − e θ − e −θ ⎤
⎡ 1 ⎤ ∞ 1 1 = ⎢ 2 ⎥
Z⎢ ⎥ = Σ z −n × 2 × z 2 2 θ −θ
⎣ z − z (e + e ) + 1 ⎦
⎣ (n + 2)!⎦ n = 0 (n + 2)! z

z − ( n+ 2)
∞ z ⎡ 2 z − (e θ + e − θ ) ⎤
= z Σ
2
= ⎢ 2 θ −θ ⎥
n = 0 ( n + 2) 2 ⎣ z − z (e + e ) + 1 ⎦

Look for the SIA GROU P LOGO on the TITLE COVER before you buy
UNIT-5 (Z-Transform) 5.3
Where,
z ⎡ 2 z − 2 cosh θ ⎤ Z(n) = Z(n2n)
= ⎢ ⎥
2 ⎣ z 2 − 2 z cosh θ + 1⎦
∞ ∞ n
⎛2⎞
(Q 2 cosh θ = e θ + e − θ )
Z(n2n) = ∑
n=0
n2n z −n = ∑ n⎜ ⎟
n=0 ⎝ ⎠
z

⎛ z − cosh θ ⎞ 2 3
∴ Z(cosh nθ) = z ⎜ ⎟ 2 ⎛2⎞ ⎛2⎞
⎝ z − 2 z cosh θ + 1 ⎠
2
= + 2⎜ ⎟ + 3⎜ ⎟ + .....
z ⎝z⎠ ⎝z⎠
(v) an + bn
Given that,
2⎡ ⎤
2
⎛2⎞ ⎛2⎞
un = an + bn = ⎢1 + 2⎜ ⎟ + 3⎜ ⎟ + .....⎥
Taking Z-transform on both sides,
z⎢
⎣ ⎝z⎠ ⎝z⎠ ⎥⎦
Z(un) = Z(an + bn)
−1
Z(a + bn) = Z(an) + Z(bn)
n 2 ⎡ 2⎤ 2 z2
= 1− ⎥ =
z z z ⎢⎣ z⎦ z ( z − 2) 2
= +
z −a z −b 2z
=
nπ ( z − 2) 2
(vi) π n – 4 sin – 7a
4 (ii) an2 + bn + c
Given that, Z(n) = a Z(n2) + b Z(n) + c Z(1)
First consider, Z(n2)

un = nπ – 4 sin – 7a From recurrrence formula, we get,
4
d
By applying Z-transform on both sides, we get, Z(n P ) = − z Z ( n P −1 )
dz
⎛ nπ ⎞ Where,
Z(un) = z ⎜ nπ − 4 sin − 7a ⎟ P =2
⎝ 4 ⎠
d z
⎛ nπ ⎞ Z(n2) = − z
dZ ( z − 1) 2
= z(nπ) – z ⎜ 4 sin ⎟ – z(7a)
⎝ 4 ⎠
− z[( z − 1) 2 − 2( z − 1) z ]
Z(n2) =
⎛ nπ ⎞ [( z − 1) 2 ]2
= π z(n) – 4z ⎜ sin ⎟ – 7a z(1)
⎝ 4 ⎠
z2 + z
z sin( π / 4) z = ... (1)
= π.
z
–4× 2 – 7a . ( z − 1)3
( z − 1) 2
z − 2 z cos(π / 4) + 1 z −1

⎛ 1 ⎞
∴ Z(n) = ∑ nz
n=0
−n

4 z⎜ ⎟
zπ ⎝ 2⎠ 7 az
= − − 1 2 3
( z − 1) 2
⎛ 1 ⎞ ( z − 1) = + + + .....
z 2 − 2z⎜ ⎟ +1 z z 2 z3
⎝ 2⎠
1⎛ 2 3 ⎞
= ⎜1 + + 2 + ..... ⎟
πz 4z 7az z⎝ z z ⎠
= – 2 –
( z − 1) 2
z 2 − 2z + 2 z −1 1⎛ 1⎞
−2
z
Z(n) = ⎜1 − ⎟ = ... (2)
Q2. Find the Z-transform of the sequences {x(n)} z⎝ z⎠ ( z − 1) 2
where x(n) is,
Also,
(i) n.2n
(ii) an2 + bn + c. ∞

Ans: Z(1) = ∑1 z
n=0
−n

(i) n.2n

Given that,
∑z
1
= n
x(n) = n2n n=0

SPECTRUM ALL-IN-ONE JOURNAL FOR ENGINEERING STUDENTS SIA GROUP


5.4 MATHEMATICS-II [JNTU-ANANTAPUR]
1 1 z ( z − cos 3)
= 1+ + 2 + ..... ⇒ Z[cos 3n] = ... (2)
z z z − 2 z cos 3 + 1
2

1 z For, Z[cos n], θ = 1


= = ... (3)
1−
1 z −1 z ( z − cos 1)
z ⇒ Z[cos n] = ... (3)
z − 2 z cos 1 + 1
2

On substituting the values from equations (1), (2) and


On substituting equations (2) and (3) in equation (1),
(3) in the given equation, we get,
we get,
⎡ z2 + z ⎤ ⎡ z ⎤ ⎡ z ⎤ 1 ⎡ z ( z − cos 3) ⎤ 3 ⎡ z ( z − cos 1) ⎤
Z(n) = a ⎢ 3⎥
+ b⎢ 2⎥
+ c⎢ ⎥ Z[cos3n] = +
⎢⎣ ( z − 1) ⎥⎦ ⎢⎣ ( z − 1) ⎥⎦ ⎣ z − 1⎦ 4 ⎢⎣ z 2 − 2 z cos 3 + 1 ⎥⎦ 4 ⎢⎣ z 2 − 2 z cos 1 + 1 ⎥⎦

Q3. Find the Z-transform of, Q4. State and prove the linearity property of
Z-transform. Model Paper-I, Q1(i)
(i) 2n + 3
Ans:
(ii) cos 3n. Model Paper-III, Q1(j)
Linearity Property: If x, y, z are any constants and un, vn, wn
Ans:
are any distinct functions, then
(i) Let, un = 2n + 3 Z(xun + yvn – zwn) = xZ(un) + yZ(vn) – Z(wn)
By applying Z-transform on both sides, we get, Proof: From the definition of Z-transform,
Z(un) = Z(2n + 3)

= Z(2n) + Z(3) Z(x un + yvn – zwn) = Σ (xun + yvn – zwn) z–n
n =0
= 2.Z(n) + 3 Z(1)
∞ ∞ ∞
z z = x Σ un z–n + y Σ vn z–n – z Σ wn z–n
= 2. +3 n =0 n =0 n =0
( z − 1) 2 z −1
= xZ(un) + yZ(vn) – zZ(wn)

2z 3z Since, Z (un ) = Σ un z
−n
= + n=0
( z − 1) 2 z −1
∴ Z(xun + yvn – zwn) = xZ(un) + yZ(vn) – Z(wn)
(ii) cos 3n = 4cos3n – 3cosn
Q5. State and prove damping rule.
⇒ 4cos3n = cos3n + 3cosn
Ans:
cos 3n + 3 cos n Damping Rule (or) Change of Scale Property
⇒ cos3n =
4
1. If Z(un) = u (z ) then Z(a–n un) = u (az)
1 3
⇒ cos3n = cos 3n + cos n
4 4 ∞

On applying Z-transform on both sides, we get, Z(un)= ∑u


n=0
n
z − n = u (z )

⎡1 3 ⎤
Z[cos3n] = Z ⎢ cos 3n + cos n ⎥ ∞ ∞
⎣4 4 ⎦
⇒ Z(a–n un) = ∑ a − n u n z −n = ∑ u (az)
n=0
n
−n

⎡1 ⎤ ⎡3 ⎤ n=0
= Z ⎢ cos 3n ⎥ + Z ⎢ cos n ⎥
⎣4 ⎦ ⎣4 ⎦
= u (az )
1 3
= Z [cos 3n]+ Z [cos n] Hence, the Z-transform of a– n un is equal to the Z-transform
4 4
of un when z is replaced by az.
1 3
⇒ Z[cos3n] = Z [cos 3n] + Z [cos n] ... (1) ⎛z⎞
4 4 2. If Z(un) = u (z ) , then Z(an un) = u ⎜ ⎟
⎝a⎠
Z ( Z − cos θ)
As, Z[cos nθ] = ∞

∑u
2
Z − 2 Z cos θ + 1 z −n
Z(un)= n
For, Z[cos 3n], θ = 3 n= 0

Look for the SIA GROU P LOGO on the TITLE COVER before you buy
UNIT-5 (Z-Transform) 5.5
∞ ∞ n ∞
⎛a⎞
∑ ∑ ∑ n . x ( n) z
d
Z(anun) = a n un z −n = un ⎜ ⎟ −1
= −z .
−n
− z. X ( z)
n= 0 n= 0 ⎝z⎠ n = −∞
dz

∞ −n = z[(n x (n))]
⎛z⎞
= ∑
n =0
un ⎜ ⎟
⎝a⎠
⎛z⎞
= u⎜ ⎟
⎝a⎠ ∴ nx(n) ←⎯→ – z
z d
X(z)
dz
az
Q6. Show that, Z(nan) = . Model Paper-II, Q1(i) In this property, ROC does not change.
(z − a) 2 Division by n
Ans: To prove, Statement
z
az ⎧ f (n) ⎫

Z(nan) = −1
If Z{f(n)} = f(z) then z ⎨ ⎬ = − z f ( z ) dz
(z - a) 2 ⎩ n ⎭ 0
Let, un = n.an
Proof
z Consider,
Z(n) = = U(z)
( z − 1) 2 ∞
⎧ f (n) ⎫

f ( n) − n
z⎨ ⎬= z
Applying scaling property, ⎩ n ⎭ n
n = −∞
i.e., If Z(un) = U(z) then Z(an. un) = U(z/a)
∞ ⎡ z ⎤
Where, U(z) =
z
( z − 1) 2
= ∑ ⎢ 0 ∫
f (n) ⎢− z n −1dz ⎥

n = −∞ ⎣ ⎦

⎡ z/a ⎤ z ∞

∫ ∑ f ( n) z
−1 −n
∴ Z(n.an) = ⎢ 2⎥ =− z dz
⎣ ( z a − 1) ⎦ 0 n = −∞

z z
× a2

= −1
a × ( z − a) 2 = − z f ( z )dz
0

az
∴ Z(n.an) = z
( z − a) 2 ⎧ f (n) ⎫

−1
∴ z⎨ ⎬ = − z f ( z )dz
⎩ n ⎭
Q7. Write the properties of multiplication by n and 0
division by n of Z-transforms. Model Paper-III, Q1(i)
Q8. Discuss about the initial and final value
Ans: theorems of Z-transforms.
Multiplication by n OR
∞ State and prove the initial and final value
X(z) = ∑ x (n ) z
n = −∞
−n
theorems of Z-transforms.
Ans:
On differentiating on both sides with respect to ‘z’, we get, (i) Initial Value Theorem of Z-transform

If Z(f (n)) = f (z), then Lt f (z) = f (0)


d ⎡ ⎤
∞ z →∞

d
( X ( z )) = ⎢ x( n ) z − n ⎥ ∞
dz dz ⎢⎣ n = −∞ ⎥⎦
Z[f(n)] = f(z) = ∑ f ( n) z
n =0
−n


= f (0) + f (1)z–1 + f (2)z–2 + ...

d d
⇒ ( X ( z )) = x( n ) ( z − n )
dz n = −∞
dz 1 1
= f(0) + f (1). + f (2) 2 +...

z z
= ∑ x( n ) − n . ( z
n = −∞
− n −1
)
As z → ∞, Lt f (z) = f (0) ⇒ f (0) = Lt f (z)
z →∞ z →0

SPECTRUM ALL-IN-ONE JOURNAL FOR ENGINEERING STUDENTS SIA GROUP


5.6 MATHEMATICS-II [JNTU-ANANTAPUR]

f (1) = Lt z[f (z) – f (0)]


z →∞

f (2) = Lt z2[f (z) – f (0) – f (1) z–1]


z →∞

f (3) = Lt z3[f (z) – f (0) – f (1) z–1 – f (1)2 z–2] ....


z →∞

(ii) Final Value Theorem of Z-Transform

If Zf (n) = f (z), then nLt


→∞
f (n) = Lt (z – 1)f (z)
z →1


Z[f(n + 1) – f (n)] =
∑ [ f (n + 1) − f (n)] z
n=0
–n


⇒ Z[f (n + 1)] – z[f (n)] = ∑ [ f ( n + 1) − f ( n) ]z
n =0
–n

⇒ Z[f (z) – f (0)] – f(z) = ∑ [ f (n + 1) − f (n)]z


n= 0
−n


⇒ f (z) (z – 1) – z.f (0) = ∑ [ f (n + 1) − f (n)]z
n= 0
−n

Taking the limits as z → 1, we get,


Lt [f (z) (z – 1)] – f (0) =
z →1 ∑ [ f (n + 1) − f (n)]
n=0

[
= Lt ( f (1) − f (0)) + ( f (2) − f (1)) + ... + ( f (n + 1) − f (n))]
n →∞

= nLt [ f ( n + 1) − f (0)]
→∞

∴ Lt ( z − 1) f ( z ) − f (0) = nLt f (n) − f (0)


z →1 →∞

So, Lt f (n) = Lt ( z − 1) f ( z )
n→∞ z →1

⎧⎪ z 2 + 2 ⎫⎪
Q9. Find Z–1 ⎨ ⎬. Model Paper-II, Q1(J)
⎪⎩ (z − 1) 2 ⎪⎭

Ans: Given that,

⎧⎪ z 2 + 2 ⎫⎪
Z −1 ⎨ ⎬
⎪⎩ ( z − 1) 2 ⎪⎭

z2 + 2 A B
Considering, = + ... (1)
( z − 1) 2 ( z − 1) ( z − 1) 2

z2 + 2 A( z − 1) + B
=
( z − 1) 2
( z − 1) 2

⇒ z2 + 2 = A(z – 1) + B ... (2)

Look for the SIA GROU P LOGO on the TITLE COVER before you buy
UNIT-5 (Z-Transform) 5.7
On substituting Z = 1 in equation (2), we get,

∴B = 3

On comparing constant terms, we get,


2 =–A+B
2 =–A+3
2–3 =–A
–1 = – A

∴ A=1

On substituting the values of A and B in equation (1), we get,

z2 + 2 1 3
= +
( z − 1) 2 ( z − 1) ( z − 1) 2

⎡ z2 + 2 ⎤ ⎛ 1 ⎞ ⎛ 3 ⎞
Z–1 ⎢ 2⎥=Z ⎜
–1 ⎟ + Z–1 ⎜⎜ ⎟

⎢⎣ ( z − 1) ⎥⎦ ⎝ z −1 ⎠ ⎝ ( z − 1)
2

⎡ −1 ⎛ 1 ⎞ n −1 ⎤
⎢Q Z ⎜ ⎟ = a u ( n − 1) and ⎥
⎢ ⎝ z−a⎠ ⎥
⎛ 1 ⎞ ⎛ 1 ⎞
= Z–1 ⎜ ⎟ + 3Z–1 ⎜⎜ ⎟

⎢ ⎛ 1 ⎞ ⎥
⎝ z −1 ⎠ ⎝ ( z − 1)
2
⎠ ⎢ Z −1 ⎜ ⎟ = ( n − 1) a n − 2 u ( n − 2) ⎥
⎢⎣ ⎜ ( z − a) 2 ⎟ ⎥⎦
⎝ ⎠
= 1n–1 u(n – 1) + 3((n–1)1n–2 u(n – 2))
= u(n – 1) + 3(n – 1) u(n – 2)
Q10. State and prove convolution theorem.
Ans: Model Paper-I, Q1(j)

Statement
If Z–1[f (z)] = f (α) and Z–1[g(z)] = g(α)
Then,
α
Z–1[f(z).g(z)] = f (α)* g(α) = Σ f(β) g(α – β)
β=0

∞ ∞
Proof : f(z) = Σ f(α) z–α, g(z) = Σ g(α) z–α
α =0 α =0

∴ f(z) g(z)= [f (0) + f (1) z–1 + f (2) z–2 + ..... + f (α) z–α +.....] × [g(0) + g(1) z–1 + g(2) z–2 +.....+g(α) z–α +.....]
= [f(0) g(0) + f(0) g(1) z–1 + ..... + f(0) g(α) z–α + ..... + f(α) g(0) z–α + f(α) g(1) z–α + f(α) g(α) z–α]

= Σ [f (0) g(α) + f (1) g(α–1) + f (2) g(α–2) +.... + f (α) g(0) ] z–α
α =0

Applying transform on both sides, we get


f(z) g(z) = Z [f (0) g(α) + f (1) g(α–1) +....+ f (α) g(0) ]
= Z–1 [f(z).g(z)] = f(0) g(α) + f(1) g(α – 1) +..... + f(α) g(0)
= f(0) g(α–0) + f(1) g(α–1) +......+f(α) g(α– α)
α
= Σ f(β) g(α–β).
β=0

SPECTRUM ALL-IN-ONE JOURNAL FOR ENGINEERING STUDENTS SIA GROUP


5.8 MATHEMATICS-II [JNTU-ANANTAPUR]

PART-B
ESSAY QUESTIONS WITH SOLUTIONS
5.1 INTRODUCTION
⎡ 1 ⎤
Q11. Find Z ⎢ ⎥
⎣ (n + 2)(n − 1) ⎦
Ans: Given equation is,

⎛ 1 ⎞
To find Z ⎜⎜ ⎟⎟
⎝ (n + 2)(n − 1) ⎠

1
Consider, ... (1)
( n + 2)(n − 1)
On solving equation (1) using partial fractions, we get,
1 A B
= +
( n + 2)(n − 1) ( n + 2) ( n − 1)

1 A(n − 1) + B (n + 2)
=
( n + 2)(n − 1) ( n + 2)(n − 1)
1 = A(n – 1) + B(n + 2) ... (2)
On substituting n = –2 in equation (2), we get,
1
1 = –3A ⇒ A = –
3
On substituting n = 1 in equation (2), we get,
1
1 = –3B ⇒ B =
3

1⎡ 1 1 ⎤
∴ Equation (2) changes as −

3 ⎣ (n − 1) n + 2 ⎥⎦
On applying Z-transform, we get,

Z(un) = ∑u z
n =0
n
−n

1 ⎡ 1 1 ⎤
Z⎢ −
3 ⎣ n − 1 n + 2 ⎥⎦
=

1⎧ ⎛ 1 ⎞ ⎛ 1 ⎞⎫
= ⎨Z ⎜ ⎟ − Z⎜ ⎟⎬ ... (3)
3 ⎩ ⎝ n −1⎠ ⎝ n + 2 ⎠⎭

⎛ 1 ⎞ z −n
Z⎜ ⎟=
⎝ n −1 ⎠

n =0
n −1

⎛ z +2 z +3 ⎞
= z–1 ⎜ z .1 − + + .....⎟
⎜ 2 3 ⎟
⎝ ⎠

⎛ z ⎞
= z–1 log ⎜ −1 ⎟ ... (4)
⎝z ⎠

Look for the SIA GROU P LOGO on the TITLE COVER before you buy
UNIT-5 (Z-Transform) 5.9
∞ 1 −αi −1
⎛ 1 ⎞ z −n

Consider, [e z ]
Z⎜ ⎟= n+2
2i
⎝n+2⎠ n =0
1 1
[ e − α i z −1 ] = {1 + (e–αi z–1) + (e–αi z–1)2 + ....}
2i 2i
1 z −1 z −2 z −3
= + + + + .....
2 3 4 5 1 ⎡ 1 ⎤
= ⎢ − αi −1 ⎥ ... (3)
2i ⎣ 1 − e z ⎦
⎛ 1 z −1 z −2 ⎞
= ⎜⎜ + + + .....⎟

On substituting equations (2) and (3) in equation (1), we get,
⎝2 3 4 ⎠ 1 ⎧ 1 1 ⎫
= ⎨ αi −1
− − αi −1 ⎬
2i ⎩1 − e z 1− e z ⎭
⎛ 1 z −1 z −2 ⎞
= z2 ⎜ + + + ..... − z ⎟ ⎧⎪ 1 − e −αi z −1 − 1 + e αi z −1 ⎫⎪
⎜2 3 4 ⎟ 1
⎝ ⎠ = ⎨ ⎬
2i ⎪⎩ (1 − e αi z −1 )(1 − e −αi z −1 ) ⎪⎭
⎛ z ⎞ ⎧⎪ ⎫⎪
= z2 log ⎜ −1 ⎟ – z ... (5) 1 (e αi − e − αi ) z −1
⎝z ⎠ = ⎨ αi − αi −1 −2 ⎬
2i ⎪⎩1 − (e + e ) z + z ⎪⎭
On substituting the values of equations (4) and (5) in
equation (3), we get, On multiplying and dividing (−e αi + e − αi ) by ‘2’, we get,

⎛ 1 ⎞ 1 ⎡ −1 ⎛ z ⎞ 2 ⎛ z ⎞ ⎤ ⎛ e αi − e −αi ⎞ −1
Z ⎜⎜ ⎟⎟ = ⎢ z log⎜ ⎟ − z log⎜ −1 ⎟ + z ⎥ ⎜ ⎟z
⎝ (n − 1)(n + 2) ⎠ 3 ⎣ ⎝ z −1 ⎠ ⎝z ⎠ ⎦ ⎜
⎝ 2i ⎟

=
Q12. Find the Z-transform of sin α k, k ≥ 0. ⎛ e αi + e −αi ⎞ −1 1
1 − 2⎜⎜ ⎟z +

Ans: Given that, 2 z2
⎝ ⎠
U(k) = sin αk, k ≥ 0
[Taking LCM in denominator]

Z{U(k)} = ∑U
k =0
kz
−k
⎛ sin α ⎞
⎜ ⎟
∞ ⎝ Z ⎠ (sin α ) Z
= 2 = 2
Z{sin αk}= ∑ sin(αk ) z
k =0
−k
z − 2 cos αz + 1 Z − 2 cos α + 1
z2

⎧⎪ e iαk − e −iαk ⎪⎫ − k
⇒ Z{sin αk} = ∑ ⎨⎪⎩
k =0
2i
⎬z
⎪⎭ = 2
z sin α
z − 2 cos α + 1
∞ iαk ∞ − iαk Q13. Find the Z-transforms of:
∑ ∑
e e
= z −k − z −k (i) (n – 1)2
k =0
2i k =0
2i
(ii) e–an cosnθ θ.
∞ ∞
e iαk − k e −iαk − k Ans:
= ∑
k =0
2i
z −
k =0
2i
z∑ (i) (n – 1)2
Z[(n – 1)2] = Z[n2 – 2n + 1]
∞ ∞ = Z[n2] – 2Z[n] + Z[1]
∑ ∑
1 1
= (e iα z −1 ) k − (e −iα z −1 ) k ... (1) Consider,
2i k =0
2i k = 0
Z[1] = Z[f(n)]
Consider, By the definition of Z-transform,

∑ (e
1 i α −1 k ∞
z )
2i k =0 Z[f(n)] = ∑ f (n).z
n =0
−n


1

1
iα −1 k
(e z ) = {1 + eαi z–1 + (eαi z–1)2 + .... } ∞
2i 2i
k =0

1 ⎧
∴ Z[1] = ∑1.z −n

1 ⎫ n =0
= ⎨ ⎬ ... (2)
2i ⎩1 − e αi z −1 ⎭ = 1 + z–1 + z–2 + z–3 + ......
⎧ 1 ⎫ 1 1 1
⎨Q1 + a + a + ... + ∞ = = 1+ + 2 + 3 + .....
2

⎩ 1− a ⎭ z z z

SPECTRUM ALL-IN-ONE JOURNAL FOR ENGINEERING STUDENTS SIA GROUP


5.10 MATHEMATICS-II [JNTU-ANANTAPUR]

1 ⎡ z − 1 − 2z ⎤
Z[1] =
1 = − z ( z − 1) ⎢ 4 ⎥
1− ⎣⎢ ( z − 1) ⎦⎥
z

a ⎡ − z −1 ⎤ z2 + Z
Since, Sum of geometric series is . Here a = 1 = − z ⎢ 3⎥ =
⎣⎢ ( z − 1) ⎦⎥ ( z − 1)
3
1− r
1 ∴ Z[(n – 1)2 = Z[n2] – 2Z[n] + Z[1]
and r =
z
z2 + z ⎡ z ⎤ ⎡ 2 ⎤
z − 2⎢ +
∴ Z[1] = = 2⎥ ⎢ ⎥
( z − 1) ⎣⎢ ( z − 1) ⎦⎥ ⎣ z − 1⎦
3
z −1
The recurrence formula of Z-transform is given by,
z ⎡ z +1 z ⎤
= ⎢ − + 1⎥
d ( z − 1) ⎣⎢ ( z − 1) 2
z − 1 ⎦⎥
Z[np] = − z [ z ( n p −1 )] ... (1)
dz

On substituting p = 1 in equation (1), we get, z ⎡ z + 1 − 2( z − 1) + ( z − 1) 2 ⎤


= ⎢ ⎥
z − 1 ⎣⎢ ( z − 1) 2 ⎦⎥
d
Z[n] = − z [ Z ( n 0 )]
dz
=
z
( z − 1) 3
[z + 1 − 2z + 2 + z 2
− 2z + 1]
d
= −z [Z (1)]
dz
=
z
( z − 1) 3
[z 2
− 3z + 4 =] z 3 − 3z 2 + 4 z
( z − 1) 3
d ⎡ z ⎤ ⎡ z ⎤
= −z ⎢Q z (1) = z − 1 ⎥
dz ⎢⎣ z − 1 ⎥⎦ ⎣ ⎦ z 3 − 3z 2 + 4 z
∴ Z[(n – 1) ] = 2
( z − 1)3
⎡ ( z − 1).1 − z (1 − 0) ⎤ z (ii) θ]
[e–an cos nθ
= − z⎢ ⎥ = 2
( z − 1) ⎥⎦ ( z − 1)
2
⎢⎣
Z[e–an cos nθ] = Z [(e − a ) − n . cos nθ]

z Since,
∴ Z[n] = 2
( z − 1) z ( z − cos θ)
Z[cos(θ)] =
z 2 − 2 z cos θ + 1
On substituting p = 2 in equation (1), we get,
From the change of scale property or the damping rule,
d If Z[x(n)] = X(z)
Z[n2] = − z [ z ( n 2−1 )]
dz Then,
Z[a– n.x(n)] = X(az)
d
= −z [ z ( n)] Here,
dZ
x(n) = cos nθ

⎡ And a = e– a
d ⎡ Z ⎤ z ⎤
= −Z ⎢ 2⎥
⎢Q Z (n) = ⎥
dZ ⎣ (Z − 1) ⎦ ⎢⎣ ( z − 1) 2 ⎥⎦ (e a .z )[(e a .z ) − cos θ]
∴ Z[(ea)–n.cos(nθ)] =
(e .z ) 2 − 2(e a .z ). cos θ + 1
a

⎡ ( z − 1) 2 .1 − z.2( z − 1).(1) ⎤
= − z⎢ ⎥ ze a .[ z.e a − cos θ]
⎣⎢ ( z − 1) 4 ⎦⎥ =
z 2 .e 2a − 2 ze a cos θ + 1

Look for the SIA GROU P LOGO on the TITLE COVER before you buy
UNIT-5 (Z-Transform) 5.11

⎧ 1 ⎫ z 5.2 INVERSE Z-TRANSFORM


Q14. Show that Z ⎨ ⎬ = z log .
⎩ n + 1 ⎭ z −1
⎛ z 2 − 3z ⎞
Q15. Find Z–1 ⎜⎜ ⎟.

Ans: To prove, ⎝ (z + 2)(z − 5) ⎠

⎧ 1 ⎫ z z 2 − 3z Az Bz
Z⎨ Ans: Let, = −
⎬ = z log ( z + 2)( z − 5) z + 2 z −5
⎩ n +1⎭ z −1

Consider L.H.S, z 2 − 3z Az( z − 5) − Bz ( z + 2)


⇒ =
( z + 2)( z − 5) ( z + 2)( z − 5)
⎧ 1 ⎫
Z⎨ ⎬ = z{f(n)} ⇒ z2 – 3z = Az2 – 5Az – Bz2 – 2Bz
⎩ n +1⎭
⇒ z2 – 3z = z2(A – B) – z(5A + 2B)
By the definition of Z-transform, On comparing L.H.S and R.H.S, we get,
⇒ A – B =1 ... (1)
∞ ⇒ 5A + 2B = 3 ... (2)
Z{f(n)} = ∑ f (n).z
n =0
−n
From equation (1), we get,
A= 1 + B
On substituting A = 1 + B in equation (2), we get,

⎡ 1 ⎤

⎧ 1 ⎫
Z⎨ ⎬ =
⎩ n +1⎭
∑ ⎢⎣ n + 1⎥⎦.z −n 5(1 + B) + 2B = 3
n= 0 ⇒ 5 + 5B + 2B = 3
⇒ 5 + 7B = 3
1 −1 1 − 2 1 −3 ⇒ 7B = 3 – 5
= 1 + .z + .z + .z + ...
2 3 4 ⇒ 7B = – 2
−2
1 1 1 ∴B =
= 1+ + 2 + 3 + ... 7
2 z 3z 4z
−2
On substituting B = in equation (1), we get,
⎡1 2 3 4 ⎤ 7
1⎛1 ⎞ 1⎛1⎞ 1 ⎛1⎞
= z⎢ + ⎜ f ⎟ + ⎜ ⎟ + ⎜ ⎟ + ...⎥
2⎝ z ⎠ 3⎝ z ⎠ 4⎝ z⎠ A – B =1
⎣⎢ z ⎥⎦
2
⇒ A+ =1
⎡ ⎛ 1 ⎞⎤
7
= z ⎢− log⎜1 − ⎟⎥
⎣ ⎝ z ⎠⎦ 2 5
⇒ A =1– =
7 7
⎡ x 2 x3 x 4 ⎤
⎢Q x + + + + ... = − log(1 − x )⎥ 5
⎢⎣ 2 3 4 ⎥⎦ ∴A =
7

⎡ z 2 − 3z ⎤ ⎡ 5 ⎛ z ⎞ 2 ⎛ z ⎞⎤
⎡ ⎛ z − 1 ⎞⎤ ∴ z–1 ⎢ ⎥ = z–1 ⎢ ⎜ ⎟+ ⎜ ⎟⎥
= z ⎢− log⎜ ⎟⎥ ⎣ 7 ⎝ z + 2 ⎠ 7 ⎝ z − 5 ⎠⎦
⎣ ⎝ z ⎠⎦ ⎣⎢ ( z + 2)( z − 5) ⎦⎥

5 –1 ⎛ z ⎞ 2 ⎛ z ⎞
⎡ ⎛ z ⎞⎤ = z ⎜ ⎟ + z–1 ⎜ ⎟
7 ⎝ z+2⎠ 7 ⎝ z −5⎠
= z ⎢log⎜ ⎟⎥
⎣ ⎝ z −1 ⎠⎦
5 2
= (–2)n + (5)n
7 7
⎧ 1 ⎫ ⎛ z ⎞
∴ Z⎨ ⎬ = z log ⎜ ⎟
⎩ n +1⎭ ⎝ z −1 ⎠ =
1
[5(–2)n + 2(5)n]
7

SPECTRUM ALL-IN-ONE JOURNAL FOR ENGINEERING STUDENTS SIA GROUP


5.12 MATHEMATICS-II [JNTU-ANANTAPUR]

3z 2 + z ⎛ −8⎞ ⎛ 11 ⎞
Q16. Find the inverse Z-transform of . ⎜ ⎟z ⎜ ⎟z
(5z − 1)(5z − 2) ⎝ 5 ⎠ ⎝5⎠
x(z) = +
Ans: Model Paper-III, Q10 ⎛ 1 ⎞ ⎛ 2⎞
5⎜ z − ⎟ 5⎜ z − ⎟
⎝ 5⎠ ⎝ 5⎠
Given that,
Taking inverse Z-transform on both sides, we get,
3z 2 + z n n
x(z) =
(5 z − 1)(5 z − 2) −8 ⎛ 1 ⎞ 11 ⎛ 2 ⎞
x(n) = ⎜ ⎟ u (n) + ⎜ ⎟ u (n)
25 ⎝ 5 ⎠ 25 ⎝ 5 ⎠
z (3z + 1)
x(z) = (5 z − 1)(5 z − 2) ⎡ 1 ⎤
Q17. Find Z–1 ⎢ 3⎥
when | z | > 5. Determine the
(3 z + 1) ⎣ (z − 5) ⎦
x( z)
= (5 z − 1)(5 z − 2) region of convergence. Model Paper-I, Q10
z
Ans: Given that,
By partial fractions, we get,
(3 z + 1) A B ⎡ 1 ⎤
x( z)
= (5 z − 1)(5 z − 2) = (5z − 1) + (5 z − 2) ... (1) Z–1 ⎢ 3⎥
z ⎣⎢ ( z − 5) ⎦⎥

A(5z – 2) + B(5z – 1) = (3z + 1) ... (2) 1 1


Let, U(z) = = 3
( z − 5) 3 ⎛ 5⎞
2 z 3 ⎜1 − ⎟
On substituting z = in equation (2), we get, ⎝ z⎠
5

⎛ ⎛2⎞ ⎞ ⎛ ⎛ ⎛ 2 ⎞⎞ ⎞ ⎛ ⎛ 2 ⎞ ⎞ z −3
A⎜⎜ 5⎜ ⎟ − 2 ⎟⎟ + ⎜⎜ B⎜⎜ 5⎜ ⎟ ⎟⎟ − 1⎟⎟ = ⎜⎜ 3⎜ ⎟ + 1⎟⎟ = = z–3(1 – 5z–1)–3
⎝ ⎝5⎠ ⎠ ⎝ ⎝ ⎝ 5 ⎠⎠ ⎠ ⎝ ⎝ 5 ⎠ ⎠ (1 − 5 z −1 ) 3


⎛6 ⎞
A(0) + B(1) = ⎜ +1⎟
[
= z–3 1 + 3(5 z −1 ) + 6(5z −1 ) + 9(5 z −1 ) 3
⎝5 ⎠
( n + 1)(n + 2) ⎤
+ ...... + (5 z −1 ) n + ......⎥
⎛ 6+5⎞ 2 ⎦
⇒ B=⎜ ⎟
⎝ 5 ⎠
⎧ ( n + 1)(n + 2) ⎫
= z–3 ⎨ (5 z −1 ) n ⎬
∴B =
11 ⎩ 2 ⎭
5
... (3)
⎧ ( n + 1)(n + 2) n ⎫ − n
On comparing the coefficients of z, we get, = z–3 ⎨ 5 ⎬z
5A + 5B = 3 ⎩ 2 ⎭

⎛ 11 ⎞ ⎧ ( n + 1)( n + 2) n ⎫ −n −3
⇒ 5A + 5 ⎜ ⎟ = 3 =⎨ 5 ⎬z
⎝5⎠ ⎩ 2 ⎭
⇒ 5A + 11 = 3 On substituting ‘n’ as n – 3, we get,
⇒ 5A = 3 – 11
⎡ [( n − 3) + 1][( n − 3) + 2] ( n −3) ⎤ − ( n −3) − 3
−8 U(z) = ⎢ 5 ⎥z
∴A = ... (4) ⎣ 2 ⎦
5
On substituting equations (3) and (4) into equation (1), ⎡ ( n − 3 + 1)(n − 3 + 2) n −3 ⎤ −n
we get, U(z) = ⎢ 5 ⎥z
⎣ 2 ⎦
−8 11
x( z) 5 5 ⎡ ( n − 2)(n − 1) n −3 ⎤ −n
= + =⎢ 5 ⎥z
z (5z − 1) ( 5 z − 2) ⎣ 2 ⎦

⎛ −8⎞ ⎛ 11 ⎞
⎜ ⎟ ⎜ ⎟ ⎧ (n − 1)(n − 2) n −3
=
⎝ 5 ⎠ ⎝5⎠ ⎪ 5 , when n ≥ 3
+ ∴ Z [U(z)] = [U(n)] = ⎨
–1
2
⎛ 1⎞ ⎛ 2⎞
5⎜ z − ⎟ 5⎜ z − ⎟ ⎪⎩ 0, when n < 3
⎝ 5⎠ ⎝ 5⎠

Look for the SIA GROU P LOGO on the TITLE COVER before you buy
UNIT-5 (Z-Transform) 5.13

z2 − z ⎡ z2 − z ⎤ ⎡ z ⎤ ⎡ (−1) z ⎤
Q18. Find the inverse Z-transform of . ⎢
⇒ Z–1 ⎢ 2 ⎥ = Z ⎢ z − ( −1) ⎥ + 2Z 2⎥
–1 –1
(z + 1)2
⎣⎢ (z + 1) ⎦⎥ ⎣ ⎦ ⎣⎢ ( z − (−1)) ⎦⎥
Ans: Given function is,
= (–1)n + 2n (–1)n
z ( z − 1)
= (–1)n (1 + 2n)
( z + 1) 2
z2 − z z ( z − 1) ⎧⎪ z3 ⎫⎪
⇒ = 2 Q19. Find Z–1 ⎨ ⎬.
( z + 1) 2
( z + 1) ⎪⎩ (z + 1)(z − 1)2 ⎪⎭
z −1 A B Ans: Given function is,
Consider, += ... (1)
( z + 1) ( z + 1)
2
( z + 1) 2
⇒ z – 1 = A (z + 1) + B z3
⇒ z–1=Az+A+B ( z + 1)( z − 1) 2
Equating the coefficients of z on both sides, we get,
z3
∴A =1 Let, F(z) =
( z + 1)( z − 1) 2
Equating the constants on both sides, we get,
A + B = –1 Z–1[F(z)] = ?
On substituting A = 1 in the above equation, we get,
⎧⎪ z3 ⎫⎪
1 + B = –1 Z–1{F(z)} = Z–1 ⎨ 2⎬
⎪⎩ ( z + 1) ( z − 1) ⎪⎭
B = –1 –1 = – 2

∴B = −2 z3
∴ F(z) =
On substituting A = 1 and B = –2 is equation (1), we get, ( z + 1)( z − 1) 2

z −1 1 −2 F (z) z2
= +
( z + 1) 2 z + 1 (z + 1)2 = ... (1)
z ( z + 1)( z − 1) 2
z −1 1 2 Taking partial fractions of the R.H.S term of equation (1),
⇒ = –
( z + 1) 2
z + 1 (z + 1)2 we get,

On multiplying by ‘z’ on both sides, we get, z2 A B C


= + +
( z + 1)( z − 1) 2
z + 1 z − 1 ( z − 1) 2
z ( z − 1) ⎛ 1 2 ⎞
= z ⎜⎜ − ⎟

(z + 1)2 z + 1 + 1) 2 ⇒ z2 = A[(z –1)2] + B[(z + 1) (z –1)] + C [z +1]
⎝ ( z ⎠
⇒ z2 = A [(z – 1)2] + B [(z2 –1)] + C [z + 1] ... (2)
z2 − z z 2z On substituting z = 1 in equation (2), we get,
⇒ = –
( z + 1) 2
z + 1 ( z + 1) 2 12 = 0 + 0 + C [1 +1]
By applying inverse z-transforms on both sides, we get, ⇒ 1 = 2C

1
⎡ z2 − z ⎤ ⎡ z 2z ⎤ ∴C =
⎢ ⎢ − 2⎥
2
Z–1 2⎥ =Z
–1
⎢⎣ (z + 1) ⎥⎦ ⎣⎢ z + 1 ( z + 1) ⎦⎥
On substituting z = – 1 in equation (2), we get,
⎡ z ⎤ ⎡ 2z ⎤ (–1)2 = A [(–1–1)2] + 0 + 0
= Z–1 ⎢ ⎥ – z–1 ⎢ 2⎥ ⇒
⎣ z +1 ⎦ ⎣ ( z + 1) ⎦
1 = 4A

1
⎡ z ⎤ ∴A =
⎡ z ⎤ 4
= Z–1 ⎢ z +1⎥ – 2Z–1 ⎢ 2⎥
⎣ ⎦ ⎣⎢ (z + 1) ⎦⎥
Comparing the coefficients of z2 on both sides of equa-
tion (2), we get,
⎡ z ⎤ ⎡ az ⎤
= an and Z–1 ⎢ 1 =A+B
As, Z–1 ⎢ 2 ⎥ = n.a
n

⎣z −a⎦ ⎢⎣ ( z − a) ⎦⎥ ⇒ B = 1 –A

SPECTRUM ALL-IN-ONE JOURNAL FOR ENGINEERING STUDENTS SIA GROUP


5.14 MATHEMATICS-II [JNTU-ANANTAPUR]
On substituting z = – 3 in equation (3), we get,
1
⇒ B=1– 1 = A(– 3 + 8) + B( – 3 + 3)
4
⇒ 1 = A(5) + 0
3 1
∴B = ⇒ A =
4 5
On substituting z = – 8 in equation (3), we get,
F ( z) 1/ 4 3/ 4 1/ 2
∴ = + + 1 = A(– 8 + 8) + B(– 8 + 3)
z z + 1 z − 1 ( z − 1) 2
⇒ 1 = A(0) + B(– 5)

F (z) 1 ⎡ 1 ⎤ 3 ⎡ 1 ⎤ 1 ⎡ 1 ⎤ −1
⇒ = ⎢ + + ⎢ ⎥ ⇒ B=
z 4 ⎣ z + 1 ⎥⎦ 4 ⎢⎣ z − 1⎥⎦ 2 ⎢⎣ ( z − 1) 2 ⎥⎦ 5

1 −1
On substituting A = ,B= in equation (2), we get,
1⎡ z ⎤ 3⎡ z ⎤ 1⎡ z ⎤ 5 5
⇒ F(z) = ⎢ + + ⎢ ⎥ ... (3)
4 ⎣ z + 1 ⎥⎦ 4 ⎢⎣ z − 1⎥⎦ 2 ⎢⎣ ( z − 1) 2 ⎥⎦
⎛1⎞ ⎛ −1 ⎞
⎜ ⎟ ⎜ ⎟
Applying inverse Z-transform on equation (3), we get, 1 ⎝5⎠ + ⎝ 5 ⎠
= ... (4)
( z + 3)( z + 8) ( z + 3) ( z + 8)
1 −1 ⎡ z ⎤ 3 −1 ⎡ z ⎤ 1 −1 ⎡ z ⎤
Z–1 [F(z)] = Z ⎢ ⎥ + Z ⎢ z − 1⎥ + 2 Z ⎢ 2⎥
4 ⎣ z + 1⎦ 4 ⎣ ⎦ ⎣⎢ ( z − 1) ⎦⎥ On substituting equation (4) in equation (1), we get,

⎛1⎞ ⎛ −1 ⎞
1 n 3 n 1 ⎜ ⎟ ⎜ ⎟
= ( −1) + (1) + n F (z) ⎝5⎠ + ⎝ 5 ⎠
4 4 2 =
z ( z + 3) ( z + 8)
⎡ −1 ⎡ z ⎤ ⎡ z ⎤ ⎡ z ⎤ ⎤
⎢Q Z ⎢ ⎥ = −a n , Z −1 ⎢ ⎥ = a n , Z −1 ⎢ ⎥ = n ⎥ 1 z 1 z
⎢⎣ ⎣z + a⎦ ⎣z − a⎦ ⎣ ( z − 1) ⎦
2
⎥⎦ ⇒ F(z) = . − .
5 z +3 5 z +8
Applying inverse Z-transform on both sides of above
(−1) n 3 n
= + + equation, we get,
4 4 2
−1 ⎡ 1 z 1 z ⎤
⎧⎪ ⎫⎪ (−1) n 3 n Z–1|F(z)| = z ⎢ . − . ⎥
z3 ⎣5 z + 3 5 z + 8 ⎦
∴Z–1 ⎨ ⎬= + +
⎪⎩ ( z + 1)( z − 1) 2 ⎪⎭ 4 4 2
⎡ z ⎤ 1 −1 ⎡ z ⎤ 1 −1 ⎡ z ⎤
⇒ z −1 ⎢ 2 ⎥ = 5 .z ⎢⎣ z + 3 ⎥⎦ − 5 .z ⎢⎣ z + 8 ⎥⎦
⎧ z ⎫ ⎣ z + 11z + 24 ⎦
Q20. Evaluate Z–1 ⎨ 2 ⎬.
⎩ z + 11z + 24 ⎭ 1 1
= .( −3) n − .(−8) n
Ans: Given function is, 5 5

⎧ z ⎫ ⎧ z ⎫
Z −1 ⎨ 2 ⎬ [Q z[an] =
z
⇒ z–1 ⎨ ⎬ = an]
⎩ z + 11z + 24 ⎭ z−a ⎩z − a⎭

z 1
Consider, 2 = [( −3) n − ( −8) n ]
z + 11z + 24 5
Let,
⎡ z ⎤ 1
∴ z −1 ⎢ 2 ⎥ = [( −3) − ( −8) ]
n n
z ⎣ z + 11z + 24 ⎦ 5
F(z) = 2
z + 11z + 24
z
Q21. Find the inverse Z-transform of 2 .
F ( z) 1 z + 8z + 15
⇒ = ... (1)
z ( z + 3)( z + 8) Ans: Given function is,
Applying partial fractions on R.H.S, we get, z
f(z) =
1 A B z + 8 z + 15
2

= + ... (2)
( z + 3)( z + 8) ( z + 3) ( z + 8) f (z) 1
⇒ = 2
⇒ 1= A(z + 8) + B(z + 3) ... (3) z z + 8 z + 15

Look for the SIA GROU P LOGO on the TITLE COVER before you buy
UNIT-5 (Z-Transform) 5.15

f (z) 1 z2 + z
⇒ = 2 Q22. Find the inverse Z-transform of .
z z + 5 z + 3 z + 15 (z − 1)2

f (z) 1
⇒ = z2 + z
z ( z + 5)( z + 3) Ans: Let, F(z) =
( z − 1) 2
Taking partial fractions of the R.H.S term, we get,
z ( z + 1)
1 A B ⇒ F(z) =
= + ( z − 1) 2
( z + 5)( z + 3) z + 5 z + 3
F (z ) z +1
⇒ 1= A[z + 3] + B[z + 5] ⇒ =
z ( z − 1) 2
On substituting z = – 3, we get,
1 = A[0] + B[– 3 + 5] Taking partial fractions of R.H.S term, we get,

⇒ 1 = 2B z +1 A B
2 =
+
( z − 1) z − 1 ( z − 1) 2
1
∴B =
2 ⇒ z + 1 = A[z – 1] + B ... (1)
On substituting z = 1, we get,
On substituting z = – 5, we get,
⇒ 1 + 1 = A[0] + B
⇒ 1 = A[– 5 + 3] + B[0]
⇒ 1 = – 2A ∴B = 2

Comparing the constants of equation (1), we get,


1
∴A = − –A+B = 1
2
A =B–1
−1 1 ⇒ A =2–1
f (z) 2 + 2
∴ =
z z +5 z +3 ∴A=1

f (z) −1 ⎛ 1 ⎞ 1 ⎛ 1 ⎞ F ( z) 1 2
⇒ = ⎜ ⎟+ ⎜ ⎟ ∴ = +
z 2 ⎝ z + 5⎠ 2 ⎝ z + 3⎠ z z − 1 ( z − 1) 2

−1 ⎛ z ⎞ 1 ⎛ z ⎞ z ⎡ z ⎤
⇒ f(z) = ⎜ ⎟+ ⎜ ⎟ ⇒ + 2⎢
2 ⎝ z + 5⎠ 2 ⎝ z + 3⎠ F(z) = 2⎥
z −1 ⎣ ( z − 1) ⎦
Hence,
−1 ⎡ − 1 ⎛ z ⎞ 1 ⎛ z ⎞⎤
∴ Z–1[f(z)] = z ⎢ ⎜ ⎟+ ⎜ ⎟⎥
⎣ 2 ⎝ z + 5 ⎠ 2 ⎝ z + 3⎠ ⎦
⎡ z ⎡ z ⎤⎤
Z–1[F(z)]= Z–1 ⎢ z − 1 + 2 ⎢ 2 ⎥⎥
⎡ z ⎤ − 1 −1 ⎛ z ⎞ 1 −1 ⎛ z ⎞ ⎣⎢ ⎣ ( z − 1) ⎦ ⎦⎥
⇒ Z–1 ⎢ 2 ⎥ = z ⎜ ⎟+ z ⎜ ⎟
⎣ z + 8 z + 15 ⎦ 2 ⎝ z + 5⎠ 2 ⎝ z + 3⎠
−1 ⎡ z ⎤ −1 ⎡ z ⎤
−1 =Z ⎢ ⎥ + 2Z ⎢ ( z − 1) 2 ⎥
=
1
( −5) n + ( −3) n ⎣ z − 1⎦ ⎣ ⎦
2 2
⎡ −1 ⎡ z ⎤ ⎤
⎢Q Z ⎢ z a ⎥ = (a )
n

⎡ −1 ⎛ z ⎞ n⎤ ⎢ ⎣ − ⎦ ⎥
⎢Q z ⎜⎝ ⎟ = (−a) ⎥
z + a⎠ ⎢ ⎡ z ⎥
⎣ ⎦ n⎤
⎢ Z −1 ⎢ = n ( a ) ⎥ ⎥
⎣ ( z − a)
2
⎢⎣ ⎦ ⎥⎦
1
= [(−3) n − (−5) n ]
2 = (1)n + 2n (1)n = [1 + 2n](1)n
SPECTRUM ALL-IN-ONE JOURNAL FOR ENGINEERING STUDENTS SIA GROUP
5.16 MATHEMATICS-II [JNTU-ANANTAPUR]

5.3 PROPERTIES – DAMPING RULE – SHIFTING RULE


Q23. Find the Z-transform of the following,
(i) (n + 1)2
(ii) sin(3n + 5).
Ans:
(i) (n + 1)2
Given function is,
(n + 1)2
By the definition of Z-transform,
Z(n + 1)2 = z(n2 + 2n + 1)

= Σ (n2 + 2n + 1) z–n
n =0

∞ ∞ ∞
⎡ z2 + z z z ⎤
⎢Q z (n ) = = =
2
; z ( n ) ; z (1) ⎥
= Σ n2 z–n + 2 Σ nz–n + Σ 1 z–n ⎣⎢ ( z − 1) 3 ( z − 1) 2 z − 1 ⎥⎦
n =0 n =0 n =0

By linearity property, we get,


Z(n + 1)2 = Z(n2) + 2Z(n) + Z(1)

z2 + z z z z 2 + z + 2 z ( z − 1) + z ( z − 1) 3
= + 2. + =
( z − 1) 3 ( z − 1) 2 z − 1 ( z − 1) 3

z 2 ( z + 1)
∴ Z(n + 1) 2 =
( z − 1) 3
(ii) sin(3n + 5)
Given that,
un = sin(3n + 5)
By the definition of Z-transforms,
Z[sin (3n + 5)]= Z(sin 3n cos 5 + cos 3n sin 5) [Q sin (A + B) = sin A cos B + cos A sin B]
∞ ∞
= cos 5 Σ sin 3n z–n + sin 5 Σ cos 3n z–n
n =0 n =0

By linearity property,
Z[sin (3n + 5)] = cos 5 Z(sin3n) + sin 5 Z(cos3n)
Using formula of Z(sin nθ), Z(cos nθ), we get,
z sin 3 z ( z − cos 3)
Z[sin (3n + 5)]= cos 5 + sin 5. 2
z − 2 z cos 3 + 1
2
z − 2 z cos 3 + 1
Q24. Find the linearity property of Z-transforms,
⎛ nπ π ⎞
(i) cos ⎜ + ⎟
⎝ 2 4⎠

⎛ nπ ⎞
(ii) cosh ⎜ + θ⎟ .
⎝ 2 ⎠
Ans: Model Paper-I, Q11

⎛ nπ π ⎞
(i) cos ⎜ + ⎟
⎝ 2 4⎠
Given that,

⎛ nπ π ⎞
cos ⎜ + ⎟
⎝ 2 4⎠
By the definition of Z-transforms,

Look for the SIA GROU P LOGO on the TITLE COVER before you buy
UNIT-5 (Z-Transform) 5.17

⎛ ⎛ nπ π ⎞ ⎞ ⎛ nπ π nπ π⎞
Z ⎜⎜ cos⎜ + ⎟ ⎟⎟ = Z ⎜ cos cos − sin sin ⎟
⎝ 2 4 ⎠⎠ ⎝ 4⎠
[Q cos( A + B) = cos A cos B − sin A sin B ]
⎝ 2 4 2

∞ ⎛ nπ π nπ π⎞
= Σ ⎜ cos cos − sin sin ⎟ z–n
n =0 ⎝ 2 4 4 4⎠

π ∞ nπ –n π ∞ nπ –n
= cos Σ cos z – sin Σ sin z
4 n =0 2 4 n =0 2

π ⎛ nπ ⎞ π ⎛ nπ ⎞
= cos z ⎜ cos ⎟ – sin z ⎜ sin ⎟
4 ⎝ 2 ⎠ 4 ⎝ 2 ⎠
Using Z(sin nα) and Z(cos nα), we get,

⎛ ⎛ π⎞ π ⎞
⎜ z ⎜ z − cos ⎟ z sin ⎟
⎛ ⎛ nπ π ⎞ ⎞ 1 ⎜ ⎝ 2⎠ 2 ⎟ ⎡Q cos 90° = 0⎤
Z ⎜⎜ cos⎜ + ⎟ ⎟⎟ = − ⎢ sin 90° = 1 ⎥
⎝ ⎝ 2 4 ⎠⎠
⎜ π π ⎟
2 ⎜ z 2 − 2 z cos + 1 z 2 − 2 z cos + 1 ⎟ ⎣ ⎦
⎝ 2 2 ⎠

1 ⎜⎛ z − z ⎞⎟
2
z ( z − 1)
= ⎜ 2 ⎟=
2 ⎝ z +1 z +1⎠
2
2 ( z 2 + 1)

⎛ nπ ⎞
(ii) cosh ⎜ + θ⎟
⎝ 2 ⎠
Given that,

⎛ nπ ⎞
u n = cosh ⎜ + θ⎟
⎝ 2 ⎠
By the definition of Z-transforms,

⎛ nπ ⎛ nπ ⎞ ⎞
⎛ ⎛ nπ ⎞⎞
− +θ
⎜ e 2 + θ + e ⎜⎝ 2 ⎟⎠ ⎟ ⎡ e x + e− x ⎤
Z ⎜⎜ cosh ⎜ + θ ⎟ ⎟⎟ = Z ⎜ ⎟ ⎢Q cosh x = ⎥
⎝ ⎝ 2 ⎠⎠ ⎜

2 ⎟
⎟ ⎣ 2 ⎦
⎝ ⎠

⎛ nπ ⎛ nπ ⎞ ⎞
1 ⎡e θ Σ
− +θ nπ − nπ ⎤
∞ ⎜ e 2 + θ + e ⎜⎝ 2 ⎟⎠ ⎟ ∞ ∞
⎢ 2 z − n + e −θ Σ e 2 z − n ⎥
= Σ ⎜ ⎟ –n
z = e
n =0 ⎜

2 ⎟
⎟ 2 ⎣⎢ n =0 n= 0
⎦⎥
⎝ ⎠

1 ⎡ θ ⎛ nπ ⎞ − θ ⎛ − nπ ⎞⎤
= ⎢e Z ⎜ e 2 ⎟ + e Z ⎜ e 2 ⎟⎥
⎜ ⎟ ⎜ ⎟⎥
2 ⎣⎢ ⎝ ⎠ ⎝ ⎠⎦

⎛ nπ ⎞ ⎡ z ⎤
⎜ 2 ⎟ z
⎢Q Z (a ) = z − a ⎥
n
∴ Z ⎜e ⎟ =
⎝ ⎠
π ⎣ ⎦
z −e2

⎛ − nπ ⎞ z
Z⎜e 2 ⎟ =
⎜ ⎟ z − e −π 2
⎝ ⎠

SPECTRUM ALL-IN-ONE JOURNAL FOR ENGINEERING STUDENTS SIA GROUP


5.18 MATHEMATICS-II [JNTU-ANANTAPUR]

⎛ ⎞
⎛ ⎛ nπ ⎞⎞ 1 ⎜ z −θ z ⎟ z ⎡ eθ e− θ ⎤
Z ⎜⎜ cosh ⎜ + θ ⎟ ⎟⎟ = ⎜ eθ . π
+e π ⎟ = ⎢ π/2
+ ⎥
⎝ ⎝ 2 ⎠⎠ 2 ⎜ − ⎟ 2 ⎢⎣ z − e z − e − π / 2 ⎦⎥
⎝ z −e2 z −e 2 ⎠

z ⎡ eθ ( z − e − π / 2 ) + e − θ ( z − e π / 2 ) ⎤ z ⎡ ze θ − e θ e − π / 2 + ze − θ − e − θ e π / 2 ⎤
= ⎢ ⎥ = ⎢ ⎥
2 ⎢⎣ z 2 − z (e π / 2 + e − π / 2 ) + 1 ⎥⎦ 2 ⎢⎣ z 2 − z (e π / 2 + e − π / 2 ) + 1 ⎥⎦

⎛ ⎛ ⎛π ⎞⎞⎞
⎜ θ −θ ⎜ (π 2 −θ ) −⎜⎝ 2 − θ ⎟⎠ ⎟ ⎟
⎜ z ( e + e ) − ⎜⎜ e + e ⎟⎟ ⎟ z 2 cosh θ − z cosh ⎛⎜ π − θ ⎞⎟
z ⎜ ⎝ ⎠⎟ ⎝2 ⎠
= ⎜ ⎟=
2⎜ ⎛ π

π ⎞ ⎟ ⎛ π ⎞
z 2 − z⎜ e 2 + e 2 ⎟ + 1 z 2 − 2 z cosh ⎜ ⎟ + 1
⎜ ⎜ ⎟ ⎟ ⎝2⎠
⎜ ⎝ ⎠ ⎟
⎝ ⎠

Q25. Find the Z-transform of,


(i) cos(n + 1)θθ

(ii) sinh .
2
Ans:
(i) θ
cos(n + 1)θ
Given that,
un = cos(n + 1)θ
Applying Z-transform on both sides, we get,
Z[un] = Z[cos(n + 1)θ]

= Z[cos nθ cos θ – sin nθ sin θ] Q cos (A + B) = cos A cos B – sin A sin B

∞ ∞

= ∑[cos nθ cos θ − sin nθ sin θ]Z


n =0
−n
Q Z [an ] = ∑a Z
n =0
n
−n

∞ ∞
= cos θ ∑
n =0
cos nθZ − n − sin θ ∑sin nθZ
n =0
−n

= cos θ Z[cos nθ] – sin θ Z[sin nθ]


Since,
Z [ Z − cos α] Z sin α
Z[cos nα] = and Z[sin nα] = 2
Z − 2 Z cos α + 1
2
Z − 2 Z cos α + 1
Then,

Z ( Z − cos θ) Z sin θ
Z[un] = cos θ. – sin θ.
Z − 2 Z cos θ + 1
2 Z − 2 Z cos θ + 1
2

Z 2 cos θ − Z cos 2 θ Z sin 2 θ


= − 2
Z − 2Z cos θ + 1 Z − 2Z cos θ + 1
2

Z 2 cos θ − Z cos 2 θ − Z sin 2 θ


=
Z 2 − 2Z cos θ + 1

Z 2 cos θ − Z (cos 2 θ + sin 2 θ)


= Q cos 2 θ + sin 2 θ = 1
Z 2 − 2Z cos θ + 1

Look for the SIA GROU P LOGO on the TITLE COVER before you buy
UNIT-5 (Z-Transform) 5.19

Z 2 cos θ − Z Z ( Z cos θ − 1) ⎡ π −π ⎤
= = 2 Z ⎢ e2 −e 2 ⎥
Z − 2Z cos θ + 1
2
Z − 2 Z cos θ + 1 =
2 j ⎢⎢ 2 −
π π π π



⎣ Z − Ze − Ze + e
2 2 2 2
Z ( Z cos θ − 1) ⎦
∴ Z (cos( n + 1)θ) =
Z 2 − 2 Z cos θ + 1
⎡Q e0 = 1⎤
⎣ ⎦
⎛ nπ ⎞
(ii) sinh ⎜ ⎟
⎝ 2⎠
⎡ ⎤
Given that,
⎢ ⎛π⎞ ⎥
2 j sin h⎜ ⎟
⎛ nπ ⎞ Z ⎢ ⎝ ⎠
2 ⎥
un = sinh⎜ ⎟ = ⎢ ⎥
⎝ 2 ⎠ 2j ⎢ ⎛ π
− ⎞
π

⎜ 2 2 ⎟
⎢ Z − Z ⎜ e + e ⎟ + 1⎥
2

Applying Z-transform on both sides, we get,


⎣⎢ ⎝ ⎠ ⎦⎥
⎛ ⎛ nπ ⎞ ⎞
Z(un) = Z ⎜⎜ sinh ⎜ ⎟ ⎟⎟ ⎡Q 2 j sinh θ = eθ − e −θ ⎤
⎝ ⎝ 2 ⎠⎠ ⎣ ⎦

⎡ nπ − nπ ⎤
⎢e − e 2
2 ⎥ ⎡ ⎛π⎞ ⎤
= Z⎢ ⎥ ⎢ 2 j sin h⎜ ⎟ ⎥
⎢ 2j ⎥ Z
⎢ ⎝2⎠ ⎥
⎣ ⎦ =
2j ⎢ 2 ⎛π⎞ ⎥
⎢ Z − Z 2 cos h⎜ 2 ⎟ + 1⎥
⎣ ⎝ ⎠ ⎦
1 ⎡ 2
nπ − nπ ⎤

= Z ⎢e − e 2 ⎥
2j ⎢ ⎥⎦ ⎡Q 2 cosh θ = eθ + e −θ ⎤
⎣ ⎣ ⎦

1 ⎛⎜ 2 ⎞⎟ 1 ⎛⎜ 2 ⎞⎟
nπ − nπ

= Z e − Z e ⎡ ⎛π⎞ ⎤
2 j ⎜⎝ ⎟ 2j ⎜
⎠ ⎝

⎠ ⎢ 2 j sin h⎜ ⎟ ⎥
Z
⎢ ⎝2⎠ ⎥
=
2j ⎢ 2 ⎛π⎞ ⎥
⎛ ⎞ ⎛ ⎞ ⎢ Z − 2Z cos h⎜ 2 ⎟ + 1⎥
1 ⎜ Z ⎟ 1 ⎜ Z ⎟ ⎣ ⎝ ⎠ ⎦

2 j ⎜⎜ nπ ⎟ ⎜ − nπ ⎟
=
⎟ 2 j ⎜ ⎟
⎝Z −e 2 ⎠ ⎝ Z −e 2 ⎠
⎛ π⎞
Z sin h ⎜ ⎟
⎡ Z ⎤ ⎝2⎠
⎢Q Z (e ) =
an
a ⎥ =
⎢ Z −e ⎥ ⎛ π⎞
Z 2 − 2Z cos h⎜ ⎟ + 1
⎢ − an Z ⎥ ⎝2⎠
⎢⎣ Z (e ) = Z − e− a ⎥⎦

⎛ ⎞
1 ⎜ Z Z ⎟ ⎛ π⎞
= ⎜ − ⎟ Z sin h ⎜ ⎟
2j ⎜ π −π
⎟ ⎡ ⎛ nπ ⎞ ⎤ ⎝2⎠
⎝ Z − e2 Z − e 2 ⎠ Z ⎢sin h⎜ ⎟⎥ =
⎣ ⎝ ⎠⎦
2 ⎛ π⎞
Z 2 − 2Z cos h⎜ ⎟ + 1
⎛ ⎞ ⎝2⎠
Z ⎜ 1 1 ⎟

2 j ⎜⎜ −π ⎟
= π

⎝ Z −e2 Z −e 2 ⎠ ⎡Q cos h 90o = 0⎤
Z (1) ⎢ ⎥
⎡ ⎤ = o
⎢ π π ⎥
Z 2 − 2Z (0) + 1 ⎣⎢ sin h 90 = 1 ⎦⎥

Z ⎢ Z −e 2 −Z +e2 ⎥
= ⎢ ⎥
2 j ⎢⎛ π ⎞⎛ −π ⎞
⎥ ⎛
⎢ ⎜⎜ Z − e 2 ⎟⎟ ⎜⎜ Z − e 2 ⎟⎟ ⎥
⎛ nπ ⎞ ⎞ Z
∴ Z ⎜⎜ sin h ⎜ ⎟ ⎟⎟ = 2
⎣⎢ ⎝ ⎠⎝ ⎠ ⎦⎥ ⎝ ⎝ ⎠⎠ Z + 1
2

SPECTRUM ALL-IN-ONE JOURNAL FOR ENGINEERING STUDENTS SIA GROUP


5.20 MATHEMATICS-II [JNTU-ANANTAPUR]

az 2 + a 2 z z
Q26. Show that, Z(n2 an) = . Z[cos nθ – i sin nθ]=
(z − a) 3 z − e −iθ
Ans: To prove, z z + e iθ
= ×
az 2 + a 2 z z − e −iθ z − e −iθ
Z(n2 an) =
( z − a)3 z ( z − e iθ )
=
Let, z 2 − z (e iθ + e −iθ ) + 1
un = n2 an
z ( z − cos θ − i sin θ)
2
z2 + z =
z 2 − 2 z cos θ + 1
Z(n ) = = U(z)
( z − 1) 3
z ( z − cos θ) z sin θ
Applying scaling property, Z[cos nθ – i sin nθ] = –i 2
z − 2 z cos θ + 1
2 z − 2 z cos θ + 1
i.e., if Z(un) = U(z) then Z(an. un) = U(z/a) Equating real and imaginary parts, we get,
z2 + z z ( z − cos θ)
Here, U(z) = ∴Z[cos nθ] =
( z − 1) 3 z − 2 z cos θ + 1
2

Q28. State and prove shifting rule.


∴ Z(n2.an) =
(z a ) + (z a )
2
Ans:
3 (i) Shifting un to the Right
⎛z ⎞
⎜ − 1⎟ Statement
⎝a ⎠ If Z(un) = u(z), then
Z(un – r) = z–r u(z) (k > 0)
z 2 + az Proof
= × a3 By definition,
a 2 ( z − a)3

a ( z + az )
2 Z(un – r) = ∑u n −k z −n
∴ Z(n2.an) = n =0
( z − a )3
= u0 z–k + u1 z–(k + 1) +.....
= u0 z–k + u1 z–k – 1 + u2 z–k –2 + u3 z–k – 3 +.....
z(z − cos θ)
θ) =
Q27. Show that, Z(cos nθ . = u0 z–k + u1 z–k z–1 + u2 z–k z–2 + u3z–k z–3 + ...
z 2 − 2zcos θ + 1 = z–k [u0 + u1 z– 1 + u2 z–2 + u3 z– 3 +...]
Ans: To prove, ⎡∞ ⎤
z ( z − cosθo
−k
∑ −n
= z ⎢ u n z ⎥ = z–k u(z)
Z(cos nθ) = ⎣⎢ n =0 ⎦⎥
z − 2az cosθ + 1
2
(ii) Shifting un to the Left
z Statement
Z(1) = = U(z) If, Z(un) = u(z), then
z −1
Z(un + k) = zk[u(z) – u0 – u1 z–1 – u2 z–2 –...... – uk – 1 z–(k – 1)]
Consider, Proof
Z(e–inθ) = Z[(e–inθ) .1] By definition,
Z[(e–iθ)n .1] is in the form of Z[an. un]

Where, a = e–iθ and un = 1.
Applying scaling property,
Z(un + k) = ∑u
n =0
n+k z −n

i.e., if Z(un) = U(z)


Then Z(an . un) = U(z/a) ∞

z
= z
k
∑u
n =0
n+k z −( n + k )
∴ Here U(z) = U(1) =
z −1
⎡ ∞ k −1 ⎤
z e − iθ = z k
⎢ ∑ u n z −n
− ∑
u n z −n ⎥
∴ Z[(e–iθ)n.1] = ⎣⎢ n =0 n= 0 ⎦⎥
z e −i θ − 1
∴ Z(un + k) = zk[u(z) – u0 – u1 z–1 – u2 z–2 –....... –uk – 1 z–(k – 1)]

Look for the SIA GROU P LOGO on the TITLE COVER before you buy
UNIT-5 (Z-Transform) 5.21
n n+3
Q29. Find Z(2.3 + 5.n) deduce Z[2.3 +5(n + 3)] us-
⎡ u ⎤
ing shifting theorem. u2 = zLt
→∞
z2 ⎢U ( z ) − u 0 − 1 ⎥
Ans: To find, ⎣ z⎦
Z(2.3n + 5.n)
∴ Z(2.3 + 5.n) = Z(2.3n) + Z(5.n)
n
⎡ 2 z 2 + 4 z + 12 ⎤
= 2Z(3n) + 5Z(n) = z →∞ z ⎢
Lt 2 − 0 − 0⎥
⎣ ( z − 1)
4
1

Z
= 2. + 5. ... (1)
Z −3 ( Z − 1) 2
⎡ 2⎛ 4 12 ⎞ ⎤

⎜Q Z (a n ) =
1 ⎞
⎟ ⎢ z ⎜2 + + 2 ⎟⎥
z2 ⎢ ⎝
− z z ⎠⎥
⎜ Z a ⎟ = z Lt
⎜ Z ⎟
→∞ ⎢ ⎛ 1⎞ ⎥
4
⎜ Z ( n ) = ⎟ ⎢ z 4 ⎜1 − ⎟ ⎥
⎝ ( Z − 1) 2 ⎠ ⎣⎢ ⎝ z ⎠ ⎦⎥
Applying shifting theorem,
i.e., Z(un) = U (Z ) 2 + 4 z + 12 z 2
= z Lt =2
⇒ Z(un-k) = z–k U (Z )
→∞
(1 − 1 z )4
From equation (1), we get,
2 Z 2z 2 + 5z + 14
+ 5. = Z(2.3n–(–3) + 5(n – (–3))) Q31. If U(z) = find u2 and u3 .
Z + ( −3) ( Z + (−1)) 2 (z − 1) 4
= Z(2.3n+3 + 5(n + 3)).
Ans: Given that,
5.4 INITIAL AND FINAL VALUE THEOREMS

2z 2 + 4z + 12 2 z 2 + 5 z + 14
Q30. If Z(un) = find u2. U(z) =
(z − 1) 4
Model Paper-II, Q10 ( z − 1) 4
Ans: Given that,
u0 = zLt U(z)
2 z 2 + 4 z + 12 →∞
Z(un) =
( z − 1) 4
2 z 2 + 5 z + 14
From initial value theorem, = z Lt
→∞
( z − 1) 4
u0 = zLt
→∞
U(z) = zLt
→∞
Z(un)

2 z 2 + 4 z + 12 z 2 ( 2 + 5 z + 14 z 2 )
= zLt
= zLt →∞ 4
z (1 − 1 z )
4
→∞
( z − 1) 4

⎛ 4 12 ⎞
z2 ⎜ 2 + + 2 ⎟ = zLt
(
1 . 2 + 5 z + 14 z
2
) =0
⎝ z z ⎠
= zLt
→∞ 4
→∞
z 2
(1 − 1 z ) 4

⎛ 1⎞
z 4 ⎜1 − ⎟
⎝ z⎠
u1 = z Lt
→∞
[z[u(z) – u0]]
2
2 + 4 z + 12 z 1
= z Lt . =0
→∞
(1 − 1 z )4
z
= z Lt z⎢
⎡ 2 z 2 + 5 z + 14
− 0


⎣ ( z − 1)
→∞ 4

u 1 = zLt
→∞
z[U(z) – u0]

⎡ 2 z 2 + 4 z + 12 ⎤ = zLt
(
z × z 2 + 5 z + 14 z
2 2
)
Lt
= z →∞ ⎢
z − 0⎥ →∞ 4
z (1 − 1 z )
4

⎣ ( z − 1)
4

2 + 4 z + 12 z 2 1
Lt
(
1 2 + 5 z + 14 z 2 )
= zLt = z →∞
(1 − 1 z )4 =0
→∞ (1 − 1 z )4 . z = 0 z

SPECTRUM ALL-IN-ONE JOURNAL FOR ENGINEERING STUDENTS SIA GROUP


5.22 MATHEMATICS-II [JNTU-ANANTAPUR]

u2 = zLt
→∞
[z2(u(z) – u0 – u1 z–1)]

⎡ 2 ⎛ 2 z 2 + 5 z + 14 ⎞⎤
= zLt ⎢ z ⎜⎜ − 0 − 0 ⎟⎟⎥
⎢⎣ ⎝ ( z − 1)
4
→∞
⎠⎥⎦

z × z ( 2 + 5 z + 14 z )
2 2 2
= z Lt
→∞
=2
4
z (1 − 1 z )
4

u 3 = z Lt
→∞
z3 [U(z) – u0 – u1 z–1 – u2 z–2]

⎡ 2 z 2 + 5 z + 14 2⎤ ⎡ 2 z 4 + 5 z 3 + 14 z 2 − 2( z − 1) 4 ⎤
Lt
= z →∞ z3 ⎢ − 0 − 0 ⎥ = Lt z ⎢
3 ⎥
⎣ ( z − 1)
4
z 2 ⎦ z →∞ ⎣ z 2 ( z − 1) 4 ⎦

⎡ 2 z 4 + 5z 3 + 14 z 2 − 2( z 4 − 4 z 3 + 6 z 2 − 4 z + 1) ⎤
Lt
= z →∞ z ⎢
3 ⎥
⎣ z 2 ( z − 1) 4 ⎦

⎡ 2 z 4 + 5z 3 + 14 z 2 − 2 z 4 + 8 z 3 − 12 z 2 + 8z − 2 ⎤
= z Lt
→∞
z3 ⎢ ⎥
⎣ z 2 ( z − 1) 4 ⎦

⎡13 z 3 + 2 z 2 + 8z − 2 ⎤ ⎡13 + 2 / z + 8 / z 2 − 2 / z 3 ⎤
Lt
= z →∞ z 3 ⎢ ⎥ = Lt z ×z ⎢
3 3 ⎥ = 13
⎣ z 2 ( z − 1) 4 ⎦ z →∞ ⎣ z × z (1 − 1 / z )
2 4 4

z z
Q32. If Z {Un} = + 2 , find the Z-transform of Un+2.
z −1 z +1
Ans: Given that,
z z
Z{Un} = + 2
z −1 z +1
Z{ Un+2} = ?
Let,
z z
U(z) = +
z −1 z 2 +1

z ( z 2 + 1) + z ( z − 1)
⇒ U(z) =
( z − 1)( z 2 + 1)

z3 + z + z2 − z
⇒ U(z) =
( z − 1)( z 2 + 1)

z3 + z2
U(z) =
( z − 1)( z 2 + 1)
By shifting theorem, we get,
Z{Un+2} = z2 {U(z) –U(0) –U (1) z–1} ... (1)
By initial value theorem,
If, Z {Un} = U(z), then,
lim U(z) = U(0)
z →∞

∴ U(0) = zlim
→∞
U(z)

Look for the SIA GROU P LOGO on the TITLE COVER before you buy
UNIT-5 (Z-Transform) 5.23

And U (1) = zlim


→∞
z [U(z) – U(0)]

z2 + z3
U (0) = zlim
→∞ ( z − 1)( z 2 + 1)

z 3 (1 / z + 1)
= zlim
→∞ ⎛ 1 ⎞⎛ 1 ⎞
z 3 ⎜1 − ⎟⎜1 + 2 ⎟
⎝ z ⎠⎝ z ⎠

1+1/ z
= zlim
→∞ ⎛ 1 ⎞
(1 − 1 / z )⎜1 + 2 ⎟
⎝ z ⎠

1+ 0
= =1
(1 − 0)(1 + 0)
∴ U(0) = 1

U(1)= Lim
z→∞
z [U (z) – U (0)]

⎡ z 2 + z3 ⎤
= z →∞ z ⎢
Lim − 1⎥ [Q U (0) = 1]
⎢⎣ ( z − 1)( z + 1) ⎥⎦
2

⎡ z 2 + z 3 − ( z − 1)( z 2 + 1) ⎤
= Lim
z→∞ ⎢
z ⎥
⎣⎢ ( z − 1)( z 2 + 1) ⎦⎥

⎡ z 2 + z 3 − ( z 3 + z − z 2 − 1) ⎤
= Lim
z →∞ ⎢
z ⎥
⎣⎢ ( z − 1)( z 2 + 1) ⎦⎥

⎡ z 2 + z 3 − z 3 − z + z 2 + 1⎤
= Lim
z →∞ ⎢
z ⎥
⎢⎣ ( z − 1)( z 2 + 1) ⎥⎦

⎡ ⎤
z 2z − z + 1 ⎥
2
= Lim
z →∞ ⎢
⎣⎢ ( z − 1)( z + 1) ⎦⎥
2

⎡ 2z 3 − z 2 + z ⎤
= Lim
z →∞ ⎢ ⎥
⎣⎢ ( z − 1)( z + 1) ⎦⎥
2

⎛ 1 1 ⎞
z3⎜ 2 − + 2 ⎟
⎝ z z ⎠
= Lim
3⎛ 1⎞⎛ 1 ⎞
z →∞
z ⎜1 − ⎟ ⎜1 + 2 ⎟
⎝ z ⎠⎝ z ⎠

1 1
2−
+
z z2
= Lim
z→∞ ⎛ 1⎞⎛ 1 ⎞
⎜1 − ⎟ ⎜1 + 2 ⎟
⎝ z⎠⎝ z ⎠

SPECTRUM ALL-IN-ONE JOURNAL FOR ENGINEERING STUDENTS SIA GROUP


5.24 MATHEMATICS-II [JNTU-ANANTAPUR]

2−0+0
=
(1 − 0)(1 + 0)
=2
∴ U(1) = 2
From equation (1), we get,

⎧⎪ z 2 + z 3 ⎫
−1 ⎪
Z {Un+ 2} = z2 ⎨ − 1 − 2 z ⎬
⎪⎩ ( z − 1)( z 2 + 1) ⎪⎭

⎧⎪ z 2 + z 3 2 ⎫⎪
= z2 ⎨ − − 1⎬
⎪⎩ ( z − 1)( z + 1) z ⎪⎭
2

⎧⎪ ( z 2 + z 3 ) × z − 2[( z − 1)( z 2 + 1)] − 1[ z ( z − 1)( z 2 + 1)] ⎫⎪


= z2 ⎨ ⎬
⎪⎩ z ( z − 1)( z 2 + 1) ⎪⎭

⎧⎪ ( z 3 + z 4 ) − 2[ z 3 + z − z 2 − 1] − {z ( z 3 + z − z 2 − 1)} ⎫⎪
=z⎨ ⎬
⎪⎩ ( z − 1)( z 2 + 1) ⎪⎭

⎧⎪ z 3 + z 4 − 2 z 3 − 2 z + 2 z 2 + 2 − z 4 − z 2 + z 3 + z ⎫⎪ ⎧⎪ z 2 − z + 2 ⎫⎪
=z⎨ ⎬ = z ⎨ ⎬
⎪⎩ ( z − 1)( z 2 + 1) ⎪⎭ ⎪⎩ ( z − 1)( z 2 + 1) ⎪⎭

⎧⎪ z 2 − z + 2 ⎫⎪
∴ Un+2 = z ⎨ ⎬
⎪⎩ ( z − 1)( z 2 + 1) ⎪⎭

3z 2 − 4z + 7
Q33. If is the Z-transform of f(n), then find f(0), f(1) and f(2).
(z − 1) 3
Ans: Given that,

3z 2 − 4 z + 7
Z[f(n)] = = F(z)
( z − 1)3

To find,
f(0) = ?
f(1) = ?
f(2) = ?

⎡ 4 7⎤
z 2 ⎢3 − + 2 ⎥
⎣ z z ⎦
F(z) = 3
⎡ 1⎤
z 3 ⎢1 − ⎥
⎣ z⎦

⎡ 4 7 ⎤
⎢3− + ⎥
1⎢ z z2 ⎥
⇒ F(z) = ⎢ ⎥
z ⎛ 1 ⎞3
⎢ ⎜1 − ⎟ ⎥
⎣⎢ ⎝ z ⎠ ⎦⎥

Look for the SIA GROU P LOGO on the TITLE COVER before you buy
UNIT-5 (Z-Transform) 5.25
From initial value theorem,

f(0) = Lim F (z )
z →∞

⎡ 4 7 ⎤
⎢3 − + 2 ⎥
1⎢ z z ⎥ =0
= Lim ⎢ 3 ⎥
z →∞ z
⎢ ⎛⎜1 − ⎞⎟
1

⎢⎣ ⎝ z ⎠ ⎥⎦

∴ f ( 0) = 0

f(1)= Lim z[ F ( z ) − f (0)]


z→∞

⎡ ⎛ ⎞ ⎤
⎢ ⎜ 3− 4 + 7 ⎟ ⎥ 4 7
3− + 2
1⎜ ⎟ ⎥ 3− 0+ 0
= Lim z ⎢⎢ ⎜ z z2
3 ⎟ − 0⎥ = Lim
z z
= =3
z→∞ z (1 − 0) 3
⎢ ⎜ ⎛⎜1 − 1 ⎞⎟
3
⎟ ⎥ z →∞ ⎛ 1 ⎞
⎢⎣ ⎜⎝ ⎝ z ⎠ ⎟ ⎥ ⎜1 − ⎟
⎠ ⎦ ⎝ z⎠

∴ f (1) = 3

−1
f(2) = Lim z [ F ( z ) − f (0) − f (1) z ]
2
z →∞

⎡ ⎛ ⎞ ⎤
⎢ ⎜ 3− 4 + 7 ⎟ ⎥
2 ⎢1 ⎜ z z2 ⎟ −1 ⎥
= Lim z ⎢ ⎜
z→∞ 3 ⎟ − 0 − 3z ⎥
z
⎢ ⎜ ⎛⎜1 − 1 ⎞⎟ ⎟ ⎥
⎢⎣ ⎜⎝ ⎝ z ⎠ ⎟
⎠ ⎥⎦
⎡ 4 7 ⎤
⎢3 − + 2
2 ⎥ 2
z ⎢ z z ⎥ − 3z
= Lim
z →∞ z ⎢ 3 ⎥ z
⎢ ⎛⎜1 − ⎞⎟
1

⎣⎢ ⎝ z ⎠ ⎦⎥

⎡ 4 7 ⎤
⎢3 − + 2 ⎥
= Lim z⎢ z z ⎥ − 3z
z →∞ ⎢ 3 ⎥
⎢ ⎛⎜1 − ⎞⎟
1

⎢⎣ ⎝ z⎠ ⎥⎦
3
7 ⎡ 1⎤
3z − 4 + − 3z ⎢1 − ⎥
z ⎣ z⎦
= Lim
z →∞ 3
⎛ 1⎞
⎜1 − ⎟
⎝ z⎠

7 ⎡ ⎛1⎞ ⎛1⎞ ⎛1⎞ ⎤


2 3
3z − 4 + − 3z ⎢1 − 3⎜ ⎟ + 3⎜ ⎟ − ⎜ ⎟ ⎥
z ⎢⎣ ⎝ z ⎠ ⎝ z ⎠ ⎝ z ⎠ ⎥⎦
Lim
= z→∞ 3
⎛ 1⎞
⎜1 − ⎟
⎝ z⎠
SPECTRUM ALL-IN-ONE JOURNAL FOR ENGINEERING STUDENTS SIA GROUP
5.26 MATHEMATICS-II [JNTU-ANANTAPUR]
Q35. Using convolution theorem, find the inverse Z-
7 9 ⎛ 1 ⎞
3 z − 4 + − 3 z + 9 − + 3⎜ 2 ⎟ 1
z z ⎝z ⎠ transform of .
= Lim
z →∞ 3 ⎛ 1 −1 ⎞ ⎛ 1 −1 ⎞
⎛ 1⎞ ⎜1− z ⎟ ⎜1− z ⎟
⎜1 − ⎟ ⎝ 2 ⎠⎝ 4 ⎠
⎝ z⎠
2 3
5− + 2 Ans: Let,
Lim z z 5−0+0 1
→ ∞ 3
= z
⎛ 1 ⎞ = (1 − 0) 3 = 5 U(z) =
⎛ 1 −1 ⎞⎛ 1 −1 ⎞
⎜1 − ⎟ ⎜⎜1 − z ⎟⎟⎜⎜1 − z ⎟⎟
⎝ z⎠
⎝ 2 ⎠⎝ 4 ⎠
∴ f ( 2) = 5
1 1
5.5 CONVOLUTION THEOREM =
⎛ 1 1 ⎞⎛ 1 1⎞
=
⎛ 1 ⎞⎛ 1 ⎞
⎜⎜1 − ⎟⎟⎜⎜1 − ⎟ ⎜⎜1 − ⎟⎟⎜⎜1 − ⎟⎟
⎧⎪ ⎝ 2 z ⎠⎝ 4 z ⎟⎠ ⎝ 2 z ⎠⎝ 4 z ⎠
z2 ⎪⎫
Q34. Evaluate Z–1 ⎨ ⎬ using convolution
⎪⎩ (z − 1) (z − 3) ⎪⎭
1 1
theorem. Model Paper-II, Q11 = (2 z − 1)(4 z − 1) = (2 z − 1)(4 z − 1)
2 z.4 z 8z 2
⎧⎪ z2 ⎫⎪
Ans: To find Z–1 ⎨ ⎬
⎪⎩ ( z − 1) ( z − 3) ⎪⎭ 8z 2
U(z) =
(2 z − 1)(4 z − 1)
z ⎧ z ⎫
Z(aα) = (or) Z–1 ⎨ ⎬ = aα U ( z) 8z
z−a ⎩z − a⎭ =
( 2 z − 1)(4 z − 1)
z
By applying partial fractions, we get,
⎧ z ⎫
Considering, Z–1 ⎨ ⎬ = 1α = 1 and
⎩ z −1⎭ U ( z)
=
8z
... (1)
z (2 z − 1)(4 z − 1)
⎧ z ⎫
Z–1 ⎨ ⎬ = 3α A
+
B
⎩ z − 3⎭ =
(2 z − 1) (4 z − 1)

⎧⎪ ⎫⎪ ⎧⎪ z 8z = A(4z – 1) + B(2z – 1)
z2 z ⎫⎪
∴ Z–1 ⎨ ⎬ = Z–1 ⎨ ⎬ 8z = 4Az – A + 2Bz – B ... (2)
⎪⎩ ( z − 1) ( z − 3) ⎪⎭ ⎪⎩ z − 1 z − 3 ⎪⎭
Comparing the coefficients of ‘z’ equation (2), we get,
8 = 4A + 2B ... (3)
α
= ∑ 1 .3
β=0
β α–β Comparing the constant terms in equation (2), we get,
0=–A–B
⇒ A + B =0
α Multiplying, the above equation by 2, we get,
= ∑3
β=0
α–β
⇒ 2A + 2B = 0 ... (4)
On solving equations (3) and (4), we get,
= 3α + 3α–1 + 3α–2 + .... +30 4A + 2B = 8
2A + 2B = 0
The above expression is in G.P series. – – –
8
2A = 8 ⇒ A = =4
3α +1 (3α +1 − 1) ⎡ a(1 − r ) ⎤
n +1 2
= ⎢Q S n = ⎥
2.3α +1 ⎢⎣ 1 − r ⎥⎦ ∴A=4
On substituting the value of A in equation (4), we get,
1
= (3α+1 –1) 2(4) + 2B = 0
2
8 + 2B = 0

Look for the SIA GROU P LOGO on the TITLE COVER before you buy
UNIT-5 (Z-Transform) 5.27
2B = –8
−8
B=
2

∴ B = −4
Equation (1) can now be written as,
U ( z) 8z
=
z (2 z − 1)(4 z − 1)

U ( z) 4 (− 4)
= +
z 2z − 1 4z − 1
4z 4z
U(z) = −
2z − 1 4z − 1

4z 4z
= −
⎛ 1 ⎞ ⎛ 1⎞
2⎜⎜ z − ⎟⎟ 4⎜⎜ z − ⎟⎟
⎝ 2⎠ ⎝ 4⎠

2z z
= −
1 1
z− z−
2 4

⎛ ⎞ ⎛ ⎞
⎜ ⎟ ⎜ ⎟
z z
U(z)= 2⎜ ⎟−⎜ ⎟
⎜ 1⎟ ⎜ 1⎟
⎜ z− ⎟ ⎜z− ⎟
⎝ 2⎠ ⎝ 4⎠
Applying Z–1 on both sides,

⎛ ⎞ ⎛ ⎞
⎜ ⎟ ⎜ ⎟ ⎡ −1 ⎛ z ⎞

z ⎟ − z −1 ⎜ z ⎢Q z ⎜⎜ ⎟⎟ = a n ⎥
Z–1[U(z)]= 2 z ⎜ ⎟
−1
⎜ 1⎟ ⎜ 1 ⎟ ⎣⎢ ⎝ z−a⎠ ⎦⎥
⎜z− ⎟ ⎜z− ⎟
⎝ 2⎠ ⎝ 4 ⎠

n n
⎛1⎞ ⎛1⎞
= 2⎜⎜ ⎟⎟ − ⎜⎜ ⎟⎟
⎝2⎠ ⎝4⎠

⎡ ⎤
⎢ ⎥ n n
−1 ⎢ 1 ⎥ ⎛1⎞ ⎛1⎞
⇒ z
⎢⎛ 1 = 2⎜⎜ ⎟⎟ − ⎜⎜ ⎟⎟
−1 ⎞⎛ 1 −1 ⎞ ⎥ ⎝2⎠ ⎝4⎠
⎢ ⎜⎜1 − z ⎟⎟⎜⎜1 − z ⎟⎟ ⎥
⎢⎣ ⎝ 2 ⎠⎝ 4 ⎠ ⎥⎦

⎡ z2 ⎤ ⎡ z2 ⎤
Q36. Using convolution theorem evaluate Z–1 ⎢ ⎥ and deduce Z
–1
⎢ ⎥.
⎣⎢ (z − a)(z − b) ⎦⎥ ⎣⎢ (z − 4)(z − 5) ⎦⎥

⎡ z2 ⎤ ⎡ z z ⎤
Ans: Z–1 ⎢ ⎥ = Z–1 ⎢ . ⎥
⎣ ( z − a )( z − b) ⎦ ⎣z −a z −b⎦

⎡ z ⎤ ⎡ z ⎤
Z–1 ⎢ ⎥ = an and Z–1 ⎢ ⎥ = bn
⎣z − a⎦ ⎣z −b⎦

SPECTRUM ALL-IN-ONE JOURNAL FOR ENGINEERING STUDENTS SIA GROUP


5.28 MATHEMATICS-II [JNTU-ANANTAPUR]
⎡ z2 ⎤
∴ Z–1 ⎢ ⎥ = anbn
⎣ ( z − a)( z − b) ⎦
n
= Σ am.bn–m [From convolution theorem]
m =0

m
⎛a⎞
n
= bn Σ ⎜ ⎟
m=0 ⎝ b ⎠

⎡ ⎛a⎞ ⎛a⎞ ⎛a⎞


2 3
⎛a⎞
n

= bn ⎢1 + ⎜ ⎟ + ⎜ ⎟ + ⎜ ⎟ + ... + ⎜ ⎟ ⎥ [Which is in a geometric series]
⎢⎣ ⎝b⎠ ⎝b⎠ ⎝b⎠ ⎝b⎠ ⎥⎦

n +1
⎛a⎞
⎜ ⎟ −1
n ⎝b⎠
=b ×
⎛a⎞
⎜ ⎟ −1
⎝b⎠

a n +1 − b n +1
=
a −b
Consider, a = 4 and b = 5,

⎡ z2 ⎤ 4 n +1 − 5 n +1
Z–1 ⎢ ⎥=
⎣ ( z − 4)( z − 5) ⎦ −1
= 5n+1 – 4n+1
Q37. Using convolution theorem evaluate,
3 3
⎛ z ⎞ ⎛ z ⎞
Z–1 ⎜ ⎟ and deduce Z–1 ⎜ ⎟ .
⎝z−a⎠ ⎝ z − 1⎠
Ans: Given function is,
3
⎛ z ⎞
⎜ ⎟
⎝ z−a⎠
3
⎛ z ⎞
Z–1 ⎜ ⎟ =?
⎝ z−a⎠
3
⎛ z ⎞
Z–1 ⎜ ⎟ =?
⎝ z −1⎠
Since,

⎛ z ⎞ n
Z–1 ⎜ ⎟= a
⎝z−a⎠
2
⎛ z ⎞ ⎛ z z ⎞
Z–1 ⎜ ⎟ = Z–1 ⎜ . ⎟
⎝ z−a⎠ ⎝ z−a z−a⎠
= an.an
n
= Σ am.an–m [From convolution theorem]
m=0

Look for the SIA GROU P LOGO on the TITLE COVER before you buy
UNIT-5 (Z-Transform) 5.29
n ⎡ Z2 ⎤ ⎡ Z Z ⎤
= an Σ am . a–m −1
∴ Z ⎢ ⎥ = Z −1 ⎢ .
m=0 − − ⎥
⎣⎢ ( Z 3)( Z 4 ) ⎦⎥ ⎣Z −3 Z − 4⎦
= an (1 + 1 + 1 + ... +1)
⎡ Z ⎤ −1 ⎡ Z ⎤
= an(n + 1) = Z −1 ⎢ ⎥ * Z ⎢Z − 4⎥
⎣ Z − 3⎦ ⎣ ⎦

⎛ z ⎞
3
⎛ z2 z ⎞ = 3n * 4n
Z–1 ⎜ ⎟ = Z–1 ⎜⎜ . ⎟

⎝ z − a ⎠ ⎝ ( z − a ) 2
z − a ⎠ n

= [an (n + 1)] an
= ∑3
m =0
m
.4 n − m

n
= Σ am (m + 1) an–m n
m=0 = ∑3
m =0
m
.4 n .4 − m

n
= an Σ (m + 1) n
3m
m=0 =4
n
∑4
m=0
m
= an(1 + 2 + 3 + .... + (n +1))
n m
3
⎛3⎞
∴ Z–1 ⎜
⎛ z ⎞
⎟ = an . 1 (n + 1) (n + 2) = 4n ∑ ⎜ ⎟
m =0 ⎝ ⎠
4
⎝ z−a⎠ 2
Consider, a = 1,
n
⎡ ⎛ 3 ⎞1 ⎛ 3 ⎞ 2 ⎛3⎞ ⎤
n
= 4 ⎢1 + ⎜ ⎟ + ⎜ ⎟ + ...... + ⎜ ⎟ ⎥
⎛ z ⎞
3
1 ⎢⎣ ⎝ 4 ⎠ ⎝ 4 ⎠ ⎝ 4 ⎠ ⎥⎦
Z–1 ⎜ ⎟ = 1n . (n + 1) (n + 2)
⎝ z −1⎠ 2
⎡ ⎛ ⎛ 3 ⎞ n +1 ⎞ ⎤
1 ⎢1× ⎜1 − ⎜ ⎟ ⎟ ⎥
= (n + 1) (n + 2) n
⎢ ⎜⎝ ⎝ 4 ⎠ ⎟⎠ ⎥
2 = ⎢
4 ⎥
⎢ 3 ⎥
1−
Q38. State convolution theorem and using it find the ⎢ 4 ⎥
⎣⎢ ⎦⎥
z2
inverse Z-transform of .
(z − 3)(z − 4) ⎡ a(1 − r n +1 ) ⎤
⎢Q Sum of n terms in G . P is, S n = ⎥
⎣⎢ 1− r ⎥⎦
Ans:
Convolution Theorem ⎡ 3n +1 ⎤
n +1
Statement = 4 ⎢1 − n +1 ⎥
⎢⎣ 4 ⎥⎦
If Z–1[f(z)] = f(n) and Z–1[g(z)] = g(n)
Then, Z–1[f(z). g(z)] = f(n) * g(n) n +1 3n +1
=4 − .4 n +1 = 4 n +1 − 3 n +1
4 n +1
n

= ∑ f (m).g (n − m)
n =0
5.6 SOLUTION OF DIFFERENCE EQUATION
BY Z-TRANSFORMS
To find, Q39. Solve the difference equation, using Z-transform
y(k + 2) – 5y(k + 1) + 6y(k) = 5n, given y(0) = 0, y(1) = 0.
⎡ Z2 ⎤ Ans: Given Z-transform is,
Z −1 ⎢ ⎥ using convolution theorem.
⎣⎢ ( Z − 3)( Z − 4) ⎦⎥ yn+2 – 5yn+1 + 6yn = 5n

Since, ⎡ y ⎤ z
⇒ z2 ⎢ y ( z ) − y 0 − 1 ⎥ –5z [y(z) – y0] + 6y(z) =
⎣ z⎦ z − 5
⎡ Z ⎤
∴ Z −1 ⎢ ⎥ =a
n
⎣Z −a⎦ Given, y0 = 0, y1 = 0

SPECTRUM ALL-IN-ONE JOURNAL FOR ENGINEERING STUDENTS SIA GROUP


5.30 MATHEMATICS-II [JNTU-ANANTAPUR]
Q40. Solve the difference equation using Z-transform
z
⇒ 2
y(z) [z –5z + 6] = x(n+2) – 3xn+1 – 10xn = 0, given x(0) = 1, x(1) = 0.
z −5
Ans: Given that,
z xn+2 – 3xn+1 – 10xn = 0
⇒ y(z) =
( z − 5)( z 2 − 5 z + 6) Applying Z-transform to the above equation,
z(xn+2) – 3z(xn+1) – 10z(xn) = 0
z
=
( z − 5)( z − 2)( z − 3) ⎡ x ⎤
⇒ z2 ⎢ x ( z ) − x 0 − 1 ⎥ – 3z[x(z) – x0]–10x(z) = 0
⎣ z⎦
A B C
= + + ⇒ x(z) [z2 – 3z – 10] – x0[z2 – 3z] – zx1 = 0 ... (1)
z − 2 ( z − 3) ( z − 5)
Given, x0 = 1 and x1 = 0
A( z − 3) ( z − 5) + B( z − 2) ( z − 5) + C ( z − 2) ( z − 3)
On substituting the above values in the equation (1),
= (z2 – 8z + 15) A + B (z2 – 7z + 10) + C(z2 – 5z + 6) we get,
= z2 (A + B + C) + z (–8A – 7B – 5C) + (15A + 10B + 6C) x(z) [z2 – 3z – 10] – [z2 – 3z] – 0 = 0
On comparing, ⇒ x(z) [z2 – 3z – 10] = z2 – 3z
A + B + C = 0. z 2 − 3z
–8A – 7B – 5C = 1. ⇒ x(z) =
z 2 − 3z − 10
⇒ –3A – 2B – [5A + 5B + 5C] = 1
z ( z − 3) z ( z − 3)
⇒ –3A – 2B – 5(A + B + C) = 1 ⇒ x(z) = =
z 2 − 3 z − 10 ( z − 5)( z + 2)
⇒ –3A – 2B = 1. ... (1)
Similarly, x( z) z−3
=
15A + 10B + 6C = 0. z ( z − 5) ( z + 2)
9A + 4B + 6 (A + B + C) = 0. By partial fraction expansion, we get,
⇒ 9A + 4B = 0. ... (2)
x( z ) A B z −3
Multiplying equation (1) by ‘2’ and adding it to equation = + = ... (2)
(2), we get, z ( z − 5) ( z + 2) ( z − 5 ) ( z + 2)
−6 A − 4 B = 2 (z – 3) = A(z + 2) + B(z – 5) ... (3)
9 A + 4B = 0 On substituting z = –2, we get,
2 ⇒ –2 –3 = A(–2 + 2) + B(–2 – 5)
3A = 2 ∴A=
3 ⇒ –5 = 0 – 7B
On substituting the value of A in equation (2) we get. ⇒ –5 = –7B
⎛2⎞ 18 5
9⎜⎜ ⎟⎟ + 4B = 0 ⇒ + 4B = 0 ∴B =
⎝3⎠ 3 7

On substituting z = 5 in equation (3), we get,


−3
∴B = 5 – 3 = A(5 + 2) + B(5 – 5)
2
⇒ 2 = 7A + 0
Since, A + B + C = 0
⇒ 2 = 7A
2 3
⇒ – +C=0 2
3 2 ∴A=
7
5
∴C = ∴ Equation (2) becomes,
6
2 5
2 ⎡ z ⎤ 3 ⎡ z ⎤ 5⎡ z ⎤ x( z) 7 7

3 ⎢⎣ z − 2 ⎥⎦ 2 ⎢⎣ z − 3 ⎥⎦ 6 ⎢⎣ z − 3 ⎥⎦
y(z) = – + = +
z ( z − 5) ( z + 2)

2 n 3 n 5 n 2 z 5 z
y(z ) = (2) − (3) + (5) ⇒ x(z) = +
3 2 6 7 z −5 7 z+2

Look for the SIA GROU P LOGO on the TITLE COVER before you buy
UNIT-5 (Z-Transform) 5.31
Applying Z–1 on both sides, we get, On substituting z = 2 in equation (3), we get,
1 = A(2 + 1) + B(2 – 2)
2 n 5
xn = 5 + (–2)n ⇒ 1 = A(3) + B(0)
7 7
On substituting z = –1 in equation (3), we get,
1 = A(–1 + 1) + B(–1 –2)
2.5 n + 5(−2) n
∴ x( z ) = ⇒ 1 = A(0) + B(–3)
7
⇒ 1 = B (–3)
Q43. Solve the difference equation, using Z-transforms ⇒ B = –1/3
un +2 – un = 2n where u0 = 0, u1 = 1.
On substituting A, B values in equation (1), we get,
Ans: Model Paper-III, Q11
1 −1
Given that, u( z)
⇒ = 3 + 3
u n + 2 – u n = 2n z ( z − 2) ( z + 1)
Applying Z-transform on both sides,
Z[un+2 – un] = z[2n] u( z ) 1 −1
= +
⇒ Z(un+2) – z(un) = z[2n] z 3( z − 2) 3( z + 1)

z z z
⇒ z2(u(z) – u0 – u1z–1) – u(z) = ⇒ u(z) = –
z−2 3( z − 2) 3( z + 1)
Applying Z–1 on both sides, we get,
z
⇒ 2 2
u(z) {z –1} – u0 (z ) – u1(z) = un = Z–1[uz]
z−2
1 –1 ⎧⎪ z ⎫⎪ 1 ⎧ z ⎫
z ⎨ ⎬ – Z–1 ⎨ ⎬
⇒ u(z) {z2–1} – 0 – z = =
3
Z
⎪⎩ (z − 2) ⎪⎭ 3 ⎩ z +1⎭
z−2
[Q Given u0 = 0, u1 = 1] 1 n 1
⇒ un =
3
2 – (–1)n
3
z
⇒ u(z) {z2 – 1} = +z
z−2 1 n
⇒ un =
3
{2 – (–1)n}
z + z 2 − 2z
⇒ u(z) {z2 – 1} =
z−2 1
n
⎛ 1⎞
Q42. Solve the difference equation un+1 + un = ⎜ ⎟ ,
4 ⎝4⎠
z2 − z
⇒ u(z) {z2 – 1} = n ≥ 0, u0 = 0, u1 = 1 using Z-transforms.
z−2
Ans: Given that,
z2 − z n
∴ u(z) = 1 ⎛1⎞
( z − 2)( z 2 − 1) un+1 + un = ⎜ ⎟
4 ⎝4⎠
z ( z − 1) Applying Z-transform on both sides, we get,
=
( z − 2)( z + 1)( z − 1) ⎛ ⎛ 1 ⎞n ⎞
1 ⎜ ⎟
Z(un+1) + Z(un) = z ⎜⎜ ⎜⎜ 4 ⎟⎟ ⎟⎟
⎝⎝ ⎠ ⎠
u( z) 1 4
⇒ = ... (1)
z ( z − 2)( z + 1)
Let, Z(un) = u (z)
Solving equation (1) using partial fractions,
Then, Z(un+1) = Z( u (z) – u0) and
1 A B
= + ... (2) z
( z − 2)( z + 1) z−2 z +1 Z(an) =
z−a
1 A( z + 1) + B ( z − 2) 1 z
⇒ ( z − 2)( z + 1) = ( z − 2)( z + 1) ∴ Z[ u (z) – u0] + u (z) =
4 1
z−
⇒ 1 = A(z +1) + B (z – 2) ... (3) 4

SPECTRUM ALL-IN-ONE JOURNAL FOR ENGINEERING STUDENTS SIA GROUP


5.32 MATHEMATICS-II [JNTU-ANANTAPUR]
Z[yn + 1] = Z[y(z) – y0]
1 1
⇒ z u (z) + u (z) = [Q z(u0) = 0] Z[yn] = y(z)]
4 1
1 − z −1
4 Using u0 = 0 and u1 = 1

1
⎛ 1⎞ 1 ⇒ z2[u(z) – 0 – ] – 3z[u(z) – 0] + 2u(z) = 0
u (z) ⎜⎜ z + ⎟⎟ = 1 z
⎝ 4 ⎠ 1 − z −1
4 1
⇒ z2[u(z) – ] – 3z u(z) + 2u(z) = 0
z
1 1
u (z) = × ⇒ (z2 – 3z + 2) u(z) – z = 0
1 −1 1
1− z z+
4 4 ⇒ z(z2 – 3z – 2) u(z) = z2
⇒ (z2 – 3z – 2) u(z) = z
1 1
= × ⇒ (z2 – 2z – z – 2) u(z) = z
1 −1
1− z ⎛ 1 ⎞
z⎜⎜1 + ⎟⎟ ⇒ [(z – 2) (z – 1)] u(z) = z
4 ⎝ 4 z⎠
z
⇒ u(z) =
z −1
1 ( z − 1)( z − 2)
= ×
1 −1 1 −1
1+ z 1− z u( z) 1
4 4 ⇒ = ... (1)
z ( z − 1)( z − 2)
By partial fractions, we get,
Taking partial fractions on equation (1), we get,
2 2
u (z) = – + 1 A B
1 −1 1 −1 +
1+ z 1− z =
4 4 ( z − 1)( z − 2) z −1 z − 2
Applying Z–1 on both sides, we get, ⇒ 1 = A[z – 2] + B[z – 1] ... (2)
On substituting z = 1 in equation (2), we get,
Z–1( u (z)) = Z–1 ⎡⎢
−2 ⎤ ⎡ 2 ⎤
+ Z–1
1 −1 ⎥ ⎢ 1 −1 ⎥ 1 = A[1 – 2] + B[0]
⎢1 + z ⎥
⎣ 4 ⎦ ⎢1 − z ⎥ – A =1
⎣ 4 ⎦
∴A = −1
⎡ ⎛ 1 −1 ⎤ ⎡ 1 −1 ⎞ ⎤
−1
−1 ⎞ −1 ⎛
un = Z–1 ⎢− 2⎜⎜1 + z ⎟⎟ ⎥ + z ⎢2⎜⎜1 − z ⎟⎟ ⎥ On substituting z = 2 in equation (2), we get,
⎢ ⎝ 4 ⎠ ⎥⎦ ⎢ ⎝ 4 ⎠ ⎥⎦
⎣ ⎣ ⇒ 1 = 0 + B[2 – 1]
n n
⎛ −1 ⎞ ⎛1⎞ ∴B =1
= (–2) ⎜⎜ ⎟⎟ + 2⎜⎜ ⎟⎟
⎝ 4 ⎠ ⎝4⎠ Hence,

⎡⎛ 1 ⎞ n ⎛ − 1 ⎞ n ⎤ u( z) −1 1
= +
= 2 ⎢⎜⎜ ⎟⎟ − ⎜⎜ ⎟⎟ ⎥ , where (n ≥ 0) z z − 1 z − 2
⎢⎝ 4 ⎠ ⎝ 4 ⎠ ⎥
⎣ ⎦
⎡ z ⎤ ⎡ z ⎤
Q43. Solve the difference equation, un+2 – 3un+1 + 2 un= ⇒ U(z) = − ⎢ ⎥+⎢ ⎥ ... (3)
⎣ z − 1⎦ ⎣ z − 2 ⎦
0 given u0 = 0 and u1 = 1.
Ans: Given that, Applying inverse Z-transform on equation (2), we get,
un + 2 – 3un + 1 + 2un = 0 with u0 = 0 and u1 = 1 −1 ⎡ z ⎤ −1 ⎡ z ⎤
Z–1[U(z)] = − Z ⎢ ⎥ + Z ⎢ z − 2⎥
Applying Z-transform to the above equation, we get, ⎣ z − 1 ⎦ ⎣ ⎦
Z[un + 2] – 3Z[un + 1] + 2Z[un] = 0
⎡ −1 ⎛ z ⎞ n⎤
Z2[u(z) – u0 –
u1
] – 3z[u(z) – u0] + 2u(z) = 0 Un = – (1)n + (n)n ⎢Q Z ⎜⎝ z − a ⎟⎠ = a ⎥
z ⎣ ⎦
y1
[Q Z[yn + 2] = z2[y(z) – y0 – ] ∴Un = 2n − (1)n
z

Look for the SIA GROU P LOGO on the TITLE COVER before you buy
UNIT-5 (Z-Transform) 5.33
n
Q44. Solve un+2 + 4un+1 + 3un = 3 with u0 = 0; u1 = 1 using Z-transforms.
Ans: Given that,
un+2 + 4un+1 + 3un = 3n
Applying Z-transform on both sides,
Z[un+2 + 4un+1 + 3un] = z[3n]
⇒ Z[un+2] + 4Z[un+1] + 3Z[un] = Z[3n]

z
⇒ Z2[u(z) – u0 – u1z–1] + 4z[u(Z) – u0] + 3u(z) =
z −3

z
⇒ u(z){z2 + 4z + 3} – u0[z2 + 4z] – u1(z) =
z−3

z
⇒ u(z){z2 + 4z + 3} – 0 – z = Q Given u0 = 0, u1 = 1
z −3

z
⇒ u(z){z2 + 4z + 3} – z =
z −3

z
⇒ u(z){z2 + 4z + 3} = +z
z−3

z + z 2 − 3z
⇒ u(z){z2 + 4z + 3} =
z−3

z2 − 2z
⇒ u(z){z2 + 4z + 3} =
z −3

z 2 − 2z
∴ u(z) =
( z − 3)( z 2 + 4 z + 3)

z ( z − 2)
=
( z − 3)( z + 1)( z + 3)

u(z) ( z − 2)
= ... (1)
z ( z − 3)( z + 1)( z + 3)

Solving equation (1) using partial fractions,

u( z) ( z − 2)
=
z ( z − 3)( z + 1)( z + 3)

A B C
= + + ... (2)
( z − 3) ( z + 1) ( z + 3)

( z − 2) A( z + 1)( z + 3) + B( z − 3)( z + 3) + C ( z − 3)( z + 1)


⇒ =
( z − 3)( z + 1)( z + 3) ( z − 3)( z + 1)( z + 3)

⇒ (z – 2) = A(z+ 1)(z+ 3) + B(z – 3)(z + 3) + C(z – 3)(z + 1) ... (3)

SPECTRUM ALL-IN-ONE JOURNAL FOR ENGINEERING STUDENTS SIA GROUP


5.34 MATHEMATICS-II [JNTU-ANANTAPUR]
On substituting z = 3 in equation (3), we get,
(3 – 2) = A(3 + 1)(3 + 3) + B(3 – 3)(3 + 3) + C(3 – 3)(3 + 1)
1= A(4)(6) + B(0)(6) + C(0)(4)
1= A(24)

1
∴A=
24

On substituting z = –1 in equation (3), we get,


(–1 –2) = A(–1 + 1)(–1 + 3) + B(–4)(2) + C(–4)(0)
⇒ –3 = A(0) + B(–8) + C(0)
⇒ –3 = B(–8)

3
∴B=
8
On substituting C = –3 in equation (3), we get,
–3 – 2 = A(–3 + 1)(–3 + 3) + B(–3 + 1)(–3 + 3) + C(–3 – 3)(–3 + 1)
⇒ – 5 = 0 + 0 + C(–6)(–2)

−5
∴C =
12
On substituting A, B and C values in equation (2), we get,

1 3 −5
u(z) 24 + 8
= + 12
z ( z − 3) ( z + 1) ( z + 3)

1 ⎛ z ⎞ 3⎛ z ⎞ 5 ⎛ z ⎞
⇒ u(z) = ⎜ ⎟+ ⎜ ⎟− ⎜ ⎟
24 ⎝ z − 3 ⎠ 8 ⎝ z + 1 ⎠ 12 ⎝ z + 3 ⎠
Applying Z–1 on both sides, we get,
un = Z–1[uz]

1 −1 ⎧ z ⎫ 3 −1 ⎧ z ⎫ 5 −1 ⎧ z ⎫
= z ⎨ ⎬+ z ⎨ ⎬− z ⎨ ⎬
24 ⎩ z − 3⎭ 8 ⎩ z + 1 ⎭ 12 ⎩ z + 3⎭

1 n 3 5
= 3 + (−1) n − (−3) n
24 8 12

1 n 3 5
∴ un = 3 + (−1) n − (−3) n
24 8 12
Q45. Solve un+2 – 2un+1 + un = 3n + 5 using Z-transforms.
Ans: Given that,
un+2 – 2un+1 + un = 3n + 5
Applying Z-transform on both sides, we get,
Z[un+2] – 2Z[un+1] + Z[un] = Z[3n + 5]

⎛ 2 ⎡ y1 ⎤ ⎞
⎜Q z [ y n + 2 ] = z ⎢ y ( z ) − y 0 − z ⎥ ⎟
⎜ ⎣ ⎦⎟
⎡ u ⎤ ⎛ z ⎞ ⎛ z ⎞ ⎜ z [ y + 1] = z [ y − ( z − y 0 )] ⎟
z 2 ⎢u ( z ) − u 0 − 1 ⎥ − 2 z[u ( z ) − u0 ]+ u ( z ) = 3⎜⎜ ⎟ + 5⎜
2 ⎟
⎟ ⎜ ⎟
=
⎣ z⎦ ⎝ ( z − 1) ⎠ ⎝ z − 1 ⎠ ⎜ z [ y n ] y ( z ) ⎟
⎝⎜ ⎠⎟

Look for the SIA GROU P LOGO on the TITLE COVER before you buy
UNIT-5 (Z-Transform) 5.35

u1 3z + 5 z 2 − 5z
⇒ z2u(z) – z2u0 – z2 – 2zu(z) + 2zu0 + u(z) =
z ( z − 1) 2

u1 5z 2 − 2z
⇒ (z2 – 2z + 1)u(z) + (2z – z2)u0 – z2 =
z ( z − 1) 2

5z 2 − 2z
⇒ (z – 1)2 u(z) + (2z – z2)u0 – u1z = [Q (a – b)2 = a2 – 2ab + b2]
( z − 1) 2

5z 2 − 2 z
⇒ (z – 1)2 u(z) = − (2 z − z 2 )u0 + u1 z
( z − 1) 2

5z 2 − 2 z ( z 2 − 2z ) u1 z
⇒ u(z) = + u0 + ... (1)
( z − 1) 4
( z − 1) 2
( z − 1) 2
Let,

5z 2 − 2 z
U1 =
( z − 1) 4

z2 − 2z
U2 = u0
( z − 1) 2

z
U3 = u1
( z − 1) 2
Thus, the equation (1) can be written as,
u(z) = U1 + U2 + U3 ... (2)
Taking partial fractions to express. U1 and U2 in terms of z(1), z(n), z(n2) and z(n3).

5z 2 − 2 z A( z 3 + 4 z 2 + z ) B( z 2 + z ) Cz Dz
Let, U1 = = + + +
( z − 1) 4
( z − 1) 4
( z − 1) 3
( z − 1) 2 ( z − 1)

⎛ ⎞
⎜Q z ( a n ) = 1 ⎟
⎜ z−a ⎟
⎜ z z ⎟
⎜ z (1) = , z (n ) = ⎟
⎜ z − 1 ( z − 1) 2 ⎟
⎜ ⎟
⎜ z ( n 2 ) = z + z , z (n 3 ) = z + 4 z +
2 3 2
z⎟
⎝⎜ ( z − 1) 3 ( z − 1) 4 ⎠⎟
⇒ 5z2 – 2z = A(z3 + 4z2 + z) + B(z2 + z) (z – 1) + Cz(z – 1)2 + Dz(z – 1)3
⇒ 5z2 – 2z = A(z3 + 4z2 + z) + B(z2 + z) + C(z3 – 2z2 + z) + D(z4 – 3z3 + 3z2 – z) ... (3)
On substituting z = 1 in equation (3), we get,
5(1) – 2 = A(13 + 4(1)2 + 1)
3 = A(6)

1
∴A =
2
On comparing the coefficients of z4, we get,

∴D = 0

SPECTRUM ALL-IN-ONE JOURNAL FOR ENGINEERING STUDENTS SIA GROUP


5.36 MATHEMATICS-II [JNTU-ANANTAPUR]
Comparing the coefficients of z3, we get,
A + B + C – 3D = 0
1
+ B+C −0 =0
2

−1
⇒ B+C= ... (4)
2
Similarly, comparing the coefficients of z2, we get,
4A – 2C + 3D = 5

⎛1⎞
4⎜ ⎟ − 2C + 0 = 5
⎝2⎠
2 – 2C = 5
–2C = 5 – 2

−3
∴C =
2
On substituting the value of C in equation (4), we get,
−1 3
B= +
2 2

∴B =1

5z 2 − 2 z 1 ⎡ z 3 + 4z 2 + z ⎤ z 2 + z 3 ⎡ z ⎤
4 = ⎢ ⎥+ − ⎢ ⎥ ... (5)
( z − 1) 2 ⎢⎣ ( z − 1) 4 ⎦⎥ ( z − 1) 3 2 ⎣⎢ ( z − 1) 2 ⎦⎥

Similarly,

z 2 − 2z z z
U2 = = − ... (6)
( z − 1) 2 ( z − 1) ( z − 1) 2

Applying inverse Z-transform on equation (1), we get,

⎧ 2
−1 ⎪ 5 z − 2 z ( z 2 − 2z) u1 ⎫⎪
Un = Z–1(u(z)) = z ⎨ + u 0 + ⎬
⎪⎩ ( z − 1) 4 ( z − 1) 2 ( z − 1) 2 ⎪⎭

−1 ⎧ 5 z 2 − 2 z ⎫ −1 ⎧ z 2 − 2 z ⎫ ⎧ z ⎫
= z ⎨ 4 ⎬
+z ⎨ u + z −1 ⎨
2⎬ 0 2⎬ 1
u ... (7)
⎩ ( z − 1) ⎭ ⎩ ( z − 1) ⎭ ⎩ ( z − 1) ⎭
On substituting equation (5) and (6) in equation (7), we get,

⎡ −1 ⎡ z ⎤ ⎤
⎢Q z ⎢ ⎥ = an ⎥
⎢ ⎣z −a⎦ ⎥
⎛1 3 ⎞
U n = ⎜ n 3 + n 2 − n ⎟ + u0 (1 − n) + u1 (n) ⎢ ⎡ az ⎤ ⎥
⎝2 2 ⎠ ⎢ z −1 ⎢ = n.a n ⎥
2⎥
⎢⎣ ⎣ ( z − a) ⎦ ⎥⎦

Q46. Solve un+2 – 6un + 1 + 9un = 0.


Ans: Given that,
un+2 – 6un+1 + 9un = 0 ... (1)
Applying Z-transform to equation (1), we get,
Z[un+2] – 6Z[un+1] + 9Z[un] = 0

Look for the SIA GROU P LOGO on the TITLE COVER before you buy
UNIT-5 (Z-Transform) 5.37

⎡ u ⎤ z−6 1 −3
z 2 ⎢u ( z ) − u 0 − 1 ⎥ − 6 z[u ( z ) − u 0 ] + 9u ( z ) = 0 ∴ = +
⎣ Z⎦ ( z − 3) 2 z − 3 ( z − 3) 2

⎡ 2 ⎡ y1 ⎤ ⎤ Hence,
⎢Q z[ y n + 2 ] = z ⎢ y ( z ) − y0 − ⎥ ⎥
⎢ ⎣ z ⎦⎥
⎢ z[ y n +1 = z ] y ( z − y 0 ) ⎥ u( z ) ⎡ 1 3 ⎤ 1
⎢ ⎥ =⎢ − 2⎥ 0
u + u1
⎢ z[ y n ] = y ( z ) ⎥ z ⎢⎣ z − 3 ( z − 3) ⎥⎦ ( z − 3) 2
⎢ ⎥
⎣ ⎦

u( z) ⎡ 1 3 ⎤ 1⎡ 3 ⎤
⇒ =⎢ − 2⎥ 0
u + ⎢ ⎥u1
2 2 2 ⎛ u1 ⎞ z ⎢⎣ z − 3 ( z − 3) ⎥⎦ 3 ⎢⎣ ( z − 3) 2 ⎥⎦
⇒ z u ( z ) − z u 0 − z ⎜ ⎟ − 6 zu ( z ) + 6 zu 0 + 9u ( z ) = 0
⎝ z ⎠

⇒ (z2 – 6z + 9)u(z) – (z2 – 6z)u0 – zu1 = 0 ⎡ z 3z ⎤ 1 ⎡ 3z ⎤


⇒ u(z) = ⎢ − 2⎥ 0
u + ⎢ ⎥u1 ... (3)
⇒ (z2 – 6z + 9)u(z) = (z2 – 6z)u0 + zu1 ⎣⎢ z − 3 ( z − 3) ⎦⎥ 3 ⎣⎢ ( z − 3) 2 ⎦⎥

( z 2 − 6 z )u0 + zu1 Applying inverse Z-transform on equation (3), we get,


⇒ u(z) =
z − 6z + 9
2

⎡ ⎛ z ⎞ −1 ⎛⎜ 3 z ⎞⎤ ⎡ ⎤
z ( z − 6)u 0 + zu1
−1
Z–1[u(z)] = ⎢ z ⎜ ⎟− z ⎜ ⎟⎥u 0 + 1 z −1 ⎢ 3 z ⎥u1
⎝ z −3⎠ 2 ⎟ 2
⇒ u(z) = ⎢⎣ ⎝ ( z − 3) ⎠⎥⎦ 3 ⎢⎣ ( z − 3) ⎥⎦
z2 − 6z + 9

u( z ) ( z − 6)u 0 + u1 n n 1 n
⇒ = ⇒ un = [(3) − n.3 ]u 0 + [ n.3 ].u1
z ( z − 3) 2 3
⎡ −1 ⎡ Z ⎤ ⎤
⎢Q Z ⎢ ⎥ = an ⎥
u( z) ( z − 6) 1 ⎣Z −a⎦
⇒ = u0 + u1 ⎢ ⎥
z ( z − 3) 2 ( z − 3) 2 ⎢ ⎡ aZ ⎤ ⎥
⎢ Z −1 ⎢ = n

2⎥
n.a
⎢⎣ ⎣ ( Z − a) ⎦ ⎥⎦
z−6
Taking partial fractions of , i.e.,
( z − 3) 2
u n = (1 − n).3n u0 + n.3n −1.u1

z−6 A B
= + Q47. By using Z-transforms, solve the difference
( z − 3) 2
z − 3 ( z − 3) 2 equation yn + 2 – 3yn + 1 + 2yn = 0 subject to the
conditions y0 = – 1 and y1 = 2.
z – 6 = A[z – 3] + B[1] ... (2)
Ans: Given that,
On substituting z = 3 in equation (2), we get,
3 – 6 = A[0] + B yn + 2 – 3yn + 1 + 2yn = 0, y0 = – 1 and y1 = 2

Applying Z-transform, we get,


∴B = −3
Z[yn + 2] – 3Z[yn + 1] + 2Z[yn] = 0
On comparing the constant terms of equation (2), we get,
– 6 = – 3A + B ⎡ y ⎤
⇒ z 2 ⎢ y ( z ) − y 0 − 1 ⎥ – 3 z[y(z) – y ] + 2 y(z) = 0
⇒ – 3A = – 6 – B ⎣ z⎦ 0

⇒ – 3A = – 6 – (– 3)
y1
⇒ – 3A = – 3 ⇒ y(z)[z2 – 3z + 2] – z2 y0 – (z)2 + 3zy0 = 0
z
∴A =1 Given that, y0 = – 1 and y1 = 2

SPECTRUM ALL-IN-ONE JOURNAL FOR ENGINEERING STUDENTS SIA GROUP


5.38 MATHEMATICS-II [JNTU-ANANTAPUR]

⎛ 2⎞
⇒ y(z)[z2 – 3z + 2] – z2 (– 1) – z2 ⎜ ⎟ + 3z(–1) = 0
⎝z⎠

⇒ y(z)[z2 – 2z – z + 2] + z2 – 2z – 3z = 0
⇒ y(z)[(z – 2) (z – 1)] + z2 – 5z = 0
⇒ y(z)[(z – 2) (z – 1)] = 5z – z2

5z − z 2
⇒ y(z) =
( z − 1)( z − 2)

y( z) 5− z
⇒ =
z ( z − 1)( z − 2)

By partial fraction expansion, we have,

y ( x) 5− z A B
⇒ = = + ... (1)
z ( z − 1)( z − 2) z −1 z − 2

⇒ 5 – z = A[z – 2] + B[z – 1] ... (2)


On substituting z = 2, in equation (2), we get,
⇒ 5 – 2 = A[0] + B[2 – 1]

∴B = 3

On substituting z = 1 in equation (2), we get,


⇒ 5 – 1 = A[1 – 2] + B[0]
⇒ –A=4

∴ A = −4

y( z) −4 3
∴ = +
z z −1 z − 2

⎡ z ⎤ ⎡ z ⎤
y(z) = – 4 ⎢ ⎥ + 3⎢ ⎥
⎣ z −1⎦ ⎣ z − 2 ⎦

Applying inverse Z-transform on both sides, we get,

−1 ⎡ z ⎤ −1 ⎡ z ⎤
Z–1[y(z)] = – 4 z ⎢
− ⎥ + 3 z ⎢ z − 2⎥
⎣ z 1 ⎦ ⎣ ⎦

⎡ −1 ⎡ z ⎤ n⎤
= – 4 (1)n + 3(2)n ⎢Q Z ⎢
− ⎥=a ⎥
⎣ ⎣ z a ⎦ ⎦

= – 4 + 3.2n
= 3(2n) – 4

Q48. Solve the difference equation yn+2 + 3yn+1 + 2yn = 0, y0 = 1, y1 = 2 by Z-transform.


Ans: Given difference equation is,
yn + 2 + 3yn + 1 + 2yn = 0 ... (1)
y0 = 1, y1 = 2

Look for the SIA GROU P LOGO on the TITLE COVER before you buy
UNIT-5 (Z-Transform) 5.39
On applying z-transform to equation (1), we get,
Z[yn + 2] + 3Z[yn + 1] + 2z[yn] = 0

⎡ y1 ⎤
⎢Q z[ y n + 2 ] = z [ y ( z ) − y 0 − z ⎥
2

⎢ ⎥
⎡ y ⎤ ⎢ z[ yn +1 ] = z[ y ( z ) − y0 ] ⎥
⇒ z 2 ⎢ y ( z ) − y0 − 1 ⎥ + 3z[ y ( z ) − y0 ] + 2 y ( z ) = 0 ⎢ z[ y ] = y ( z ) ⎥
⎣ z⎦
⎢ n ⎥
⎣ ⎦

z 2 y1
⇒ z 2 y ( z ) − z 2 y0 − + 3zy( z ) − 3zy0 + 2 y ( z ) = 0
z

y1
⇒ y ( z )[ z 2 + 3z + 2] − z 2 y0 − z 2 − 3zy 0 = 0 ... (2)
z

On substituting the values of ‘y0’ and y1 in equation (2), we get,

( 2)
⇒ y(z)[z2 + 3z + 2] – z2 (1) – z2 − 3z (1) = 0
z

⇒ y(z) [z2 + 3z + 2] – z2 – 2z – 3z = 0
⇒ y(z) [(z + 2) (z + 1)] – z2 – 5z = 0
⇒ y(z) [(z + 2) (z + 1)] = z2 + 5z

z 2 + 5z
⇒ y(z) =
( z + 1)( z + 2)

y( z) z+5
⇒ =
z ( z + 1)( z + 2)

Applying partial fractions, we get,

y( z) z+5 A B
= = + ... (3)
z ( z + 1)( z + 2) z +1 z + 2

⇒ z + 5 = A[z + 2] + B[z + 1] ... (4)


On substituting z = – 2 in equation (4), we get,
⇒ – 2 + 5 = A[0] + B[– 2 + 1]
3=–B

∴B = −3

On substituting z = – 1 in equation (4), we get,


⇒ – 1 + 5 = A[– 1 + 2] + B[0]
4 =A

∴A = 4

SPECTRUM ALL-IN-ONE JOURNAL FOR ENGINEERING STUDENTS SIA GROUP


5.40 MATHEMATICS-II [JNTU-ANANTAPUR]
On substituting the values of A and B in equation (3), we get,

y( z) 4 (−3)
= +
z z +1 z + 2

⎡ z ⎤ ⎡ z ⎤
y (z ) = 4⎢ ⎥ − 3⎢ ⎥ ... (5)
⎣ z + 1⎦ ⎣ z + 2 ⎦

On applying inverse Z-transform to the above equation, we get,

−1 ⎡ z ⎤ −1 ⎡ z ⎤
Z–1 [y(z)] = 4 z ⎢
+ ⎥ − 3z ⎢ z + 2 ⎥
⎣ z 1 ⎦ ⎣ ⎦

⎡ −1 ⎡ z ⎤ n⎤
= 4(– 1n) – 3 (– 2n) ⎢Q z ⎢
+ ⎥ = (− a ) ⎥
⎣ ⎣ z a ⎦ ⎦

∴ Z–1[y(z)] = 4(–1)n – 3 (–2)n

Look for the SIA GROU P LOGO on the TITLE COVER before you buy

You might also like